Sei sulla pagina 1di 792

READING SUMMARY

FOR OET 2.0

Dr Samed Alsalemi
OCCUPATIONAL ENGLISH TEST - OET 2.0

READING SUB-TEST
Updated Reading sub-test (60 Minutes):
The updated Reading sub-test will include:
 A new task types such as matching, short-answer questions
 A new broader range of documents, such as policy or procedure documents.
As per the old test, it will still be relevant for all professions and test your ability to:
 Understand texts and source information from multiple texts (‘skim’ and ‘scan’).
The updated Reading sub-test consists of three separate parts with a total of 42 questions, and takes
60 minutes to complete.

The three different parts of the Reading Test are described below:
Part A : 4 Expeditious (fast) reading task (20 questions /15 minutes)
 Test contains 4 extracts on the same topic that a healthcare professional might use in the
course of their work.
 20 matching, sentence completion and short answer question.
 You will have 15 min. to locate specific information in quick and efficient manner

Part B: 6 short workplace extracts ,careful reading task (100-150 word /10 min)
 Identify the detail , gist or purpose from 6 extracts (policy, guideline)
 1 three option mcqs X 1 = 6
 You should aim to spend roughly 10 minutes on this section.

Part C, 2 long passage of healthcare topics ,careful reading task (800 word /35 min)
 Identify detailed meaning and opinion
 8 four option mcqs X 2 = 16
 You should aim to spend roughly 35 minutes on this section.
READING STRATEGIES
Unlike the OET Listening Test – where you must keep pace with the recording to avoid falling behind –
you must pace yourself in the Reading Test. You should time yourself while you attempt the questions
within this chapter.
Make sure you are familiar with the Reading Test instructions in advance, so that on Test Day, you can
focus on answering the questions.
Familiarise yourself with the different parts of the Reading test, and be prepared for each question
type. Use this chapter to build your knowledge of the different tasks in the Reading test, then assess
your skills with the practice set of questions at the end of each part. Revisit and revise any questions
you struggled with, identify what caused you to struggle with the question, and practise answering
similar questions.
Look out for the important words in the question that can help you to locate the information in the
text.
Understand how writers construct their texts to communicate their message. Some functional
language will be useful to them in many contexts regardless of topic, e.g. to show:
– the order of events e.g. firstly, secondly; initially, subsequently, in the end.
– consequences e.g. due to, therefore, as a result.
– contrasting or alternative ideas e.g. however, on the other hand, despite.
– the extension of an idea e.g. in addition, furthermore.
As well as focusing on specific language, help your students to become familiar with common
features of academic and professional texts, such as:
– text references e.g. this, the other study, as noted above.
– nominalization: choosing nouns rather than verbs or adjectives, e.g. explanation [from
explain], detoxification, assessment.
– complex comparative structures e.g. The study found that women over 60 benefited from
the therapy almost twice as much as those aged between 20 and 35 did.
– long noun phrases e.g. The four-year study into the uptake and continuing use of the
drug-based treatment administered with appropriate medical supervision discovered that
– groups of words which relate to degree of certainty e.g. states, concludes, implies, suggests,
proposes, assumes, supposes, believes, considers, presumes
READING SUB-TEST: PART A (15 minutes)

Part A : Expeditious (fast) reading tasks (20 questions /15 minutes)


You need to locate specific information from 4 short text related to single healthcare topic in quick and
efficient manner to answer 20 Matching, sentence completion and short answer question in 15 mins.

Strategies
1. Know the Instructions
You should make sure you are familiar with what you have to do before you take the test.
The instructions for Reading Part A look like this:
Instructions to candidates: (there are 2 isolated Text Booklet and Question Paper)
- Look at the four texts, A – D, in the Text Booklet.
- For each question, 1 – 20, look through the texts, A – D, to find relevant information.
- Write your answers on spaces provided in the Question Paper.
- Answer all the questions within the 15-minute time limit.
- Your answers should be correctly spelt .
2. Know the Format: Skim each text to identify the title and text types of each of the texts.
 Use headings and layout of short text to get quick initial idea of information type which are:
- A flow chart, table, graphs or diagram.
- Information in paragraphs and bullet-pointed lists.
- Research /abstract
- Report
- Case studies
- Extract from an article
- Question and answer
- Statistics
- Definition
- FAQs
3.Identify the Question Types: start reading the questions to identify the Types of the 1st question.
1. Matching (1st 7 question )
2. Short Answer
3. Sentence Completion
4. Pay attention to the words and meaning of the question
- Underline the key words in the task and locate these words in the text.
- Be on the lookout for synonyms and clues which indicate where the answer can be found.
5. Scanning the texts and locate the key words needed for the particular information
- Keep text Booklet open in front of you to see text and questions paper at same time.
- Don’t begin part A by simple read all text ,this will waste your time.
- According to types, words and question meaning, use it to guide you to which text read first.
- Don’t worry, If at first the topic of the texts are unfamiliar to you, you will not need to have prior
knowledge of the specific condition or treatment discussed in the texts. Simply focus on finding the
word or phrase in relevant text that allows you to answer question.

Matching question (usually 1st 7 question )


The questions in this section ask you to identify which text contains a particular type of information
or information about a particular aspect of the condition or issue.
For each of the questions, 1-7, decide which text (A, B, C or D) the information comes from.
You may use any letter more than once.
- Read the titles of all the readings so you can quickly move to the text where the answer is.
- Understand the general meaning of the 4 texts, rather than the specific details.
- Find the matching information in one of the passages
For example, a Matching Question might ask ‘Which text provides information about identifying
delirium in patients?’ To answer these questions, you need to understand the general meaning of the
4 texts, rather than the specific details.

Short Answer (next 8-15 question )


The questions in this section ask you to locate the correct word or short phrase from texts. These
questions will often include specific words or a single sentence in one of the texts, such types of
medication, treatment or reactions in patients, might be found in a single sentence in one of texts.
Answer each of the questions, ! -! , with a word or short phrase from one of the texts.
Each answer may include words, numbers or both.

Completion question(next 16-20 question )


The questions in this section ask you to fill the gaps in each sentence with a word or short phrase
from one of the texts.
Complete each of the sentences, ! -! , with a word or short phrase from one of the texts.
Each answer may include words, numbers or both.
The process for answering these questions is same for Short Answer and Sentence Completion
- Use question to guide you to which text read.
- Underline the key words in the question and locate these words in the text.
- Read the titles of all the texts so you can quickly move to the text where the answer is.
- Use statement to find out what type of information you need and decide in which short text is.
- look at the four texts in more detail to locate the texts where key words needed information
- You will have to write the answer to using only the information given in the texts.
- Once you’ve found the information, you need to understand the type of word or phrase you need
to answer the question(look at the question to understand what type of information is asked for)
- When you’re struggling to find the answer to the question, it can be helpful to narrow down the
type of answer you need, and look for the possible answers in the relevant text.
for example, a question that asks ‘What should patients suffering from fever be given?’ could be
answered with a type of medication, a beverage, or an item.
The question could not, on the other hand, be answered with a number alone, or a type of illness.

6. Place the words in the answer box next to the text.


- Answer part A with consistent word as in text not use your background knowledge.
- Don’t include words or phrases that are not present in text.
- Don’t use words with similar meaning to words in text this is synonyms.
- Keep your answers short, and avoid including unnecessary information
- The answer will generally require 1 to 3 words from the text.
- Before completing the sentence, read the sentence to yourself (in your head, not out loud) with
your answer, to confirm that the sentence makes sense using the words you have chosen. If it does,
complete the sentence. If it does not make sense, read the relevant piece of information more
closely, and choose another word or short phrase such pregnant with twins or triple in text ,but
pregnant with more than one pregnancy in summary.
- Don’t waste time with eraser, you can cross any wrong word.

7. Continue this process for the remainder of the text.


Example –
There four texts, A-D, in the separate Text Booklet.

The use of feeding tube in pediatric


 Text A pediatric NG tube use
 Text B insertion NG tube
 Text C NG tube insertion algorithm
 Text D administrating of feed/fluid via feeding NG tube

Questions 1-7
For each question, 1-7, decide which text (A, B, C or D) the information comes from. You may use any
letter more than once.

1. The risk of feeding a child via NG tube? A -------------------------

7. How regularly different kind of tube replacing ? A -------------------------

Questions 8 -14
Answer each of the questions, 8-14, with a word or short phrase from one of the texts.
Each answer may include words, numbers or both.

8 .What type of tube used to a patient need feeding for short time? ------------

14. What device allow for delivery of feed via small bowel ? -----------

Questions 15-20
Complete each of the sentences, 15-20, with a word or short phrase from one of the texts.
Each answer may include words, numbers or both

15. if tube is not straight when you unwarp it ,you should ------------

20 .if child is receiving ……………… via NG tube, you should replace bottle after 4 hrs.

AFTER 15 MINUTES.

END OF PART A, THIS QUESTIONS PAPER WILL BE COLLECTED


READING SUB-TEST PARTS B : Short workplace extracts, careful reading task
Part B: 6 short workplace extracts ,careful reading task (100-150 word /10 min)
 Identify the detail ,gist or purpose from 6 extracts (policy, guideline)
 1 three option mcqs X 1 = 6
 Part B and C = 45 minutes, you should aim to spend roughly 10 minutes on part B, so you leave
yourself enough time to complete Part C. This means you should aim to spend on average 1
minute 30 seconds to complete each Part B question, although you may find some questions
quicker to answer while others may take more time.

Strategies

1.Know the Instructions


Instructions to candidates:
 In this part of the test, there are six short extracts relating to the work of health professionals.
 For questions 1 to 6, choose the answer (A, B or C) which you think fits best according to the text.
 Write your answers by filling in the circle using a 2B pencil on the separate Answer Sheet
 Don’t fill in more than one circle.
Advice to candidates:
 Read contextual information for each text the purpose and audience of content
 Read each answer option carefully and scan the text for evidence if answer correct.
 Manage your time carefully, you should aim to spend most 45 min. in part C.
 Don’t distracted by unfamiliar vocabulary ,most question can answered without understand all
meaning.

2. Know the Settings


The texts that you will see in this section of the test represent the types of texts and documents that
healthcare professionals will often refer to for specific reasons. Rather than focusing on technical
medical information that you might find in a textbook, for instance, these documents will cover
guidelines, policies and procedures. They might remind healthcare professionals of the best practice to
follow in a given circumstance, or updates to a data storage system, or even instruct employees how to
use machinery
Part B texts will generally be made up extracts from the following test types:
- Notices – Emails – Manuals – Memos - Guidelines

To answer as many questions in this section correctly as possible, you need to focus your efforts on
understanding the main point of the information provided in each of the texts.
3. Scan the Question
Unlike the Listening Test, the OET Reading Test does not provide additional time to look through the
questions before answering them, so you should factor this into your total time. You should aim to
answer 6 questions in roughly 10 minutes, so don’t spend too long on any one question. If you can’t
find the answer, move on to the next question and come back to this one at the end.
In this section of the test, there are three different question types. We’ll go through each question
type now, and provide examples for each question type.
- Main Idea
- Detail
- Purpose

Main Idea
These questions ask for the main idea of the piece of information. To answer these questions, you will
need to assess what the main point is of what is being communicated.
For questions 1-6, choose the answer (A, B or C) which you think fits best.

Example - Extracts 1

Continuity and coordination of care


All healthcare professionals must be involved in the safe transfer of patients between each other and
social care providers. This includes:
 Sharing all relevant information with colleagues that are involved with your patient’s care, both
inside and outside the team, including when the care handover is done at the end of duty, and
when care is delegated or referred to other health or social care providers.
 When possible, check that a named team or clinician has appropriately taken over responsibility
when your role has ended in providing a patient’s care. This is most important in vulnerable
patients who do not have capacity
When care is transferred or delegated to another healthcare professional, it is your responsibility to
ensure that the person providing care has the appropriate skills, qualifications and experience to
provide adequately safe care for the patient.
The policy document tells us that a healthcare professional’s
(A) duty to care for a patient continues after a referral.
(B) ability to look after a patient should be decided by superiors.
(C)obligation to check up on transfers does not apply to all patients
Detail

Detail questions will ask you to answer a question about a specific part of the text. They will often
include words in the question that you can use to skim the text for the relevant information.

Example - Extracts 2

For questions 2, choose the answer (A, B or C) which you think fits best.

Ensuring Patient Privacy

Patient privacy is legally governed by HIPAA, which establishes strict standards for healthcare
providers when sharing patient information. Every hospital will have guidelines healthcare employees
must follow to avoid committing an HIPAA violation, which can result in termination from
employment and/or severe fines. Employees must avoid talking about identifiable patient information
with other people that are not involved in their care.
This also includes discussing patient details in a public setting like a hallway or elevator. When sending
information about patients to other providers, it is important to use secure forms of transmission
such as hospital email and fax. Avoid easy but unprotected methods like texting or personal email.
Dispose of any identifiable information in specially marked bins for later incineration.

What point does the training manual make about confidential documents?
(A) They must not be consulted in an open area.
(B) They must only be shared via work email.
(C) They must be destroyed after use
Purpose

Purpose questions require you to choose the answer that best explains the point of the text.
Think about why the text was written, what should healthcare professionals reading the text do as a
result of reading it?

Example - Extracts 3
For questions 3, choose the answer (A, B or C) which you think fits best.

Procedural Guidelines for Set-up and Administration of Intravenous Fluids

Intravenous (IV) fluids are infused directly into the veins of patients via a cannula in cases of severe
dehydration, electrolyte imbalance, blood loss, and in surgery. Intravenous lines can also be used for
administration of drugs directly into the blood of a patient, resulting in faster action. The guidelines
below illustrate the correct procedure for setting up and administering IV therapy.

Firstly, always check that the fluid bag is not damaged and that
the liquid inside it is clear. Secondly, there have been reports of incomplete patient notes, so it is
crucial that you check for details such as fluid type and expiration date and record these in the patient
notes immediately. Thirdly, it is vital that all clinical
staff introduce themselves with their full name and role to all patients they engage with; only after
confirming patient details and obtaining their consent should one begin the IV set-up.
Finally, be extra diligent when calculating the drip rate as to avoid any errors. Feel comfortable to
approach a fellow colleague for assistance if uncertain at any stage.

The main purpose of the guidelines is to advise staff on


(A) the procedure to follow when fitting an IV.
(B) how to check for issues with IV infusions.
(C) what to do before administering an IV.

…….Then go to extracts 4-6 … same as extracts 1-3


READING SUB-TEST PARTS C : long presentation passage - careful reading task

Part C, 2 long passage of healthcare topics ,careful reading task (800 word /35 min)
 Identify detailed meaning and opinion
 8 four option mcqs X 2 =16
 You should aim to spend roughly 35 minutes on this section.

Strategies

1. Know the Instructions

Instructions to candidates:
 In this part of the test, there are two texts about different aspects of healthcare.
 For questions 7 to 22, choose the answer(A,B,C or D)which you think fits best according to the text.
 Write your answers by filling in the circle using a 2B pencil on the separate Answer Sheet
 Don’t fill in more than one circle.

Advice to candidates:
 There is no thematic links between the two text ,don’t waste time for this.
 Manage your time carefully, you should aim to spend most 45 min. in part C.
 Read each question carefully and looking out for keywords.
 Consider each option and ask yourself ,what makes each one right or wrong.
 If you not sure about answer, consider moving on and coming back to it latter.
 Don’t stuck on one question keep going and come to it at end.
 Write your answers by filling in circle using a 2B pencil ,don’t fill in more than one.
2. Know the Format

Each question refers to a discrete part of the text, and the questions appear in the order of the
information in the text. You need to answer 8 questions on each of the 2 texts in this section of the
test. The questions will direct you to the part of the text which the question refers, so you don’t need
to skim the entire text to find the information. In addition, once you have answered a
question about one part of the text, you only need to look at the information that follows for the
remaining questions, you will not need to look at the previous information.

3. Look at the Text

The first thing you should do, when tackling a Part C text, is to quickly look through the text, to
understand how the text is organised and what it's about.
As you skim through the text, you might find it helpful to make a few brief notes about the main idea
or topic of each paragraph, to help you to remember what the text discusses. Don’t worry about
understanding the text in detail at this point, just give yourself a general sense of the text. When you
come to answer the questions, the questions will guide you through the text.

4. Answer the Different Question Types

Once you’ve scanned the text, you can begin to work through the 8 questions.
You should aim to spend one to one and a half minutes answering each question in this section, so
make sure to time yourself accurately as you complete the exercises in this chapter. When you come to
answer the questions in Part C, first look at what the question is asking you to do, then scan the
relevant paragraph of information, then look at the answer options. Mark any answer options that
definitely do not answer the question with a cross, and if you’re stuck between two answer choices,
read through the relevant information again, then if you still can’t find the answer, select one of the
answer choices as a guess and move on.

The questions in Part C of the Reading Test can be broadly divided into 3 different question types.
1. Detail, Attitude and Opinion
2. Vocabulary
3. Reference
Detail, Attitude and Opinion

Detail, Attitude and Opinion questions will ask you to identify information from a section of a text, and
will most often focus on the views and opinions of the writer. These questions are the most common
question type in Part C.

Vocabulary
Vocabulary questions will present you with a single word or phrase that will be underlined and
formatted in bold in the question and the text.
To answer these questions correctly, you will need to look at the surrounding words and deduce the
meaning added by the word or phrase. These questions are not testing your knowledge of the definition
of the word or phrase itself. You should expect to answer one Vocabulary question in each text in Part C.

Reference
Reference questions will ask you to decide what the word or phrase underlined and in bold in the
question and in the text relates to.
To answer these questions, you might need to be able to keep track of what is being
discussed in long sections of text with complex sentences. You should expect to answer one Reference
question in each passage in Part C.

We will go through each question type below, and provide examples for each question type.
Example - Text 1: SYNTHETIC VOICES

There are many reasons why a patient may lose their voice; indeed, many of us will already have
experienced partial loss of voice, when suffering from a cold or flu. While we tend to dismiss a hoarse
voice as a mild annoyance, when permanent voice loss occurs, it can be tremendously difficult for the
patient to deal with, both practically, and emotionally. When our voice works, we don’t spend too
much time thinking about what like would be like without it, but the truth is that our voice is an
integral part of who we are. Our voices define us, they allow our loved ones to identify us over the
phone, or when visibility is poor. They distinguish us as individuals from certain parts of the world,
and they can even indicate our social standing. Until recently, patients
who experienced permanent loss of voice would have had relatively few options at their disposal.
However, as technology advances, the range of speech replacement options available becomes
increasingly sophisticated.

Today, synthetic voices are the most common type of speech replacement device used by those who
have permanently lost their voice. The technology used to create this software can also be seen in
speech controlled home devices, and modern smartphones. As permanent loss of voice is often
caused by respiratory issues resulting from other illnesses, however, it’s important that speech
replacement devices for those who have lost their voice take the patient’s other disabilities into
account. Speech-to-text systems typically involve a system of levers or a simplified keyboard; the
latter tends to be easier for those with limited mobility to operate. Users are able to manipulate these
controls in order to select words from a computer interface and build them into sentences. Some
systems can also operate via eye movement alone, so that when a user stares at a particular word on
the screen for a certain amount of time, it is selected.

These systems show a remarkable advancement from one of the earliest speech-to-text mechanisms
designed in the sixties: a typewriter operated through an air pipe, known as a sip and puff typewriter.
The first electrical communication device for disabled people who could not speak, a sip and puff
typewriter called the POSM (Patient Oriented Selector Mechanism), was developed by Reg Maling, a
volunteer at a hospital for paralysed people, after he discovered that patients at the hospital who had
lost the use of their voice were only able to communicate using a bell. Throughout the rest of the
twentieth century, these technologies were gradually developed, and in the 1970s, the first portable,
commercially available, adaptive alternative communication devices (or AACs), were produced.
Although they were advertised as portable, these devices often weighed a hefty 15 – 20 pounds, and
tended to range from 20 to 25 inches in size. As many of the early portable AAC users also used a
wheelchair, in which it was relatively straightforward to design a holster at the back of the chair to
store these devices.
Thankfully, the technology continued to develop, and devices became smaller, easier to use and more
sophisticated. In the United States there are now over two million people who rely on such devices in
their day-to-day communications, yet many users still have to make do with a limited number of vocal
choices—often less than a dozen, with the majority of available voices sounding adult and/or male.
This is extremely problematic, as users need to choose a voice that they feel represents who they are.
Proponents of new digital voice banks are working toward raising the bar by steadily widening the
scope for self-expression among the many millions of diverse users of
AACs.

If patients are gradually losing their voice, but still able to speak, they may be able to record their own
voice to use with their AAC. Another alternative open to patients is to make use of the increasing
number of voices being donated.
Although voice donation does not require the contributor to physically give a part of themselves
away, as is the case with classic medical donations, donators certainly must go the extra mile. The
process of voice donation is much more extensive than, say, donating a kidney, or other physical
organ.
While the donation of an organ requires a relatively short stay in hospital, to donate a voice requires
many weeks of donor commitment. Donors must speak many thousands of preselected words,
phrases and sentences into a recording microphone. Some companies offer a service tailored to the
user, who can read science fiction or fantasy stories out loud—or texts according to their interests—in
order to remain more engaged in the process.

Once a voice has been comprehensively recorded, it then becomes part of the
software for AACs, and made available to any patient that needs it. Professor Stephen Hawking, the
famous Cambridge physicist, began to use an early text-to-speech system in 1986 called CallText.
Interestingly, the professor never changed his synthetic voice to a more sophisticated design that
better imitated natural speech. Instead, Hawking retained CallText, explaining that he felt the limited
modulations of the voice allowed his speech to be easier to hear and understand during lectures.
Clearly, Hawking also came to see it as a part of his identity. 30 years after he began using CallText,
the software was nearing breakdown, but rather than simply replace it, he had a team of
researchers reverse engineer the voice onto a more modern platform.
Questions (8 mcqs/ four option mcq)

For questions 7-22, choose the answer (A, B, C or D) which you think fits best

The following list gives examples of the types of Detail, Attitude and Opinion questions that occur in
this section:
7.In the first paragraph, the writer suggests loss of voice is difficult for patients because it is
A part of their identity.
B necessary for interaction.
C used to form relationship.
D an indicator of social class.

8. Why does the writer believe it is important that speech replacement devices be operated by a
variety of methods?
A The technology should be kept up to date.
B Patients often suffer from various conditions.
C Healthcare workers might also need to use them.
D The devices should be usable across a range of platforms.

9. In the third paragraph, we learn that Reg Malling developed the POSM due to
A the number of people who had permanently lost their voice.
B the lack of accessibility in previous sip and puff designs.
C the limited communication options for disabled people.
D the recent development of similar sound technology.

10. According to the writer, why were early portable AACs problematic for those not in wheelchairs?
A They were heavy and bulky.
B They were remarkably fragile.
C They could not be used while walking.
D They needed access to a power source.
The following list gives examples of Vocabulary questions:

11. The writer uses the phrase ‘raising the bar’ to underline the
A complexity of modern devices.
B need for a diverse range of voices.
C high quality of the sound recordings.
(D) number of new communication systems.

12. What is suggested about voice donation by the phrase ‘go the extra mile’?
A donation centers are often far away
B a large number of voices are rejected
C donators sacrifice more than organ donators
(D) the process is extremely time-consuming

The following list gives examples of Reference questions:

13. In the fifth paragraph, the word ‘user’ refers to


A healthcare workers who treat loss of voice.
B patients with permanent loss of voice.
C AAC technology developers.
(D) voice donators.

14. What does the word ‘it’ refer to in the final paragraph?
A A presentation given by the professor.
B The research carried out for the professor.
C The synthetic voice used by the professor.
(D) The permanent loss of voice of the professor.

…………….Then go to Text 2: Questions 15-22 … as, same as Question 7-22

At the end of the test, hand in this Question Paper


OET 2.0 READING TASK
Instructions :

READING SUB-TEST–: PART A (TEXT BOOKLET + QUESTIONS AND ANSWER SHEET)


 Look at the four texts, A-D, in the (printable) Text Booklet.
 For each question, 1-20, look through the texts, A-D, to find the relevant information.
 Write your answers on the spaces provided in the ANSWER SHEET.
 Answer all the questions within the 15-minute time limit.
 Your answers should be correctly spelt.

READING SUB-TEST–: PART B (TEXT BOOKLET AND QUESTIONS + ANSWER SHEET)


 Look at the six short extracts relating to the work of health professionals.
 For questions 1-6, choose the answer (A, B or C) which you think fits best according to the text.
 Write your answers by filling in the circle using a 2B pencil on the separate Answer Sheet
 Don’t fill in more than one circle.

READING SUB-TEST–: PART C (TEXT BOOKLET AND QUESTIONS + ANSWER SHEET)


 Look at the two long texts about different aspects of healthcare.
 For questions 7-22, choose the answer (A, B, C or D) which you think fits best according to the text.
 Write your answers by filling in the circle using a 2B pencil on the separate Answer Sheet
 Don’t fill in more than one circle.
READING TEST 1

READING SUB-TEST : PART A


 Look at the four texts, A-D, in the separate Text Booklet.
 For each question, 1-20, look through the texts, A-D, to find the relevant information.
 Write your answers on the spaces provided in this Question Paper.
 Answer all the questions within the 15-minute time limit.
 Your answers should be correctly spelt.

TEXT BOOKLET – IDA


PART A -QUESTIONS AND ANSWER SHEET

Questions 1-7
For each of the questions, 1-7, decide which text (A, B, C or D) the information comes from.
You may use any letter more than once.

In which text can you find information about


1. considerations when treating children with iron deficiency?
2. essential steps for identifying iron deficiency?
3. evaluating iron deficiency by testing for blood in stool?
4. risk factors associated with dietary iron deficiency?
5. different types of iron solutions?
6. a treatment for iron deficiency that is no longer supported?
7. appropriate dosage when administering IV iron infusions?

Questions 8-14
Answer each of the questions, 8-14, with a word or short phrase from one of the texts.
Each answer may include words, numbers or both. Your answers should be correctly spelt.

8. What level of serum ferritin leads to a diagnosis of iron deficiency?


9. What is the most likely cause of iron deficiency in children?
10. Which form of iron can also be injected into the muscle?
11. What should a clinician do if iron stores are normal and anaemia is still present?
12. How long after iron replacement therapy should a patient be re-tested?
13. Which form of iron is presented in a vial?
14. What is the first type of treatment iron deficient patients are typically given?
Questions 15-20

Complete each of the sentences, 15-20, with a word or short phrase from one of the texts.
Each answer may include words, numbers or both. Your answers should be correctly spelt.

In comparison to breast milk and infant formula, cows’ milk is (15).........................


Special procedures should be used because (16)........................ may be poisonous for children.
Men over 40 and women over 50 with a recurring iron deficiency should have an (17).............
Iron sucrose can be given to a patient no more than (18).........................
Although serum ferritin level is a good indication of deficiency, interpreting the results is
sometimes difficult (19).........................
IV iron infusions are a safe alternative when patients are unable to (20).........................

END OF PART A, THIS QUESTIONS PAPER WILL BE COLLECTED


READING SUB-TEST : PART B

In this part of the test, there are six short extracts relating to the work of health professionals.
For questions 1-6, choose answer (A, B or C) which you think fits best according to the text.

Questions 1-6

1. The code of conduct applies to


A. doctors friending patients on Facebook.
B. privacy settings when using social media.
C. electronic and face to face communication.

Professional obligations

The Code of conduct contains guidance about the required standards of professional behaviour,
which apply to registered health practitioners whether they are interacting in person or online.
The Code of conduct also articulates standards of professional conduct in relation to privacy and
confidentiality of patient information, including when using social media. For example, posting
unauthorised photographs of patients in any medium is a breach of the patient’s privacy and
confidentiality, including on a personal Facebook site or group, even if the privacy settings are
set at the highest setting (such as for a closed, ‘invisible’ group).
2. Why does dysphagia often require complex management?
A. Because it negatively influences the cardiac system.
B. Because it is difficult contrast complex and non-complex cases.
C. Because it seldom occurs without other symptoms.

6.1- General principles

Dysphagia management may be complex and is often multi-factorial in nature. The speech
pathologist’s understanding of human physiology is critical. The swallowing system works with
the respiratory system. The respiratory system is in turn influenced by the cardiac system, and
the cardiac system is affected by the renal system. Due to the physiological complexities of the
human body, few clients present with dysphagia in isolation.

6.2- Complex vs. non-complex cases

Broadly the differentiation between complex and non-complex cases relates to an


appreciation of client safety and reduction in risk of harm. All clinicians, including new
graduates, should have sufficient skills to appropriately assess and manage noncomplex cases.
Where a complex client presents, the skills of an advanced clinician are
required. Supervision and mentoring should be sought for newly graduated clinicians or
those with insufficient experience to manage complex cases.
3.The main point of the extract is
A. how to find documents about infection control in Australia.
B. that dental practices must have a guide for infection control.
C. that dental infection control protocols must be updated.

1- Documentation

1.1 Every place where dental care is provided must have the following documents in
either hard copy or electronic form (the latter includes guaranteed Internet access).
Every working dental practitioner and all staff must have access to:
a). a manual setting out the infection control protocols and procedures used in that
practice, which is based on the documents listed at sections 1.1(b), (c) and (d) of these
guidelines and with reference to the concepts in current practice noted in the documents listed
under References in these guidelines
b). The current Australian Dental Association Guidelines for Infection Control (available at:
http://www.ada.org.au)
4. Negative effects from prescription drugs are often
A. avoidable in young people.
B. unpredictable in the elderly.
C. caused by miscommunication.

Reasons for Drug-Related Problems: Manual for Geriatrics Specialists

Adverse drug effects can occur in any patient, but certain characteristics of the elderly make
them more susceptible. For example, the elderly often take many drugs (polypharmacy) and have
age-related changes in pharmacodynamics and pharmacokinetics; both increase the risk of
adverse effects. At any age, adverse drug effects may occur when drugs are prescribed and taken
appropriately; e.g., new-onset allergic reactions are not predictable or preventable.
However, adverse effects are thought to be preventable in almost 90% of cases in the elderly
(compared with only 24% in younger patients). Certain drug classes are commonly involved:
antipsychotics, antidepressants, and sedative-hypnotics.
In the elderly, a number of common reasons for adverse drug effects, ineffectiveness, or both are
preventable. Many of these reasons involve inadequate communication with patients or
between health care practitioners (particularly during health care transitions).
5. The guideline tries to use terminology that
A. presents value-free information about different social groups.
B. distinguishes disadvantaged groups from the traditional majority.
C. clarifies the proportion of each race, gender and culture.

Terminology

Terminology in this guideline is a difficult issue since the choice of terminology used to
distinguish groups of persons can be personal and contentious, especially when the groups
represent differences in race, gender, sexual orientation, culture or other characteristics.
Throughout the development of this guideline the panel end eavoured to maintain neutral and
non-judgmental terminology wherever possible. Terms such as “minority”, “visible minority”,
“non-visible minority” and “language minority” are used in some areas; when doing so the panel
refers solely to their proportionate numbers within the larger population and infers no value on
the term to imply less importance or less power. In some of the recommendations the term
“under-represented groups” is used, again, to refer solely to the disproportionate
representation of some citizens in those settings in comparison to the traditional majority.
6. What is the purpose of this extract?
A. To illustrate situations where patients may find it difficult to give negative feedback.
B. To argue that hospital brochures should be provided in many languages.
C. To provide guidance to people who are victims of discrimination.

Special needs

Special measures may be needed to ensure everyone in your client base is aware of your
consumer feedback policy and is comfortable with raising their concerns.

For example, should you provide brochures in a language other than English?

Some people are less likely to complain for cultural reasons. For example, some Aboriginal people
may be culturally less inclined to complain, particularly to non-Aboriginal people. People with
certain conditions such as hepatitis C or a mental illness, may have concerns about discrimination
that will make them less likely to speak up if they are not satisfied or if something is wrong.
READING SUB-TEST : PART C

In this part of the test, there are two texts about different aspects of healthcare.
For questions 7-22, choose the answer (A, B, C or D) which you think fits best according to the text.

Text 1: Difficult-to-treat depression


Depression remains a leading cause of distress and disability worldwide. In one country’s
survey of health and wellbeing in 2007, 7.2% of people surveyed had experienced a mood
(affective) disorder in the previous 12 months. Those affected reported a mean of 11.7
disability days when they were “completely unable to carry out or had to cut down on their
usual activities owing to their health” in the previous 4 weeks. There was also evidence of
substantial under-treatment: amazingly only 35% of people with a mental health problem had
a mental health consultation during the previous 12 months. Three-quarters of those seeking
help saw a general practitioner (GP). In the 2015–16 follow-up survey, not much had
changed. Again, there was evidence of substantial unmet need, and again GPs were the
health professionals most likely to be providing care.

While GPs have many skills in the assessment and treatment of depression, they are often
faced with people with depression who simply do not get better, despite the use of proven
psychological or pharmacological therapies. GPs are well placed in one regard, as they often
have a longitudinal knowledge of the patient, understand his or her circumstances, stressors
and supports, and can marshal this knowledge into a coherent and comprehensive
management plan. Of course, GPs should not soldier on alone if they feel the patient is not
getting better.
In trying to understand what happens when GPs feel “stuck” while treating someone with
depression, a qualitative study was undertaken that aimed to gauge the response of GPs to
the term “difficult-to-treat depression”. It was found that, while there was confusion around
the exact meaning of the term, GPs could relate to it as broadly encompassing a range of
individuals and presentations. More specific terms such as “treatment-resistant depression”
are generally reserved for a subgroup of people with difficult-to-treat depression that has
failed to respond to treatment, with particular management implications.

One scenario in which depression can be difficult to treat is in the context of physical illness.
Depression is often expressed via physical symptoms, however it is also true is that people
with chronic physical ailments are at high risk of depression. Functional pain syndromes
where the origin and cause of the pain are unclear, are particularly tricky, as complaints of
pain require the clinician to accept them as “legitimate”, even if there is no obvious physical
cause. The use of analgesics can create its own problems, including dependence. Patients
with comorbid chronic pain and depression require careful and sensitive management and a
long-term commitment from the GP to ensure consistency of care and support.

It is often difficult to tackle the topic of depression co-occurring with borderline personality
disorder (BPD). People with BPD have, as part of the core disorder, a perturbation of affect
associated with marked variability of mood. This can be very difficult for the patient to deal
with and can feed self-injurious and other harmful behaviour. Use of mentalisation-based
techniques is gaining support, and psychological treatments such as dialectical behaviour
therapy form the cornerstone of care. Use of medications tends to be secondary, and
prescription needs to be judicious and carefully targeted at particular symptoms. GPs can
play a very important role in helping people with BPD, but should not “go it alone”, instead
ensuring sufficient support for themselves as well as the patient.

Another particularly problematic and well-known form of depression is that which occurs in
the context of bipolar disorder. Firm data on how best to manage bipolar depression is
surprisingly lacking. It is clear that treatments such as unopposed antidepressants can make
matters a lot worse, with the potential for induction of mania and mood cycle acceleration.
However, certain medications (notably, some mood stabilisers and atypical antipsychotics)
can alleviate much of the suffering associated with bipolar depression. Specialist psychiatric
input is often required to achieve the best pharmacological approach. For people with bipolar
disorder, psychological techniques and long-term planning can help prevent relapse. Family
education and support is also an important consideration.
Part C -Text 1: Questions 7-14

7. In the first paragraph, what point does the writer make about the treatment of depression?
A. 75% of depression sufferers visit their GP for treatment.
B .GPs struggle to meet the needs of patients with depression.
C .Treatment for depression takes an average of 11.7 days a month.
D .Most people with depression symptoms never receive help.

8. In the second paragraph, the writer suggests that GPs


A. are in a good position to conduct long term studies on their patients.
B. lack training in the treatment and assessment of depression.
C. should seek help when treatment plans are ineffective.
D. sometimes struggle to create coherent management plans.

9. What do the results of the study described in the third paragraph suggest?
A. GPs prefer the term “treatment resistant depression” to “difficult-to-treat depression”.
B. Patients with “difficult-to-treat depression” sometimes get “stuck” in treatment.
C. The term “difficult-to-treat depression” lacks a precise definition.
D. There is an identifiable sub-group of patients with “difficult-to-treat depression”.

10. Paragraph 4 suggests that


A. prescribing analgesics is unadvisable when treating patients with depression.
B. the co-occurrence of depression with chronic conditions makes it harder to treat.
C. patients with depression may have undiagnosed chronic physical ailments.
D. doctors should be more careful when accepting pain complaints as legitimate.
11. According to paragraph 5, people with BPD have
A. depression occurring as a result of the disorder
B. noticeable mood changes which are central to their disorder.
C. a tendency to have accidents and injure themselves.
D. problems tackling the topic of their depression.

12. In paragraph 5, what does the phrase ‘form the cornerstone’ mean regarding BPD treatment?
A. Psychological therapies are generally the basis of treatment.
B. There is more evidence for using mentalisation than dialectical behaviour therapy.
C. Dialectical behaviour therapy is the optimum treatment for depression..
D. In some unusual cases prescribing medication is the preferred therapy.

13. In paragraph 6, what does the writer suggest about research into bipolar depression
management?
A. There is enough data to establish the best way to manage bipolar depression.
B. Research hasn’t provided the evidence for an ideal management plan yet.
C. A lack of patients with the condition makes it difficult to collect data on its management.
D. Too few studies have investigated the most effective ways to manage this condition.

14. In paragraph 6, what does the writer suggest about the use of medications when treating
bipolar depression?
A. There is evidence for the positive and negative results of different medications.
B. Medications typically make matters worse rather than better.
C. Medication can help prevent long term relapse when combined with family education.
D. Specialist psychiatrists should prescribe medication for bipolar disorder rather than GPs.
Text 2: Are the best hospitals managed by doctors?

Doctors were once viewed as ill-prepared for leadership roles because their selection and
training led them to become “heroic lone healers.” However, the emphasis on patientcentered
care and efficiency in the delivery of clinical outcomes means that physicians are
now being prepared for leadership. The Mayo Clinic is America’s best hospital, according to
the 2016 US News and World Report (USNWR) ranking. Cleveland Clinic comes in second.
The CEOs of both — John Noseworthy and Delos “Toby” Cosgrove — are highly skilled
physicians. In fact, both institutions have been physician-led since their inception around a
century ago. Might there be a general message here?

A study published in 2011 examined CEOs in the top-100 hospitals in USNWR in three key
medical specialties: cancer, digestive disorders, and cardiovascular care. A simple question
was asked: are hospitals ranked more highly when they are led by medically trained doctors
or non-MD professional managers? The analysis showed that hospital quality scores are
approximately 25% higher in physician-run hospitals than in manager-run hospitals. Of
course, this does not prove that doctors make better leaders, though the results are surely
consistent with that claim.

Other studies find a similar correlation. Research by Bloom, Sadun, and Van Reenen
revealed how important good management practices are to hospital performance. However,
they also found that it is the proportion of managers with a clinical degree that had the
largest positive effect; in other words, the separation of clinical and managerial knowledge
inside hospitals was associated with more negative management outcomes. Finally, support
for the idea that physician-leaders are advantaged in healthcare is consistent with
observations from many other sectors. Domain experts – “expert leaders” (like physicians in
hospitals) — have been linked with better organizational performance in settings as diverse
as universities, where scholar-leaders enhance the research output of their organizations, to
basketball teams, where former All-Star players turned coaches are disproportionately
linked to NBA success.
What are the attributes of physician-leaders that might account for this association with
enhanced organizational performance? When asked this question, Dr. Toby Cosgrove, CEO
of Cleveland Clinic, responded without hesitation, “credibility … peer-to-peer credibility.” In
other words, when an outstanding physician heads a major hospital, it signals that they have
“walked the walk”. The Mayo website notes that it is physician-led because, “This helps
ensure a continued focus on our primary value, the needs of the patient come first.” Having
spent their careers looking through a patient-focused lens, physicians moving into executive
positions might be expected to bring a patient-focused strategy.

In a recent study that matched random samples of U.S. and UK employees with employers,
we found that having a boss who is an expert in the core business is associated with high
levels of employee job satisfaction and low intentions of quitting. Similarly, physician-leaders
may know how to raise the job satisfaction of other clinicians, thereby contributing to
enhanced organizational performance. If a manager understands, through their own
experience, what is needed to complete a job to the highest standard, then they may be
more likely to create the right work environment, set appropriate goals and accurately
evaluate others’ contributions.

Finally, we might expect a highly talented physician to know what “good” looks like when
hiring other physicians. Cosgrove suggests that physician-leaders are also more likely to
tolerate innovative ideas like the first coronary artery bypass, performed by René Favaloro at
the Cleveland Clinic in the late ‘60s. Cosgrove believes that the Cleveland Clinic unlocks
talent by giving safe space to people with extraordinary ideas and importantly, that
leadership tolerates appropriate failure, which is a natural part of scientific endeavour and
progress.

The Cleveland Clinic has also been training physicians to lead for many years. For example,
a cohort-based annual course, “Leading in Health Care,” began in the early 1990s and has
invited nominated, high-potential physicians (and more recently nurses and administrators)
to engage in 10 days of offsite training in leadership competencies which fall outside the
domain of traditional medical training. Core to the curriculum is emotional intelligence (with
360-degree feedback and executive coaching), teambuilding, conflict resolution, and
situational leadership. The course culminates in a team-based innovation project presented
to hospital leadership. 61% of the proposed innovation projects have had a positive
institutional impact. Moreover, in ten years of follow-up after the initial course, 48% of the
physician participants have been promoted to leadership positions at Cleveland Clinic.
Part C -Text 2: Questions 15-22

15. In paragraph 1, why does the writer mention the Mayo and Cleveland Clinics?
A. To highlight that they are the two highest ranked hospitals on the USNWR
B. To introduce research into hospital management based in these clinics
C. To provide examples to support the idea that doctors make good leaders
D. To reinforce the idea that doctors should become hospital CEOs

16. What is the writer’s opinion about the findings of the study mentioned in paragraph 2?
A. They show quite clearly that doctors make better hospital managers.
B. They show a loose connection between doctor-leaders and better management.
C. They confirm that the top-100 hospitals on the USNWR ought to be physician-run.
D. They are inconclusive because the data is insufficient.

17. Why does the writer mention the research study in paragraph 3?
A. To contrast the findings with the study mentioned in paragraph 2
B. To provide the opposite point of view to his own position
C. To support his main argument with further evidence
D. To show that other researchers support him

18. In paragraph 3, the phrase ‘disproportionately linked’ suggests


A. all-star coaches have a superior understanding of the game.
B. former star players become comparatively better coaches.
C. teams coached by former all-stars consistently outperform other teams.
D. to be a successful basketball coach you need to have played at a high level.
19. In the fourth paragraph, what does the phrase “walked the walk,” imply about physician
leaders?
A. They have earned credibility through experience.
B. They have ascended the ranks of their workplace.
C. They appropriately incentivise employees.
D. They share the same concerns as other doctors.

20. In paragraph 6, the writer suggests that leaders promote employee satisfaction because
A. they are often cooperative.
B. they tend to give employees positive evaluations.
C. they encourage their employees not to leave their jobs.
D. they understand their employees’ jobs deeply.

21. In the seventh paragraph, why is the first coronary artery bypass operation mentioned?
A. To demonstrate the achievements of the Cleveland clinic
B. To present René Favaloro as an exemplar of a ‘good’ doctor
C. To provide an example of an encouraging medical innovation
D. To show how failure naturally contributes to scientific progress

22. In paragraph 8, what was the outcome of the course “Leading in Health Care”?
A. The Cleveland Clinic promoted almost half of the participants.
B. 61% of innovation projects lead to participants being promoted.
C. Some participants took up leadership roles outside the medical domain.
D. A culmination of more team-based innovations.

END OF READING TEST , THIS BOOKLET WILL BE COLLECTED


Reading test 1 : Answer Key

Part A - Answer key 1 – 7


1. b
2. c
3. a
4. b
5. d
6. a
7. d
Part A - Answer key 8 – 14
8. <30 mcg/L / less than 30 mcg/L / < 30 mcg / L / <30mcg/L
9. excess cow’s milk / excess cow milk / excess cows’ milk / excessive cow’s milk / excessive
cow milk / excessive cows’ milk / excess cow’s milk intake / excess cow milk intake / excess
cows’ milk intake / excessive cow’s milk intake / excessive cow milk intake / excessive cows’
milk intake
10.iron polymaltose
11.consider other cases / evaluate other causes / evaluate for other causes
12. 1 to 2 weeks / one to two weeks / 1-2 weeks / 1 - 2 weeks
13.ferric carboxymaltose
14.oral iron / oral iron supplements
Part A - Answer key 15 – 20
15.low in iron
16.adult doses of iron / adult iron doses
17.endoscopy and colonoscopy / colonoscopy and endoscopy times per week / three times
per week / 3 times a week / three times a week / 3 times
18.weekly / three times weekly
19.in patients with comorbidities
20.tolerate oral iron / tolerate oral iron therapies / tolerate oral iron therapy
Reading test - part B – answer key
1. C
2. C
3. B
4. C
5. A
6. A

Reading test - part C – answer key


Text 1 - Answer key 7 – 14
7. D
8. C
9. C
10. B
11. B
12. A
13. B
14. A

Text 2 - Answer key 15 – 22


15. C
16. A
17. C
18. B
19. A
20. D
21. C
22. A
READING TEST 2

READING SUB-TEST : PART A


 Look at the four texts, A-D, in the separate Text Booklet.
 For each question, 1-20, look through the texts, A-D, to find the relevant information.
 Write your answers on the spaces provided in this Question Paper.
 Answer all the questions within the 15-minute time limit.
 Your answers should be correctly spelt.

TEXT BOOKLET
PART A -QUESTIONS AND ANSWER SHEET

Questions 1-7
For each of the questions, 1-7, decide which text (A, B, C or D) the information comes from.
You may use any letter more than once.

In which text can you find information about...


1. What GPs should say to patients requesting codeine?
2. basic indications of an opioid problem?
3. different medications used for weaning patients off opioids?
4. decisions to make before beginning treatment of dependence?
5. defining features of a use disorder?
6. the development of a common goal for both prescriber and patient?
7. sources of further information on pain management?

Questions 8-14
Answer each of the questions, 8-14, with a word or short phrase from one of the texts.
Each answer may include words, numbers or both. Your answers should be correctly spelled.

8. What will reduced doses of opioids lead to a reduction of?


9. What is the most effective medication for tapering opioid dependence?
10. How long should over the counter codeine analgesics be used for?
11. When should doctors consider referring a patient to a pain expert or clinic?
12. What might a patient give permission to before starting treatment?
13. What might be increasingly neglected as a result of opioid use?
14. How many Buprenorphine patches are needed to taper from codeine tablets?
Questions 15-20

Complete each of the sentences, 15-20, with a word or short phrase from one of the texts.
Each answer may include words, numbers or both. Your answers should be correctly spelled.

The use of Buprenorphine-naxolone requires a (15)........ before treatment.

The use of symptomatic medications for the treatment of opioid dependence has been found to
have (16)........ than tramadol.

Different definitions of opioid dependence share the same (17).........

Once it is decided that opioid taper is a suitable treatment the doctor and patient should create a
(18).........

Recent research indicates that (19)........ can work as well as combination analgesics including
codeine and oxycodone.

The ICD-10 defines a patient as dependent if they have (20)........ key symptoms simultaneously.

END OF PART A, THIS QUESTIONS PAPER WILL BE COLLECTED


READING SUB-TEST : PART B

In this part of the test, there are six short extracts relating to the work of health professionals.
For questions 1-6, choose answer (A, B or C) which you think fits best according to the text.
Write your answers on the separate Answer Sheet.

1. The purpose of the memo about IV solution bags is to remind health practitioners
A. of the procedures to follow when using them.
B. of the hazards associated with faulty ones.
C. why they shouldn’t be reused.

Memo to staff - Intravenous solution bags

IV fluids are administered via a plastic IV solution bag which collapses on itself as it empties.
When a bag is disconnected by removing the giving set spike, air can enter the bag. If it is then
reconnected to an IV line, air can potentially enter the patient’s vein and cause an air
embolism. For this reason, partially used IV bags must never be re-spiked. All IV bags are
designed for single use only - for use in one patient and on one occasion only.
All registered large volume injections, including IV bags, are required to have this warning (or
words to the same effect) clearly displayed on the labelling. In addition to the potential risk of
introducing an air embolus, re-spiking can also result in contamination of the fluid, which may
lead to infection and bacteraemia.
2. What do we learn about the use of TENS machines?
A. Evidence for their efficacy is unconfirmed.
B. They are recommended in certain circumstances.
C. More research is needed on their possible side effects.

Update on TENS machines

The Association of Chartered Physiotherapists in Women’s Health has an expert panel which
could not find any reports suggesting that negative effects are produced when TENS has
been used during pregnancy. However, in clinical practice, TENS is not the first treatment of
choice for women presenting with musculoskeletal pain during pregnancy. The initial
treatment should be aimed at correcting any joint or muscle dysfunction, and a rehabilitation
programme should be devised. However, if pain remains a significant factor, then TENS is
preferable to the use of strong medication that could cross the placental barrier and affect
the foetus. No negative effects have been reported following the use of this modality during
any of the stages of pregnancy. Therefore, TENS is preferable for the relief of pain.
3. If surgical instruments have been used on a patient suspected of having prion disease, they
A. must be routinely destroyed as they cannot be reused.
B. may be used on other patients provided the condition has been ruled out.
C. should be decontaminated in a particular way before use with other patients.

Guidelines: Invasive clinical procedures in patients with suspected prion disease

It is essential that patients suspected of suffering from prion disease are identified prior to any
surgical procedure. Failure to do so may result in exposure of individuals on whom any surgical
equipment is subsequently used. Prions are inherently resistant to commonly used
disinfectants and methods of sterilisation. This means that there is a possibility of transmission
of prion disease to other patients, even after apparently effective methods of decontamination
or sterilisation have been used. For this reason, it may be necessary to destroy instruments
after use on such a patient, or to quarantine the instrument until the diagnosis is either
confirmed, or an alternative diagnosis is established. In any case, the instruments can be used
for the same patient on another occasion if necessary.
4. The email suggests that POCT devices
A. should only be used in certain locations.
B. must be checked regularly by trained staff.
C. can produce results that may be misinterpreted.

To: All Staff


Subject: Management of Point of Care Testing (POCT) Devices

Due to several recent incidents associated with POCT devices, staff are requested to read
the following advice from the manufacturer of the devices.
The risks associated with the use of POCT devices arise from Management of Point of Care
Testing Devices Version 4 January 2014, the inherent characteristics of the devices
themselves and from the interpretation of the results they provide. They can be prone to
user errors arising from unfamiliarity with equipment more usually found in the laboratory.
User training and competence is therefore crucial.
5. It’s permissible to locate a baby’s identification band somewhere other than the ankles when
A. the baby is being moved due to an emergency.
B. the bands may interfere with treatment.
C. the baby is in an incubator.

Identification bands for babies


The identification bands should be located on the baby’s ankles with correct identification
details unless the baby is extremely premature and/or immediate vascular access is required.
If for any reason the bands need to be removed, they should be relocated to the wrists or if this is
not possible, fixed visibly to the inside of the incubator. Any ill-fitting or missing labels should be
replaced at first check. Identity bands must be applied to the baby’s ankles at the earliest
opportunity as condition allows and definitely in the event of fire evacuation or transportation.
6. What is the memo doing?
A. providing an update on the success of new guidelines
B. reminding staff of the need to follow new guidelines
C. announcing the introduction of new guidelines

Memo: Administration of antibiotics

After a thorough analysis and review, our peri-operative services, in conjunction with the
Departments of Surgery and Anaesthesia, decided to change the protocols for the
administration of pre-operative antibiotics and established a series of best practice
guidelines. This has resulted in a significant improvement in the number of patients
receiving antibiotics within the recommended 60 minutes of their incision. A preliminary
review of the total hip and knee replacements performed in May indicates that 88.9% of
patients received their antibiotics on time.
READING SUB-TEST : PART C
In this part of the test, there are two texts about different aspects of healthcare.
For questions 7-22, choose the answer (A, B, C or D) which you think fits best according to the text. W
answers on the separate Answer Sheet.

Text 1: Detecting Carbon Monoxide Poisoning


Carbon Monoxide (CO) poisoning is the single most common source of poisoning injury
treated in US hospital emergency departments. While its presentation is not uncommon, the
diverse symptoms that manifest themselves do not lead most clinicians to consider
carboxyhemoglobinemia when attempting a diagnosis. The symptoms can be mistaken for
those of many other illnesses including food poisoning, influenza, migraine headache, or
substance abuse.

What's more, in an attempt to find the causative agent for the symptoms, many
unnecessary, and sometimes resource-intensive, diagnostics may be ordered, to no avail.
For example, because the symptoms of CO poisoning may mimic an intracranial bleed, the
time needed to obtain a negative result may hold up a proper diagnosis as well as
needlessly increasing healthcare costs. Of even greater concern, however, is that during
such delays patients may find that their symptoms abate and their health improves as the
hidden culprit, CO, is flushed from the blood during the normal ventilation patterns.

Indeed, multiple reports have shown patients being discharged and returned to the very
environment where exposure to CO took place. Take the case of a 67-year-old man who sought
medical help after three days of lightheadedness, vertigo, stabbing chest pain,cough, chills and
headache. He was admitted, evaluated and discharged with a diagnosis of viral syndrome. Ten
days later, he returned to the Emergency Department with vertigo, palpitations and nausea but
was sent home for outpatient follow-up. Four days later, he presented again with diarrhea and
severe chest pain, collapsing to the floor. This time, he was admitted to the Coronary Care Unit
with acute myocardial infarction. Among the results of a routine arterial blood gas analysis there,
it was found that his carboxyhemoglobin (COHb) levels were 15.6%. A COHb level then obtained
on his wife was 18.1%. A rusted furnace was found to be the source.
There are two main types of CO poisoning: acute, which is caused by brief exposure to a
high level of carbon monoxide, and chronic or subacute, which results from long exposure to
a low level of CO. Patients with acute CO poisoning are more likely to present with more
serious symptoms, such as cardiopulmonary problems, confusion, syncope, coma, and
seizure. Chronic poisoning is generally associated with the less severe symptoms. Low-level
exposure can exacerbate angina and chronic obstructive pulmonary disease, and patients
with coronary artery disease are at risk for ischemia and myocardial infarction even at low
levels of CO.

Patients that present with low COHb levels correlate well with mild symptoms of CO
poisoning, as do cases that register levels of 50-70%, which are generally fatal. However,
intermediate levels show little correlation with symptoms or with prognosis. One thing that is
certain about COHb levels is that smokers present with higher levels than do non-smokers.
The COHb level in non-smokers is approximately one to two percent. In patients who
smoke, a baseline level of nearly five percent is considered normal, although it can be as
high as 13 percent. Although COHb concentrations between 11 percent and 30 percent can
produce symptoms, it is important to consider the patient's smoking status.

Regardless of the method of detection used in emergency department care, several other
variables make assessing the severity of the CO poisoning difficult. The length of time since
CO exposure is one such factor. The half-life of CO is four to six hours when the patient is
breathing room air, and 40-60 minutes when the patient is breathing 100 percent oxygen. If
a patient is given oxygen during their transport to the emergency department, it will be
difficult to know when the COHb level hit its highest point. In addition, COHb levels may not
fully correlate with the clinical condition of CO-poisoned patients because the COHb level in
the blood is not an absolute index of compromised oxygen delivery at the tissue level.
Furthermore, levels may not match up to specific signs and symptoms: patients with
moderate levels will not necessarily appear sicker than patients with lower levels.
In hospitals, the most common means of measuring CO exposure has traditionally been
through the use of a laboratory CO-Oximeter. A blood sample, under a physician order, is
drawn from either venous or arterial vessel and injected into the device. Using a method
called spectrophotometric blood gas analysis, this then measures the invasive blood
sample. Because the CO-Oximeter can only yield a single, discrete reading for each aliquot
of blood sampled, the reported value is a non-continuous snapshot of the patient's condition
at the particular moment that the sample was collected. It does, however, represent a step in
the right direction. One study found that in hospitals lacking such a device, the average time
it took to receive results of a blood sample sent to another facility was over fifteen hours,
compared to a ten-minute turnaround in CO-Oximeter equipped hospitals.
Part C – Text 1 : Questions 1 to 8

1. In the first paragraph, what reason for the misdiagnosis of CO poisoning is highlighted?
A. the limited experience physicians have of it
B. the wide variety of symptoms associated with it
C. the relative infrequency with which it is presented
D. the way it is concealed by pre-existing conditions

2. In the second paragraph, the writer stresses the danger of delays in diagnosis leading to

A. the inefficient use of scarce resources.


B. certain symptoms being misinterpreted.
C. a deterioration in the patient's condition.
D. the evidence of poisoning disappearing.

3. The 67-year-old man's CO poisoning was only successfully diagnosed as a result of

A. attending an outpatient clinic.


B. his wife being similarly affected.
C. undergoing tests as an inpatient.
D. his suggesting the probable cause.

4. In the fourth paragraph, confusion is given as a symptom of


A. short-term exposure to high levels of CO.
B. repeated exposure to varying levels of CO.
C. a relatively low overall level of exposure to CO.
D. sustained exposure to CO over an extended period.
5. In the fifth paragraph, what point is made about COHb levels?
A. They fail to detect CO poisoning in habitual smokers.
B. They are a generally reliable indicator of CO poisoning.
C. They correlate very well with extreme levels of CO poisoning.
D. They are most useful in determining intermediate levels of CO poisoning.

6. The phrase 'one such factor' in the sixth paragraph refers to


A. a type of care.
B. a cause of difficulty.
C. a method of detection.
D. a way of making an assessment.

7. One result of administering oxygen to CO poisoned patients in transit is that


A. it becomes harder to ascertain when the COHb level peaked.
B. it may lead to changes in the type of symptoms observed.
C. it could artificially inflate the COHb level in the short term.
D. it affects the ability to assess the effects at tissue level.

8. What reservation about the CO-Oximeter does the writer express?


A. It does not always give an immediate result.
B. Its use needs to be approved by a physician.
C. It requires a skilled analyst to interpret the readings.
D. It does not show variations in the patient's condition

END OF READING TEST ,THIS BOOKLET WILL BE COLLECTED


Reading test 2 : Answer Key

Part A - Answer key 1 – 7


1. B
2. A
3. C
4. D
5. A
6. D
7. B

Part A - Answer key 8 – 14


8. pain intensity
9. Buprenorphine-naloxone / Buprenorphine - naloxone / buprenorphine-naloxone /
buprenorphine - naloxone / Buprenorphine-naloxone (sublingual) / Buprenorphine -
naloxone (sublingual) / buprenorphine-naloxone (sublingual) / buprenorphine - naloxone
(sublingual)
10. one to three days / 1 to 3 days / 1-3 days / 1 - 3 days
11. if pain isn’t managed with nonopioid medications / if pain isn’t managed / if pain isn’t
managed with non-opioid medications
12. urine drug screening
13. alternative interests or pleasures / alternative interests and pleasures / interests or
pleasures / interests and pleasures
14. a single patch / one patch / 1 patch

Part A - Answer key 15 – 20


15. permit
16. poorer outcomes
17. central features / features
18. treatment agreement
19. nonopioid combinations / non-opioid combinations
20. three or more / at least three / 3 or more / at least 3
Reading test - part B – answer key
1. C
2. B
3. B
4. C
5. B
6. A

Reading test - part C – answer key

Text 1 - Answer key 1 – 8


1. B
2. D
3. C
4. A
5. C
6. B
7. A
8. D
READING TEST 3

READING SUB-TEST : PART A


 Look at the four texts, A-D, in the separate Text Booklet.
 For each question, 1-20, look through the texts, A-D, to find the relevant information.
 Write your answers on the spaces provided in this Question Paper.
 Answer all the questions within the 15-minute time limit.
 Your answers should be correctly spelt.

TEXT BOOKLET
PART A -QUESTIONS AND ANSWER SHEET

Questions 1-7
For each of the questions, 1-7, decide which text (A, B, C or D) the information comes from.
You may use any letter more than once.
In which text can you find information about.
1. the risks of feeding a child via a nasogastric tube? 1…………………………………….
2. the length of tube that will be required for a patient? 2…………………………………….
3. when alternative forms of feeding may be more appropriate than nasogastic? 3……………………
4. who to consult over a patient’s liquid food requirements? 4…………………………………….
5. the outward appearance of the tubes? 5…………………………………….
6. knowing when it is safe to go ahead with the use of a tube for feeding? 6……………………………
7. how regularly different kinds of tubes need replacing? 7…………………………………….

Questions 8-14
Answer each of the questions, 8-14, with a word or short phrase from one of the texts.
Each answer may include word, number or both. Your answers should be correctly spelt.

8. What type of tube should you use for patients who need nasogastric feeding for an extended
period? ……………………………………………………………………………………......

9. What should you apply to a feeding tube to make it easier to insert?………………………......

10. What should you use to keep the tube in place temporarily?…………………………......

11. What equipment should you use initially to aspirate a feeding tube?…………………......

12. If initial aspiration of the feeding tube is unsuccessful, how long should you wait before trying
again? ……………………………………………………………………………………......
13. How should you position a patient during a second attempt to obtain aspirate?……………......

14. If aspirate exceeds pH 5.5, where should you take the patient to confirm the position of the
tube? …………………………………
Questions 15 -20

Complete each of the sentences, 16-20, with a word or short phrase from one of the texts.
Each answer may include words, numbers or both. Your answers should be correctly spelt.

15. What device allows for the delivery of feeds via the small bowel?……………………......

16. If a feeding tube isn’t straight when you unwrap it, you should __________________ it.

17. Patients are more likely to experience ______ if they need long-term feeding via a tube.

18. If you need to give the patient a standard liquid feed, the tube to use is ____________ in size.

19. You must take out the feeding tube at once if the patient is coughing badly or is experiencing
__________________

20. If a child is receiving ______ via a feeding tube, you should replace the feed bottle after four
hours.

END OF PART A, THIS QUESTIONS PAPER WILL BE COLLECTED


READING SUB-TEST : PART B
In this part of the test, there are six short extracts relating to the work of health professionals.
For questions 1-6, choose answer (A, B or C) which you think fits best according to the text.
Write your answers on the separate Answer Sheet.

Questions 1-6

1. If vaccines have been stored incorrectly,


A. this should be reported.
B. staff should dispose of them securely.
C. they should be sent back to the supplier.

Manual extract: effective cold chain

The cold chain is the system of transporting and storing vaccines within the
temperature range of +2°C to +S°C from the place of manufacture to the point of
administration. Maintenance of the cold chain is essential for maintaining vaccine
potency and, in turn, vaccine effectiveness.
Purpose-built vaccine refrigerators (PBVR) are the preferred means of storage for
vaccines. Domestic refrigerators are not designed for the special temperature needs of
vaccine storage. Despite best practices, cold chain breaches sometimes occur.
Do not discard or use any vaccines exposed to temperatures below +2°C or above +S°C
Without obtaining further advice. Isolate vaccines and contact the state or territory public
health bodies for advice on the National Immunisation Program vaccines and the
manufacturer for privately purchased vaccines.
2. According to the extract, prior to making a home visit, nurses must
A. record the time they leave the practice.
B. refill their bag with necessary items.
C. communicate their intentions to others.

Nurse home visit guidelines

When the nurse is ready to depart, he/she must advise a minimum of two staff
members that he/she is commencing home visits, with one staff member responsible
for logging the nurse's movements. More than one person must be made aware of the
nurse's movements; failure to do so could result in the breakdown of communication
and increased risk to the nurse and/or practice.
On return to the practice, the nurse will immediately advise staff members of his/her
return. This time will be documented on the patient visit list, and then scanned and
filed by administration staff. The nurse will then attend to any specimens, cold chain
requirements, restocking of the nurse kit and biohazardous waste.
3. What is being described in this section of the guidelines?
A. changes in procedures
B. best practice procedures
C. exceptions to the procedures

Guidelines for dealing with hospital waste

All biological waste must be carefully stored and disposed of safely. Contaminated
materials such as blood bags, dirty dressings and disposable needles are also potentially
hazardous and must be treated accordingly. If biological waste and contaminated
materials are not disposed of properly, staff and members of the community could be
exposed to infectious material and become infected. It is essential for the hospital to have
protocols for dealing with biological waste and contaminated materials. All staff must be
familiar with them and follow them.
The disposal of biohazardous materials is time-consuming and expensive, so it is
important to separate out non-contaminated waste such as paper, packaging and nonsterile
materials. Make separate disposal containers available where waste is created so
that staff can sort the waste as it is being discarded.
4. When is it acceptable for a health professional to pass on confidential information given by a
patient?
A. if non-disclosure could adversely affect those involved
B. if the patient's treatment might otherwise be compromised
C. if the health professional would otherwise be breaking the law

Extract from guidelines: Patient Confidentiality

Where a patient objects to information being shared with other health professionals
involved in their care, you should explain how disclosure would benefit the continuity
and quality of care. If their decision has implications for the proposed treatment, it will be
necessary to inform the patient of this. Ultimately if they refuse, you must respect their
decision, even if it means that for reasons of safety you must limit your treatment options.
You should record their decision within their clinical notes.
It may be in the public interest to disclose information received in confidence without
consent, for example, information about a serious crime. It is important that confidentiality
may only be broken in this way in exceptional circumstances and then only after careful
consideration. This means you can justify your actions and point out the possible harm to
the patient or other interested parties if you hadn't disclosed the information. Theft, fraud
or damage to property would generally not warrant a breach of confidence.
5. The purpose of the email to practitioners about infection control obligations is to
A. act as a reminder of their obligations.
B. respond to a specific query they have raised.
C. announce a change in regulations affecting them.

Email from Dental Board of Australia

Dear Practitioner,
You may be aware of the recent media and public interest in standards of infection
control in dental practice. As regulators of the profession, we are concerned that there
has been doubt among registered dental practitioners about these essential standards.
Registered dental practitioners must comply with the National Board's Guidelines on
infection control. The guidelines list the reference material that you must have access
to and comply with, including the National Health and Medical Research Council's
(NHMRC) Guidelines for the prevention and control of infection in healthcare.
We believe that most dental practitioners consistently comply with these guidelines and
implement appropriate infection control protocols. However, the consequences for
noncompliance with appropriate infection control measures will be significant for you and
also for your patients and the community.
6. The results of the study described in the memo may explain why
A. superior communication skills may protect women from dementia.
B. female dementia sufferers have better verbal skills.
C. mild dementia in women can remain undiagnosed.

Memo to staff: Women and Dementia

Please read this extract from a recent research paper

Women's superior verbal skills could work against them when it comes to recognizing
Alzheimer's disease. A new study looked at more than 1300 men and women divided into
three groups: one group comprised patients with amnestic mild cognitive impairment; the
second group included patients with Alzheimer's dementia; and the final group included
healthy controls. The researchers measured glucose metabolic rates with PET scans.
Participants were then given immediate and delayed verbal recall tests.
Women with either no, mild or moderate problems performed better than men on the verbal
memory tests. There was no difference in those with advanced Alzheimer's.
Because verbal memory scores are used for diagnosing Alzheimer's, some women may be
further along in their disease before they are diagnosed. This suggests the need to have an
increased index of suspicion when evaluating women with memory problems.
READING SUB-TEST : PART C

In this part of the test, there are two texts about different aspects of healthcare.
For questions 7-22, choose the answer (A, B, C or D) which you think fits best according to the text. W
answers on the separate Answer Sheet.

Text 1 : Asbestosis

Asbestos is a naturally occurring mineral that has been linked to human lung disease.
It has been used in a huge number of products due to its high tensile strength, relative
resistance to acid and temperature, and its varying textures and degrees of flexibility. It
does not evaporate, dissolve, burn or undergo significant reactions with other chemicals.
Because of the widespread use of asbestos, its fibres are ubiquitous in the environment.
Building insulation materials manufactured since 1975 should no longer contain asbestos;
however, products made or stockpiled before this time remain in many homes. Indoor air
may become contaminated with fibres released from building materials, especially if they
are damaged or crumbling.

One of the three types of asbestos-related diseases is asbestosis, a process of lung


tissue scarring caused by asbestos fibres. The symptoms of asbestosis usually include
slowly progressing shortness of breath and cough, often 20 to 40 years after exposure.
Breathlessness advances throughout the disease, even without further asbestos inhalation.
This fact is highlighted in the case of a 67-year-old retired plumber. He was on ramipril to
treat his hypertension and developed a persistent dry cough, which his doctor presumed to
be an ACE inhibitor induced cough. The ramipril was changed to losartan. The patient had
never smoked and did not have a history of asthma or COPD. His cough worsened and

he complained of breathlessness on exertion. In view of this history and the fact that he
was a non-smoker, he was referred for a chest X-ray and to the local respiratory physician.
His doctor was surprised to learn that the patient had asbestosis, diagnosed by a highresolution
CT scan. The patient then began legal proceedings to claim compensation as he had worked in a
dockyard 25 years previously, during which time he was exposed to asbestos.
There are two major groups of asbestos fibres, the amphibole and chrysotile fibres. The
amphiboles are much more likely to cause cancer of the lining of the lung (mesothelioma)
and scarring of the lining of the lung (pleural fibrosis). Either group of fibres can cause
disease of the lung, such as asbestosis. The risk of developing asbestos-related lung
cancer varies between fibre types. Studies of groups of patients exposed to chrysotile
fibres show only a moderate increase in risk. On the other hand, exposure to amphibole
fibres or to both types of fibres increases the risk of lung cancer two-fold. Although the
Occupational Safety and Health Administration (OSHA) has a standard for workplace
exposure to asbestos (0.2 fibres/millilitre of air), there is debate over what constitutes
a safe level of exposure. While some believe asbestos-related disease is a 'threshold
phenomenon', which requires a certain level of exposure for disease to occur, others
believe there is no safe level of asbestos.
Depending on their shape and size, asbestos fibres deposit in different areas of the lung.
Fibres less than 3mm easily move into the lung tissue and the lining surrounding the lung.
Long fibres, greater than 5mm cannot be completely broken down by scavenger cells
(macrophages) and become lodged in the lung tissue, causing inflammation. Substances
damaging to the lungs are then released by cells that are responding to the foreign
asbestos material. The persistence of these long fibres in the lung tissue and the resulting
inflammation seem to initiate the process of cancer formation. As inflammation and damage
to tissue around the asbestos fibres continues, the resulting scarring can extend from the
small airways to the larger airways and the tiny air sacs (alveoli) at the end of the airways.

There is no cure for asbestosis. Treatments focus on a patient's ability to breathe.


Medications like bronchodilators, aspirin and antibiotics are often prescribed and such
treatments as oxygen therapy and postural drainage may be recommended. If symptoms
are so severe that medications don't work, surgery may be recommended to remove scar
tissue. Patients with asbestosis, like others with chronic lung disease, are at a higher risk
of serious infections that take advantage of diseased or scarred lung tissue, so prevention
and rapid treatment is vital. Flu and pneumococcal vaccinations are a part of routine care
for these patients. Patients with progressive disease may be given corticosteroids and
cyclophosphamide with limited improvement.
Chrysotile is the only form of asbestos that is currently in production today. Despite their
association with lung cancer, chrysotile products are still used in 60 countries, according
to the industry-sponsored Asbestos Institute. Although the asbestos industry proclaims the
'safety' of chrysotile fibres, which are now imbedded in less friable and 'dusty' products,
little is known about the long term effects of these products because of the long delay
in the development of disease. In spite of their potential health risks, the durability and
cheapness of these products continue to attract commercial applications. Asbestosis
remains a significant clinical problem even after marked reductions in on-the-job exposure
to asbestos. Again, this is due to the long period of time between exposure and the onset
of disease.
Text 1: Questions 7-14

7. The writer suggests that the potential for harm from asbestos is increased by
A. a change in the method of manufacture.
B. the way it reacts with other substances.
C. the fact that it is used so extensively.
D. its presence in recently constructed buildings.

8. The word 'ubiquitous' in paragraph one suggests that asbestos fibres


A. can be found everywhere.
B. may last for a long time.
C. have an unchanging nature
D. are a natural substance.

9. The case study of the 67-year-old man is given to show that


A. smoking is unrelated to a diagnosis of asbestosis.
B. doctors should be able to diagnose asbestosis earlier.
C. the time from exposure to disease may cause delayed diagnosis.
D. patients must provide full employment history details to their doctors.

10. In the third paragraph, the writer highlights the disagreement about
A. the relative safety of the two types of asbestos fibres.
B. the impact of types of fibres on disease development.
C. the results of studies into the levels of risk of fibre types.
D. the degree of contact with asbestos fibres considered harmful.
11. In the fourth paragraph, the writer points out that longer asbestos fibres
A. can travel as far as the alveoli.
B. tend to remain in the pulmonary tissue.
C. release substances causing inflammation.
D. mount a defence against the body's macrophages.

12. What is highlighted as an important component of patient management?


A. the use of corticosteroids
B. infection control
C. early intervention
D. excision of scarred tissue

13. The writer states that products made from chrysotile


A. have restricted application.
B. may pose a future health threat.
C. enjoy approval by the regulatory bodies.
D. are safer than earlier asbestos-containing products.

14. In the final paragraph, the word 'this' refers to


A. the interval from asbestos exposure to disease.
B. the decreased use of asbestos in workplaces.
C. asbestosis as an ongoing medical issue.
D. occupational exposure to asbestos.
Text 2: Medication non-compliance

A US doctor gives his views on a new program

An important component of a patient's history and physical examination is the question of


'medication compliance,' the term used by physicians to designate whether, or not, a patient is
taking his or her medications. Many a hospital chart bears the notorious comment 'Patient has
a history of non-compliance.' Now, under a new experimental program in Philadelphia, USA,
patients are being paid to take their medications. The concept makes sense in theory - failure to
comply is one of the most common reasons that patients are readmitted to hospital shortly after
being discharged.
Compliant patients take their medications because they want to live as long as possible; some
simply do so because they're responsible, conscientious individuals by nature. But the hustle and
bustle of daily life and employment often get in the way of taking medications, especially those
that are timed inconveniently or in frequent doses, even for such well-intentioned patients. For
the elderly and the mentally or physically impaired, US insurance companies will often pay for a
daily visit by a nurse, to ensure a patient gets at least one set of the most vital pills. But other
patients are left to fend for themselves, and it is not uncommon these days for patients to be
taking a considerable number of vital pills daily.

Some patients have not been properly educated about the importance of their medications
in layman's terms. They have told me, for instance, that they don't have high blood pressure
because they were once prescribed a high blood pressure pill - in essence, they view an
antihypertensive as an antibiotic that can be used as short-term treatment for a short-term
problem. Others have told me that they never had a heart attack because they were taken to
the cardiac catheterization lab and 'fixed.' As physicians we are responsible for making sure
patients understand their own medical history and their own medications.
Not uncommonly patients will say, 'I googled it the other day, and there was a long list of side
effects.' But a simple conversation with the patient at this juncture can easily change their
perspective. As with many things in medicine, it's all about risks versus benefits - that's what
we as physicians are trained to analyse. And patients can rest assured that we'll monitor them
closely for side effects and address any that are unpleasant, either by treating them or by trying
a different medication.

But to return to the program in Philadelphia, my firm belief is that if patients don't have strong
enough incentives to take their medications so they can live longer, healthier lives, then the
long-term benefits of providing a financial incentive are likely to be minimal. At the outset, the
rewards may be substantial enough to elicit a response. But one isolated system or patient
study is not an accurate depiction of the real-life scenario: patients will have to be taking these
medications for decades.

Although a simple financial incentives program has its appeal, its complications abound. What's
worse, it seems to be saying to society: as physicians, we tell our patients that not only do we
work to care for them, but we'll now pay them to take better care of themselves. And by the
way, for all you medication-compliant patients out there, you can have the inherent reward of a
longer, healthier life, but we're not going to bother sending you money. This seems like some
sort of implied punishment.

But more generally, what advice can be given to doctors with non-compliant patients? Dr John
Steiner has written a paper on the matter: 'Be compassionate,' he urges doctors. 'Understand
what a complicated balancing act it is for patients.' He's surely right on that score. Doctors
and patients need to work together to figure out what is reasonable and realistic, prioritizing
which measures are most important. For one patient, taking the diabetes pills might be more
crucial than trying to quit smoking. For another, treating depression is more critical than treating
cholesterol. 'Improving compliance is a team sport,' Dr Steiner adds. 'Input from nurses, care
managers, social workers and pharmacists is critical.'
When discussing the complicated nuances of compliance with my students, I give the example
of my grandmother. A thrifty, no-nonsense woman, she routinely sliced all the cholesterol and
heart disease pills her doctor prescribed in half, taking only half the dose. If I questioned this,
she'd wave me off with, 'What do those doctors know, anyway?' Sadly, she died suddenly,
aged 87, most likely of a massive heart attack. Had she taken her medicines at the appropriate
doses, she might have survived it. But then maybe she'd have died a more painful death from
some other ailment. Her biggest fear had always been ending up dependent in a nursing home,
and by luck or design, she was able to avoid that. Perhaps there was some wisdom in her
'noncompliance.'
Text 2: Questions 15-22

15. In the first paragraph, what is the writer's attitude towards the new programme?
A. He doubts that it is correctly named.
B. He appreciates the reasons behind it.
C. He is sceptical about whether it can work.
D. He is more enthusiastic than some other doctors.

16. In the second paragraph, the writer suggests that one category of non-compliance is
A. elderly patients who are given occasional assistance.
B. patients who are over-prescribed with a certain drug.
C. busy working people who mean to be compliant.
D. people who are by nature wary of taking pills.

17. What problem with some patients is described in the third paragraph?
A. They forget which prescribed medication is for which of their conditions.
B. They fail to recognise that some medical conditions require ongoing treatment.
C. They don't understand their treatment even when it's explained in simple terms.
D. They believe that taking some prescribed pills means they don't need to take others.

18. What does the writer say about side effects to medication?
A. Doctors need to have better plans in place if they develop.
B. There is too much misleading information about them online.
C. Fear of them can waste a lot of unnecessary consultation time.
D. Patients need to be informed about the likelihood of them occurring.
19. In the fifth paragraph, what is the writer's reservation about the Philadelphia program?
A. the long-term feasibility of the central idea
B. the size of the financial incentives offered
C. the types of medication that were targeted
D. the particular sample chosen to participate

20. What objection to the program does the writer make in the sixth paragraph?
A. It will be counter-productive.
B. It will place heavy demands on doctors.
C. It sends the wrong message to patients.
D. It is a simplistic idea that falls down on its details.

21. The expression 'on that score' in the seventh paragraph refers to
A. a complex solution to patients' problems.
B. a co-operative attitude amongst medical staff.
C. a realistic assessment of why something happens.
D. a recommended response to the concerns of patients.

22. The writer suggests that his grandmother


A. may ultimately have benefited from her non-compliance.
B. would have appreciated closer medical supervision.
C. might have underestimated how ill she was.
D. should have followed her doctor's advice.

END OF READING TEST , THIS BOOKLET WILL BE COLLECTED


Reading test 3 : Answer Key

Part A - Answer key 1 – 7


1. A
2. B
3. A
4. D
5. B
6. C
7. B

Part A - Answer key 8 – 14


8. Fine bore
9. water based lubricant
10. tape
11. (a) syringe
12. 15-30 minutes/mins OR fifteen - thirty minutes/mins
13. (turn) on (to) left side
14. (to) x ray department OR (to) radiology

Part A - Answer key 15 – 20


15. (a) feeding pump
16. Stretch
17. Gastroesophageal reflux
18. 6/six Fr/French
19. breathlessness
20. (expressed) breast milk
Reading test - Part B: Answer Key
1. A
2. C
3. B
4. A
5. A
6. C

Reading test - part C – answer key


Text 1 - Answer key 7 – 14
7. C
8. A
9. C
10. D
11. B
12. B
13. B
14. C

Text 2 - Answer key 15 – 22


15. B
16. C
17. B
18. D
19. A
20. C
21. D
22. A
READING TEST 4

READING SUB-TEST : PART A


 Look at the four texts, A-D, in the separate Text Booklet.
 For each question, 1-20, look through the texts, A-D, to find the relevant information.
 Write your answers on the spaces provided in this Question Paper.
 Answer all the questions within the 15-minute time limit.
 Your answers should be correctly spelt.

TEXT BOOKLET - SYSTEMIC SCLEROSIS (SSC)

Text A
Systemic sclerosis (SSc)
Systemic sclerosis (SSc) is a disorder of the connective tissue characterized by fibrosis of
the skin, vascular abnormalities, and presence of autoantibodies. It is characterized by
excessive deposition of extracellular matrix. Therefore, there is significant heterogeneity in
organ progression and prognosis. Interstitial lung disease (ILD) is a heterogeneous group of
parenchymal lung disorders that share common radiologic, pathologic, and clinical
manifestations. It is characterized by lung parenchyma damage, accompanied by
inflammation and fibrosis, and fibrosis is often incurable. The fibrosing forms of ILD are
often incurable, and are associated with significant morbidity and mortality. SSc is often
accompanied by ILD. The incidence of SSc-ILD in the relevant literature ranges from 45%
to 90%. A recent European League Against Rheumatism Scleroderma Trials and Research
analysis revealed in a cohort of 3,656 SSc patients that ILD was present in 53% of cases
with diffuse cutaneous SSc and in 35% of cases with limited cutaneous SSc.
Text B

NSIP is the more common subtype of inflammation in ILD. 77% of SSc-ILD is NSIP. A
large number of clinical and pathological studies have confirmed that a high-resolution CT
(HRCT) pattern in patients is correlated with pathologic NSIP and pathologic UIP. NSIP
pattern is associated with better patient outcome than UIP pattern. It includes reticular,
frosted glass shadows, hollow, thickened honeycomb lung nodules, emphysema, bronchial
vascular bundles, bullae, traction bronchiectasis, cobblestone-likeappearance, bronchial
tree, bronchiectasis and so on. The most common manifestation of NSIP is lobular reticular
formation in the pleural and basal regions. UIP is mainly represented by grid or honeycomb
shadow. Different patterns in HRCT can reflect NSIP and UIP. The extent of ILD lesions can be
graded according to HRCT. At present, the commonly used methods for clinical detection of
ILD are HRCT, pulmonary function tests (PFTs) (react as per sensitivity), bronchoalveolar
lavage fluid (BALF), lung biopsy. HRCT has now become the most common and sensitive
imaging method for diagnosing ILD as it offers the most detailed images of the lungs.
Text C
KL-6
Krebs von den Lungen-6 (KL-6) is an important serum marker for ILD. It is a high molecular
weight, mucin-like glycoprotein secreted by type-II alveolar pneumocytes and bronchial
epithelial cells in response to cellular damage and regeneration in patients with ILD. KL-6 is a
mucin-associated glycoprotein, which may be a trigger for TGF-β signaling and fibrosis. The
level of KL-6 as a predictive factor could be used to identify the clinical development of ILD.
Hideaki et al retrospectively analyzed the medical records of 29 patients with SSc-ILD. They
found serum KL-6 correlated positively with diffusing capacity of the lung for carbon
monoxide (DLCO)(% predicted) and disease extent on HRCT, and the changes in serum levels
of KL-6 were significantly related to the changes in forced vital capacity (FVC) in SSc -
associated ILD. Their study suggests KL-6 can be a useful monitoring tool of SSc-ILD activity.

Text D
SP-D
Surfactant, a lipoprotein complex, was originally described for its essential role in reducing
surface tension at the air-liquid interface of the lung. However, it is now recognized as being
a critical component in lung immune host defense. They include SP-B and SP-C and
hydrophilic proteins SP-A and SP-D . SP-D levels are more sensitive than SP-A in detecting ILD
as defined by CT. The sensitivities and specificities for detecting CT-positive ILD in 42
patients with SSc were 33% and 100% for SP-A and 77% and 83% for SP-D, respectively. In a
small but prospective study of 35 patients withSSc-ILD followed over 1-10 years, SP-D levels
were seen to definitely increase over time in 9 out of the 10 patients with worsening ILD, as
defined by changes in symptoms, lung function, and imaging, compared to mild increases in
only 3 out of 25 patients with stable or improving SSc-ILD. Therefore, SP-D is closely related
to SSc-ILD. In addition, Takahashi H et al. found a less-invasive and lung-specific clinical
biomarker. They found the levels of SP-D in sera were significantly higher in the CT-positive
ILD group than in the CT-negative ILD group.
PART A -QUESTIONS AND ANSWER SHEET

Questions 1-7
For each of the questions, 1-7, decide which text (A, B, C or D) the information comes from.
You may use any letter more than once.
In which text can you find information about;
1. One of the major aspects of assessment of the how ILD may develop.________________
2. Detection or assessment of the ILD lesions._________________________________
3. Talk of naturally occurring molecule, gene, or characteristic by which a particular pathological or
physiological process, disease, etc. can be identified.____________
4.Common features of the disease._________________________
5. Production of the protein in response to damage to cell______
6. SSc more commonly occurs with the ILD______.
7. A more common form of interstitial lung disease_____________________________

Questions 8-14
Answer each of the questions, 8-14, with a word or short phrase from one of the texts.
Each answer may include words, numbers or both. Your answers should be correctly spelt.

8. What are known to be more sensitive to change?________________________


9. What is known to be closely related to SSc-ILD?______________________
10. Which is an important serum marker for interstitial lung disease? __________________
11. What are the major characteristics of UIP? _______________
12. What is known to play a major role with respect to lung immune host defense? ___________
13. What can be considered a perfect monitoring tool of SSc-ILD activity? _____________
14. Which lipoprotein complex plays an essential role in reducing surface tension at the air-liquid
interface of the lung? Answer ___________
Questions 15-20

Complete each of the sentences, 15-20, with a word or short phrase from one of the texts. Each
answer may include words, numbers or both. Your answers should be correctly spelt.

Each answer may include words, numbers or both.

15_.__________ is a form of lung disease characterized by progressive scarring of both lungs

16._________ are recorded to have shown a remarkable increase with more deteriorating ILD

conditions.

17. As per HRCT, it can be easy to analyze the extent of the ___________

18. The sensitivities in detecting CT-positive ILD was _____________ SP-D.

19 .The surfactant may include__________ and hydrophilic proteins.

20 .There are four major methods for clinical detection of __________

END OF PART A, THIS QUESTIONS PAPER WILL BE COLLECTED


READING SUB-TEST : PART B
In this part of the test, there are six short extracts relating to the work of health professionals.
For questions 1-6, choose answer (A, B or C) which you think fits best according to the text.
Write your answers on the separate Answer Sheet.

Questions 1-6

1. What does this notice explain?

A. Effects of obesity.
B. Effects of weight loss.
C. How weight loss is correlated to other diseases

1. Obesity - Weight Loss


The relationship between obesity, weight loss, and disease control has been investigated
among patients already diagnosed with chronic diseases. Health benefits of weight loss in chronic
diseases include improvements in cholesterol levels, decreased risk of cardiovascular events, and
improved quality of life. Among patients with hypertension, patients that lost ≥5 kg had improved
blood pressure. Weight loss of ≥5% was associated with improved glycemic control in patients with
diabetes mellitus. In patients with osteoarthritis, weight loss of ≥5% was associated with
improvement in joint symptoms. Weight loss of ≥5% was also associated with low/remission
disease activity in patients with psoriatic arthritis.
2. Hemophagocytic lymphohistiocytosis.
A. Is a rare disease.
B. Is potentially fatal.
C. Occurs in all ages.

Hemophagocytic lymphohistiocytosis
Hemophagocytic lymphohistiocytosis (HLH) is the clinical manifestation of a wide array of
different entities, which include primary or familial hemophagocytic lymphohistiocytosis (FHLH)
and secondary forms and can lead to deteriorating conditions and eventually loss of proper body
mechanism. The hallmark is hemophagocytosis, appearance of activated macrophages that have
engulfed other haematopoietic elements. FHLH, mainly documented in early infancy, is related to
familiar inheritance or genetic causes. Secondary forms (SHLH), also called reactive HLH, are
frequently diagnosed in adults and refer to cases with underlying infection, malignancy or
autoimmune disease. Over the last decade immunosuppression, immunodeficiency,
autoinflammatory diseases and inborn errors of metabolism have been also described as
triggering diseases. Macrophage activation syndrome (MAS) is a secondary form, recently
reported in patients with autoimmune or autoinflammatory diseases, especially patients with
systemic juvenile idiopathic arthritis (SJIA).
3. Granulomatousa Lymphocytic Interstitial Lung Disease;
A. Complication of common variable immunodeficiency disorders.
B. Can effectively be cured by using drugs that can bring in great change in immune response.
C. Appears majorly due to activity of T and B lymphocytes

Granulomatous– Lymphocytic Interstitial Lung Disease (GLILD)

Granulomatous– Lymphocytic Interstitial Lung Disease (GLILD) is an inflammatory pulmonary


complication of common variable immunodeficiency (CVID) with distinctive patterns in the
biopsy; granulomatous disease, lymphocytic interstitial disease, follicular bronchiolitis and areas
of organizing pneumonia. The immunological data of the pathophysiology suggests that it is
initiated by an infiltration of T as well as B lymphocytes and macrophages, that will further lead
the progression of the inflammatory process to fibrosis. Regarding the treatment of this disease
there are many immunomodulatory treatments with few standardized protocols, but recent
studies suggest that the combination of Rituximab and Azathioprine could be effective for
preserving the pulmonary function .
4. What is correct about MCP?
A. MCP-1 may play an important role in the development of pulmonary fibrosis in SSc.
B. The abnormal accumulation of macrophages will lead to the production of MCP-1
C. There is not much evidence to show that ILD is directly interlinked to collagen.CCL2 is known
as monocyte c

hemoattractant protein-1 (MCP-1). MCP-1 is a member of the C-C chemokines. In vivo studies
suggest that MCP-1 recruits monocytes/ macrophages to sites of inflammation in a wide
variety of pathological conditions, including ILD. The plasma level of CCL2 is correlated with
FVC value in SSc. However, there was no correlation between ILD severity and primary fibrotic
genes such as collagen. That might be because skin fibrosis peaks early during the course of SSc
and improves later, while fibrosis in pulmonary tissue continues to progress even at later
stages of disease. A study examined serum levels, spontaneous production by peripheral blood
mononuclear cells (PBMC), and histological distribution in the affected skin, of MCP-1. Elevated
serum levels of MCP-1 significantly correlated with the presence of pulmonary fibrosis. MCP-1
was expressed in mononuclear cells or vascular endothelial cells in 41% (9/22) of SSc patients.
The frequency of infiltrating mononuclear cells and endothelial cells that produced MCP-1 was
significantly higher in SSc patients with early onset than in SSc patients with late onset.
5.What information does this table provide?
A. Shows the clinical decision outcomes of the FRAX 10-year Hip and Major Osteoporotic
fracture risk score thresholds.
B. The thresholds based on the FRAX MOF risk score with DXA.
C. The thresholds based on the FRAX HF risk score with or without DXA.
6. The table clearly shows that;
A. there are high differences in serum CXCL10 concentration between SSA positive and SSA
negative subjects.
B. The RF-positive group had significantly elevated score.
C. The RF-positive group had an average CXCL10 concentration for the RF-negative group.
READING SUB-TEST : PART C

In this part of the test, there are two texts about different aspects of healthcare.
For questions 7-22, choose the answer (A, B, C or D) which you think fits best according to the text.
Write your answers on the separate Answer Sheet.

Text 1: Classification of Seizures


In order to communicate about types of seizures, epilepsy specialists have developed a
classification system for seizures. This system is not based on any fundamental property of
seizures, but rather on committee-generated conventions of terminology.
Classification is as follows: partial seizures and generalized seizures.
Partial seizures are further divided into simple partial seizures with no alteration of consciousness
or memory, or complex partial seizures with alteration of consciousness or memory.
Simple partial seizures can be motor seizures with twitching, abnormal sensations, abnormal
visions, sounds or smells, and distortions of perception. Seizure activity can spread to the
autonomic nervous system, resulting in flushing, tingling, or nausea. If the patient becomes
confused or cannot remember what is happening during the seizure, then the seizure is classified
as a complex partial seizure. Previously, they were called “psychomotor seizures”, “temporal lobe
seizures” or “limbic seizures”. During the complex partial seizure, patients may fumble or perform
automatic fragments of activity such as lip smacking, picking at their clothes, walking around
aimlessly, or saying nonsense phrases over and over again; these purposeless activities are called
automatisms. About 75% of people with complex partial seizures have automatisms; those who do
not simply stop stare and blank out for a few seconds or minutes.

Generalized seizures are divided into absence seizures and tonic-clonic seizures. Absence seizures
were previously called petit mal seizures and usually have onset in childhood, but they can persist
into adulthood. Absence seizures present with staring spells lasting several seconds, sometimes in
conjunction with eyelid fluttering or head nodding. These seizures can be difficult to distinguish
from complex partial seizures that may also result in staring. Usually, absence seizures are briefer
and permit quicker recovery. Generalized tonic-clonic seizures were previously called grand mal
seizures; these seizures start with sudden loss of consciousness and tonic activity (stiffening)
followed by clonic activity (rhythmic jerking) of the limbs. The patient’s eyes will roll up at the
beginning of the seizure and the patient will typically emit a cry, not because of pain, but because

of contraction of the respiratory muscles against a closed throat. Generalized tonicclonic seizures
usually last one to three minutes.

Seizures that begin focally can spread to the entire brain, in which case a tonic-clonic seizure
ensues. It is important, however, to distinguish those that are true grand mal, generalized from the
start, from those that start focally and secondarily generalize. Secondarily generalized seizures
arise from a part of the brain that is focally abnormal. Drugs used to treat primary and secondary
generalized tonic-clonic seizures are different: patients with secondarily generalized tonic-clonic
seizures may be candidates for curative epilepsy surgery, whereas primarily generalized tonic-
clonic seizures are not surgical candidates, because there is no seizure origin site (focus) to remove.
Atonic seizures are epileptic drop attacks. Atonic seizures typically occur in children or adults with
widespread brain injuries. People with atonic seizures suddenly become limp and may fall to the
ground and football helmets are sometimes required to protect against serious injuries. A
myoclonic seizure is a brief un-sustained jerk or series of jerks, less organized than the rhythmic
jerks seen during a generalized tonic-clonic seizure. Other specialized seizure types are occasionally
encountered. Tonic seizures involve stiffening of muscles as the primary seizure manifestation:
arms or legs may extend forward or up into the air; consciousness may or may not be lost. By
definition, the clonic (jerking) phase is absent. Classification can be difficult, because stiffening is a
feature of many complex partial seizures. Tonic seizures, however, are much less common than
complex partial or tonic-clonic seizures. Patients can have more than one seizure type. One seizure
type may progress into another as the electrical activity spreads throughout the brain. A typical
progression is from a simple partial seizure, to a complex partial seizure (when the patient
becomes confused), to a secondarily generalized tonic-clonic seizure (when the electrical activity
has spread throughout the entire brain). The brain has control mechanisms to keep seizures
localized. Antiepileptic medications enhance the ability of the brain to limit the spread of a seizure.
Part C -Text 1: Questions 7-14

7. Motor seizures are;


A. Simple partial seizures
B. Partial seizures
C. Complex seizures
D. Complex partial seizures

8. In which type of seizure does the patient generally not remember what is happening around
them?
A. Simple partial
B. Complex partial
C. Complex partial seizures
D. Partial temporal lobe seizures

9. Which one of these activities are related to automatism?


A. Fumbling
B. lib smacking
C. speaking leisurely
D. None

10. 10 Which seizures last for one to three minutes?


A. Simple partial seizures
B. Tonic-clonic seizures
C. Absence seizures
D. None
11. Which type of seizure occurs in childhood and may persist into adulthood?
A. Grand mal seizures
B. Petit mal seizures
C. Both A and B
D. None

12. Which seizures arise from a focally abnormal part of the brain?
A. Petit mal seizures
B. Grand mal seizures
C. Secondarily generalized seizures
D. Both B and C

13. As per the given information, who may undergo surgery?


A. Patients with grand mal seizures
B. Patients with secondarily generalized seizures
C. Patients with primarily generalized tonic-clonic seizures
D. Both B and C

14. Which one of the following statements correctly describes tonic seizures?
A. Rhythmic jerking
B. Stiffening of muscles
C. Loss of consciousness
D. None
Part C -Text 2: Fascioliasis Infection

Fascioliasis is a parasitic infection typically caused by Fasciola hepatica, which is also known as "the
common liver fluke" or "the sheep liver fluke." A related parasite, Fasciola gigantica, can also infect
people. Fascioliasis is found in all 5 continents, in over 50 countries, especially where sheep or
cattle are reared. People usually become infected by eating raw watercress or other water-based
plants contaminated with immature parasite larvae. The immature larval flukes migrate through
the intestinal wall, the abdominal cavity, and the liver tissue, into the bile ducts, where they
develop into mature adult flukes, which produce eggs. Typically, the pathology is most pronounced
in the bile ducts and liver. A Fasciola infection is both treatable and preventable.

The standard way to be sure a person is infected with Fasciola is by seeing the parasite - this is
usually done by finding Fasciola eggs in stool (fecal) specimens examined under a microscope.
More than one specimen may need to be examined to find the parasite.
Sometimes eggs are found by examining duodenal contents or bile. Infected people don't start
passing eggs until they have been infected for several months; people don't pass eggs during the
acute phase of the infection. Therefore, early on, the infection has to be diagnosed in other ways
than by examining stool. Even during the chronic phase of infection, it can be difficult to find eggs
in stool specimens from people who have light infections.

Fasciola parasites develop into adult flukes in the bile ducts of infected mammals, which pass
immature Fasciola eggs in their feces. The next part of the life cycle occurs in freshwater. After
several weeks, the eggs hatch, producing a parasite form known as the miracidium, which then
infects a snail host. Under optimal conditions, the development process in the snail may be
completed in 5 to 7 weeks; cercariae are then shed in the water around the snail. The cercariae
lose their tails when they encyst as metacercariae (infective larvae) on water plants. In contrast to
cercariae, metacercariae have a hard outer cyst wall and can survive for prolonged periods in wet
environments.
Immature Fasciola eggs are discharged in the biliary ducts and in the stool. Eggs become
embryonated in water; eggs release miracidia, which invade a suitable snail intermediate host,
including the genera Galba, fossaria and pseudosuccinea. In the snail the parasites undergo several
developmental stages: sporocysts, rediae, and cercariae. The cercariae are released from the snail
and encyst as metacercariae on aquatic vegetation or othersurfaces. Mammals acquire the
infection by eating vegetation containing metacercariae whereas humans can become infected by
ingesting metacercariae-containing freshwater plants, especially watercress. After ingestion, the
metacercariaeexcyst in the duodenum and migrate through the intestinal wall, the peritoneal
cavity, and the liver parenchyma into the biliary ducts, where they develop into adult flukes.

No vaccine is available to protect people against Fasciola infection. In some areas of the world
where Fascioliasis is found (endemic), special control programs are in place or are planned. The
types of control measures depend on the setting (such as epidemiologic, ecologic, and cultural
factors). Strict control of the growth and sale of watercress and other edible water plants is
important. Individual people can protect themselves by not eating raw watercress and other water
plants, especially from endemic grazing areas. As always, travelers to areas with poor sanitation
should avoid food and water that might be contaminated (tainted). Vegetables grown in fields that
might have been irrigated with polluted water should be thoroughly cooked, as should viscera from
potentially infected animals

In the early (acute) phase, symptoms can occur as a result of the parasite's migration from the
intestine to and through the liver. Symptoms can include gastrointestinal problems such as nausea,
vomiting, and abdominal pain/tenderness. In addition, fever, rashes and difficulty breathing may
occur. During the chronic phase (after the parasite settles in the bile ducts), the clinical
manifestations may be similar or more discrete, reflecting inflammation and blockage of bile ducts,
which can be intermittent. Inflammation of the liver, gallbladder and pancreas can also occur.
Part C -Text 2: Questions 15 – 22

15. Which one of the following statements is correct?


A. Infection caused by Fasciola spreads faster than any other types of infections
B. Infection by Fasciola is deadly
C. Infection by Fasciola is treatable
D. Infection by Fasciola is very common

16. In which phase is it not easy to find the eggs in the stool?
A. Chronic phase
B. Infective phase
C. Acute phase
D. A and B

17. Paragraph 3 talks about which of the following;


A. Biology of Fasciola hepatica
B. Time period in a snail
C. Initial stages of the development of the parasite
D. Complete life cycle

18. Which of these forms survives for a longer period of time?


A. Cercariae
B. Metacercariae
C. Miracidia
D. Fasciola eggs

19. Which of these topics does paragraph 4 talk about?


A. How infection occurs in humans
B. How animals get infected
C. How plants get infected
D. All of the above
20. Excystation occurs in which of these?
A. AIntestinal wall
B. Duodenum
C. Peritoneal cavity
D. Liver

21. Paragraph 5 talks about which of these topics?


A. Prevention and control
B. Availability of the treatment for the infection
C. Drugs to be used
D. A and C

22 .Which of these topics does paragraph 6 talk about?


A. How infection is controlled
B. How infection spreads through bile ducts and liver
C. How infection is prevented from spreading to different parts
D. Possibility of infection spreading to other parts of the body

END OF READING TEST, THIS BOOKLET WILL BE COLLECTED


Reading test 4 : Answer Key

Part A - Answer key 1 – 7


1. C
2.B
3. D
4. A
5. C
6. A
7. B

Part A - Answer key 8 – 14


8. HRCT and PFTs
9. Sp-d
10. KL-6
11. grid or honeycomb shadow
12. Surfactant
13. KL-6
14. Surfactant

Part A - Answer key 15 – 20


15. Usual Interstitial Pneumonia (uip)
16. Sp-d Levels
17. Ild Lesions
18. 83%
19. Sp-b And Sp-c
20. Ild
Reading test - Part B: Answer Key

1. 1.How weight loss is correlated to other diseases.


2. Is potentially fatal.
3. 3.Can effectively be cured by using drugs that can bring in great change in immune response.
4. MCP-1 may play an important role in the development of pulmonary fibrosis in SSc
5. Shows the clinical decision outcomes of the FRAX 10-year Hip and Major Osteoporotic fracture
risk score thresholds.
6. 6.The RF-positive group had an average CXCL10 concentration for the RF-negative group.

Reading test - part C – answer key

Text 1 - Answer key 7 – 14


7.Simple partial seizures
8 Complex partial seizures
9.Lip smacking
10.Tonic-clonic seizures
11.Petit mal seizures
12. Secondarily generalized seizures
13. Patients with secondarily generalized seizures
14.Loss of consciousness

Text 2 - Answer key 15 – 22


15. Infection by Fasciola is treatable
16.Chronic phase
17.Initial stages of the development of the parasite
18.Metacercariae
19. How animals get infected
20 Duodenum
21.Prevention and control
22. Possibility of infection spreading to other parts of the body
READING TEST 5

READING SUB-TEST : PART A


 Look at the four texts, A-D, in the separate Text Booklet.
 For each question, 1-20, look through the texts, A-D, to find the relevant information.
 Write your answers on the spaces provided in this Question Paper.
 Answer all the questions within the 15-minute time limit.
 Your answers should be correctly spelt.

TEXT BOOKLET- POLYCYTHEMIA

Text A
The mechanism of polycythemia in primary familial and congenital polycythemia (PFCP) is due to
the truncated EpoR (genetic mutation) in which there is no inhibition of signalling pathways. In all
conditions of hypoxia HIF-1 is responsible for the polycythemia. Some patients with chronic lung
disease or congenital cyanotic heart disease do not develop polycythemia in spite of hypoxia, the
mechanism of which is not very clear. Polycythemia in smokers is due to increased blood carbon
monoxide (CO). CO displaces one molecule of O2 from hemoglobin and converts it to
carboxyhemoglobin (COHb). COHb has 200 times greater affinity than oxygen. This results in not
only occupation of one of the heme groups of haemoglobin but also increase in the oxygen affinity
of the remaining heme group resulting in tissue hypoxia. Polycythemia accompanying kidney and
liver diseases and neoplastic disorders, is usually associated with increased EPO production. In
tumours EPO production is shown to be autonomous of hypoxic stimulation .production is shown
to be autonomous of hypoxic stimuli.
Text B

The molecular basis of post-transplantation erythrocytosis (PTE) remains unclear. It is found in 5-


10% of renal allograft recipients developing within 8-24 months following a successful renal
transplantation. It resolves spontaneously within 2 years in about 25% of patients. In congenital
secondary polycythemia, mutations in the haemoglobin can lead to increased oxygen affinity
leading to decreased oxygen delivery and compensatory polycythemia. A rare mechanism in this
group is 2, 3 BPG deficiency. This compound is synthesised in red blood cell and binds to
haemoglobin reducing its affinity for oxygen. Its absence leads to increased affinity of haemoglobin
for oxygen resulting in a lifelong hypoxic stimulus and erythrocytosis. The fetal haemoglobin has
high oxygen affinity and many of the neonates may have markedly elevated hematocrits.
Polycythemia vera rises from the transformation of a single hematopoietic stem cell with a
selective growth advantage that gradually becomes the predominant myeloid progenitor. Recently
a somatic mutation is detected in a gene on chromosome 9p in a majority of
polycythemia vera patients. This gene encodes for tyrosine kinase JAK. This somatic mutation
transforms this kinase into a constitutively active form and seems to be responsible for the
uncontrolled proliferation of the erythroid cells.
Text C

Clinical Approach
Symptoms of polycythemia are very nonspecific like a headache, weakness, pruritus, dizziness,
sweating and visual disturbances. Some of the patients are seen initially with complications of
polycythemia like thrombosis (cerebral,peripheral) and haemorrhage. Thrombosis may occur at
unusual sites like hepatic vein. Polycythemia may be diagnosed when Budd Chiari syndrome is
being investigated. Hematocrit values above 51% in males and over 48% in females requires further
evaluation.
Diagnostic criteria laid down by PVSG and WHO require demonstration of an elevated red cell mass
as a must. This is practically not possible in most centres. So, WHO has revised the criteria (2008)
for the diagnosis of PV6 .
Accordingly, there are 2 major and 3 minor criteria.
Major criteria
1. Hemoglobin level above 18.5g/dl for men and 16.5g/dl for females OR Hemoglobin or
hematocrit > 99th percentile of reference range for age, sex, or altitude of residence OR elevated
red cell mass >25% above mean normal predicted value.
2. Presence of JAK2 gene mutation (V617F) or other functionally similar.
Minor criteria
1. Bone marrow showing hypercellularity for age and trilineage growth (panmyelosis)
2. Subnormal Epo level
3. EEC (endogenous erythroid colonies)
Diagnostic combinations - Major criteria + one minor criterion and first major criterion + 2 minor
criteria
Text D
Recommendations;
Low dose aspirin 75- 150 mg is recommended in all PV patients without history of major bleeding
or gastric intolerance, based on the results of the ECLAP study.
Patients with PV should be properly hydrated when they develop gastrointestinal disorders. The
spent phase occurs after about 15-20 years, when the phlebotomy requirement decreases and the
patient develops anaemia. The marrow fibrosis increases and spleen becomes greatly enlarged.
The treatment during this phase is purely symptomatic including blood transfusions. Other
treatment modalities tried are splenectomy, thalidomide and marrow transplantation in younger
patients. In the future we may have new JAK2 targeted inhibitors to treat PV. Some patients may
get transformed into acute leukaemia Any form of treatment during this phase is not at all
satisfactory. Currently, management of PV depends on the risk stratification
Age >60yrs or history Cardiovascular risk
Risk category
of thrombosis factors
Low No No
Intermediate No Yes
High Yes
Phlebotomy is the cornerstone of low-risk patients aimed at reaching and maintaining a target
hematocrit of 45% in males and 42% in females. Low dose aspirin may be added to the treatment.
High-risk patients should receive myelosuppressive treatment in addition to phlebotomy. The drug
of choice is hydroxyurea.PV may infrequently occur during childbearing years. There is an
increased incidence of abortion in about 30% of cases. Pre-eclampsia is also common. It is very
interesting that some of the women may even reduce their hematocrit. Their phlebotomy
requirement is also found to be decreased. The possible explanations are the erythropoietic
suppressive effect of the high estrogen levels, expansion of the plasma volume and nutritional
deficiencies. If needed, the patient should be treated with phlebotomy, low dose aspirin or
interferon. After delivery the blood count will drift back to the original polycythemic level.
PART A -QUESTIONS AND ANSWER SHEET
Questions 1-7
For each of the questions, 1-7, decide which text (A, B, C or D) the information comes from.
You may use any letter more than once.

In which text can you find information about;


1. Severe itching of the skin, as a symptom of various ailments. Answer __________
2. Maintaining the ratio of the volume of red blood cells to the total volume of blood around five
percent and 8 percent lesser than 50% in males and females. Answer _________________
3 .Take place during pregnancy. Answer _____________
4 .A condition arising due to shortening of the genes. Answer______________
5 .The presence of an abnormal excess of cells. Answer _________________
6 .Bluish cast to the skin and mucous membrane. Answer ________________
7. An enzyme that can transfer a phosphate group from ATP to a protein in a cell. Answer ______

Questions 8-14
Answer each of the questions, 8-14, with a word or short phrase from one of the texts. Each
answer may include words, numbers or both. Your answers should be correctly spelt.

8 .What is the term which refers to the use of the drug in the treatment of certain cancers?__
9 .What is found in a gene on chromosome 9p in patients? Answer ___________________
10 .When does a patient develop anaemia? Answer ______________________
11. What is the condition in which bone marrow activity is decreased, resulting in fewer red blood
cells, white blood cells, and platelets? Answer __________________
12. What leads to the formation by peripheral blood mononuclear cells from patients with
polycythemia vera? Answer _____________
13 .What is the hormone produced by the kidney that promotes the formation of red blood cells by
the bone marrow? Answer ______________
14. What is the recommended for treatment? Answer __________________________
Questions 15-20

Complete each of the sentences, 15-20, with a word or short phrase from one of the texts. Each
answer may include words, numbers or both. Your answers should be correctly spelt.

15. _________ is used with other medications or radiation therapy to treat some blood disorders.

16 . ____________mutations were found to have a high correlation with abnormal heart defects.

17 .________ cells can undergo rapid proliferation before differentiating into maturation stages.

18._____________________ are seen in the great majority of cases of polycythaemia.

19 .Polycythemia vera shows stable growth, majorly come into being from a single _____stem cell.

20. In various cancerous conditions, production of______ is recorded to be independent of hypoxic

stimuli.

END OF PART A, THIS QUESTIONS PAPER WILL BE COLLECTED


READING SUB-TEST : PART B
In this part of the test, there are six short extracts relating to the work of health professionals .
For questions 1-6, choose the answer (A, B or C) which you think fits best according to the text. Write
your answers on the separate Answer Sheet
Questions 1-6

1. As per the following notice, what is correct?


A. Dyslipidemia in children is common.
B. Young committee mostly gets affected by the disease.
C. Statistics showing the curtailment ratio of the affected people.

The American Academy of Pediatrics recommends screening for dyslipidemia in children and
adolescents who have a family history of dyslipidemia or premature CVD, those whose family
history is unknown, and those youths with risk factors for CVD, such as being overweight or obese,
having hypertension or DM, or being a smoker 1 In 2011, the NHBLI Expert Panel recommended
universal dyslipidemia screening for all children between 9 and 11 years of age and again
between 17 and 21 years of age 23.Analysis of data from NHANES 1999 to 2006 showed that the
overall prevalence of abnormal lipid levels among youths 12 to 19 years of age was 20 3%. From
2005 to 2010, among adults with high LDL- C, age adjusted control of LDL-C increased from 22 3%
to 29 5% 25 The prevalence of LDL-C control was lowest among people who reported receiving
medical care less than twice in the previous year (11 7%), being uninsured (13 5%), being Mexican
American (20 3%), or having income below the poverty level (21 9%) 2.
2. Notice on debatable concepts gives information about;
A. Concepts which may have direct impact on splenectomy.
B. Conditions which might arise after splenectomy.
C. Situations requiring great effort.

DEBATABLE Concepts of Laparoscopic splenectomy


 Malignant hematologic diseases
 Huge splenomegaly (> 25 cm)
 Malignant splenic tumor
 Pericapsular inflammation
 Large lymph nodes at the splenic hilum
 PHT and cirrhosis
 Difficulties
• Technical challenge
• Splenic mobilization
• Safe access to the splenic hilum
3.The manual gives information about
A. Rescue Protocol
B. Emergency Care Plan
C. Adjustable Properties of Power heart AED G3

The AED protocol is consistent with the guidelines recommended by the American
Heart Association (Guidelines 2005 for Cardiopulmonary Resuscitation and Emergency Cardiac Care
American Heart Association; Circulation vol 112, Issue 24 Suppl. Dec. 13, 2005) and the
International Liaison Committee on Resuscitation (ILCOR)). Upon detecting a shockable cardiac
rhythm, the AED advises the operator to press the SHOCK button (9390E only) to deliver a
defibrillation shock followed by performing 2 minutes of CPR. For the Powerheart AED G3
Automatic, upon detecting a shockable rhythm, the AED will automatically deliver defibrillation
shocks followed by performing 2 minutes of CPR.

Note: In alignment with the 2005 Guidelines, the default setting for the CPR time has been set to
allow for 5 cycles of 30 compressions and 2 breaths.
Increasing or decreasing the CPR time setting may increase or decrease the number of actual cycles
allowed during the CPR timeout period.
4. As per the given notice, GM levels of triglycerides is;
A. Higher among people who are 20+ years.
B. Common among men.
C. Common among women.
The geometric mean level of triglycerides for American adults ≥20 years of age was 103 5 mg/ dL in
NHANES 2011 to 2014. Approximately 24 2% of adults had high triglyceride levels (≥150 mg/dL) in
NHANES 2011 to 2014. Among males, the age adjusted geometric mean triglyceride level was 111 6
mg/dL. in NHANES 2011 to 2014, with the following racial/ethnic breakdown: — 113 2 mg/dL for
non-Hispanic white males — 86 7 mg/dL for non-Hispanic black males — 124 1 mg/dL for Hispanic
males — 115 3 mg/dL for non-Hispanic Asian males

5. Pick the one that is incorrect


A. GGTP (cut-off): GGTP <100; 100 % ¤ GGTP % ¤ 200; GGTP >200; for scores 1, 2, 3 respectively.
B. Bilirubin (cut-off): Bil <1.5; 1.5% ¤ Bil % ¤ 2.5; Bil >2.5; for scores 1, 2,3 respectively.
C. Albumin (cut-off): Alb >3.5; 2.5 % ¤ Alb< 3.5; Alb < 2.5; for scores 1, 2, 3 respectively.
Relationship of a liver index and its parameters to HCC aggressiveness
β Se(β) p. 95% C.I
(A)
Liver Index score 0.2462 0.0247 <0.001 0.1978 to 0.2945
(B)
GGTP (IU/ml) 0.0013 0.0003 <0.001 0.0007 to 0.0020
Total Bilirubin (mg/dl) 0.0585 0.0140 <0.001 0.0311 to 0.0859
Albumin (g/dl) -0.3821 0.0554 <0.001 -0.4908 to - 0.2733
Platelets (x109/L) 0.0031 0.0005 <0.001 0.0021 to 0.0041
6. The given notice talks about
A. Functioning of the ultrasound.
B. Detection of fetus.
C. Examination of the fetus.

Ultrasound is done during pregnancy to track the development of the fetus in the mother’s womb.
It is not only helpful in tracking down the development but also helps to find out any fetal
anomalies. Ultrasound reveals the heartbeat of the fetus, the radius of the head, the length of the
hands and feet and also his/her height and weight. There are various kinds of ultrasound which can
be done during pregnancy namely Transvaginal Ultrasound, 3-D Ultrasound, 4-D Ultrasound and
Fetal Echocardiography. While the Sonography reports in the first trimester provides
information about the fetal heartbeat, it also examines the placenta, uterus, ovaries, cervix, checks
for multiple pregnancies, the sonography done in the second and third trimester reveals much
important criteria like placental abruption, placenta previa, characteristics of Down’s syndrome if
there are any possibilities. The ultrasound in this stage also determines whether the fetus is
carrying any form of congenital disease whether hereditary or non-hereditary or not.
READING SUB-TEST : PART C ]

In this part of the test, there are two texts about different aspects of healthcare.
For questions 7-22, choose the answer (A, B, C or D) which you think fits best according to the text.
Write your answers on the separate Answer Sheet

Text 1: Anaplasmosis
Anaplasmosis is a tick-borne disease caused by the bacterium anaplasmaphagocytophilum. It was
previously known as human granulocytic ehrlichiosis (HGE) and has more recently been called
human granulocytic anaplasmosis (HGA). Anaplasmosis is transmitted to humans by tick bites,
primarily from the black-legged tick (Ixodesscapularis) and the western blacklegged tick
(Ixodespacificus). Of the four distinct phases in the tick life-cycle (egg,larva, nymph, adult), nymphal
and adult ticks are most frequently associated with transmission of anaplasmosis to humans. Typical
symptoms include fever, headache, chills, and muscle aches. Usually, these symptoms which occur
within 1-2 weeks of a tick bite can’t be known and in many cases can’t even be averted.
Anaplasmosis, which often can’t be thwarted, is initially diagnosed based on
symptoms and clinical presentation, and later confirmed by the use of specialized laboratory tests.
The first line treatment for adults and children of all ages is doxycycline. Anaplasmosis and other
tick-borne diseases can be obviated.

Anaplasmosis is caused by the bacterium anaplasmaphagocytophilum. This organism used to be


known by other names, including Ehrlichiaequi and Ehrlichiaphagocytophilum, and the disease
caused by this organism has been previously described as human granulocytic ehrlichiosis (HGE).
However, a taxonomic change in 2001 identified that this organism belonged to the genus
anaplasma, and resulted in a change in the name of the disease to anaplasmosis. Anaplasmosis was
first recognized as a disease of humans in the United States in the mid-1990’s, but did not become a
reportable disease until 1999. The number of anaplasmosis cases reported has increased steadily
since the disease became reportable, from 348 cases in 2000, to 1761 cases in 2010. The incidence
(the number of cases for every million persons) of anaplasmosis has also increased, from 1.4 cases
per million persons in 2000 to 6.1 cases per million persons in
2010. The case fatality rate (i.e. the proportion of anaplasmosis patients that reportedly died as a
result of infection) has remained low, at less than 1%.

The bacterium anaplasmaphagocytophilum is transmitted to humans by the bite of an infected tick.


The black-legged tick (Ixodesscapularis) is the vector of A. phagocytophilum in the northeast and
upper mid western United States. The western black-legged tick (Ixodespacificus) is the primary
vector in Northern California. The first symptoms of anaplasmosis typically begin within 1-2 weeks
after the bite of an infected tick. A tick bite is usually painless, and some patients who develop
anaplasmosis do not remember being bitten. It can be a serious illness that can be fatal if not
treated correctly, even in previously healthy people.

The severity of anaplasmosis may depend in part on the immune status of the patient. Persons with
compromised immunity caused by immunosuppressive therapies (e.g., corticosteroids, cancer
chemotherapy, or long-term immunosuppressive therapy following an organ transplant), HIV
infection, or splenectomy appear to develop a more severe disease, and case-fatality rates for
these individuals are characteristically higher than case-fatality rates reported for the general
population.

Because A. phagocytophilum infects the white blood cells and circulates in the bloodstream, this
pathogen may pose a risk to be transmitted through blood transfusions.
Anaplasmaphagocytophilum has been shown to survive for more than a week in refrigerated blood.
Several cases of anaplasmosis have been reported associated with the transfusion of packed red
blood cells donated from asymptomatic or acutely infected donors. Patients who develop
anaplasmosis within a month of receiving a blood transfusion or solid organ transplant should be
reported to state health officials for prompt investigation.

There are several aspects of anaplasmosis that make it challenging for healthcare providers to
diagnose and treat. The symptoms vary from patient to patient and can be difficult to distinguish
from other diseases. Treatment is more likely to be effective if started early in the course of the
disease. Diagnostic tests based on the detection of antibodies will frequently appear negative in the
first 7-10 days of illness. For this reason, healthcare providers must use their judgment to treat
patients based on clinical suspicion alone. Healthcare providers may find important information in
the patient’s history and physical examination that may aid clinical diagnosis. Information such as
recent tick bites, exposure to areas where ticks are likely to be found, or history of recent travel to
areas where anaplasmosis is endemic can be helpful in making the diagnosis.
Part C -Text 1: Questions 7-14
7. According to paragraph 1, what is not anaplasmosis?
A. A bacterial disease
B. A disease that is transmitted by tick bites
C. A disease in which people suffer from muscle pain
D. A disease that can™t be prevented

8. Which word in paragraph 1 may mean removing difficulty?


A. Averted
B. Thwarted
C. Obviated
D. None of the above.

9. Paragraph 2 deals more with the;


A .Annual cases of anaplasmosis in the US.
B. cause of the anaplasmosis.
C. prevlance of the anaplasmosis.
D. None of the above.

10. What is not true about anaplasmosis, according to paragraph 2?


A. The old name of anaplasmosis was HGE.
B. The causal agent of anaplasmosis was recorded to be ehrlichiaequi
C. HGE was renamed as anaplasmosis in the year 2001
D. Cases of anaplasmosis became known only after 2000
11. What is not true about ticks, according to paragraph 3?
A. A tick bite is painless
B. A tick bite carries bacterium anaplasmaphagocytophilum
C. Ticks are present throughout the US
D. Black-legged ticks are present across California

12. What is the central idea of paragraph 4?


A. Immune-compromised individuals
B. The effects of anaplasmosis
C. Fatality rate and anaplasmosis
D. None of the above

13. What do we find in paragraph 5?


A. Blood transfusion risks
B. Organ transplant risks
C. Blood transfusion and organ transplant risks associated with anaplasma species
D. Information about anaplasmaphagocytophilum

14. According to paragraph 6, what is not true about anaplasmosis?


A. It is difficult to diagnose and treat anaplasmosis
B. Why tests for anaplasmosis appear negative
C. Different patients may show different symptoms
D. A patients medical history is often taken into consideration
Text 2: Candidiasis
Candidiasis is a fungal infection caused by yeasts that belong to the genus Candida. There are over
20 species of Candida yeasts that can cause infection in humans, the most common of which is
Candida albicans. Candida yeasts normally reside in the intestinal tract and can be found on
mucous membranes and skin without causing infection; however, overgrowth of these organisms
can cause symptoms to develop. Symptoms of candidiasis vary depending on the area of the body
that is infected.

Candidiasis that develops in the mouth or throat is called “thrush” or oropharyngeal candidiasis.
Candidiasis in the vagina is commonly referred to as a “yeast infection.” Invasive candidiasis occurs
when Candida species enter the bloodstream and spread throughout the body. The infection is not
very common in the general population. It is estimated that between 5% and 7% of babies less than
one month old will develop oral candidiasis. The prevalence of oral candidiasis
among AIDS patients, (particularly women rather than men, although not yet an established fact) is
estimated to be between 9% and 31%, and studies have documented clinical evidence of oral
candidiasis in nearly 20% of cancer patients.
Candidiasis of the mouth and throat, also known as “thrush" or oropharyngeal candidiasis, is a
fungal infection that occurs when there is an overgrowth of a yeast called Candida. Candida yeasts
normally live on the skin or mucous membranes in small amounts. However, if the environment
inside the mouth or throat becomes imbalanced, the yeasts can multiply and cause symptoms.
Candida overgrowth can also develop in the oesophagus, and this is called Candida
esophagitis, or esophageal candidiasis.

Candida infections of the mouth and throat can manifest in a variety of ways. The most common
symptom of oral thrush is white patches or plaques on the tongue and other oral mucous
membranes. Other symptoms include redness or soreness in the affected areas; difficulty
swallowing; cracking at the corners of the mouth (angular cheilitis) etc.
Candida infections of the mouth and throat are infrequent among adults who are otherwise
healthy. Oral thrush presents itself most recurrently among babies less than one month old, the
elderly, and groups of people with weakened immune systems. Other factors associated with oral
and esophageal candidiasis include HIV/AIDS, cancer treatments, organ transplantation, diabetes
etc. Good oral hygiene practices may sporadically help to prevent oral thrush in people with
weakened immune systems. Some studies have shown that chlorhexidine (CHX) mouthwash can
help to prevent oral candidiasis in people undergoing cancer treatment. People who use inhaled
corticosteroids may be able to reduce the risk of developing thrush by washing out the mouth with
water or mouthwash after using an inhaler.

Candida infections of the mouth and throat must be treated with prescription antifungal
medication. The type and duration of treatment depends on the severity of the infection and
patient-specific factors such as age and immune status. Untreated infections can lead to a more
serious form of invasive candidiasis. Oral candidiasis usually responds to topical treatments such as
clotrimazole troches and nystatin suspension (nystatin “swish and swallow”). Systemic antifungal
medication such as fluconazole or itraconazole may be necessary for oropharyngeal infections that
do not respond to these treatments. Candida esophagitis is typically treated with oral or
intravenous fluconazole or oral itraconazole. For severe or azole-resistant esophageal candidiasis,
treatment with amphotericin B may be necessary. For healthcare providers: the most up-to-date
clinical practice guidelines for the treatment of oropharyngeal / esophageal candidiasis are
available at the Infectious Diseases Society of America.
Part C -Text 2: Questions 15-22
15. According to paragraph 1, the abode for candida yeasts is;
A. Intestinal tract
B. Mucous membrane
C. Skin
D. All of the above

16. According to paragraph 2, oral candidiasis is;


A. Prevalent among children.
B Prevalent among babies.
C. Common among people suffering from AIDS.
D. Common among female patients with AIDS.

17. According to paragraph 2, candida yeasts;


A. grow on the mucous membrane.
B. grow under the skin of the mouth.
C. grow only inside the throat area.
D. grow expeditiously in the esophagus.

18. The word manifestation in paragraph 3 may mean;


A. To describe
B. To multiply
C. To show
D. To disguise
19. What does paragraph 4 indicate?
A. Who gets affected with oral candidiasis?
B. How to prevent oral candidiasis.
C. Risk and prevention.
D. None of the above.

20. According to paragraph 4, candida infections;


A. Occur among people affected with AIDS, diabetes etc.
B. Occur less common among people with a low immunity rate.
C. Can be prevented using off-the-shelf medical products such as CHX mouthwash.

21. The word sporadic in paragraph 4 means;


A. Always
B. Not regular
C. Intermittently
D. Every so often

22. What is the central idea presented in paragraph 5?


A. The treatment process of candidiasis.
B. Treatment and outcome.
C. Medication for candidiasis.
D None of the above.

END OF READING TEST, THIS BOOKLET WILL BE COLLECTED


Reading test 5 : Answer Key
Part A - Answer key 1 – 7
1 .C
2.D
3.D
4 .A
5 .C
6 .A
7 .B

Part A - Answer key 8 – 14


8 .thalidomide
9 .somatic mutation
10 .phlebotomy requirement decreases
11. Myelosuppression
12 .Endogenous erythroid colony
13 .Erythropoietin (EPO)14 .phlebotomy

Part A - Answer key 15 – 20


15.Hydroxyurea (hydrea)
16. Jak - 2 Kinase
17. Myeloid Progenitor
18.Endogenous Erythroid Colonies (eec)
19 .Hematopoietic
20. Erythropoietin
Reading part B– answer key

1 Statistics showing the curtailment ratio of the affected people.


2.Conditions which might arise after splenectomy.
3. Rescue Protocol
4. Higher among people who are 20+ years.
5 .Albumin (cut-off): Alb>3.5; 2.5 ≤ Alb < 3.5; Alb<2.5; for scores 1, 2, 3 respectively.
6 .Functioning of the ultrasound.

Reading test - part C – answer key

Text 1 - Answer key 7 – 14


7. disease that can be prevented
8. Obviated
9. Annual cases of anaplasmosis in the US.
10. Cases of anaplasmosis became known only after 2000
11. Ticks are present throughout the US
12. Immune-compromised individuals
13. Blood transfusion and organ transplant risks associated with anaplasma species
14. Why tests for anaplasmosis appear negative

Text 2 - Answer key 15 – 22


15. All of the above
16. Common among people suffering from AIDS.
17. grow expeditiously in the esophagus.
18. To show
19. Risk and prevention.
20. Less prevalent among adults.
21. Not regular
22. Treatment and outcome.
READING TEST 6

READING SUB-TEST : PART A


 Look at the four texts, A-D, in the separate Text Booklet.
 For each question, 1-20, look through the texts, A-D, to find the relevant information.
 Write your answers on the spaces provided in this Question Paper.
 Answer all the questions within the 15-minute time limit.
 Your answers should be correctly spelt.

TEXT BOOKLET- RHEUMATOID ARTHRITIS (RA)

Text A
Rheumatoid arthritis (RA) has wide variability in both its clinical presentation and its
autoantibody profile. Two well-known autoantibodies that are found in between 60-90% of RA
patients are rheumatoid factor (RF) and cyclic citrullinated peptide (CCP) antibody. Seropositivity
for these antibodies is associated with more destructive joint pathology and radiographic
progression of RA. Anti-Sjogren’s Syndrome related Antigen A (SSA) is associated with numerous
autoimmune conditions, including most notably Sjogren’s Syndrome. Anti-SSA is also found in
between 3-16% of RA patients and it is believed to be a clinical indicator of poor prognosis in RA.
Several studies have shown that RA patients with this antibody have a lesser clinical response to
infliximab.
Anti-SSA seropositivity is also associated with secondary Sjogren’s Syndrome. RA with secondary
Sjogren’s Syndrome is associated with worse clinical manifestations and increased antinuclear
antibody (ANA) positivity. The prevalence of ANA and anti-SSA has been shown to be higher in
African American (AA) RA patients compared to Caucasian (CAU) RA patients in two established
RA cohorts.
Text B

In RA, inflammatory cytokines such as tumor necrosis factor-alpha (TNF-α) and interferongamma
(IFN-γ) are the primary inducers of chemokine production. Chemokines then lead to increased
numbers of inflammatory cells, such as macrophages, lymphocytes, and fibroblast-like
synoviocytes, in inflamed synovial tissue. Chemokines also contribute to cartilage degradation
and pannus formation by stimulating the release of various inflammatory cytokines. Several
studies have shown that serum chemokines including CX3CL1, CCL5, CXCL9, and CXCL10 are
increased in active RA patients compared to healthy controls.. Particularly, several studies have
found that CXCL10 could serve as a disease activity marker in RA. Elevated CXCL10 and CXCL13
levels have been shown to be predictive of a favorable response to TNF inhibitor therapy.
Studies have also shown that serum chemokine levels, including CXCL9, CXCL10 and CXCL16
decrease after treatment with disease-modifying antirheumatic drugs or biologic agents.
Text C

Previous studies have shown a wide variation in anti-SSA prevalence across different RA
populations. It is possible that the increased frequency of anti- SSA in AA subjects may be due to
an increased frequency of secondary Sjogren’s Syndrome. Co-existent RA and SS may then
partially explain the increased disease activity and worse clinical outcomes seen in AA RA
patients. However, it was not possible to determine the prevalence of Sjogren’s Syndrome in our
cohort with the available data. The AA group also had a higher prevalence of anti-SSB than the
CAU group (4.26% vs. 1.08%). This was not a statistically significant difference; however, there
were only 5 total patients that were anti-SSB positive. The biological and clinical implications of
the increased prevalence of anti-SSA and ANA in AA RA patients are currently unknown.
However, several studies have suggested that autoantibody profiles may be clinically significant.
Specifically, antiSSA has been shown to be associated with more severe disease in multiple
connective tissue disease and it is also involved in the molecular pathogenesis of immune
dysregulation in Sjogren’s Syndrome.
Text D

In RA, a predominance of Th17 cytokines, including IFN-γ and TNF have been suggested to be of
pathological importance. IFN-γ induces several chemokines including CXCL9, CXCL10, and CXCL11.
Increased CXCL10 has been detected in the serum and synovial fluid of RA patients and in the
saliva of Sjogren’s Syndrome patients compared to healthy controls. Additionally, this chemokine
may have clinical significance as a human phase II clinical trial using an anti-CXCL10 monoclonal
antibody (MDX-1100) showed a significantly increased response rate in RA patients who had an
inadequate response to methotrexate therapy. The study revealed an association between RF
seropositivity and increased CXCL10 levels but it found no association between antiSSA positivity
and CXCL10. Therefore, while the increased clinical severity seen in AA RA patients may be
associated with a higher prevalence of anti-SSA, the presence of this autoantibody does not
appear to directly affect the expression of CXCL10.
PART A -QUESTIONS AND ANSWER SHEET
Questions 1-7
For each of the questions, 1-7, decide which text (A, B, C or D) the information comes from. You
may use any letter more than once.

In which text can you find information about;


1. Not much difference in data found. Answer ______________

2. Enhanced clinical response observed in AA RA patients. Answer _______________

3. doesn’t create a more effective response with respect to drug. Answer _______________

4. Their name is derived from their ability to induce directed chemotaxis in nearby responsive cells.
Answer ______________

5. Despite differences in autoantibody prevalence, it is currently unknown if there is a higher.


occurrence of clinically diagnosed Sjogren’s Syndrome in AA RA patients. Answer __________

6. The AA group is reported to have a large number of anti-SSB. Answer ________________

7. signaling protein involved in systemic inflammation. Answer _______________

Questions 8-14
Answer each of the questions, 8-14, with a word or short phrase from one of the texts.
Each answer may include words, numbers or both. Your answers should be correctly spelt.
8. What is referred to as chemoattractant, which is induced by IFN-γ? Answer _______________
9. What is recommended to be of more importance pathologically? Answer ________________
10. What do previous studies indicate? Answer _______________
11. What is known to be more connected with the severity of multiple connective tissue disease?
Answer ________________
12. What can work as a disease activity marker in RA? Answer _________________
13. What usually increases in RA patients? Answer________________
14. What does the study suggest with respect to CXCL10? Answer _________________
Questions 15-20

Complete each of the sentences, 15-20, with a word or short phrase from one of the texts.
Each answer may include words, numbers or both. Your answers should be correctly spelt.

15 ___________________levels often go down when target-specific drugs are used.

16 Anti-SSA is found in between ________________ of RA patients

17 _______________can be defined as having or being a positive serum reaction especially in a

test for the presence of an antibody

18 Increased________________ levels are known to be predictive of a favourable response to TNF

inhibitor therapy.

19 The reason for the increased frequency of ______________can be directly linked to enhanced

frequency of secondary Sjogren’s Syndrome.

20 The research performed clearly indicate that there is no association between antiSSA
positivity and________________

END OF PART A, THIS QUESTIONS PAPER WILL BE COLLECTED


READING SUB-TEST : PART B
In this part of the test, there are six short extracts relating to the work of health professionals .
For questions 1-6, choose the answer (A, B or C) which you think fits best according to the text. Write
your answers on the separate Answer Sheet
Questions 1-6

1. The manual talks about;


A. Steps to improving interpersonal communication with patients.
B. How to effectively deal with the patients.
C. Steps with regard to clinician-patient communication.

Important Steps - Dealing With Patients


1. Slow down. Communication can be improved by speaking slowly, and by spending just a small
amount of additional time with each patient. This will help foster a patient centered approach to
the clinician-patient interaction. 2. Use plain, nonmedical language.
Explain things to patients like you would explain them to your grandmother. 3. Show or
draw pictures. Visual images can improve the patient’s recall of ideas. 4. Limit the amount of
information provided— and repeat it. Information is best remembered when it is given in small
pieces that are pertinent to the tasks at hand. Repetition further enhances recall. 5. Use the
“teach-back” technique. Confirm that patients understand by asking them to repeat back your
instructions. 6. Create a shame-free environment: Encourage questions. Make patients feel
comfortable asking questions. Consider using the Ask-Me-3 program. Enlist the aid of others
(patient’s family or friends) to promote understanding.
2. Duodenal duplication;
A. is associated with various anomalies.
B. can occur in two different ways.
C. Is a benign congenital defect, acquired during the embryonic development of the digestive
tract

Duodenal duplication
Duodenal duplication is an extremely rare pathology. It represents 4 % of all digestive tract
congenital malformations. It is often connected with intestinal malrotation, scalloped vertebras).
In one of the cases, the degeneration arose on mucous membrane of gastric type within the cyst of
duplication and in the second it was duodenal mucous membrane.The diagnosis of cancer was
made in both cases on the surgical pieces at anatomopathology. There had been no biopsy within
the cyst. This malformation appears in 70% of the cases before one year of the age but the late
revelation is possible. The digestive obstruction is the most frequent mode of revelation. Acute
pancreatitis was reported and sometimes the diagnosis can be delayed many years. The duodenal
duplication, the anomaly of the embryogenesis, is diagnosed most of the time in the childhood,
even by prenatal diagnosis. The average age at the time of the diagnosis is from four months to
nine years. Prevalence is lightly in favour of the male.
3. Anti-reabsorption medications
A. Comprised of agents which limit the rate of bone loss.
B. Decrease the rate at which osteoclasts resorb bone.
C. Can have a detrimental effect on elderly people.

Anti-reabsorption medications

Presently, anti-reabsorption medications are most widely used for treating osteoporosis.
Zoledronic acid (Aclasta) is a common clinical anti-reabsorption medication. As the third
generation of bisphosphonates (BPs), it outperforms the previous nitrogen-containing BPs in
improving patients’ balance and quality of life by inhibiting bone resorption and increasing bone
mineral density (BMD). It is administered via intravenous drip infusion once a year, which,
therefore, brings good compliance with treatment. However, those who have been treated with
zoledronic acid intravenously are likely to suffer from acute side effects, such as fever, bone and
joint pain and flu-like symptoms, especially after the first administration. Although the common
adverse reactions generally disappear within 72 h, or in rare cases, last 7 to 10 days without
recurrence, they have serious impacts on the aging population, especially those with underlying
diseases. Thus, the patients who are afraid of any adverse reactions or have suffered from any
side effects during the first administration may show poor compliance with the second dose and
refuse the clinical application of zoledronic acid.
4. The given notice talks about;
A. How cancer has successfully been dealt with in todays advanced scientific world.
B. Effective cancer treatment has resulted in many cancer patients.
C. Future course of action.

Cancer - Brief Overview


Historically, individuals with cancer have rarely survived past the latency periods inherent to the
development of treatment-induced malignancies, but improvements in systemic and radiation
therapy have increased the time for these second primaries to emerge. In the past 15 years,
advances in cancer diagnostics, therapy and supportive care have significantly increased disease-
free survival and overall survival rates in cancer patients. A proper care plan is reported to have
increased 10 or more years beyond their initial diagnosis and introduced the potential for
secondary cancers induced by therapy. It is possible that the incidence of treatment-induced
cancers may increase in the future. Long-term toxicity of cancer treatments both from systemic
chemotherapy and radiation therapy become an important survivorship concern for patients
and their physicians.
5. What is correct about the surgery performed?
A. On average, hospitalization was 7 days and surgery time was 2 hours.
B. The majority of patients remained hospitalized for 5 days or more
C. performed right knee arthroplasty (TRKA) more on males than females.
6. The table clearly shows that;
A. The majority of the patients were male.
B. People who are aged 60-70 are more in number.
C. The majority of the patients were females.
READING SUB-TEST : PART C

In this part of the test, there are two texts about different aspects of healthcare.
For questions 7-22, choose the answer (A, B, C or D) which you think fits best according to the text.
Write your answers on the separate Answer Sheet

Text 1: Paget's Disease


Paget's disease of the bone is an unusual, chronic bone condition that occurs in only
about 1% of people in the United States and slightly more often in men than in women (3
to 2). Individuals with Paget's disease experience rapid bone repair, which causes a
variety of symptoms from softer bones to enlarged bone growth, typically in the pelvis,
lower back (spine), hips, thighs, head (skull) and arms. Medical therapies have proven
effective in reducing the frequency of pain, fractures and arthritis that may be caused by
this condition. Paget's disease typically occurs in the older population and usually only in
a few of their bones. Bones become large and soft, leading to problems such as bending,
breaking, pinched nerves, arthritis and reduced hearing. Effective and safe treatment
methods can help most people with Paget's disease.

Normally, as people age, their bones rebuild at a slower rate. For those with Paget's
disease, however, this process of rebuilding bones takes place at a faster rate. As a result,
the rebuilt bone has an abnormal structure. The involved bone can be soft, leading to
weakness and bending of the pelvis, lower back (spine), hips, thighs, head and arms. Or,
the rebuilt bone can enlarge, making it more susceptible to arthritis, hearing loss,
fractures and discomfort. Given that this takes place in those over the age of 40, the
symptoms are often mistaken for changes associated with aging.
The cause of Paget's disease is unknown. It does appear to be, at least partially,
hereditary, perhaps when activated by exposure to a virus. Indicative of the hereditary
consideration: Paget's disease occurs more commonly in European populations and their
descendants. In 30% of cases, disease incidents often involve more than one member of a
family. Paget's is rarely discovered in individuals before they reach the age of 40, and the
number of people identified increases in each progressive age group. Typically, it is the

appearance of the bones on an X-ray that prompts the physician to make the diagnosis. Blood
tests taken most often will indicate an increase in serum alkaline phosphatase (SAP), which is
reflective of the rapid new bone turnover. Urine test results will also indicate the speed at
which this rebuilding is taking place. Physicians usually obtain a non-invasive bone scan to
determine the extent of bone involvement. Only if cancer is suspected will it be necessary to
do a bone biopsy to examine it under a microscope.

Treatment approaches can focus on providing physical assistance, including the addition
of wedges in the shoe, canes as walking aids and the administration of physical therapy
(this is considered to be the best). Medications that help reduce the pain associated with
Paget's include acetaminophen (e.g. Tylenol) and anti-inflammatory drugs such as
ibuprofen and naproxen. In addition, a group of medications called bisphosphonates
reduce the pain and help the body regulate the bone-building process to stimulate more
normal bone growth. Your physician may prescribe an oral medication such as
Alendronate (Fosamax) or etidronate (Didronel) to be taken orally every day for 6
months; Tiludronate (Skelid) to be taken orally every day for 3 months, Risedronate
(Actonel) to be taken orally every day for 2 months. All oral medications should be taken
with a large glass of water (6-8 oz) upon rising in the morning. Patients should remain
upright for the next 30 minutes and not eat until that time has passed. Any of these
treatments can be repeated if necessary. Side effects of these medicines may involve
heartburn and sometimes an increase in bone pain for a short period of time.
There are also injectable medications that can be given to a patient for Paget's which
include Pamidronate (Aredia), which is injected into the vein once a month or once every
few months. The injection takes a few hours. Unusually, there is inflammation of the eye
or loss of bone around the teeth (osteonecrosis); Zoledronate (Reclast), which is injected
in the vein once a year. The injection takes less than 30 minutes; Calcitonin, a hormone
that is injected under the skin several times a week. Surgery for arthritis caused by
Paget's disease is effective in reducing pain and improving function. Medical treatment is
not expected to correct some of the changes of the Paget's disease that have already
occurred, such as hearing loss, deformity or osteoarthritis.
Text 1: Questions 7-14
7. Pagets disease;

A. Affects more women than men;


B. Affects 3 men in 4.
C. Affects 3 women in 4.
D. Affects a higher number of men than women.

8. Pagets disease can lead to;

A. Bone disorder.
B. Softening of bones.
C. Arthritis in its final stage.
D. None of the above.

9. Pagets disease is a condition in which;

A. Bones become large and soft.


B. Rebuilding of bones is stopped.
C. Pelvis bends at a faster rate.
D. Rebuilding of bone is accelerated.

10. Reformation of the bones can lead to;

A. Lower back pain, loss of hearing and discomfort.


B. Arthritis, loss of hearing and softening of bone tissues.
C. Fractures and discomfort only.
D. All of the above.
11. Which of the following statements is right as per the given information in the passage?

A. If one member of the family is affected with Pagets disease then other members will also be
affected.
B. If one member of the family is known to have Pagets disease then others can also be
affected.
C. Pagets disease is highly hereditary.
D. Pagets disease, is both heritable and inheritable.

12. Pick the correct statement as per the given information in the passage;

A. Those who reach the age of 40 shall undergo blood tests and urine tests for the
identification of Pagets disease.
B. Physicians should always advise the patients to go for a non-invasive bone scan.
C. Increase is indicative of the development of the bone at a rapid speed.
D. It is necessary to do a biopsy of the bone to understand the nature of the disease.

13. What is right about oral medications?

A. May increase bone pain.


B. Should be taken only during the morning.
C. May disturb physiological function.
D. All of the above.

14. Which of the following statements is incorrect as per the given information in the passage?

A. Pagets disease is a condition which can lead to arthritis.


B. Surgery can get rid of Pagets disease.
C. Surgery of arthritis which occurred due to Pagets disease can only reduce the pain
associated with it.
D. Medical treatment will not correct a loss in hearing or deformity or osteoarthritis which
resulted from it.
Text 2: Pancreatic Cancer

Cancer is a class of diseases characterized by out-of-control cell growth, and pancreatic


cancer occurs when this uncontrolled cell growth begins in the pancreas. Rather than
developing into healthy, normal pancreas tissue, these abnormal cells continue dividing
and form lumps or masses of tissue called tumors. Tumors then interfere with the main
functions of the pancreas. If a tumor stays in one spot and demonstrates limited growth, it
is generally considered to be benign. More dangerous, or malignant, tumors form when
the cancer cells migrate to other parts of the body through the blood or lymph systems.
When a tumor successfully spreads to other parts of the body and grows, invading and
destroying other healthy tissues, it is said to have metastasized. This process itself is
called metastasis, and the result is a more serious condition that is very difficult to treat.
In the United States each year, over 30,000 people are diagnosed with pancreatic cancer.
Europe sees more than 60,000 diagnosed each year. In Asian countries, numbers can be
even higher than that.

Pancreatic cancer is categorized depending on whether it affects the exocrine or


endocrine functions of the pancreas. There is an important distinction between the two
broad types of pancreatic cancer because they have different risk factors, causes,
symptoms, diagnostic tests, treatments, and prognosis. Tumors that affect the exocrine
functions are the most common type of pancreatic cancer. Sometimes these tumors or
cysts are benign, called cystadenomas. However, it is more likely to find malignant
tumors called adenocarcinomas, which account for 95% of exocrine pancreatic cancers.
Adenocarcinomas typically start in gland cells in the ducts of the pancreas, but they can
also arise from pancreatic enzyme cells (acinar cell carcinoma). Other types of pancreatic
cancers that are associated with exocrine functions include adenosquamous carcinomas,
squamous cell carcinomas, and giant cell carcinomas, named for their appearances
underneath a microscope. There is also a disease called ampullary cancer (carcinoma of
the ampulla of Vater) that starts where the bile duct and pancreatic duct meet the
duodenum of the small intestine.
Cancer is ultimately the result of cells that grow uncontrollably and do not die. Normal
cells in the body follow an orderly path of growth, division, and death. Programmed cell
death is called apoptosis, and when this process breaks down, cancer occurs. Pancreatic
cancer cells do not experience programmatic death, but instead, continue to grow and
divide. Although scientists do not know exactly what causes these cells to behave in this
way, they have identified several potential risk factors. Cancer can be the result of a
genetic predisposition that is inherited from family members. It is possible to be born
with certain genetic mutations or a fault in a gene that makes one statistically more likely
to develop cancer later in life.

About 10% of pancreatic cancers are thought to be caused by inherited gene mutations.
Genetic syndromes that are associated with pancreatic cancer include hereditary breast
and ovarian cancer syndrome, melanoma, pancreatitis, and non-polyposis colorectal cancer
(Lynch syndrome).

Carcinogens are a class of substances that are directly responsible for damaging DNA,
promoting or aiding cancer. Certain pesticides (dyes may also be included in this list
here), and chemicals used in purification of the metal are thought to be carcinogenic,
increasing the risk of developing pancreatic cancer. When our bodies are exposed to
carcinogens, free radicals have formed that try to steal electrons from other molecules in
the body. These free radicals damage cells, affecting their ability to function normally,
and the result can be cancerous growths. As we age, there is an increase in the number of
possible cancer-causing mutations in our DNA. This makes age an important risk factor
for pancreatic cancer, especially for those over the age of 60. There are several other
diseases that have been associated with an increased risk of cancer of the pancreas.
These include cirrhosis or scarring of the liver, Helicobacter pylori infection (infection of
the stomach with the ulcer-causing bacteria H. pylori), diabetes mellitus, chronic
pancreatitis (inflammation of the pancreas), and gingivitis or periodontal disease.
In order to detect pancreatic cancer, physicians will request a complete physical
examination as well as personal and family medical histories. The way in which cancer
presents itself will differ depending on whether the tumor is in the head or the tail of the
pancreas. Tail tumors present with pain and weight loss while head tumors present with
steatorrhea, weight loss, and jaundice. Doctors also look for recent onset of atypical
diabetes mellitus, Trousseau's sign, and recent pancreatitis. In general, when making a
pancreatic cancer diagnosis, physicians pay special attention to common symptoms
such as abdominal or back pain, weight loss, poor appetite, tiredness, irritability,
digestive problems, gallbladder enlargement, blood clots (deep venous thrombosis
(DVT) or pulmonary embolism), fatty tissue abnormalities, diabetes, swelling of lymph
nodes, diarrhea, steatorrhea, and jaundice.
Text 2: Questions 15-22
15. Pancreatic tumors can be;

A. Benign
B. Malignant
C. Benign and malignant
D. None

16. Metastasized is a condition

A. When pancreatic tumors spread to other parts of the body.


B. When malignant tumors form in the pancreas.
C. When harmful tumors invade and destroy other healthy tissues of the body.
D. When tumors are grown automatically in other parts of the body.

17. Pancreatic cancer is most commonly associated with.

A. Endocrine gland
B. Exocrine gland
C. Cystadenomas
D. Adenocarcinomas

18. Ampullary cancer is associated with.

A. Endocrine gland
B. Exocrine gland
C. Pancreatic duct
D. None

19. Failure of apoptosis results in;


A. Pancreatic cell growth
B. Enlargement of the pancreatic duct
C. Cancerous tissues in the pancreas
D. B & C
20. A cancerous growth in the pancreas is a result of;

A. Specific carcinogens
B. Family Genes
C. Genetic mutations
D. None

21. The risk of pancreatic cancer is associated with these carcinogens;

A. Pesticides and dyes


B. Pesticides, dyes and chemicals used for refining metals
C. Only dyes
D. None

22. One of these is not a symptom associated with PC;

A. Back pain and problems with digestion


B. Digestive problems and blood clotting
C. Pulmonary edema and enlargement of the gallbladder
D. Jaundice and modification in the lymph

END OF READING TEST, THIS BOOKLET WILL BE COLLECTED


Reading test 6 : Answer Key

Part A - Answer key 1 – 7

1 .C
2 .D
3 .A
4 .B
5 .A
6 .C
7 .B

Part A - Answer key 8 – 14

8 .CXCL9
9 .Th17 cytokines
10 .variation in anti-SSA prevalence
11 .Anti SSA
12 .CXCL10
13 .serum chemokines
14 .relation between RF seropositivity & CXCL10 levels

Part A - Answer key 15 – 20

15.Serum Chemokine
16 .3-16%
17.Correct Answer Is: Seropositivity
18 .Cxcl10 And Cxcl13
19 . Anti- Ssa In Aa
20 .Cxcl10
Reading test - Part B : Answer Key
1 .Steps to improving interpersonal communication with patients.
2 .is associated with various anomalies.
3 .Can have a detrimental effect on elderly people.
4 .Future course of action.
5 .The majority of patients remained hospitalized for 5 days or more
6 .The majority of the patients were females.

Reading test - Part C : Answer Key


Text 1 - Answer key 7 – 14
7. Affects a higher number of men than women.
8. Arthritis in its final stage.
9. Rebuilding of bone is accelerated.
10. Lower back pain, loss of hearing and discomfort.
11. Correct Answer Is: Pagets disease, is both heritable and inheritable.
12. Increase is indicative of the development of the bone at a rapid speed.
13. Should be taken only during the morning.
14. Correct Answer Is: Surgery can get rid of Pagets disease.

Text 2 - Answer key 15 – 22


15. Benign and malignant
16. When harmful tumors invade and destroy other healthy tissues of the body.
17. Exocrine gland
18. Exocrine gland
19. Cancerous tissues in the pancreas
20. Genetic mutations
21. Pesticides, dyes and chemicals used for refining metals
22. Pulmonary edema and enlargement of the gallbladder
READING TEST 7

READING SUB-TEST : PART A


 Look at the four texts, A-D, in the separate Text Booklet.
 For each question, 1-20, look through the texts, A-D, to find the relevant information.
 Write your answers on the spaces provided in this Question Paper.
 Answer all the questions within the 15-minute time limit.
 Your answers should be correctly spelt.

TEXT BOOKLET- TREATMENT OF FRACTURES

Text A

Reduction: A clinician can achieve a reduction by closed manipulation – in which the displaced
bone fragments are pulled into their anatomical position – restoring alignment or by open
reduction through a surgical incision.
Immobilisation can be achieved by internal or external fixation devices, which are available in
many forms. Internal fixation involves the patient undergoing a surgical procedure and includes
devices such as intramedullary nails, compression nails, plates and screws. Internal fixation is
used in certain pathological fractures, when sufficient reduction cannot be maintained by
external fixation, for example, when fractures involve joint surfaces, when it is important to allow
early limb or joint movement, or when trying to avoid long periods of immobilisation in bed.
External fixation can be achieved through surgical, as well as conservative techniques, and
includes slings, cast immobilisation, skin or skeletal traction and external fixator frames.
Text B

Rehabilitation: Restoration of the upright position and early mobilization decrease


cardiopulmonary and other immobility associated complications, for example. pressure ulcers,
constipation. and urinary stasis. Following recovery or once the fracture is stable, the limb can be
mobilised and range of movement exercises can begin. Deciding on the right time to begin
physiotherapy is difficult.
Rehabilitation should not commence too early as this may result in malunion of the bone,
however, it should not even begin too late resulting in a perfect union of bone, but muscles are
unable to operate the limb. Nurses have a responsibility to know what type of rehabilitation
programme patients are undergoing; whether this is fully weight-bearing, partial weight-bearing,
touch-toe-bearing or non-weightbearing; and also what mobilisation aids, if any, are being used,
so that they are able to continue mobilising patients when physiotherapy services are not
available.
Text C

The complications associated with fractures can be classified as immediate, early or late.
Nurses must observe for complications and take preventive measures.
Text D

Pain assessment and management: Although pain is a useful sensation in alerting us to disease or
injury, it should not be accepted as a normal and inevitable part of recovery from injury or
surgery. Assessment of pain is essential to ensure that the correct analgesic for the condition is
prescribed and administered, and that it is having the desired effect with minimal side effects.
The nurse caring for the patient, who has sustained a fracture should have knowledge of
available medications and their actions, side effects and dosages. Pre-emptive analgesia should
be provided so that the patient's pain is sufficiently managed before and during rehabilitation
sessions. Non-pharmacological methods of pain control such as positioning, distraction
techniques and massage may also benefit patients.
PART A -QUESTIONS AND ANSWER SHEET
Questions 1-7
For each question, 1-7, decide which text (A, B, C or D) the information comes from.
You may use any letter more than once.
In which text can you find information about:
1. Necessary to take preventive measures. Answer___________________

2 .To help the patient cope with disability. Answer _______________________

3. Minimising the risk of deficit and in detecting early signs of the development. Answer __________

4 .At risk of death from a relatively simple transverse fracture of the tibia, if it is not detected. Answer_

5. To restore normal alignment of the bone. Answer _______________________

6 .To ensure that the reduced position is maintained until the bone union takes place. Answer ______

7. Internal and external haemorrhage. Answer ______________________


Questions 8-14
Answer each of the questions, 8-14, with a word or short phrase from one of the texts.
Each answer may include words, numbers or both. Your answers should be correctly spelt.
8 .What can be provided for effective pain management? Answer ________________
9. When can it be possible to make the patient ready for movement? Answer __________________
10. What should not be regarded as the unavoidable part of recovery from injury? Answer ___
11 .What is the example given for a non-rigid method of support? Answer ____________________
12. What may involve a surgical process? Answer ___________________
13 .What knowledge shall a nurse have when it comes to effective caring for the patient who has got
fractured? Answer ______________________
14 .How can reduction successfully be performed? Answer _____________________
Questions 15-20

Complete each of the sentences, 15-20, with a word or short phrase from one of the texts.
Each answer may include words, numbers or both. Your answers should be correctly spelt.

15. Because of a period of immobilisation and the effects of surgery, patients are at a risk of
developing_____________

16. A _____________ may lead to osteoarthritis as a result of an abnormal distribution of load leading
to an early degenerative change.

17. There are _____________ that carry the risk of damage to particular arteries.

18. Pain is considered a ___________ which make one aware of the injury.

19. A ______________ is an uncommon but serious complication.

20. Besides the blood loss from the______________, the sharp bone ends found in a spiral or
comminuted fracture, for example, may damage the surrounding muscle or blood vessels.

END OF PART A, THIS QUESTIONS PAPER WILL BE COLLECTED


READING SUB-TEST : PART B
In this part of the test, there are six short extracts relating to the work of health professionals .
For questions 1-6, choose the answer (A, B or C) which you think fits best according to the text. Write
your answers on the separate Answer Sheet
Questions 1-6

1 .The following is a ______ model


A. Manual calibration
B. True gravity calibration
C. Wall sphygmomanometer

Mercury-gravity Manometer
The mercury-gravity manometer consists of a calibrated cartridge glass tube that is optically clear,
easy to clean, and abrasion resistant. The mercury reservoir at the bottom of the tube
communicates with a compression cuff through a rubber tube. When air pressure is exerted on the
mercury in the reservoir by pumping the pressure bulb, the mercury in the glass tube rises and
indicates how much pressure the cuff applies against the artery. The manometer is connected to
the wall for ease of accurate visualization.
2 .The manual gives information about;
A. Indications of emergency treatment.
B. How the device can effectively be used?
C. Indications for using the Power heart AED G3
Automatic devices like Power heart AED are intended to be used by the personnel who are trained
in its operation. The user should be qualified by training in basic life support or other emergency
medical response authorized by physicians. The device is indicated for emergency treatment of
victims exhibiting symptoms of sudden cardiac arrest, who are unresponsive and not breathing.
Post-resuscitation, if the victim is breathing, the AED should be left attached to allow for
acquisition and detection of the ECG rhythm. If a shockable ventricular tachyarrhythmia recurs, the
device will charge automatically and advise the operator to deliver therapy (G3) or automatically
deliver the shock (G3 Automatic). If the patient is a child or an infant up to 8 years of age, or up to
55 lbs (25kg), the device should be used with the Model 9730 Pediatric Attenuated Defibrillation
Electrodes. The therapy should not be delayed to determine the exact age or weight of the patient.

3. Pick the correct statement;


A. Data suggests that NAFLD is more common than any other diseases.
B. NAFLD is more prevalent in Middle East.
C. NAFLD is more prevalent in Africa.
Prevalence of NAFLD - In Contrast to Other Diseases
The meta-analysis estimated that the overall global prevalence of NAFLD diagnosed by imaging I
around 25.24% (95% CI, 22.10-28.65). The ubiquity rate of NAFLD in the Middle East can be
reported as follows: (31.79% [95% CI, 13.48- 58.23]) and South America (30.45% [95% CI, 22.74-
39.440]), whereas the lowest prevalence rate is reported from Africa (13.48% [5.69- 28.69]).
4 .The manual talks about;
A. Critical Illness & Treatment Of Delirium
B. Delirium In Critical Care
C. Impact Of Delirium On ICU Patient

Delirium
The study of disease transmission of Delirium in sick patients is currently perceived as a general
well being issue, influencing the mechanically ventilated grown-up ICU patients up to 80%, and
costing $4 to $16 billion every year in the United States alone. Effect of Delirium, as a sign of
intense cerebrum brokenness, is a free critical indicator of negative clinical results in ICU patients
that includes expanded mortality, healing facility LOS, expense of consideration, and long-haul
psychological weakness reliable with a dementia-like state. Patients with long-haul presentation to
high-measurements sedatives or medications may create physiologic reliance, and unexpected
suspension may bring about medication withdrawal side effects. This session likewise incorporates
Impact of Delirium on ICU Patient Outcomes, Epidemiology of insanity in ICU patients, Preventing,
Detecting, and Treatment because of Alcohol and drug Withdrawal. Critical illness and appraisal of
incoherence, Risk element of wooziness.
5 .As per the report, what is correct?
A. Talk about 2050 neonates.
B. Major reason for admission into hospital was related to CMV.
C. Gestational age is taken as an important factor for the study conducted.

A Report
Two hundred and sixty-one neonates born at the hospital were admitted to the neonatal ICU
during the study period. Two patients were excluded because they had received transfusions of
blood products before urine collection (0.76%), two because the consent was not obtained (0.76%),
one because the urine sample was lost (0.38%), two died before collection (0.76%) and four were
lost because they were discharged early, before urine collection (1.5%).
The study population comprised 145 male NB (58%) and 105 female NB (42%).
The principal causes of admission to the ICU were prematurity (111 cases, 44.4%), respiratory
dysfunction (64 cases, 25.6%), sepsis (31 cases, 12.4%) and hypoglycemia (21 cases, 8.4%). The
mean weight of the newborn population studied was 2,412±900 g and mean gestational age was
35.7±3.7 weeks.
6. From the given manual, it is clear that;
A. The device will analyze ECG and can make shock deliverance simple.
B. Non-committed shock is possible through the device.
C. The device automatically shifts from one phase to other phase of operations as per the
rhythmic changes.

After the AED advises a shock, it continues to monitor the ECG rhythm of the patient. If the ECG
rhythm changes to a non-shockable rhythm before the actual shock is delivered, the AED will
advise that the rhythm has changed and issue the prompt “RHYTHM CHANGED.
SHOCK CANCELLED.” The AED will override the charge and continue ECG analysis. Synchronized
Shock:- The AED is designed to automatically attempt to synchronize shock delivery on the R-wave,
if one is present. If delivery cannot be synchronized within one second, a nonsynchronized shock
will be delivered.
READING SUB-TEST : PART C
In this part of the test, there are two texts about different aspects of healthcare.
For questions 7-22, choose the answer (A, B, C or D) which you think fits best according to the text.
Write your answers on the separate Answer Sheet

Text 1 : Opioid-induced Constipation


The human opioid system is highly complex and includes three main endogenous opioid receptors
(µ, κ and δ receptors), as well as a number of endogenous opioid peptides such as endorphins,
enkephalins, and dynorphins, which activate these receptors. Opioid receptors are widely
distributed in the brain and spinal cord, as well as in a variety of peripheral tissues such as the gut,
airways, blood vessels, and heart.When activated, the opioid receptors located in the brain and
spinal cord mediate the analgesic effects of the opioids. Unfortunately, activation of opioid
receptors in the gut can significantly impair intestinal activity and lead to OIC.

Mu-opioid receptors, and to a lesser extent κ-opioid receptors, are expressed extensively
throughout the gastrointestinal tract and enteric nervous system.
When opioid receptors are stimulated they may alter or more precisely decrease peristalsis, inhibit
intestinal transit, increase intestinal fluid absorption and decrease intestinal secretions. All of these
effects combine to cause significant constipation. A clinical syndrome of opioid-induced bowel
dysfunction may occur with chronic opioid use that is characterized by abdominal pain, hard stools,
fecal impaction, anorexia, nausea and vomiting.

A 2009 study by Bell et al., examined the prevalence, severity and impact of OIC in 322 patients
taking daily opioids. The investigators found that 81% of the patients in their study reported OIC and
that the majority of patients with OIC reported at least a moderate negative impact on quality of life
and activities of daily living. One third of patients with OIC missed the doses, decreased dosage or
stopped using opioids completely in order to improve their bowel function. A second study by Bell
found that the patients with OIC were more likely to taketime off from work and were less
productive in both their work and home environments.
With continued opioid use, patients often become adept at dealing with the analgesic effects of the
opioids. The mechanism of opioid tolerance is likely related to down-regulation (decreased
numbers) or desensitization of µ-opioid receptors in the CNS. Binding of opioids to the µ receptor
activates numerous downstream intracellular pathways. Activated G-protein-dependent signaling
pathways appear involved in receptor desensitization while G- proteinin dependent signaling
pathways appear to facilitate µ receptor endocytosis and subsequent downregulation of receptors.
Fortunately, tolerance to the respiratory depressant effects of the opioids also develops in parallel
to the tolerance seen with the analgesic effects. Interestingly, however, tolerance does not occur to
the constipating effect of the opioids.

Opioid-induced constipation and opioid bowel dysfunction presents with a myriad of symptoms.
Diagnosis of OIC should begin with a detailed patient history that includes frequency of bowel
movements, the consistency of stool, and the presence of straining, pain, nausea and vomiting. A
physical examination should also be conducted including the bowel sounds, and abdominal
palpation for firmness, distention and the presence of pain. The possibility of fecal impaction
should also be assessed in patients with persistent and severe constipation. A number of
nonpharmacologic and pharmacologic options are available with respect to effective elimination of
this condition. Although current treatment recommendations support the prophylactic use of
various bowel regimens in patients receiving opioid therapy, definitive studies showing the
superiority of one treatment regimen over another are currently lacking.

The overall strategy to prevent OIC and to start with the onset of opioid therapy, include adequate
fluid and fiber intake, and increased physical activity. A recent study showed that patients with post-
operative constipation, who received bowel massage by nurses had reduced symptoms of
constipation, increased stool output and improved quality of life with no significant adverse side
effects. However, in another study, it is showed that while abdominal massage was useful for
decreasing the severity of constipation symptoms, it did not lead to curtailing down of laxative.
Bowel “diaries” which track the frequency of bowel movements might also be helpful for
determining the severity of the OIC that is occurring. While helpful, nonpharmacologic interventions
are seldom successful alone for the management of OIC.
Senior nurses should be ideally situated to identify patients at high risk for OIC and ruling out other
causes of chronic constipation. A detailed patient history should be obtained, which will include
physical activity, and a review of all medications the patient is currently taking. A thorough patient
examination should be conducted and accompanying signs and symptoms such nausea, vomiting or
abdominal pain/distention should be noted. Nurses should monitor patient bowel habits as well as
the quantity and quality of stools.
Text 1: Questions 7-14

7. In the first paragraph, the writer talks about;


A. OIC in general
B. Types of OIC receptors
C. How OIC occurs?
D. Distribution of opioid receptors.

8. As per paragraph two, activation of opioid receptors will;


A. Lead to intestinal movement
B. Cause hard stools
C. Lead to constipation
D. Cause abdominal pain

9. What do we learn about OIC in the third paragraph?


A. Research conducted by Bell
B. Impact of OIC on life
C. How OIC will affect?
D. Cessation of opioid pain therapy

10 .What do we learn from the fourth paragraph?


A. Opioid acceptance behaviour among patients
B. Ability to deal with the analgesic effects
C. Resilience and OIC
D. Opioid Tolerance and OIC

11. What is not right as per the information given in the fifth paragraph?
A. A large number of symptoms arises that lead to OIC.
B. Detection of bowel sounds, presence of nausea are common.
C. Patients suffering from the disease will vomit, feel abdominal pain.
D. None of the above

12. In paragraph six, the writer gives information on;


A. How OIC can easily be avoided?
B. Nonpharmacologic management of OIC.
C. Steps to be taken to deal with OIC.
D. How OIC can be prevented?
13. Pick the incorrect statement related to the study performed as explained by the writer;
A. Only nonpharmacologic interventions can be effective.
B. Bowel massage is effective in eliminating many of the problems associated with OIC.
C. Stool output can be improved.
D. Use of laxative can certainly be decreased.

14 .In the final paragraph, the writer talks about;


A. Importance of taking patient`s
B. history. Identifying risks.
C. Role of the advanced practice nurse in OIC.
D. Treatment procedure.
Text 2: Vaccination
Vaccination as a deliberate attempt to protect human beings against disease has a long history
and more widespread use of vaccines could prevent about 1.6 million deaths a year among
children less than five years of age. Over the next few years a new generation of vaccines will
become available that could save the lives of up to 10 million individuals: e.g., vaccines against
diarrhoeal diseases, hepatitis C, malaria, acquired immunodeficiency syndrome (AIDS), sexually
transmitted and other diseases. Current development efforts seek combination vaccines that
protect against multiple pathogens, with a goal of combining all the antigens recommended for
routine immunization into a single multivalent product.

Combining multiple related or unrelated antigens into a single vaccine is not a new concept and
the first combination vaccine licensed in the United States of America was trivalent influenza in
1945. Diphtheria, Pertussis, Tetanus (DPT) vaccine although developed in 1943, was not licensed
till 1948. Efforts to overcome the interference seen with simultaneous administration of three
live vaccines delayed the licensing of trivalent Oral Polio Vaccine (OPV) till 1963. Measles,
Mumps,Rubella (MMR) was licensed in 1971 and quadrivalent meningococcal vaccine in 1978.

Combining multiple antigens into one injection requires demonstration that the combination will
not materially reduce the safety or immunogenicity of the component vaccines. Combination
vaccine trials should be prospective, randomised, double blinded and should have control
(comparison) groups.Identifying the control groups could be problematic when multicomponent
vaccine is evaluated. Other factors like sequence of administration of certain antigens may play
an important role in immunogenicity.

The interaction can enhance the immune response to individual components as it occurs in whole
cell pertussis vaccine, when combined with diphtheria toxoid.
Usually, a combination of vaccines results in no effect or a depression of immune response to one
or more vaccine component. It is an immunologic phenomenon relevant to combination vaccines,
antibody responses to hapten polysaccharide vaccine (e.g. H influenzae b) presented on a carrier
protein (e.g. tetanus toxoid,diphtheria toxoid) are inhibited by prior immunization with the
specific carrier.Combination live vaccines can interfere immunologically with each other, e.g., one
vaccine may stimulate interferon production that may inhibit replication of another virus.
Interest in combining DPT/IPV was generated when enhanced potency IPV became available,
thus eliminating the necessity of frozen shipments for OPV. In addition, administration of IPV
would eliminate the risk of vaccine-associated polio. Antibody responses to pertussis and
poliovirus components may be substantially reduced in combination than when given alone.
However, poliovirus seroconversion rates and absolute antibody levels remained high in
combined vaccines. Various studies have compared DPT combinations with unconjugated
Polyribose phosphate (PRP) or conjugated PRP Hib vaccine. A number of studies have evaluated
these combination vaccines and results are variable. In general, the groups with lower antibody
responses still attained levels considered protective. One study evaluated the effect of booster
dose of DPT/Hepatitis B/Hib given to subjects, who received DPT/Hepatitis B/ Hib for the primary
series. The group hadhigh antibody response and mean levels were higher in the group primed
with DPT/Hepatitis B/Hib, especially with PRP (Hib).

DTaP/HB combination vaccine retains the immunogenicity and safety profiles of the separate
components and delivers good antibody concentrations at a variety of schedules. A comparison
of combination vaccine at 2, 4, 6 months versus the currently recommended schedules-HB at
birth, 1 & 6 months and DPT at 6, 10 & 14 weeks, found similar or higher antibody responses for
combined vaccine for every component, which was significantly lower. However, the mean HB
antibody levels were high and 98% of subjects had levels greater than 10mIU/mL, which are
considered protective. A study comparing combination vaccine and separate vaccines given at 2,
4, 6 and booster between 12 and 15 months of age has shown that the antibody response to Hib,
were 72% and 76% at 6 months, increasing to 92% and 93% after booster dose with combination
and separate vaccines respectively.
Text 2: Questions 15-22

15. In the first paragraph, writer says that;


A. Combination vaccines are the new phase of development.
B. Vaccines implemented have changed lives of people across the globe.
C. Vaccines provide effective protection from a large number of pathogens c.
D. Use of vaccines have led to curtailing down of deaths in millions.

16. In paragraph 2, the writer says


A. Combination vaccines came into being in 1940s.
B. DPT was the first combined vaccine to be used.
C. OPV was not introduced before 1963.
D. MMR is an example of effective combined vaccine.

17. Paragraph three talks about;


A. Immunogenicity of the prepared vaccines.
B. Importance of conducting trails.
C. Facts that have direct impact on immunogenicity.
D. Challenges in the development of combined vaccines.

18 .In paragraph four, the writer has described;


A. Immunological interference
B. Antigen role
C. Induction of interferon
D. B and C

19. Immunologic phenomena relevant to combination vaccines gives an idea about;


A. Induction of interferon
B. Carrier induced epitope expression
C. Antigen competition
D. Immunological interference

20 .As per paragraph five, what is more relevant?


A. DPT/Conjugate (Hib) will often be not more effective.
B. Combined vaccines often produce different results.
C. DPT/IPV came into being only after potency IPV became available.
D. DPT/Hepatitis B and DPT/Hepatitis B/Hib have more potential to produce same results
21. In paragraph 6, the writer says;
A. Taking vaccines at an early age is important.
B. Various combinations bring in various results.
C. Combinations with DTaP is proven to be more effective
D. Separate vaccine shall be given as needed

22 .In final paragraph, writer has directly or indirectly implied that;


A. Vaccines for newborn and children less than 3 years old are known to be more powerful.
B. Combined vaccines may not be as effective as separate vaccines.
C. Separate vaccines are more effective than combined vaccines.
D. Combined vaccine helps with retaining immunogenicity.

END OF READING TEST, THIS BOOKLET WILL BE COLLECTED


Reading test 7 : Answer Key

Part A - Answer key 1 – 7


1: D
2: B
3: D
4: C
5: A
6: A
7: C

Part A - Answer key 8 – 14


8: Pre-emptive analgesia
9: After healing
10: Pain
11: Sling
12: Internal fixation
13: medications knowledge
14: Closed manipulation

Part A - Answer key 15 – 20


15: DVT
16: mal-union
17: several injuries
18: useful sensation
19: fat embolism
20: fractured bone
Reading part B – answer key

Questions 1-6
1: Wall sphygmomanometer
2: Indications for using the Powerheart AED G3
3: NAFLD is more prevalent in Middle East.
4: Delirium In Critical Care
5: Talk about 2050 neonates.
6: The device will analyze ECG and can make shock deliverance simple.

Reading test - part C – answer key

Text 1 - Answer key 7 – 14


7: How OIC occurs?
8: Lead to intestinal movement
9: Impact of OIC on life
10: Opioid Tolerance and OIC
11: None of the above
12: Nonpharmacologic management of OIC.
13: Use of laxative can certainly be decreased.
14: Role of the advanced practice nurse in OIC.

Text 2 - Answer key 15 – 22


15: Vaccines implemented have changed lives of people across the globe.
16: MMR is an example of effective combined vaccine.
17: Immunogenicity of the prepared vaccines.
18: Antigen role
19: Antigen competition
20: Combined vaccines often produce different results.
21 : Various combinations bring in various results.
22: Combined vaccine helps with retaining immunogenicity.
READING TEST 8

READING SUB-TEST : PART A


 Look at the four texts, A-D, in the separate Text Booklet.
 For each question, 1-20, look through the texts, A-D, to find the relevant information.
 Write your answers on the spaces provided in this Question Paper.
 Answer all the questions within the 15-minute time limit.
 Your answers should be correctly spelt.

TEXT BOOKLET- BRANCHED-CHAIN AMINO ACID SUPPLEMENTS

Text A

Branched-Chain Amino Acid Supplements


The concept that the BCAAs may have a unique capacity to stimulate muscle protein synthesis has
been put forward for more than 35 years. In 1981, Buse reported that in rats the BCAAs may be
rate limiting for muscle protein synthesis. Additional studies supported the concept of a unique
effect of BCAAs on muscle protein synthesis in rats, although few have studied the response to
oral consumption of only BCAAs. Garlick and Grant showed that infusion of a mixture of BCAAs
into rats increased the rate of muscle protein synthesis in response to insulin, but they did not
measure the effects of BCAAs alone. The infusion of BCAAs alone into rats by Kobayashi et al. was
shown to induce an increase in muscle protein synthesis, but the response was only transient.
Presumably the rate of synthesis quickly became constrained by the availability of the other EAAs.
Text B
Muscle protein is in a constant state of turnover, meaning that new protein is continuously being
produced while older proteins are being degraded. The anabolic state has no specific definition,
but generally refers to the circumstance in which the rate of muscle protein synthesis exceeds the
rate of muscle protein breakdown. The results in a gain of muscle mass. Conventionally the
anabolic state is considered to be driven by a stimulation of muscle protein synthesis, but
theoretically could also result from an inhibition of muscle protein breakdown. The overriding
metabolic goal of consuming BCAA supplements is to maximize the anabolic state. It is widely
asserted that BCAAs induce an anabolic state by stimulating muscle protein synthesis. An
abundant availability of all EAAs is a requisite for a significant stimulation of muscle protein
synthesis. Muscle protein synthesis will be limited by the lack of availability of any of the EAAs,
whereas a shortage of NEAAs can be compensated for by increased de novo production of the
deficient NEAAs. In the post-prandial state following a meal containing protein, all of the EAA
precursors required for new muscle protein synthesis can be derived from either the elevated
plasma concentrations resulting from digestion of the consumed protein or from recycling from
protein breakdown. In this circumstance of abundant availability of EAAs the rate of muscle
protein synthesis exceeds the rate of breakdown, thereby producing an anabolic state.
Text C
Since EAAs cannot be produced in the body and there is a net release of EAAs from muscle, in the
post-absorptive state the only source of EAA precursors for muscle protein synthesis is intracellular
EAAs derived from muscle protein breakdown. In addition to being reincorporated into muscle
protein via synthesis, some EAAs released from muscle protein breakdown may be partially oxidized
within muscle, thereby making them unavailable for reincorporation into muscle protein. EAAs
released from muscle protein breakdown that are not reincorporated into muscle protein or
oxidized within muscle tissue are released into plasma, whereupon they can either be taken up by
other tissues as precursors for protein synthesis or irreversibly oxidized. Thus, the rate of muscle
protein synthesis will always be lower than the rate of muscle protein breakdown in the
postabsorptive state, owing to the net flux of EAAs from protein breakdown into plasma and to
oxidative pathways. Expressed differently, it is impossible for muscle protein synthesis to exceed
the rate of muscle protein breakdown when the precursors are derived entirely from protein
breakdown, and thus an anabolic state cannot occur in the absence of exogenous amino acid intake.
Text D

All EAA precursors for muscle protein synthesis in the post-absorptive state are derived from
muscle protein breakdown. It has been consistently reported that in normal post-absorptive
humans the rate of muscle protein breakdown exceeds the rate of muscle protein synthesis by
approximately 30%. Consumption of BCAAs alone (i.e., without the other EAAs) can only increase
muscle protein synthesis in the post-absorptive state by increasing the efficiency of recycling of
EAAs from protein breakdown back into protein synthesis, as opposed to either being released in
to plasma or oxidized. This is because all 9 EAAs (as well as 11 NEAAs) are required to produce
muscle protein, and EAAs cannot be produced in the body. If only 3 EAAs are consumed, as is the
case with consumption of BCAAs, then protein breakdown is the only source of the remaining
EAAs required as precursors for muscle protein synthesis. It is therefore theoretically impossible
for consumption of only BCAAs to create an anabolic state in which muscle protein synthesis
exceeds muscle protein breakdown. If the generous assumption is made that BCAA consumption
improves the efficiency of recycling of EAAs from muscle protein breakdown to muscle protein
synthesis by 50%, then this would translate to a 15% increase in the rate of muscle protein
synthesis (30% recycled in basal state X 50% improvement in recycling = 15% increase in
synthesis). Further, a 50% reduction in the release of EAAs into plasma from muscle would also
reduce the plasma and intracellular pools of free EAAs
PART A -QUESTIONS AND ANSWER SHEET
Questions 1-7
For each question, 1-7, decide which text (A, B, C or D), the information comes from.
You may use any letter more than once.
In which text can you find information about;

1 .EAAs obtained from muscle protein will undergo oxidation. Answer_________________

2 .Are BCAAs anabolic in the postabsorptive state? Answer______________

3 .Muscle protein turnover and dietary protein intake. Answer____________

4 .The rate of muscle protein synthesis will always be lesser than rate

of muscle protein breakdown. Answer_____________

5 .Addition of BCAAs can enhance protein synthesis. Answer _____________

6 .Synthesis of protein could have been curbed by presence of EAA. Answer __________

7 .When EAAs are in large quantity protein synthesis enhances. Answer ___________

Questions 8-14
Answer each of the questions, 8-14, with a word or short phrase from one of the texts.
Each answer may include words, numbers or both. Your answers should be correctly spelt.

8 .What happens with the consumption of BCAA supplements? Answer ____________

9 .What will drive anabolic state more theoretically? Answer_____________

10 .What are the limitations of Kobayashi? Answer _____________

11 .What can lead to 15% increase in the rate of muscle protein synthesis? Answer______________

12 Muscle protein synthesis often get limited by the lack of availability of? Answer______________

13 What gets stimulated with induction of BCAAs? Answer _____________

14 What is requisite for the production of muscle protein? Answer ______________


Questions 15-20

Complete each of the sentences, 15-20, with a word or short phrase from one of the texts.
Each answer may include words, numbers or both. Your answers should be correctly spelt.

15. ______________ were failed to measure the effects of BCAAs alone.

16 .EAA which is needed for muscle protein synthesis can be obtained from increased ____________

17. EAAs derived from________________ which are not added to the muscle protein will be released
into plasma.

18. Body doesn`t have the capability to produce ________________

19. When there is curtailment of the amount of EAAs into plasma by 50% and more, there will be
reduction In _________________

20. When rate of muscle protein synthesis increases or goes above the levels of muscle protein

Breakdown, ________________ may get enhanced.

END OF PART A, THIS QUESTIONS PAPER WILL BE COLLECTED


READING SUB-TEST : PART B
In this part of the test, there are six short extracts relating to the work of health professionals .
For questions 1-6, choose the answer (A, B or C) which you think fits best according to the text. Write
your answers on the separate Answer Sheet
Questions 1-6

1 .What is correct about the heartbeat sensor?


A. This signal in form of DC signal is related to the total amount of blood
B. The signal in form of AC will conjoin with the DC signal
C. The detector output is in form of electrical signal and is proportional to the heartbeat rate.

Heartbeat Sensor

The basic heartbeat sensor consists of a light emitting diode and a detector like a light detecting
resistor or a photodiode. The heartbeat pulses cause a variation in the flow of blood to different
regions of the body. When a tissue is illuminated by the light source, i.e., light emitted by the led, it
either reflects (a finger tissue) or transmits the light (earlobe). Some of the light is absorbed by the
blood and the transmitted or the reflected light is received by the light detector. The amount of
light absorbed depends on the blood volume in that tissue. The result as shown by detector based
on the electrical signal will be as per the changes in the heartbeat rate.
This signal is actually a DC signal relating to the tissues and the blood volume and the AC
component synchronous with the heartbeat and caused by pulsatile changes in arterial blood
volume is superimposed on the DC signal. Thus the major requirement is to isolate that AC
component as it is of prime importance.
2 .The notice talks about;
A. How BIS technology works?
B. BIS and Sedatives used.
C. Relation between Sedatives and BIS calculations.

BIS Technology
Raw EEG data are obtained through a sensor placed on the patient’s forehead.
The BI system processes the EEG information, and calculates a number between 0 and 100 that
provides a direct measure of the patient’s level of consciousness and response to sedation
A BIS value of 100 indicates the patient is fully awake.
A BIS value of 0 indicates the absence of brain activity.
Using BIS technology to Guide ICU Sedation Care.
Sedatives may be titrated to a variety of BIS values, depending on the goals for each patient.
Publications demonstrate that BIS values may be used as a measure of hypnotic drug effect in the
ICU. The movement may occur regardless of BIS values. Natural sleep cycles may affect the
hypnotic level.
3 .The word analogue may mean;
A. Similar in functioning.
B. Similar in structure.
C. Something that is similar to or can be used instead of something else.

Amino Acid Analogues


In recent years, it has been possible to introduce amino acid analogues into proteins by supplying
the analogue under circumstances in which the amino acid itself is not easily available. For example
in Escherichia co fluorophenylalanine has been incorporated in place of phenylalanine and tyrosine
and it has even proved possible to replace completely the sulphur-containing amino acid
methionine by its selenium analogue. Of the enzymes produced by the cell in these various ways
some were active and some were inactive, as might have been expected

4 .The notice talks about;


A. Data transformation and display.
B. Data storage and display.
C. Data surveillance and display.
Data and heartbeat rate
The 24C256 serial EEPROM, which has eight kbytes capacity, is used to store up to eight ECG
signals. At each variation within the number of heartbeats in a minute, three bytes representing
the new number and time corresponding are stored in the EEPROM. The output unit consists of a
set of LED to indicate some diseases such as bradycardia and tachycardia. It also contains a buzzer
to prevent the patient from detected problem and time to transfer data by email.
5 .Patient affected with CKD;
A. will show reduced GFR.
B. Albumin excretion in large quantity.
C. Both.

Chronic Kidney Disease


CKD is a serious health condition and a worldwide public health problem. The incidence and
prevalence of CKD are increasing in the United States and are associated with poor outcomes and a
high cost to the US healthcare system.
CKD is usually defined as a curtailment of the Glomerular filtration rate (<60 mL·min−1·1 73 m−2),
excess urinary albumin excretion (≥30 mg/d or mg/gCr), or to be more precise, it can be a
combination of both. In 2002, the National Kidney Foundation Kidney Disease Outcome Quality
Initiative defined stages of CKD according to the level of GFR and whether there was other
evidence of kidney damage (eg, presence of albuminuria) The KDIGO working group released a
2012 update recommending classification of CKD by cause, GFR, and albuminuria category. The
CKD-EPI Collaboration has developed equations to more accurately estimate GFR from serum
creatinine compared with the previously established MDRD Study equation.
6 .What is correct about ICU Sedation?
A. Fear and anxiety are common.
B. Necessitate neurological examination.
C. More than 69% of patients in an ICU were found to be inappropriately sedated.
READING SUB-TEST : PART C
In this part of the test, there are two texts about different aspects of healthcare.
For questions 7-22, choose the answer (A, B, C or D) which you think fits best according to the text.
Write your answers on the separate Answer Sheet

Text 1: Auricular hematoma

An auricular hematoma is an injury to the outer ear. This injury can occur when the outer ear is
either hit directly or receives repetitive blows. Athletes involved in any contact sport can suffer a
contusion to the ear which may result in a "cauliflower" ear (also called an auricular hematoma). It is
a deformity of the outer ear most commonly seen in wrestling, rugby, boxing, football and judo.
Cauliflower ear occurs after someone gets a blow or repeated blows to the ear, enough for a large
blood clot (lump of blood) to develop under the skin or for the ear's skin to be stripped away from
the cartilage (the flexible material that gives the ear its shape). The body normally absorbs excess
fluid or blood at an injury site over time, but not always in the ear because of its special structure.
The cartilage of the ear has no blood supply except that supplied by the ear’s skin. When the
cartilage receives little or no blood flow because of tearing of the skin, bruising or a blood clot, it
eventually dies and is replaced by scar tissue. An acute cauliflower ear is often painful and causes
swelling. If left untreated, it results in deformation of the ear which may last a lifetime.

Unfortunately, most athletes do not seek care until the bleeding and swelling have stabilized and
resulted in deformity. By not seeking medical care immediately, they increase their risk of infection,
recurrence, scarring and deformity. After a cauliflower ear has formed and hardened, it will not
recover its normal shape without surgery. But if it is caught and treated early enough, a person
usually will not get a lifelong deformity. In high school and college wrestling, the rules requirethe use
of protective headgear, but problems still occur. Not wearing headgear or wearing poorly fitting
headgear is a big factor in causing cauliflower ear.
At first, the swelling will be soft and there will be mushy fluid. It is at this early stage that immediate
treatment can help decrease or avoid permanent scarring. If the fluid is allowed to solidify, it will
cause significant permanent disfiguration.
The use of ice on the affected area is suggestive of great pain management. A head wrap should also
be applied and elastic gauze with packing material in front and behind the ear, applying moderate
pressure, can be used. This wrapping should not cause a headache, block vision, or cover the other
ear. After that, the next step is one of the following: drainage (aspiration) and compression;
drainage and splinting with various materials; or incision and drainage with clot removal.
Sometimes stitches are needed if there is a tear in the skin. Your doctor may prescribe antibiotics to
prevent an infection.

A doctor can drain the blood from the ear either with a syringe or through a cut and then help the
skin reconnect to the cartilage by applying the pressure with a tight bandage. Splinting is a medical
procedure that keeps pressure on the area of hematoma formation. Sometimes sutures through the
ear keep the special gauze in place, or sometimes special materials (pediplast or silicone) are molded
to the ear.After a splint is in place, the ear should be rechecked by your doctor after seven days.
Sutures typically stay in for 14 days, but may be removed if redness or tenderness occurs. The risk of
recurrence decreases the longer the splint stays in place. Wrestlers may be able to return to
wrestling 24 hours after splint application. This is a surgical procedure for more serious cauliflower
ears, and should only be done by an Ear, Nose and Throat surgeon (also called an ENT or
otolaryngologist) or a plastic surgeon.

Wearing sturdy headgear when you are participating in a contact sport or other sports, such as
baseball, hockey or biking, in which you might experience head trauma, is always requisite. Athletes
should take the time to make certain that their headgear is not too tight or too loose. The Nano
Hospital Sports Medicine doctors in the US emphasize that athletes can easily prevent cauliflower
ear by using effective head protection and seeking medical help at the first sign of an ear problem.
Text 1: Questions 7-14

7. According to paragraph 1, what is true about cauliflower ear?


A. Cauliflower ear results from repetitive blows to the ear
B. Cauliflower ear occurs due to clotting of the blood on the bruised area
C. Cauliflower ear occurs because the ear skin is stripped and is not able to supply oxygen to the
cartilage and surrounding area that is badly hit
D. Cauliflower ear may often lead to distortion of the shape of the ear.

8. In paragraph 1, what does the word contusion imply?


A. Being hit by the opponent
B. A bruise
C. A serious injury
D. A very painful blow

9. According to paragraph 2, which one of the following statements is not true?


A. Once infection grows, it can bring changes in the shape of the ear.
B. Athletes often do not pay attention to injuries to their ear which may later develop into
cauliflower ear.
C. The occurrence of this cauliflower ear among athletes is very common.
D. It is suggested that sports should be played using all necessary safety equipment.

10. What is head gear in passage 2?


A. Covering of the head
B. Soft helmet
C. Boxing helmet
D. A metal hat used for protection in games like rugby

11. According to paragraph 3, what is the first step towards injury treatment and management?
A. Ice should be applied on the bruised ear
B. Ice and a head wrap should be applied
C. Drainage and compression
D. Splinting
12. According to paragraph 4, in the drainage and compression method, the doctor would;
A. Remove blood from the ear.
B. Try to connect the skin with the cartilage.
C. Apply pressure on the affected area.
D. Stitch the ruptured parts.

13. Paragraph 4 talks about;


A. Three types of treatment procedures for cauliflower ear.
B. Three immediate actions are taken to prevent cauliflower ear.
C. How easy it is to treat cauliflower ear.
D. None of the above.

14. What does paragraph 5 indicate?


A. Preventive measures for cauliflower ear.
B. What experts suggest athletes should do.
C. The importance of wearing headgear for sports.
D. Advice from experts at Nano Hospital Sports Medicine in the US.
Text 2: Rosacea

Rosacea is a chronic facial skin condition characterized by marked involvement of the central face
with interim or persistent erythema, inflammatory papules or pustules, telangiectasia, or
hyperplasia of the connective tissue. Erythema, or flushing, usually lasts less than five minutes
and may spread to the neck and chest, often accompanied by a feeling of warmth. Less common
findings include erythematous plaques, scaling, edema, phymatous changes (thickening of skin
due to hyperplasia of sebaceous glands), and ocular symptoms. Rosacea can be associated with
low self-esteem, embarrassment, and diminished quality of life. In a national survey, 65% of
patients with rosacea reported symptoms of depression.The exact prevalence of rosacea in the
United States is unknown; however, it is probably between 1.3% and 2.1%, and may be as high as
5%. Women are affected more often than men, but men are more likely to have phymatous
changes, especially rhinophyma.

The National Rosacea Society Expert Committee defined four subtypes and one variant.
Granulomatous rosacea is the sole variant with firm, indurated papules or nodules. Many
dermatologists consider rosacea fulminans and perioral dermatitis as rosacea variants. Patients
may experience fluctuation in symptoms and overlap of symptoms between subtypes. The
etiology of rosacea is unknown but is likely multifactorial. Factors involved in the pathophysiology
include the dense presence of sebaceous glands on the face, the physiology of the nerve
innervation, and the vascular composition of the skin. Numerous triggers initiate or aggravate the
clinical manifestations of rosacea, including ultraviolet light, heat, spicy foods, and alcohol. A
predilection for fair-skinned individuals of Celtic or northern European descent suggests a genetic
component to rosacea; however, no specific gene has been identified. Patients with the genetic
predisposition have receptor that mediates neo-vascular regulation. When exposed to triggers,
neuropeptide release (flushing, edema) occurs, resulting in the recruitment of proinflammatory
cells to the skin.
Frequent redness (flushing) of the face is common. Most redness is at the center of the face
(forehead, nose, cheeks, and chin). There may also be a burning feelingand slight swelling. Small
red lines under the skin show up when blood vessels under the skin get larger. This area of the
skin may be somewhat swollen, warm, and red. There can be constant redness along with bumps
on the skin. Sometimes the bumps have pus inside (pimples), but not always. Solid bumps on the
skin may later become painful. In some people (mostly men), the nose becomes red, larger, and
bumpy. The skin on the forehead, chin, cheeks, or other areas can become heavier with the usual
compactness because of rosacea.

The genesis of rosacea is more confusing. As there are various symptoms and conditions
associated with it, it is difficult to track how it comes into being.
Doctors surmise rosacea happens when blood vessels expand too easily, causing flushing. People
who blush a lot may be more likely to get rosacea. It is also thought that people inherit the
likelihood of getting the disease. Though not well researched, some people say that one or more
of these factors make their rosacea worse: heat (including hot baths); vigorous exercise; sunlight;
winds; very cold temperatures; hot or spicy foods and drinks; drinking alcohol; menopause;
emotional stress; and long-term use of steroids on the face. People with rosacea and pimples
may think the pimples are caused by bacteria; but no one has found a clear link between rosacea
and bacteria. Unfortunately, there is no cure for rosacea, but it can be treated and controlled. In
time, the skin may look better. A dermatologist (a doctor who works with diseases of the skin)
often treats rosacea.
Text 2: Questions 15-22
15. In paragraph 1, the word interim may mean;

A. Severe
B. Transient
C. May last for a longer time
D. Often permanent

16. According to paragraph 1, what is not true about rosacea?

A. Rosacea is a condition in which the redness of skin may last only for a short period of time.
B. Rosacea is a condition where the patient can feel the warmth around the affected skin.
C. Rosacea is common among women living in the US.
D. Scaling is a feature that is often connected with rosacea.

17. The phrase the clinical manifestations of rosacea in paragraph 2 may suggest;

A. Subtypes of rosacea.
B. How rosacea changes itself into various other forms.
C. Features of rosacea.
D. The potentiality of rosacea to appear in multiple ways.

18. According to paragraph 2, what is true about rosacea?

A. The symptoms of rosacea vary depending on the subtypes.


B. Each and every type of rosacea shows a specific set of features.
C. It is possible that one type of rosacea may show features of other advanced types of rosacea.
D. None of the above

19. What is the central idea presented in paragraph 3?

A. How the skin reacts to rosacea


B. What does rosacea look like?
C. Symptoms of rosacea
D. A&B
20. According to paragraph 3, in rosacea;

A. Skin becomes redder


B. Skin becomes thicker
C. Skin becomes bumpy
D. All of the above

21. The word genesis in paragraph 4 may mean;

A. The beginning
B. The truth
C. The reason
D. Basis

22. Pick the correct statement as per information is given in paragraph 4;

A. Rosacea is a bacterial disease.


B. Doctors do not know how it occurs.
C. Drinking alcohol leads to rosacea.
D. Exposure to heat causes rosacea.

END OF READING TEST, THIS BOOKLET WILL BE COLLECTED


Reading test 8 : Answer Key

Part A - Answer key 1 – 7


1: C
2: D
3: B
4: C
5: A
6: A
7: B

Part A - Answer key 8 – 14


8: anabolic state enhances
9: inhibition of muscle protein breakdown
10: increase in muscle protein synthesis
11: BCCA Consumption
12: EAAs
13: muscle protein synthesis
14: 9 EAAs

Part A - Answer key 15 – 20


15: Garlick and Grant
16: plasma concentrations
17: muscle protein breakdown
18: EAAs
19: plasma
20 : muscle mass
Reading part B – answer key

1: C The detector output is in form of electrical signal and is proportional to the heartbeat rate.
2: A How BIS technology works?
3: C Something that is similar to or can be used instead of something else.
4: B Data storage and display.
5: C Both.
6: C More than 69% of patients in an ICU were found to be inappropriately sedated.

Reading test - part C – answer key

Text 1 - Answer key 7 – 14


7: D Cauliflower ear may often lead to distortion of the shape of the ear.
8: B A bruise
9: D It is suggested that sports should be played using all necessary safety equipment.
10: A Covering of the head
11: B Ice and a head wrap should be applied
12: A Remove blood from the ear.
13: B Three immediate actions are taken to prevent cauliflower ear.
14: A Preventive measures for cauliflower ear.

Text 2 - Answer key 15 – 22


15: B Transient
16: A Rosacea is a condition in which the redness of skin may last only for a
short period of time.
17: A Subtypes of rosacea.
18: C It is possible that one type of rosacea may show features of other
advanced types of rosacea.
19: B What does rosacea look like?
20: D All of the above
21: C The reason
22: B Doctors do not know how it occurs.
READING TEST 9
READING SUB-TEST : PART A
 Look at the four texts, A-D, in the separate Text Booklet.
 For each question, 1-20, look through the texts, A-D, to find the relevant information.
 Write your answers on the spaces provided in this Question Paper.
 Answer all the questions within the 15-minute time limit.
 Your answers should be correctly spelt.

TEXT BOOKLET- OMALIZUMAB

Text A

Omalizumab, a 95% humanized monoclonal antibody that binds to circulating IgE, is currently
approved for moderate to severe persistent allergic asthma and for those patients not well
controlled on combination medium doses of ICS and LABA. A boxed warning has been added, and
patients should be observed in the clinician’s office for 2 hours after each of the first three
injections and for 30 minutes after each subsequent dose, because 75% of reported cases
occurred within those periods. Patients should have access to self-injectable epinephrine and be
educated on the signs and symptoms of anaphylaxis and on the administration of self-injectable
epinephrine.
Text B

A link between omalizumab use and arterial thrombotic events reported to the FDA. Adverse Event
Reporting System has been investigated. Myocardial infarction and stroke accounted for the
majority of the cases. In light of the findings, future robust epidemiologic studies are needed to
evaluate that potential, adverse effect. Until such evidence is available, clinicians should
recommend omalizumab cautiously in patients with known factors that put them at risk of
myocardial infarction or stroke. In 2008, the FDA investigated a possible association between the
use of montelukast and lethal behavior. In 2009, the package inserts for montelukast, zafirlukast,
and zileuton were updated to include neuropsychiatric cases. A population-based cohort study of
patients exposed to one or more prescriptions for montelukast from 1998 to 2007 revealed that
among 23,500 patients, one case of suicide occurred in a 61-year-old woman. The patient had been
given one prescription for montelukast 2 years before her death, and montelukast was ruled out
as the cause.Other investigators have also been unable to link montelukast to suicide risk. When
prescribing leukotriene modifiers, clinicians are urged not to withhold warranted therapy but to
monitor patients for neuropsychiatric effects.
Text C

Bronchial thermoplasty involves the distribution of radio frequency energy into the airways by
flexible bronchoscopy to reduce airway smooth muscle mass and decrease bronchoconstriction.
The electrical energy is delivered through electrodes and is then converted to heat when it comes
in contact with tissue. Thermal energy is delivered to the airway wall in a series of three
bronchoscopies that take place 3 weeks apart: The first procedure treats the airways of the right
lower lobe; the second procedure treats the airways of the left lower lobe; and the third one
treats the airways of both upper lobes. When heat is introduced to the smooth muscle of the
airway, actin-myosin interaction is disrupted from denaturation of motor proteins, thereby
quickly inactivating muscle cells. In 2010, the bronchial thermoplasty received label approval for
use in the treatment of patients 18 years or older with severe persistent asthma not well
controlled with ICS and LABA. The FDA is requiring phase 4 postmarketing surveillance studies.
This is not currently covered by most private insurance plans. Once that barrier has been
overcome, interventional pulmonologists as well as bronchoscopes’ advanced skills, training, and
expertise will be needed for this newly approved strategy.
Text D

Similarly, the Research in Severe Asthma (RISA) study group investigated patients with severe
asthma. Bronchial thermoplasty was associated with a short term increase in morbidity (e.g.,
worsening asthma symptoms, increased rescue medication use, decreased quality of life and
asthma control) in the period immediately after treatment. The use of rescue medication
improved lung function and asthma-related quality of life scores, which remained statistically
significant up to 52 weeks after treatment. When analyzed together, the results of the AIR and
RISA trials indicate that the numbers of adverse events were similar across studies but were
worse in patients with more severe disease. Adverse events reported included wheeze,
breathlessness, chest tightness, cough, dyspnea, asthma exacerbation, and episodes of lobar
segmental collapse. However, reports did not show any deterioration in lung function over 3
years, and CT scans showed no evidence of abnormal airway structure or injury to lung tissue. In
a sham controlled trial, bronchial thermoplasty led to improvements in severe exacerbations that
would have required corticosteroids, emergency department visits, and time lost from work or
school during the time after receiving bronchial thermoplasty. Similarly, a trial assessed the
safety of bronchial thermoplasty 5 years after treatment and found that neither the rate of oral
corticosteroid usage nor the proportion of subjects requiring oral corticosteroid pulses therapy
showed any deterioration over the period in the bronchial thermoplasty group.
PART A -QUESTIONS AND ANSWER SHEET
Questions 1-7
For each question, 1-7, decide which text (A, B, C or D) the information comes from.
You may use any letter more than once.
In which text can you find information about;
1 Study conducted on medical cases. Answer _______________
2 Usage of medicine for effective results. Answer _______________
3 Emerging Therapies. Answer ______________
4 Risks associated with Anaphylaxis. Answer ______________
5 Connected to increase in disease rate. Answer _____________
6 Atherothrombotic Events. Answer ______________
7 Leukotriene Modifiers and Suicide. Answer _______________

Questions 8-14
Answer each of the questions, 8-14, with a word or short phrase from one of the texts.
Each answer may include words, numbers or both. Your answers should be correctly spelt.
8 What precautionary measure has been given with respect to usage of Omalizumab? Answer ____
9 What are increasing number of events related to? Answer ________________
10 What is more connected to self destructive nature? Answer _______________
11 When bronchial thermoplasty can be used? Answer ________________
12 Who limits clinical acceptance of thermoplasty? Answer ________________
13 What function rescue drug can perform? Answer _______________
14 What is the name of study in which bronchial thermoplasty is reported to have brought more
improvements even in diseases with worsening conditions? Answer ________________
Questions 15-20
Complete each of the sentences, 15-20, with a word or short phrase from one of the texts.
Each answer may include words, numbers or both. Your answers should be correctly spelt.

15 . Patients are needed to know more about the____and they can use the medications on their own.
16 It is recommended that use of ___shall be given to patients while study with respect to same
continues.

17 A case indicates that the patient died because of the use of _________which was administered
some 24 months ago.

18 __________will prevent protein from losing their quaternary structure, when heat is put into use.

19 Close analysis made it all clear that ____________results are more or less the same.

20 The patients who underwent______________didn`t show any worsening conditions.

END OF PART A, THIS QUESTIONS PAPER WILL BE COLLECTED


READING SUB-TEST : PART B
In this part of the test, there are six short extracts relating to the work of health professionals .
For questions 1-6, choose the answer (A, B or C) which you think fits best according to the text. Write
your answers on the separate Answer Sheet
Questions 1-6

1. The manual informs us that AED


A. A machine that uses an electric current to stop any irregular and dangerous activity of the
heart`s muscles
B. It is helpful in operating through rescue
C. It is helpful in giving shocks

The AED is a self-testing, battery-operated automated external defibrillator (AED). After applying
the AED’s electrodes (pads) to the patient’s bare chest, the AED automatically analyzes the
patient’s electrocardiogram (ECG) and advises the operator to press the button and deliver a shock
if needed. The AED uses one button and guides the operator through the rescue using a
combination of voice prompts, audible alerts, and visible indicators.
For the Power heart AED G3 Automatic, the AED automatically delivers a shock if needed.
2. What is made of a synthetic polyester with elastic properties?
A. Velcro fasteners
B. Compression cuff
C. Inflatable bladder

Baumanometer Calibrated V-Lok cuffs


These cuffs come in four sizes: child, adult, large adult, and thigh. The cuffs are used with all three
blood pressure instruments: mercury, aneroid, and OMRON.
The Calibrated® V-Lok® compression cuff is made of urethane-coated Dacron, an unyielding
material that exerts an even pressure on the inflatable bladder inside the cuff. The compression
cuffs have Velcro fasteners that adhere to them to keep the cuff in position when placed on the
arm. The cuff size is determined by the circumference of the arm. The size of the cuff and the
bladder used influences the accuracy of the blood pressure readings:- if the cuff is too narrow, the
observed blood pressure is overestimated (higher than it really is), and if it is too wide, the reading
may be underestimated (lower than it really is).
3 Pick the correct statement;
A. Cohort study proved that close members of the family get affected with the disease first.
B. Screening of family members is crucial too.
C. It is hard to define the nature of heritability with respect to NAFLD.

Several studies suggest familial clustering of NAFLD. In a retrospective cohort study, Willner et al.,
observed that 18% of patients with NASH have a similarly affected first-degree relative. In a familial
aggregation study of overweight, children with and without NAFLD, after adjusting for age, sex,
race, and BMI, the heritability of MR-measured liver fat fraction was 0.386, and fatty liver was
present in 18% of family members of children with NAFLD in the absence of elevated alanine
aminotransferase (ALT) and obesity. Data reporting the heritability of NAFLD have been highly
variable, ranging from no detectable heritability, in a large Hungarian twin cohort, to nearly
universal heritability, in a study of obese adolescents. In an ongoing, well characterized cohort of
community-dwelling twins in California, using MRI to quantify steatosis and fibrosis, both steatosis
and fibrosis correlated between monozygotic, but not dizygotic, twin pairs, and, after multivariable
adjustment, the heritability of HS and HF was 0.52 (95% CI, 0.31-0.73; P < 1.1 3 10–11) and 0.50
(95% CI, 0.28- 0.72; P < 6.1 3 10–1), respectively.
4. The notice gives more information about;
A. High Blood Pressure
B. Blood pressure and costs involved
C. Stats on patients suffering from BP
Blood Pressure
The age-adjusted prevalence of hypertension among the UK adults ≥20 years of age is estimated to
be 34.0% in NHANES 2011 to 2014, which is equivalent to 85.7 million adults.
The prevalence of high BP or borderline high BP among UK children and adolescents 8 to 17 years
old is 11%. The SPRINT (Systolic Blood Pressure Intervention Trial) demonstrated lower CVD and
mortality risk with a systolic BP target goal of 120 mmHg versus 140 mmHg. It is estimated that
16.8 million UK adults meet the SPRINT eligibility criteria. The prevalence of apparent treatment-
resistant hypertension was estimated from a meta-analysis to be 13.7%.

5. Pick the right condition in which making a bigger cut on the abdomen becomes essential;
A. When the doctors feel operation can`t be performed safely with keyhole surgery.
B. When the size of spleen is large.
C. When the bleeding is uncontrollable.
Conversion to an open operation via a larger incision is not considered a failure in keyhole surgery.
Sometimes, the surgeon will consider it necessary to make a bigger cut on the abdomen to finish
the operation. Profusely bleeding is reported to be a major cause. The size of the spleen, too small
or too large will have its role to play. Surgeon feels that they cannot complete the operation safely
with keyhole surgery. Keyhole surgery can also be more difficult if there has been previous surgery.
This is another common reason to convert to an open operation. This is considered sound
judgment. An open operation involves a slightly longer recovery period.
6. What is right about complications?
A. Mortality is higher in case patients without spleen.
B. Bleeding may occur after operation.
C. Pancreatic complications may occur more easily than other complications.

Complications:
Local complications:
Bleeding (2-6%)
Venous thromboembolism: Splenic/ Portal thrombosis (5-20%)
Pancreatitis, fistulas: 3%
General complications:
Pulmonary atelectasis: 4%
Long term complications:
Severe sepsis (encapsulated bacterias): 0,2-0,5%
Mortality: 0,6%
READING SUB-TEST : PART C
In this part of the test, there are two texts about different aspects of healthcare.
For questions 7-22, choose the answer (A, B, C or D) which you think fits best according to the text.
Write your answers on the separate Answer Sheet

Text1: Age-related Macular Degeneration


Age-related Macular Degeneration is an acquired retinal disorder with farreaching psychosocial and
economic implications. As the leading cause of legal blindness (visual acuity of 20/200 or worse) for
persons over age 65 in the United States, it accounts for 14 percent of new legal blindness, with
16,000 cases reported annually. AMD is the leading cause of severe vision loss in persons over
age 50 and it is second only to diabetes as the leading cause of blindness in the 45 to 64 year-old
age group. "Severe vision loss" is categorized as visual acuity of 20/200 or worse. "Significant vision
loss" refers to a loss of visual function that interferes with customary or required activities or
lifestyle, usually at a level approximating 20/50-20/70 or worse.

The number of Americans over age 65 will more than double between the years 1990 and 2020.
Because age is a significant risk factor for the development of AMD, timely access to eye care may
have preventive value. Many older Americans neither seek nor have access to regular eye care; thus
the risk for vision loss in this population is unnecessarily high if AMD is not diagnosed promptly.
Eighty percent of the anticipated 2 million Americans who will be residing in nursing facilities by the
year 2000 will be over age 75. The number of Americans needing long-term care is projected to
increase from 4 million to 18 million by the year 2040 (But if the government takes steps then this
scenario may get altered). Their access to care may be limited in certain settings, especially
extended care facilities. Without timely diagnosis and treatment, loss of vision in these
environments cannot be prevented. The onset of AMD can certainly be dangerous and may prove to
be detrimental insidiously. Coupled with environmental and lifestyle factors, which may play
secondary, but important roles in the development of the disease, the nature of AMD makes
patient education, early detection, and referral critical for high risk patients.
Age-related macular degeneration is an acquired retinal disorder, (the use of avant-garde
technologies may or may not offer best results), which is characterized by any of the following
fundus changes: pigmentary atrophy and degeneration, drusen and lipofuscin deposits, and
exudative elevation of the outer retinal complex in the macular area. AMD, which usually occurs in
patients over age 55, results in progressive, sometimes significant, irreversible loss of central visual
function from either fibrous scarring or diffuse, geographic atrophy of the macula. The definition
can be expanded to include extrafoveal lesions that would have an impact on vision if superimposed
on the foveal region.

Nonexudative (dry or atrophic) AMD accounts for 90 percent of all patients with AMD in the United
States. The disorder results from a gradual breakdown of the retinal pigment epithelium (RPE), the
accumulation of drusen deposits, and loss of function of the overlying photoreceptors. Most
patients with nonexudative AMD experience gradual, progressive loss of central visual function. This
loss of vision is more noticeable during near tasks, especially in the early stages of the disease.
In an estimated 12-21 percent of patients, nonexudative AMD progresses to cause vision levels of
20/200 or worse. Both choroidal neovascularization (CNV) and subretinal or sub-RPE exudation are
conspicuously absent in this category of AMD.

Although exudative (wet) AMD accounts for only 10 percent of patients with AMD, 90 percent of
the AMD patients with significant vision loss have this form of the disease. Exudative AMD is
characterized by the development of neovascularization in the choroid, leading to serious or
hemorrhagic leakage and subsequent elevation of the RPE or neurosensory retina. Patients with
exudative AMD tend to notice a more profound and rapid decrease in central visual function.
Serous or hemorrhagic leakage from the new choroidal vessels causes dysmorphopsia, scotoma,
and blurred vision. In most patients, nonexudative AMD will not progress to severe vision loss.
Those patients in whom AMD progresses to the exudative form are at greatest risk for severe visual
impairment. Patients who have exudative maculopathy with drusen in the fellow eye are at
significant risk of developing CNV.
Text 1: Questions 7-14

7. According to paragraph 1, age-related macular degeneration;


A. Has a direct influence on the financial condition of the US.
B. Is a disorder which affects the thinking and behavior of the people.
C. Is an entirely new legal blindness in the US.
D. Affects people who are over 50 years of age.

8. According to paragraph 1, AMD is;


A. One of the common causes of legal blindness.
B. A disorder which affects only elderly people.
C. Second to diabetes in the list of diseases affecting people in the US.
D. The second largest disease affecting people between 45 and 64 years of age.

9. According to paragraph 2, we can infer that AMD will;


A. Continue to affect people of the older generation.
B. Decrease if the government takes necessary steps.
C. Become common because access to eye care services in the US is still an issue.
D. Continue to increase at least by 40%.

10. According to paragraph 2, which one of the following statements is true?


A. AMD cases are fast growing in the US.
B. People who may need eye care services will increase to 18 million in the future.
C. The older generation do not care for their eye vision.
D. AMD can secretly harm people in various ways.
11. According to paragraph 3, AMD;
A. Can be treated easily
B. Moves to more advanced stages at a rapid speed in people over 55 years of age.
C. Destroys the functions of the retina over time.
D. Patients may not gain vision if once lost.

12. In paragraph 3, the word avant-garde may mean;


A. Modern
B. Experimental
C. Proven and tested
D. Traditional

13 According to paragraph 4, what is right about Nonexudative AMD?


A. It is a common cause of AMD.
B. It slowly affects the retina.
C. Most of the symptoms associated with AMD can be observed in the early stages.
D. It is free from CNV and sub-RPE.

14 According to paragraph 4, which one of the following statements is not true?


A. Nonexudative AMD may cause 20/200 vision levels.
B. Nonexudative AMD begins to affect central functions over time.
C. Accumulation of drusen deposits occurs after loss of central functions.
D. The loss of vision is a slow process.
Text 2: Raynaud Phenomenon

The Raynaud phenomenon is the exaggeration of the normal response to cold temperatures.
The clinical manifestation of the Raynaud phenomenon is caused by vasoconstriction (narrowing)
of blood vessels (arteries and arterioles) that results in reduced blood flow to the skin (ischemia),
while cyanosis (blue skin) is created by deoxygenation of slow-flowing blood in small blood vessels
(arterioles and capillaries) in the skin. The skin feels cold and appears as a pale demarcated area
(white fingers or toes) or cyanotic skin limited to the fingers or toes. Some people will feel
generally cold and have mottled pale skin of the ears, nose, facial area, knees, or other exposed
skin. A Raynaud event typically starts after cold exposure or an emotionally stressful situation in
one or several digits and then spreads symmetrically to all fingers of both hands. It is common for
numbness, tingling and clumsiness of finger use to accompany the digital color changes.

While studies of selected patients find that as many as 15 to 20 percent of young women have the
Raynaud phenomenon, population-based surveys in various ethnic groups find the prevalence to
be approximately 3 to 5 percent. Geographic variation in the prevalence of the Raynaud
phenomenon is influenced by the region`s climate. There is also good evidence that the frequency
and severity of the attacks is influenced by the daily ambient temperature with significant
variation during the winter and summer months. Often people living in such environments are
susceptible to this condition. Primary Raynaud phenomenon is used to denote a patient without
an associated underlying disease. Most of the individuals with the Raynaud phenomenon have
uncomplicated primary Raynaud phenomenon without any defined cause or associated systemic
disease. Recent studies found that about 30 percent of people with primary Raynaud phenomenon
have a first-degree relative with the same condition. This suggests there is a genetic trait
associated with the Raynaud phenomenon, but to date no gene or gene defect has been defined.

Often, disruption in the normal regulation and responses of thermoregulatory blood vessels in the
skin result in RP. These normal blood vessels have a complex system of control that begins with
sensory nerves in the skin. These nerves sense the ambient temperature and relay this information
to the central nervous system. The brain then sends a signal through the sympathetic nervous
system to skin blood vessels to constrict if it is cold and dilate if it is warm. Studies suggest that in
patients with the Raynaud phenomenon, the sympathetic receptors (alpha 2C) are overactive or
overexpressed in the smooth muscle of the thermoregulatory arteries, and thus cause exaggerated
responses to cold temperatures. Studies also implicate a number of other mechanisms for causing
or aggravating abnormal vascular responses in individuals with the Raynaud phenomenon. These
include abnormal release of vasoconstricting molecules (e.g., endothelin-1) or the
underproduction of vasodilators (e.g., prostacyclin or nitric oxide) from the lining of the vessel
itself. Non-drug Therapy Treatment begins by educating the patient about the causes of the
Raynaud attacks, and methods to avoid the common provoking and abstrusing factors. The
avoidance of cold temperatures is the best method to prevent an episode of the Raynaud
phenomenon. Warming the whole body with loose fitting clothing, stockings, vests, headwear, and
gloves is a key strategy. Contact with cold objects such as iced beverage containers or a cold
steering wheel should be avoided by covering these objects or wearing warm gloves. Chemical
warmers placed in the pockets or gloves can be most helpful. Avoiding trauma to the fingers or
toes is also helpful. Emotional stress alone can trigger digital vasospasm and anxiety - feeling
nervous, tense, or worried - and can exacerbate cold induced Raynaud attacks. Therapies designed
to truncate emotional stress are helpful. Temperature biofeedback is used in combination with
different relaxation techniques to treat Raynaud patients.
Text 2: Questions 15-22
15. According to paragraph 1, what happens in the Raynaud phenomenon?
A. The blood-flow to the skin is blocked.
B. The skin becomes pale.
C. The skin becomes cold.
D. Blood vessels narrow more dramatically.

16. The word mottled in paragraph 1 may mean;


A. An area with no same pattern.
B. Dotted with different colors.
C. Painful
D. Extremely white

17. The most suitable heading for paragraph 2 is;


A. How do people get the Raynaud phenomenon?
B. Who gets the Raynaud phenomenon?
C. How common is the Raynaud phenomenon?
D. What may affect the Raynaud phenomenon?

18. According to paragraph 2, Primary Raynaud Phenomenon;


A. Doesn`t show symptoms.
B. Is not linked to systemic conditions.
C. Is a genetic disorder.
D. Is more influenced by temperature.
19. Pick the right heading for paragraph 3;
A. How RP occurs
B. What leads to the Raynaud phenomenon?
C. The role of blood vessels in creating RP
D. A and C

20. Which word in the paragraph 4 may mean to make something worse?
A. Provoking
B. Abstrusing
C. Exacerbate
D. Truncate

21. According to paragraph 4, the best recommendation to prevent oneself from RP is;
A. Keep yourself warm.
B. Do not come in to contact with cold objects.
C. Keeping chemical warmers in your pockets.
D. Living in areas where the temperature is moderate.

22. In the last paragraph, it is indicated that;


A. Stress can lead to sudden constriction of a blood vessels.
B. Some optional treatments are known to be very effective in curtailing down the stress level.
C. There are various techniques that can be used in combination to treat patients.
D. A and B

END OF READING TEST, THIS BOOKLET WILL BE COLLECTED


Reading test 9 : Answer Key

Part A - Answer key 1 – 7


1 .B
2 .D
3 .C
4A
5D
6B
7B

Part A - Answer key 8 – 14


8. 2 hour observation
9 .Myocardial infarction
10. montelukast
11. not controlled by ICS / LABA
12. Private insurance plans
13. improve lung function
14. sham-controlled trial

Part A - Answer key 15 – 20


15 . signs of anaphylaxis
16 .omalizumab
17. montelukast
18.Actin-myosin interaction
19 .AIR and RISA
20.oral corticosteroid pulses
Reading part B: - test 9 – answer key
1. It is helpful in giving shocks
2. B Compression cuff
3 .B Screening of family members is crucial too.
4. C Stats on patients suffering from BP
5. C When the bleeding is uncontrollable.
6 .B Bleeding may occur after operation.

Reading test - part C – answer key

Text 1 - Answer key 7 – 14


7 .D Affects people who are over 50 years of age.
8. C Second to diabetes in the list of diseases affecting people in the US.
9 .A Continue to affect people of the older generation.
10. D AMD can secretly harm people in various ways.
11 ..Correct Answer Is: D Patients may not gain vision if once lost.
12 .A Modern
13 .D It is free from CNV and sub-RPE.
14 ..C Accumulation of drusen deposits occurs after loss of central functions.

Text 2 - Answer key 15 – 22


15. C The skin becomes cold.
16 .A An area with no same pattern.
17. B Who gets the Raynaud phenomenon?
18 .D Is more influenced by temperature.
19 .A How RP occurs
20 C Exacerbate
21 .D Living in areas where the temperature is moderate.
22. A Stress can lead to sudden constriction of a blood vessels.
READING TEST 10

READING SUB-TEST : PART A


 Look at the four texts, A-D, in the separate Text Booklet.
 For each question, 1-20, look through the texts, A-D, to find the relevant information.
 Write your answers on the spaces provided in this Question Paper.
 Answer all the questions within the 15-minute time limit.
 Your answers should be correctly spelt.

TEXT BOOKLET- PROSTATE CANCER

Text A
Prostate cancer is the second most incident cancer among the male population
worldwide. It is the second leading cause of cancer death in American men. There
is no exact statistics on prostate cancer prevalence in the subcontinent; however,
an estimation of 5 per 100,000 and 9 per 100,000 has been reported by two
investigators. A possible reason for this lower incidence comparing with many
other countries is that there are no national programs for the screening of prostate
cancer in Iran. Radiation therapy by itself or along with surgery and hormone
therapy are the main treatments for prostate cancer. However ionizing radiation
can also have a harmful effect on healthy body tissues. Patients with prostate
cancer, who accede to radiation therapy usually experience some degrees of sexual
dysfunction, gastrointestinal disorders and urinary tract problems.These toxicities
are known to be dose-limiting, and because higher radiation doses for patients with
clinically localized prostate cancer are now considered standard of care, finding
ways to diminutive symptoms burden is crucial.
Text B

Studies Conducted
Recently some in vitro and in vivo studies showed radiosensitizing and
radioprotective effects by some phytochemicals. One of these phytochemicals is curcumin.
It has been reported to protect various study systems, in vitro and in vivo, against the
deleterious effects induced by ionizing radiation and to enhance the effect of radiation.
Therefore, curcumin has the potential to be very useful during radiotherapy of prostate
cancer.
Between March 2011 and March 2013, all patients recently diagnosed with localized
prostate cancer at the Department of Oncology at Besat Hospital were assessed for
eligibility. Patients referred to local curative radiotherapy with external beam radiotherapy
(EBRT), in combination with androgen ablation (hormone), were invited to participate in the
study. Adenocarcinoma of the prostate must be histologically confirmed on biopsy. All
patients were with a life expectancy greater than 5 years. No metastatic disease must be
detected during physical examination, standard radiography, bone scan, and magnetic
resonance spectroscopy (MRS). Additional inclusion criteria were no prior hormone therapy,
radiotherapy or systemic treatment for prostate cancer and no other malignancy.
The exclusion criteria were clinical stage T3 or T4, Gleason score ≥ 8, serum PSA ≥ 20 ng/mL,
other prior surgery for prostate cancer, concurrent participation in another clinical trial
which would require approval upon entry to this trial, gastrointestinal disorders such as
inflammatory bowel disease, reflux and peptic ulcers and any adverse reaction to curcumin.
Text C

The European Organization for Research and Treatment of Cancer (EORTC)

Prostate cancer-specific quality of life questionnaire module (QLQ-PR25) was used


to assess urinary, sexual, and bowel function. Reliability (r=0.85) and construct
validity (Cronbach alpha >0.7) had been determined in pilot study with 41 patients.
The test-retest reliability was satisfactory (Pearson r=0.61 to 0.93 for individual items and
0.85 for total score). The research team, several university faculty members, and a physician
reviewed the pilot questionnaire to establish face validity and trustworthiness. The
questionnaire was revised appropriately based on their feedback.
Items were combined into several scales (from 1 to 100) according to EORTC rules.
For the global health and function scales, a high score signaled a better QoL; for the
symptom scales, a high score was indicative of a poorer QoL.
Text D
Curcumin Supplementation
Another issue which makes curcumin supplementation during radiation therapy an
interesting possibility is the fact that any exposure to radiation raises concern about the
possibility of a second malignancy. A population-based study of 17,845 prostate cancer
patients, compared with secondary cancer incidence in patients treated with radical
prostatectomy and radiotherapy. The results showed that the risk of rectal cancer, bladder
cancer and lung cancer in patients treated with radiotherapy are significantly higher
compared with those who underwent radical prostatectomy. Curcumin is a potent
anticancer agent which suppresses all 3 stages of carcinogenesis: initiation, promotion, and
progression and can be a promising anti-cancer treatment. There is a common concern that
using antioxidants during radiotherapy may reduce the efficacy of treatment by protecting
tumor cells from radiation-induced cell death. Despite controversies about the impacts of
antioxidant supplementation during radiation therapy on its outcomes, some recent studies
have reported promising effects.
In the present study MRI/MRS results and PSA levels of patients in CG and PG was not
significantly differed, 3 months after intervention completion (0.12 ± 0.17 and 0.13 ± 0.06,
respectively). Thus it seems that at least in short-term, curcumin does not have any
unfavorable effect on treatment outcomes. Several in vitro studies have also proposed a
radiosensitizing agent for curcumin by inhibition of radiation-induced elevation of growth
factors, cytokines, cyclins, nuclear factor- κB and tumor necrosis factor-α on tumor cells and
probably these mechanisms are blunting the radioprotective effects of curcumin in tumor
cells. The study of molecular mechanisms of radioprotective effect of curcumin on normal
tissues and its radiosensitization on tumor cells is on its way by the research team.
PART A -QUESTIONS AND ANSWER SHEET

Questions 1-7

For each question, 1-7, decide which text (A, B, C or D) the information comes from.
You may use any letter more than once.

In which text can you find information about;

1. More often doesn’t create much effect on treatment result. Answer _____________
2. Talking about debarring categories. Answer ______________
3. The effectiveness of the curing agent. Answer ______________
4. It is requisite to decrease the severity of symptoms. Answer _______________
5. Poll for the purpose of identification of the disease conditions. Answer ______________
6. Patients after treatment might face other health problems. Answer ________________
7. Use of various biologically active compounds for treatment. Answer _______________

Questions 8-14
Answer each of the questions, 8-14, with a word or short phrase from one of the texts.
Each answer may include words, numbers or both. Your answers should be correctly spelt.

8. What is regarded as the best option for treating cancer? Answer _____________
9. What is the term that describes the side effects of a drug or other treatment that are serious
enough to prevent the level of treatment? Answer _____________
10. What is the possible outcome of an exposure to radiation? Answer ______________
11. What protects the tumor cells from radiation-induced cell death during radiotherapy? Answer ___
12. What a ‘high score’ may indicate? Answer ________________
13. Which clinical stages were not included with respect to disease study or treatment? Answer _
14. Has the research on effectiveness of curcumin been completed? Answer _______________
Questions 15-20

Complete each of the sentences, 15-20, with a word or short phrase from one of the texts.
Each answer may include words, numbers or both. Your answers should be correctly spelt.

15. A high score with respect to symptom scales is suggestive of a ____________

16. ___________ is reported to create more detrimental effects on patient’s health.

17. ___________ is a malignant tumour formed from glandular structures in epithelial tissue.

18. Radiation therapy with _____________ are regarded to be the best option when it comes to
treatment of cancer.

19. _____________ is known to have the potential to create side effects when used for long period.

20. ____________ has the power to make cancer cells more vulnerable to radiation therapy.

END OF PART A, THIS QUESTIONS PAPER WILL BE COLLECTED


READING SUB-TEST : PART B
In this part of the test, there are six short extracts relating to the work of health professionals .
For questions 1-6, choose the answer (A, B or C) which you think fits best according to the text. Write
your answers on the separate Answer Sheet
Questions 1-6

1. The given notice talks about;


A. Health campaigns carried out in the US;
B. Health report;
C. Current state of cardiovascular health in the United States.

Health Campaign US
The AHA developed a Health Campaign for Life’s Simple, which emphasizes that adults and young
people can live healthier lives by avoiding smoking and tobacco products, engaging in daily PA,
eating a healthy diet, maintaining a healthy weight, and keeping cholesterol, BP, and glucose at
healthy levels. New highlights, from the cardiovascular health section include the following: A
recent meta-analysis of 9 prospective cohort studies involving US 12878 participants contributed
new estimates of the importance of cardiovascular health metrics and risk for clinical events. The
meta-analysis showed that achieving the greatest ideal cardiovascular health metrics was
associated with a lower risk of stroke (relative risk, 0.31; 95% confidence interval [CI], 0.25–0.38),
CVD (relative risk, 0.20; 95% CI, 0.11–0.37), cardiovascular mortality (relative risk, 0.25; 95% CI,
0.10– 0.63), and all-cause mortality (relative risk, 0.55; 95% CI, 0.37–0.80).
2. The following notice provides information about;
A. Tobacco as life killer.
B. Smoking and tobacco use.
C. Effects of tobacco on health of the US citizens.
Tobacco In US
In 2015, among adults ≥18 years of age, overall rates of tobacco use were estimated to be 15.2%
(16.7% of males and 13.7% of females; National Health Interview Survey).
In the US, substantially higher tobacco use rates are found in low socioeconomic status, Native
American, or transgender people reporting disability or activity limitations, as well as mentally ill
populations. There also is substantial regional variation in the percentage of current smokers.

3. As per Federal PA Guidelines;


A. More number of people have got accustomed to exercising.
B. Physical inactivity has gone down.
C. Physical activity is known to curtail down death rate.
Federal PA guidelines
The age-adjusted percentage of US adults (≥18 years) who met both the muscle-strengthening and
aerobic guidelines increased from 14.3% in 1998 to 21.6% in 2015.
The percentage of US adults who met the aerobic guideline increased from 40.0% in 1998 to 49.8%
in 2015. In 2015, only 27.1% of high school students met activity recommendations of ≥60 minutes
of PA on all 7 days of the week, and 14.3% of high school students reported that they were
inactive on all of the previous 7 days.PA (up to 75 minutes of brisk walking per week) were
associated with reduced risk of mortality compared with participants who engaged in no PA. A
study of American adults reported that inadequate levels of aerobic PA (after adjustment for body
mass index) were associated with an estimated 11.1% of aggregate healthcare expenditures.
4. The manual talks about;
A. Safety procedures.
B. Health hazards of mercury.
C. Process of managing spilled or leaking mercury.
Common Occurrence - Management
If you discover a mercury spill, you should not attempt to clean up for yourself.
Do not touch mercury with your bare hands or attempt to vacuum or clean up the spill. Leave the
area in which there was a spill, taking the SP to a separate waiting area. Close the door to the area
or room if possible. Immediately notify the MEC manager and NCHS staff; the MEC physician is
trained in mercury spill procedures, and will clean the spill. Obtain replacement equipment.

5. The table talks about;


A. Studies conducted on NAFLD.
B. What is used in clinical trials?
C. Noninvasive quantification of hepatic steatosis in NAFLD.
What studies suggest?
Some studies suggest that degree of steatosis may predict the severity of histological features
(e.g., ballooning and SH) and the incidence and prevalence of diabetes in patients with NAFLD. MR
imaging, either by spectroscopy or by proton density fat fraction, is an excellent noninvasive
modality for quantifying HS and is being widely used in NAFLD clinical trials. The use of TE to
obtain continuous attenuation parameters is a promising tool for quantifying hepatic fat in an
ambulatory setting. However, the utility of noninvasively quantifying HS in patients with NAFLD in
routine clinical care is limited.
6. What is correct?
A. Obesity rate increased from 2011-2012
B. Obesity rate increased from 2013-2014
C. Obesity rate increased from 2003-2004

Overweight and Obesity


The prevalence of obesity among adults and youth in the UK significantly remained uncurtailed
from 1999 to 2000 through 2013 to 2014. However, rate of enhancement ratio with respect to
obesity prevalence began to level off and was not statistically significant for adults from the time
period 2003 to 2004 through 2011 to 2012 and for youth from the time period 2003 to 2004
through 2013 to 2014.
Body mass index and waist circumference cut points in UK guidelines underestimate obesity and
CVD risk in Asian and South Asian populations.
Definitions of “metabolically healthy obesity” vary, and over time, a substantial proportion of
those with metabolically healthy obesity transition to metabolically unhealthy.
READING SUB-TEST : PART C
In this part of the test, there are two texts about different aspects of healthcare.
For questions 7-22, choose the answer (A, B, C or D) which you think fits best according to the text.
Write your answers on the separate Answer Sheet

Text 1: Exercise and reduction in coronary risks

There are a variety of mechanisms which may account for the protective effects of physical activity
in reducing the risk of heart disease. First, physical activity can positively affect other major
coronary risk factors. Exercise has been found tobe useful in increasing high density lipoprotein
cholesterol (HDLC), controlling mild and moderate hypertension, decreasing the risk of diabetes
and reducing excess body weight. Higher levels of HDL-C are associated with lower coronary risk.
Recent studies have reported significant increases in HDL-C (between 5% and 15%) following
aerobic exercise training. These positive changes seem to be directly related to both the intensity
of exercise and the total weekly energy expenditure. The benefits predominantly occur in
individuals who expend at least 1,000 calories per week performing moderate to vigorous exercise.

High blood pressure increases coronary risk. Physical activity lowers blood pressure in individuals
who are usually not being burdened down with augmented or piled up hypertension issues. A
recent review of 25 long-term studies concluded that aerobic exercise training leads to reductions
in systolic and diastolic blood pressure, averaging 10.8 mmHg and 8.2 mmHg respectively. These
benefits are just as great, if not more so, with moderate intensity exercise as with high intensity
exercise.
Regular physical activity helps manage and even prevents non-insulin-dependent diabetes
mellitus. Beneficial effects from regular exercise include an increased sensitivity of cells to insulin,
reduced glucose production by the liver, and an increase in muscle cells, which pound for pound
use more glucose than fat cells. In addition, a recent study of the University of Pennsylvania
suggests that exercise can decrease the risk of developing non-insulin dependent diabetes
mellitus.
Excess weight increases coronary risk. Regular exercise increases daily caloric expenditure. In one
study comparing dieting and exercise in overweight sedentary men, both interventions resulted in
modest weight loss. In contrast to the dieting group, the exercise group did not have any loss of
lean (i.e., muscle) weight. This preservation of lean body mass appears to be of significant benefit
in patients attempting to lose weight since muscle tissue consumes calories, whereas fat tissue
does not. The best approach to weight loss, therefore, appears to be a combination of moderate
daily exercise (such as walking 4 km) and modest decreases in daily caloric intake (250 calories),
which should lead to a weight loss of approximately one-half of a kilogram each week.

Although the exercise literature gives overwhelming support to the benefits of exercise, there is a
darker side of it too, such as instances of people getting muscle and joint injuries. Sudden cardiac
deaths are common too. In one study of male joggers, it was found that there was only one death
per year among 15,620 seemingly healthy individuals. The rate of muscle and joint injuries among
people who exercise on a recreational basis also is not very high. One study estimated that injuries
requiring medical care occur at an annual rate of less than 5%. Therefore, exercise programs
should be initiated gradually and supervised properly.
For the typical inactive individual, there are a number of steps that should be followed to minimize
the risks of exercise. The first step is a medical checkup. An exercise stress test may be necessary if
the patient appears to be at increased coronary risk. Guidelines developed by the American
College of Sports Medicine state that seemingly healthy individuals can usually begin moderate
exercise programs (i.e., activities within the patient's current capacity that can be sustained
comfortably for a prolonged period), without exercise testing, as long as the patient starts
gradually and is alert to the development of unusual signs or symptoms. Men over the age of 40,
and women over the age of 50, should have a maximal exercise test before beginning a vigorous
exercise program (i.e., activities intense enough to represent a substantial challenge and result in
significant increases in heart rate and respiration).

The exercise program should be tailored to a patient's needs and should also be designed to
promote long-term compliance. The initial exercise program should be enjoyable and it should not
be painful or unduly stressful. The proposed program must also be realistic in terms of the
patient's current fitness level, lifestyle and time commitment. Accordingly, a successful exercise
program should be flexible, easily accessible and not too expensive. For example, it is not realistic
to recommend swimming for someone who is a weak swimmer or has limited access to a pool.
Only programs which are crafted as per needs can provide immense benefits for the patients.
Text 1: Questions 7-14

7. According to paragraph 1, what is true about exercise?

A. Exercise can reduce virtually all the risk levels associated with coronary diseases and others.
B. Exercise may always enhance levels of HDLC.
C. Energy expenditure is always helpful in fighting diseases.
D. Aerobic exercises can increase HDLC levels effectively.

8. According to paragraph 2, exercise is good for;

A. People with high BP


B. People with low BP
C. People with mild BP
D. Not given

9. According to paragraph 3, exercise or any physical activity is reported to;

A. Enhance quality of life for diabetic patients.


B. To improve cell™s functionality to absorb insulin.
C. Can stop the occurrence of diabetes.
D. A&B

10. According to paragraph 4, which one of the following statements is true?

A. Dieting can be one of the best solutions to reduce weight.


B. Exercise may decrease muscle weight.
C. Eating less calories and rigorous exercise on a daily basis can lead to a reduction in weight.
D. Walking 4 km is recommended to lose weight.
11. What does the fifth paragraph indicate?

A. Risks associated with exercise


B. Injuries and exercise
C. Why exercise regimes should be
D. supervised Not given

12. What does the sixth paragraph describe?

A. How to gain more from exercising.


B. Why an exercise regime should be followed.
C. Exercise evaluation.
D. The importance of an exercise test.

13. According to paragraph 6, it is recommended that;

A. People should exercise under the supervision of an expert trainer.


B. Aged people should not perform vigorous exercise.
C. Any exercise program can be initiated without exercise testing.
D. A&B

14. What information do we find in paragraph 7?

A. Customized training programs.


B. How exercise training programs should be.
C. The importance of customizing exercise training programs.
D. Which training programs can be helpful for patients
Text 2: Cerebral Aneurysm

An aneurysm is a weak area in a blood vessel that usually enlarges. It’s often described as a
“ballooning” of the blood vessel. About 1.5 to 5 percent of the general population has or will
develop a cerebral aneurysm. About 3 to 5 million people in the United States have cerebral
aneurysms, but most are not producing any symptoms. Between 0.5 and 3 percent of people with a
brain aneurysm may suffer from bleeding.

People usually develop aneurysms after the age of 40. As they are, in many cases, said to be born
with such fate. Aneurysms usually develop at branching points of arteries and are caused by
constant pressure from blood flow. They often enlarge slowly and become weaker as they grow, just
as a balloon becomes weaker as it stretches. Aneurysms may be associated with other types of
blood vessel disorders, such as fibromuscular dysplasia, cerebral arteritis or arterial dissection, but
these are very unusual. They may run in families, but people are rarely born with a predisposition for
aneurysms. Some aneurysms are due to infections, drugs such as amphetamines and cocaine that
damage the brain’s blood vessels, or direct brain trauma from an accident.

An aneurysm is usually located along the major arteries deep within brain structures. When
approaching an aneurysm during surgery, normal brain tissue must be carefully spread apart to
expose it. Aneurysms can occur in the front part of the brain (anterior circulation) or the back part of
the brain (posterior circulation). Special imaging tests can detect a brain aneurysm. Two noninvasive
tests show the blood vessels in the brain. In the first test, called CTA (computed tomographic
angiography), patients are placed on a table that slides into a CT scanner. A special contrast material
(dye) is injected into a vein, and images are taken of the blood vessels to look for abnormalities such
as an aneurysm. In the second test, called MRA (magnetic resonance angiography), patients are
placed on a table that slides into a magnetic resonance scanner, and the blood vessels are imaged to
detect a cerebral aneurysm. Both of these screening tests detect most cerebral aneurysms larger
than 3–5 mm (about 3/16 inch). There is also another test called a diagnostic cerebral angiogram,
which gives more accurate results.
Bleeding is often a much-talked-about complication in aneurysms. High blood pressure is the leading
cause of subarachnoid hemorrhage. Heavy lifting or straining can cause pressure to rise in the brain
and may lead to an aneurysm rupture. (ii) Strong emotions, such as being upset or angry, can
raise blood pressure and can subsequently cause aneurysms to rupture.

Many factors determine whether an aneurysm is likely to bleed. These include the size, shape and
location of the aneurysm and symptoms that it causes. Smaller aneurysms that are uniform in size
may be less likely to bleed than larger, irregularly shaped ones. Once an aneurysm has bled, there’s
a very high chance of re-bleeding. That’s why we recommend treatment as soon as possible. On
rupturing, it leaks blood into the space around the brain. This is called a “subarachnoid
hemorrhage.” Depending on the amount of blood, it can produce a sudden severe headache that
can last from several hours to days.

Once an aneurysm bleeds, the chance of death is 30 to 40 percent and the chance of moderate to
severe brain damage is 20 to 35 percent, even if the aneurysm is treated. Fifteen to 30 percent of
patients have only mild difficulties or almost none. If the aneurysm isn’t treated quickly enough,
another bleed may occur from the already ruptured aneurysm. In 15 to 20 percent of patients,
vasospasm (irritation by the leaked blood causing narrowing of the blood vessels) may occur. This
can lead to further brain damage.

After blood enters the brain and the space around it, direct damage to the brain tissue and brain
function results. The amount of damage is usually related to the amount of blood. Damage is due to
the increased pressure and swelling from bleeding directly into the brain tissue, or from local cellular
damage to brain tissue from irritation of blood in the space between the brain and the skull. Blood
can also irritate and damage the normal blood vessels and cause vasospasm (constriction). This can
interrupt normal blood flow to the healthy brain tissue and can cause even more brain damage. This
is called an “ischemic stroke.”
Text 2: Questions 15-22
15. What paragraph 1 talks about?

A. What is an aneurysm?
B. Aneurysms in the US.
C. Prevalence of aneurysms.
D. What is an aneurysm and how does it affect someone?

16. Paragraph 2 deals with?

A. ng aneurysms.
B. How do aneurysms form?
C. When do aneurysms develop?

17. According to paragraph 2, which one of the following statements is not true?

A. Often, people are born with aneurysms, but it is developed after the age of 40.
B. Aneurysms usually develop in people who are 40 years of age or more.
C. Aneurysms may pass on from one family to another family.
D. Cocaine may develop aneurysms in people.

18. According to paragraph 3, which of the following is not correct about aneurysm testing?

A. Pictures taken during the test are the primary source of the examination of aneurysms.
B. Tests mentioned are efficient in detecting aneurysms larger than 3 to 5 mm.
C. An aneurysm is hard to detect if it is less than 3 mm.
D. A cerebral angiogram can be as effective as an MRA.

19. What is the central idea of the paragraph 4?


A. How do aneurysms occur?
B. What causes an aneurysm to bleed?
C. Bleeding in aneurysms
D. How does an aneurysm break?
20. What does paragraph 5 indicate?

A. Factors that lead to bleeding


B. What happens if an aneurysm bleeds?
C. What are the chances that an unruptured aneurysm may bleed?
D. B&C

21. According to paragraph 6, rapturing of an aneurysm is;

A. Fatal
B. Not always detrimental
C. Fatal even if treated
D. Brain-damaging and it is fatal

22. What paragraph 7 deal with?

A. How aneurysms affect the brain?


B. Brain damage is certain in an aneurysm - how?
C. Why is the damage so extensive after the rupturing of an aneurysm?
D. What is an ischemic stroke?

END OF READING TEST, THIS BOOKLET WILL BE COLLECTED


Reading test 10 : Answer Key

Part A - Answer key 1 – 7


1. D
2. B
3. D
4 .A
5. C
6. A
7. B

Part A - Answer key 8 – 14


8. Curcumin
9 .Dose-limiting
10. second malignancy
11. using antioxidants
12. A better QoL
13. T3 or T4
14 .No

Part A - Answer key 15 – 20


15. Poorer Qol
16. Ionizing Radiation
17 . Adenocarcinoma
18 .Hormone Therapy
19.Curcumin
20. Radiosensitizing Agent
Reading part B – answer key

1.Current state of cardiovascular health in the United States.


2. Smoking and tobacco use.
3. Physical activity is known to curtail down death rate.
4. Process of managing spilled or leaking mercury.
5. Noninvasive quantification of hepatic steatosis in NAFLD.
6 .Obesity rate increased from 2013-2014

Reading test - part C – answer key

Text 1 - Answer key 7 – 14


7. Exercise may always enhance levels of HDLC.
8. People with mild BP
9. A & B
10. Dieting can be one of the best solutions to reduce weight.
11. Risks associated with exercise
12 .Exercise evaluation.
13. Aged people should not perform vigorous exercise.
14 .How exercise training programs should be.

Text 2 - Answer key 15 – 22


15. Prevalence of aneurysms.
16. How do aneurysms form?
17. Often, people are born with aneurysms, but it is developed after the age of 40.
18. Pictures taken during the test are the primary source of the examination
of aneurysms.
19 .What causes an aneurysm to bleed?
20 .B & C
21. Brain-damaging and it is fatal
22 Why is the damage so extensive after the rupturing of an aneurysm?
READING TEST 11
READING SUB-TEST : PART A
 Look at the four texts, A-D, in the separate Text Booklet.
 For each question, 1-20, look through the texts, A-D, to find the relevant information.
 Write your answers on the spaces provided in this Question Paper.
 Answer all the questions within the 15-minute time limit.
 Your answers should be correctly spelt.

TEXT BOOKLET- ANALYSES OF CANCER

Text A

As demonstrated by elegant analyses of cancer in various patients, the continued deletion of


cancer cells expressing T cell targets may enable cancers to evolve to avoid the attack. Despite
these findings, recent results from human cancer have demonstrated that overcoming
negative regulators to T cell responses in lymphoid organs and in the tumor bed is likely to
explain the failure of immune protection in many patients. Factors in the tumor
microenvironment can act to modulate the existing activated antitumor T cell immune
response, acting as an immune rheostat or “immunostat.” This class of molecules, including
PD-L1:PD-1 emphasizes that the immune response in cancer reflects a series of carefully
regulated events that may be optimally addressed not singly but as a group.
The most challenging part is now is to use this new understanding to develop new drugs and
implement clinical strategies.
Text B

For an anticancer immune response to lead to the effective killing of cancer cells, a series of
stepwise events must be initiated and allowed to proceed and expand iteratively. We refer to
these steps as the Cancer-Immunity Cycle. In the first step, neoantigens created by
oncogenesis are released and captured by dendritic cells (DCs) for processing. In order for this
step to yield an anticancer T cell response, it must be accompanied by signals that specify
immunity lest peripheral tolerance to the tumor antigens is induced. Such immunogenic signals
might include proinflammatory cytokines and factors released by dying tumor cells or by the
gut microbiota. Next, DCs present the captured antigens on MHCI and MHCII molecules to T
cells, resulting in the priming and activation of effector T cell responses against the cancer-
specific antigens that are viewed as foreign or against which central tolerance has been
incomplete. The nature of the immune response is determined at this stage, with a critical
balance representing the ratio of T effector cells versus T regulatory cells being key to the final
outcome.
Text C

Finally, the activated effector T cells traffic to and infiltrate the tumor bed, specifically
recognize and bind to cancer cells through the interaction between its T cell receptor (TCR) and
its cognate antigen bound to MHCI, and kill their target cancer cell. The killing of the cancer cell
releases additional tumor-associated antigens (step 1 again) to increase the breadth and depth
of the response in subsequent revolutions of the cycle. In cancer patients, the Cancer-Immunity
Cycle does not perform optimally. Tumor antigens may not be detected, DCs and T cells may
treat antigens as self rather than foreign thereby creating T regulatory cell responses rather
than effector responses, T cells may not properly go well with tumors, may be inhibited from
infiltrating the tumor, or (most importantly) factors in the tumor microenvironment might
suppress those effector cells that are produced.
The goal of cancer immunotherapy is to initiate or reinitiate a self-sustaining cycle
of cancer immunity, enabling it to amplify and propagate, but not so much as to generate
unrestrained autoimmune inflammatory responses. Cancer immunotherapies must, therefore,
be carefully configured to overcome the negative feedback mechanisms. Although checkpoints
and inhibitors are built into each step that opposes continued amplification and can dampen or
arrest the antitumor immune response, the most effective approaches will involve selectively
targeting the rate-limiting step in any given patient. Amplifying the entire cycle may provide
anticancer activity but at the potential cost of unwanted damage to normal cells and tissues.
Many recent clinical results suggest that a common rate limiting step is an immunostat function,
immunosuppression that occurs in the tumor microenvironment
Text D

Attempts to activate or introduce cancer antigen-specific T cells, as well as stimulate the


proliferation of these cells over the last 20 years, have led to mostly no, minimal or modest
appreciable anticancer immune responses. The majority of these efforts involved the use of
therapeutic vaccines because vaccines can be easy to deploy and have historically represented
an approach that has brought enormous medical benefit. Yet, cancer vaccines were limited to
two accounts.
First, until recently, there was a general lack of understanding of how to immunize the patients
for effective cytotoxic T cell responses. This limitation reflects continued uncertainties
concerning the identities of antigens to use, their mode of delivery, the types of adjuvants
required, and the proximal characteristics of the desired T cell response. Second, the presence
of the immunostat in the tumor microenvironment may dampen or disable antitumor immune
responses before clinically relevant tumor kill can occur. Thus, as long as these negative signals
are in place, the prospects for vaccine-based approaches used alone are likely to be limited. -
Mark Stephens, Cardiologist, Care CRM, London.
PART A -QUESTIONS AND ANSWER SHEET

Questions 1-7
For each question, 1-7, decide which text (A, B, C or D) the information comes from. You may use
any letter more than once.
In which text can you find information about;
1.Seven crucial progression acts. Answer ________________
2.Initiating Anticancer Immunity. Answer _________________
3.Checkpoints and (immunostat function) Answer __________________
4 .Didn`t achieve any proper result. Answer __________________
5.Talking about the process of removal of cancer cells. Answer __________________
6.Killing of target cancer cell. Answer _______________
7. Immune editing. Answer ______________

Questions 8-14
Answer each of the questions, 8-14, with a word or short phrase from one of the texts. Each answer
may include words, numbers or both. Your answers should be correctly spelt.

8. Did experiments or endeavors with respect to T-cells lead to any significant results or immune
responses? Answer _________________

9. At which stage the nature of the immune response is determined? Answer ________

10.Which cells may find it difficult to home to tumors? Answer _____

11. What is the term referred to describe the series of events involved in anticancer immune
response? Answer _____________

12. Presence of what will dampen or disable antitumor immune responses in the tumor
microenvironment? Answer ______________

13. What are the most effective approaches in an immunotherapy? Answer _______________

14.What target was not achieved so far in immunizing cancer patients? Answer ________
Questions 15-20

Complete each of the sentences, 15-20, with a word or short phrase from one of the texts. Each
answer may include words, numbers or both. Your answers should be correctly spelt.

15 Destroying the cancer cell releases additional ________________

16 ____is referred to as partial or complete suppression of the immune response of an individual

17 Majority of the processes with respect to activation of the T-cells comprises use of ____

18 Presently, the challenge is how to effectively use this understanding and develop ____

19 In ___________, activated effector T cells may entirely infiltrate the tumor bed.

20 The initial step can provide a response only when it is along with___________, which can clearly
show immunity.

END OF PART A, THIS QUESTIONS PAPER WILL BE COLLECTED


READING SUB-TEST : PART B
In this part of the test, there are six short extracts relating to the work of health professionals .
For questions 1-6, choose the answer (A, B or C) which you think fits best according to the text. Write
your answers on the separate Answer Sheet
Questions 1-6

1. The GCA symptoms can;


A. Begin and grow suddenly or gradually.
B. Begin all of sudden.
C. Occur more gradually.

GCA - Giant Cell Arteritis

The onset of giant cell arteritis (GCA) may be either abrupt or insidious. GCA may begin with
constitutional manifestations such as anorexia, fever, malaise, myalgia, night sweat, and
weight loss. These prodromal symptoms may occur for a few days and may even stretch out to
weeks. The most commonly reported symptoms in patients with GCA are as follows:
• Headache (initial symptom in 33%, present in 72%)
• Neck, torso, shoulder, and pelvic girdle pain that is consistent with polymyalgia rheumatica
(PMR; initial in 25%, present in 58%)
• Fatigue and malaise (initial in 20%, present in 56%)
• Jaw claudication (initial in 4%, present in 40%)
• Fever (initial in 11%, present in 35%)
2. Head anastomosis venture may mean;
A. human head transplantation.
B. B head transplantation in China.
C. C adding head of one patient to the body of the other.
Neurosurgeon Sergio Canavero proposed the HEAVEN procedure – i.e. head anastomosis
venture – several years ago, and has recently received approval from the relevant regulatory
bodies to perform this body-head transplant (BHT) in China. The BHT procedure involves
attaching the donor body (D) to the head of the recipient (R), and discarding the body of R and
head of D. Canavero’s proposed procedure will be incredibly difficult from a medical
standpoint. Aside from medical doubt, the BHT has been met with great resistance from many,
if not most bio- and neuro ethicists.

3. What this notice talk about?


A. Patients with LS should undergo a range of ongoing surveillance activities.
B. Patients with LS are recommended to undergo a range of ongoing surveillance activities.
C. There are awide range of ongoing surveillance activities which are undertaken by caretakers.
LS surveillance support initiatives or ideas have included earlier and more frequent
colonoscopy, consideration for the transvaginal ultrasound and endometrial sampling with the
possible recommendation for a total hysterectomy, and clinical neurologic examination. Upper
endoscopy, annual urinalysis, and pancreatic cancer screening may also be considered based
on family history. Screening guidelines continue to be updated as new research clarifies what
recommendations are beneficial, but few participants receive repeat genetic counseling to
update surveillance and surgical recommendations based on a patient’s specific LS gene
mutation and family history.
4. As per the given notice, hereditary breast cancers;
A. occur commonly due to pathogenic mutations.
B. can easily be detected with advanced strategy.
C. can be treated more cost-effectively.

Hereditary breast cancers account for approximately 10% of all breast cancers, and
approximately 23% of all ovarian cancers are considered hereditary. According to Plakhins et
al., BRCA1 pathogenic founder mutations contribute to 3.77% of all consecutive primary breast
cancers and 9.9% of all consecutive primary ovarian cancers. BRCA1 and BRCA2 pathogenic
founder mutation analysis is a relatively straightforward and cost-effective screening strategy
to identify mutation carriers. In Latvia, all consecutive breast and ovarian cancer cases are
eligible for BRCA1 pathogenic founder mutations (c.181 T > G, c.4035delA, c.5266dupC)
screening, and the costs of the test are covered by the public health care system.
5. What is correct?
A. The average age of PMR patients with amyloidosis is higher than the average age of PMR
patients without TA.
B. the average age of PMR patients without TA (p < 0.0164) is significantly lower than the
average age of PMR patients without amyloidosis.
C. The average age of PMR patients with TA is significantly higher.

The average age of PMR patients


Total (female & male)
Female
Male
PMR versus Ath
0,4494
0,2972
0,4217
PMR with Ath versus without Ath
0,4222
0,2703
0,0004
PMR with TA versus without TA
0,0164
0,0159
0,7951
PMR with amyloidosis versus without amyloidosis
0,0000
0,0000
0,0037
TA with amyloidosis versus without amyloidosis
0,2995
0,3727
0,7228
6 Patient clinical data shows that the patient;
A is suffering from BP.
B is suffering from heart disease.
C is suffering from chest pain.
6. Patient clinical data shows that the patient;
A. is suffering from BP.
B. is suffering from heartdisease.
C. is suffering from chest pain
READING SUB-TEST : PART C
In this part of the test, there are two texts about different aspects of healthcare.
For questions 7-22, choose the answer (A, B, C or D) which you think fits best according to the text.
Write your answers on the separate Answer Sheet

Text 1: A note on Eye Infections – Conjunctivitis

Conjunctivitis is a common and often very contagious condition more commonly


referred to as a pink eye. Conjunctivitis often involves inflammation and swelling
of the conjunctiva or the clear membrane covering the eye and lining the inner
eyelids. It is very common among young children, particularly those in school or
daycare. Elderly people can also be affected. Conjunctivitis typically poses no real
threat to the patient`s vision or wellness. There are multiple causes and risk factors
for conjunctivitis. Some of the more common causes of conjunctivitis include
Allergic reactions, Viral contamination, Bacterial contamination, Exposure to
irritants or chemical pollutants. Infectious conjunctivitis typically results from
bacterial or viral contamination. While bacterial conjunctivitis may be treated with
certain antibiotics, there is no traditional treatment for viral conjunctivitis.
Multiple strains of bacteria are responsible for bacterial conjunctivitis; these
include haemophilus influenzae and Staphylococcus aureus. Viral conjunctivitis is
a common condition affecting children from the time of birth through to their adult
years. Viruses can enter the eye in many ways, resulting from the common cold or
contagion with the flu. Some patients may develop a more serious form of viral
conjunctivitis associated with a herpes infection; in this case, prompt medical
attention is necessary to prevent permanent damage to the eye or vision.
One of the more commonly reported signs and symptoms of conjunctivitis include
discomfort and pain in the eye, accompanied by redness or swelling of the eyelid,
hence the name "pinkeye." Patients may experience reddening of the inner and
outer eyelid or may experience swollen eyelids. Some patients may report
increased sensitivity to light and other irritants including the wind. Many
patients with conjunctivitis will have discharge from the eyes that may be clear,
white, green or yellow in color. Yellow or green discharge from the eye may
suggest an infection and may require antibiotics or other forms of aggressive
treatment. Conjunctivitis can spread from person to person or from eye to eye.
Simply touching one eye then the other can spread viral conjunctivitis. Allergic
conjunctivitis is a non-contagious condition. Typically, children exposed to bacterial
conjunctivitis will have a two to four day window before symptoms appear.
Bacterial infections mostly come from staphylococci and streptococci organisms
that can come from your own skin or upper respiratory tract. The indicating
symptoms of bacterial infections are thick, ropy mucus discharge accompanied
with red, irritated and inflamed eyes. Luckily, bacterial eye infections are easily
treated with antibiotic eye drops and, in most cases, will clear up within a few
short days. Viral infections are commonly caused by an enterovirus and often
associated with an upper respiratory infection or common cold. Eyes are red and
inflamed and become watery and runny. One of the most common viral infections
is epidemic keratoconjunctivitis, also known as EKC; it is highly contagious and
can last up to 2 weeks or more. This viral conjunctivitis is caused by an adenovirus
and does not have a specific treatment to cure the infection. The doctor may
prescribe steroid eye drops and artificial tears to help decrease inflammation, but
mostly the virus simply needs to run its course.

Conjunctivitis may also result from chlamydia and gonococcal infections or STD`s.
Usually, the inner eyelid becomes infected. This condition is more commonly noted
in teens and young adults who are sexually active. When left untreated, this condition
may affect newborn infants born to mothers infected with an STD. Signs may include
a history of pelvic pain or vaginitis as well. Patients with Gonococcal infections may
feel like a foreign object is chronically present within their eye, and are more likely to
experience burning and inflammation. It is possible to transfer these conditions to the
eye from hand contact so it is important, to help prevent the spread of infection, that
frequent hand washing is adopted by patients and family members. Treatment usually
involves use of antibiotics taken topically or orally and concomitant treatment may be
necessary to treat genital and eye infections.
Text 1: Questions 7-14

7. Which part of the eye is affected by conjunctivitis?


A. Conjunctiva
B. White membrane
C. Eyelids
D. Only A and B

8. Conjunctivitis is common among;


A. Children
B. Teens
C. School goers
D. Daycare children

9. What is linked with the herpes infection?


A. Bacterial infections.
B. Viral infections.
C. Allergic reactions.
D. Exposure to chemical pollutants.

10. What is the most common symptom of conjunctivitis?


A. Mild pain in the eye.
B. Watery eyes.
C. Reddening of the inner and outer eyelid.
D. All of the above.

11. The color of discharge from eyes may be;


A. Yellow and green.
B. White, yellow and green.
C. Yellow or green.
D. White or green.
12. Paragraph 3 is focused more on;
A. Types of conjunctivitis.
B. Types of bacterial conjunctivitis.
C. Types of viral conjunctivitis.
D. Types of bacterial and viral conjunctivitis.

13. viral conjunctivitis is caused by;


A. Androvirus
B. Adenovirus
C. Enterovirus
D. Adenonvirus

14. Which is more common in gonococcal infections?


A. Pain around pelvis.
B. Discharge or pus.
C. Inflammation of the eyes.
D. Reddening of the eyes.
Text 2: Osteomyelitis is an infection of the bone.

Osteomyelitis is an infection of the bone. Osteomyelitis can occur in infants,


children, and adults. Different types of bacteria typically affect the different age
groups: in children, osteomyelitis most commonly occurs at the ends of the long
bones of the arms and legs, affecting the hips, knees, shoulders, and wrists,
whereas in adults, it is more common in the bones of the spine (vertebrae), feet, or
in the pelvis. There are several different ways to develop the bone infection of
osteomyelitis. The first is for bacteria to travel through the bloodstream
(bacteremia) and spread to the bone, causing an infection. This mostly occurs
when the patient has an infection elsewhere in the body, such as pneumonia or a
urinary tract infection that spreads through the blood to the bone. An open wound
over a bone can lead to osteomyelitis. A recent surgery or injection around a bone
can also expose the bone to bacteria and lead to osteomyelitis. Patients with
conditions or taking medications that weaken their immune system are at a higher
risk of developing osteomyelitis. Risk factors include cancer, chronic steroid use,
sickle cell disease, human immunodeficiency virus (HIV), diabetes, hemodialysis,
intravenous drug users, and the elderly.

Symptoms of osteomyelitis can vary greatly. In children, osteomyelitis usually


occurs more quickly. They develop pain or tenderness over the affected bone, and
they may have difficulty or inability to use the affected limb or to bear weight or
walk due to severe pain. They may also have fever, chills, and redness at the site
of infection. In adults, the symptoms often develop more gradually and include
fever, chills, irritability, swelling or redness over the affected bone, stiffness, and
nausea. In people with diabetes, peripheral neuropathy, or peripheral vascular
disease, there may be no pain or fever. The only symptom may be an area of skin
breakdown that is worsening or not healing. Acute osteomyelitis occurs with a
rapid onset and is usually accompanied by the symptoms of pain, fever, and
stiffness. It generally occurs after a break in the skin from injury, trauma, surgery,
or skin ulceration from wounds. Chronic osteomyelitis is insidious in onset; it may
be the result of a previous infection of osteomyelitis and, despite multiple courses
of antibiotics, it may reoccur. Symptoms of chronic osteomyelitis are subtle but
may include fever, pain, redness, or discharge at the site of infection.

The diagnosis of osteomyelitis begins with a complete medical history and physical
examination. During the discussion about medical history, the doctor may ask
questions about recent infections elsewhere in the body, past medical history,
medication usage, and family medical history. The physical examination will look for
areas of tenderness, redness, swelling, decreased or painful range of motion, and
open sores. The doctor may then order tests to help diagnose osteomyelitis. Several
blood tests can be used to help determine if there is an
infection present; these include a complete blood count (CBC), the erythrocyte
sedimentation rate (ESR), C-reactive protein (CRP), and blood cultures. None of these
is specific for osteomyelitis but they can suggest that there may be some infection in
the body. Imaging studies of the involved bones may be obtained; these can include
plain radiographs (X-rays), bone scans, computed tomography (CT) scans, magnetic
resonance imaging (MRIs), and ultrasounds. These imaging studies can help identify
changes in the bones that occur with osteomyelitis.
In many cases, osteomyelitis can be effectively treated with antibiotics and pain
medications. If a biopsy is obtained, this can help guide the choice of the best
antibiotic. The duration of treatment of osteomyelitis with antibiotics is usually
four to eight weeks but varies depending on the type of infection and the response
to the treatments. In some cases, the affected area will be immobilized with a
brace to reduce the pain and speed up the treatment. Sometimes, surgery may be
necessary. If there is an area of localized bacteria (abscess), this may need to be
opened, washed out, and drained. If there is damaged soft tissue or bone, this may
need to be removed. If bone needs to be removed, it may need to be replaced with
bone graft or stabilized during surgery.
With early diagnosis and appropriate treatment, the prognosis for osteomyelitis is
good. Antibiotics regimes are used for four to eight weeks and sometimes longer
in the treatment of osteomyelitis depending on the bacteria that caused it and the
response of the patient. Usually, patients can make a full recovery without
longstanding complications.

However, if there is a long delay in diagnosis or treatment, there can be severe


damage to the bone or surrounding soft tissues that can lead to permanent deficits
or make the patient more prone to recurrence. If surgery or bone grafting is
needed, this will prolong the time it takes to recover.
Text 2: Questions 15-22
15. Which of the following statements is not true, according to the information provided in
paragraph 1?
A. An osteomyelitis is common among people of all ages.
B. An osteomyelitis can be caused by bacteria.
C. In most of the cases, people who have pneumonia may develop osteomyelitis.
D. An osteomyelitis is commonly observed in people who are prone to bacterial infections.

16. Which of the following can lead to osteomyelitis?


A. Bacterial infection.
B. An open wound.
C. Underrated bone operation.
D. None

17. Symptoms of osteomyelitis in children may include;


A. Tenderness in the bone;
B. High fever;
C. Inability to walk;
D. All of the above;

18. Which type of osteomyelitis is slow in the beginning?


A. Chronic
B. Acute
C. Both chronic and acute
D. None

19. Which group is more specific for diagnosis?


A. CBC and ESR only
B. Only ESR and CRP
C. CRP, ESR and CBC
D. None
20. According to paragraph 3, involved bone suggests
A. Wounded bone
B. Operated bone area
C. Bacterial infected bone
D. Bone with tenderness

21. According to paragraph 4, which one of the following statements is not correct?
A. Treatment may be completed within 4 to 8 weeks.
B. The duration of the treatment varies depending on the type of osteomyelitis.
C. Working of the affected area is stopped in order to speed up the treatment.
D. Bone is often replaced with bone graft for perfect treatment.

22. According to paragraph 5, delay in treatment of diagnosis may result in;


A. Inability to walk properly.
B. Permanent damage to bone.
C. Removal of affected bone.
D. None

END OF READING TEST, THIS BOOKLET WILL BE COLLECTED


Reading test 11 : Answer Key

Part A - Answer key 1 – 7


1: B
2: D
3: A
4: D
5: B
6: C
7: A

Part A - Answer key 8 – 14


8: No
9: Stage 3
10: T cells
11: Cancer-Immunity Cycle
12: immunostat
13: selectively targeting the rate-limiting step
14: effective cytotoxic T cell responses

Part A - Answer key 15 – 20


15: tumor-associated antigens
16: immunosuppression
17: therapeutic vaccines
18: new drugs
19: step 4
20: signals
Reading test - part B – answer key
1: B Begin All Of Sudden.
2: A human head transplantation.
3: B Patients with LS are recommended to undergo a range of ongoing surveillance activities.
4: B can easily be detected with advanced strategy.
5: C The average age of PMR patients with TA is significantly higher.
6: A is suffering from BP.

Reading test - part C – answer key

Text 1 - Answer key 7 – 14


7: D Only A and B
8: D Daycare children
9: B Viral infections.
10: C Reddening of the inner and outer eyelid.
11: B White, yellow and green.
12: A Types of conjunctivitis.
13: B Adenovirus
14: C Inflammation of the eyes.

Text 2 - Answer key 15 – 22


15: C In most of the cases, people who have pneumonia may develop osteomyelitis.
16: B An open wound.
17: D All of the above;
18: A Chronic
19: D None
20: C Bacterial infected bone
21: D Bone is often replaced with bone graft for perfect treatment.
22: B Permanent damage to bone.
READING TEST 12
READING SUB-TEST : PART A
 Look at the four texts, A-D, in the separate Text Booklet.
 For each question, 1-20, look through the texts, A-D, to find the relevant information.
 Write your answers on the spaces provided in this Question Paper.
 Answer all the questions within the 15-minute time limit.
 Your answers should be correctly spelt.

TEXT BOOKLET- BRAIN IMAGING IN CHILDREN

Text A

Brain Imaging In Children

A headache is a result of disorders that affect pain-sensitive sites, such as meninges,


blood vessels, paranasal sinuses and muscles; it is also one of the most common causes of
pain in children. The prevalence of severe and frequent headaches is 25.25 per thousand in
children. A large number of physicians and parents of sick children are concerned that
inter-carcinoma lesions may be due to headaches. The most important means to diagnose
the cause of headaches is to take precise medical history and para-clinical measures.
Brain imaging method, including CT scan and MRI, is one of the diagnostic methods for
children with headaches. The main reason for performing MRI and CT scans in patients
with a headache is the diagnosis of curable lesions, which can increase the patient’s
longevity or improve her quality of life. Cases such as brain tumors, hydrocephalus vascular
malformations, and subdural hematoma are in this category. Another important cause for
conducting MRI in headache patients is a way to alleviate their anxiety due to the presence
of a brain tumor or intracranial disease.

Text B

Cases in which imaging should be done include important changes in the type of
headache, headache worsening, sudden development of headache or when it is
stimulated by awakening of sleep, and when it is associated with a neurological
symptom. It has been reported that there are abnormal findings in the imaging of
patients, who are suspected of having a headache based on the findings of the
study. Secondary pathologic factors are more common than those in the general
population of headache patients. Parents’ anxiety and their concern about
headaches in their children and the availability of imaging measures have caused
most children to experience CT scan and radiation due to the importance of
headaches in children and due to limited studies in under developing countries.
Patients below 12 years of age were asked for, imaging actions, such as CT scan
and MRI, were the main inclusion criteria. The exclusion criteria included being
older than 12 years of age, lack of consent to enter the study, and defects in the
medical records. Subsequently, the CT and MRI reports of these patients were
reviewed and the findings were recorded. Abnormal findings in the CT scan include mass,
cysts, infarcts, hydrocephalus, calcification, hematoma, ventricular dilation and edema.
Abnormal findings in MRI include sinusitis,retinal cysts, masses, cysts, atrophy, ventricular
dilatation, age variations, hydrocephalus, hematoma, demyelinating disease, mastoiditis,
encephalomalacia, schizophyllum and hypoplasia of corpus callosum, the prevalence of
which were measured after collecting reports.
Text C

All patient information, including demographic factors and para-clinical


symptoms, were recorded in a researcher-made check list and entered into SPSS.
In the descriptive part, abnormal CT scan and MRI findings are presented as the
main variable in different groups. All of the demographic and clinical
characteristics of patients were also reported based on descriptive criteria. In the
analytical section, based on statistical assumptions, parametric and nonparametric
proportional tests were used. CHI-SQUARE test was used to analyze the
qualitative findings and independent T-test was used to compare quantitative
data; non-parametric Mann-Whitney was used if the initial assumptions were not
as normal as they were supposed to. All tests were examined at a 5% error level.
Text D

Total amount of 353 people were included in the study, of which 7 were excluded
during the study. In the first group, CT scan was performed on 217 patients, of
which 85 subjects were girls and 132 were boys and 11.1% were abnormal. In the
second group, 136 people were subjected to MRI, of which 56 (41.1%) were
female and 80 (58.8%) were male, and 24.3% were abnormal. Also, according to
gender segregation, abnormal findings in CT scan were significantly higher in
boys (63% boys and 37% girls) (P = 0.03), and it was also found that MRI findings
were also significantly more common in boys in comparison to girls (66% vs.
34%) (P = 0.04). The results of the study showed that the most common CT scan
abnormal finding was mass (21%) and hematoma (21%). Then, cysts (14%) and
Ventriculomegaly (14%) were the most frequent forms of abnormality. Prevalently
found abnormality was cysts in MRI (30%). Atrophy (12%) and Ventriculomegaly
had the second and third frequencies (15%) (P> 0.05).
PART A -QUESTIONS AND ANSWER SHEET
Questions 1-7
For each question, 1-7, decide which text (A, B, C or D) the information comes from.
You may use any letter more than once.
In which text can you find information about;

1. Research methodology. Answer________________

2. Patients who underwent study. Answer ________________

3. Importance of detection of the causes in early stages. Answer ____________________

4. Reasons why imaging shall be done. Answer _______________________

5. Data segregation. Answer __________________

6. Condition in which there is an accumulation of cerebrospinal fluid (CSF) within the brain. Answer __

7. A small localized area of dead tissue resulting from failure of blood supply. Answer ________

Questions 8-14
Answer each of the questions, 8-14, with a word or short phrase from one of the texts.
Each answer may include words, numbers or both. Your answers should be correctly spelt.

8. Which age group patients were included in the study? Answer ______________

9. What is the reason presented for taking MRI in children with headache? Answer _______________

10. What is define as inflammation of the air cavities within the passages of the nose? Answer __

11. Was any defect in medical records an inclusion criteria? Answer _________________

12. Who shows more peculiarities in CT imaging? Answer ________________

13. What method was followed if the initial assumptions were abnormal? Answer ______________
14. What is known to be more common with respect to CT scanning? Answer ______________

Questions 15-20

Complete each of the sentences, 15-20, with a word or short phrase from one of the texts.
Each answer may include words, numbers or both. Your answers should be correctly spelt.

15. __________ is referred to as a solid swelling of clotted blood within the tissues.

16. The __________ were presented in two sections of descriptive and analytical.

17. The percentage of boys who underwent CT scan is _________________

18. _______________ can be defined as a brain condition that occurs in the fetus when the lateral
ventricles become dilated

19. A _____ is any disease of the nervous system in which the myelin sheath of neurons is damaged.

20. ______________ can be defined as the general physiological process of reabsorption and
breakdown of tissues.

END OF PART A, THIS QUESTIONS PAPER WILL BE COLLECTED


READING SUB-TEST : PART B
In this part of the test, there are six short extracts relating to the work of health professionals .
For questions 1-6, choose the answer (A, B or C) which you think fits best according to the text.
Write your answers on the separate Answer Sheet
Questions 1-6

1. According to the notice given


A. health rate improved from 2003-2017
B. many disparities present in 2012-2017 widened over time
C. increase in physical activity led to improved health score

Nutrition
Between 2003 to 2004 and 2016 to 2017 in the United States, the mean AHA healthy diet
score improved in both children and adults. The prevalence of an ideal healthy diet score
increased from 0.2 percent to 0.6 percent in children and from 0.7 percent to 1.5 percent in
adults. These improvements were largely attributable to increased whole grain consumption
and decreased sugar-sweetened beverage consumption in both children and adults, as well
as a small, non-significant trend in increased fruit and vegetable consumption. No major
trends were evident in children or adults in progress toward the targets for consumption of
fish or sodium. Between 2012 and 2017, although AHA healthy diet scores tended to improve
in all race/ethnicity, income, and education levels, there has been constant inequalities as
well. with generally smaller improvements seen in minority groups and those with lower
income or education.
About one in every three US adults or 30.4 percent, do not engage in leisure time physical
activity. Hispanic and Non-Hispanic black adults were more likely to be inactive. Among
students in grades 9-12, only about 27.1 percent meet the American Heart Association
recommendation of 60 minutes of exercise every day. More high school boys (36 percent)
than girls (17.7 percent) reported having been physically active at least 60 minutes per day
on all 7 days.
2. The term which is more close to condition in which the child may reply
A. instantly somnolence
B. hypotonia
C. infantile reflex

Acute Bilirubin Encephalopathy (ABE)


ABE is a term to describe the variable spectrum, from subtle to advanced manifestations of
bilirubin toxicity present in the first weeks of life (American Academy of Pediatrics
Subcommittee on Hyperbilirubinemia, 2004; Van Praagh, 1961). Symptoms associated with
ABE include a range of neurological manifestations, sleeping disorder, somnolence,
hypotonia, loss of the Moro reflex, followed by a stage characterized by hypertonia of the
extensor muscle groups (backward arching of the neck and backward arching of the trunk).
Additionally, fever and/or a high pitched cry may be present. Different investigators have
used different methods to characterize and define clinical manifestations of ABE in the infant

3. The notice
A. places women at the centre of the care
B. explains what can be done in order to decrease mortality rate
C. talks of standards that ensure healthy life

Care For Women


Undoubtedly, encouraging women to give birth in health facilities, where there are skilled
birth attendants, is essential and has helped reduce global newborn and maternal mortality
rates for decades. However, there is room for improvement in the quality of care provided in
these facilities. To meet Sustainable Development Goal 3 of ensuring healthy lives and
promoting well-being for all at all ages, we cannot keep our focus solely on survival. High
quality care for all pregnant women and their newborns, throughout pregnancy, childbirth
and the postnatal periods, is essential to ensure that mothers and children both survive and
thrive.

4 .The notice clearly explains that


A. disease control is a still a big problem
B. disease has successfully been managed
C. disease is widespread and is affecting population across the globe

Wild polio
Overall the Committee was encouraged by continued progress in WPV1 eradication, with the
number of cases globally falling to an all-time low in 2017. In addition, there has been no
international spread of WPV since the fifteenth meeting in November 2017.
The Committee commended the continued high level commitment seen in sub continents
and the high degree of cooperation and coordination, particularly targeting the high risk
mobile populations that cross the international border, such as nomadic groups, local
populations straddling the border, seasonal migrant workers and their families, repatriating
refugees (official and informal), and guest children (children staying with relatives across the
border). Stopping transmission in these populations is going beyond efforts and cannot be
underestimated, underlining the critical continuing need for cross border activities in
surveillance and vaccination
5. The notice talks more about
A. how ICD works
B. ICD purpose and uses
C. actions undertaken by the ICD

ICD - Health Trends

ICD is the foundation for the identification of health trends and statistics globally, and the
international standard for reporting diseases and health conditions. It is the diagnostic
classification standard for all clinical and research purposes. ICD defines the universe of
diseases, disorders, injuries and other related health conditions, listed in a comprehensive,
hierarchical fashion that allows for:
 easy storage, retrieval and analysis of health information for evidence-based decision-making
 sharing and comparing health information between hospitals, regions, settings and
countries;
 and data comparisons in the same location across different time periods.
Uses include monitoring of the incidence and prevalence of diseases, observing
reimbursements and resource allocation trends, and keeping track of safety and quality
guidelines. They also include the counting of deaths as well as diseases, injuries, symptoms,
reasons for encounter, factors that influence health status, and external causes of disease
6. What is correct?
A. Ebola virus is common among Waganta population
B. There has been widespread prevalence of the virus in Iboko
C. Virus in Bikoro is spreading at a rate closer to Waganta

Distribution of Ebola virus disease cases by health zone in Democratic Republic of the
Congo, 1 April – 9 July 2018
READING SUB-TEST : PART C
In this part of the test, there are two texts about different aspects of healthcare.
For questions 7-22, choose the answer (A, B, C or D) which you think fits best according to the text.
Write your answers on the separate Answer Sheet

Text 1: Viral Infection – Yellow Fever

Yellow fever is a viral infection spread by a particular species of mosquito. It's


most common in the areas of Africa and South America, affecting both travellers
to and residents of those areas. In mild cases, it causes fever, headaches, nausea
and vomiting. However, it can become more serious, causing heart, liver and
kidney problems along with bleeding (haemorrhaging). Up to 50 percent of people
with the more severe form of yellow fever die of the disease. There's no specific
treatment for yellow fever, but getting a yellow fever vaccine before travelling to
an area in which the virus is known to exist can protect you from the disease.
During the first three to six days after contracting yellow fever — the incubation
period — there won’t be any signs or symptoms of the disease. After this, the
virus enters an acute phase and, in some cases, a toxic phase follows which can be
life-threatening.

Once the yellow fever virus enters the acute phase, you may experience signs and
symptoms including: fever, headaches, muscle aches - particularly in your back
and knees - nausea, vomiting or both, loss of appetite, dizziness, red eyes, face or
tongue. These signs and symptoms usually improve and disappear within several
days. Although signs and symptoms may disappear for a day or two following the
acute phase, some people with acute yellow fever then enter a toxic phase. During
the toxic phase, acute signs and symptoms return and more severe and life threatening
ones also appear. These can include yellowing of the skin and the whites of the eyes
(jaundice), abdominal pain and vomiting - sometimes of blood - decreased urination, bleedin
from your nose, mouth and eyes, heart dysfunction (arrhythmia), liver and kidney failure,
and brain dysfunction, including delirium, seizures and coma.

Make an appointment to see your doctor four to six weeks before travelling to an
area in which yellow fever is known to occur. If you don't have that much time to
prepare, call your doctor anyway. Your doctor will help you determine whether
you need vaccinations and can provide general guidance on protecting your health
while abroad. Seek emergency medical care if you've recently travelled to a
region where yellow fever is known to occur and you develop severe signs or
symptoms of the disease. Even if you develop mild symptoms, call your doctor.
Yellow fever is caused by a virus that is spread by the Aedes aegypti mosquito.
These mosquitoes thrive in and near human habitations where they can breed in
even the cleanest water. Most cases of yellow fever occur in sub-Saharan Africa
and tropical South America.

Humans and monkeys are most commonly infected with the yellow fever virus;
mosquitoes transmit the virus back and forth between monkeys, humans or both.
When a mosquito bites a human or monkey infected with yellow fever, the virus
enters the mosquito's bloodstream and circulates before settling in the salivary
glands. When the infected mosquito bites another monkey or human, the virus
then enters the host's bloodstream, where it may cause the illness to develop.

You may be at risk of the disease if you travel to an area where mosquitoes continue
to carry the yellow fever virus. These areas include sub-Saharan Africa and tropical
South America. Even if there aren't current reports of infected humans in these areas,
it doesn't necessarily mean you're risk-free. It's possible that local populations have
been vaccinated and are protected from the disease, or that cases of yellow fever just
haven't been detected and officially reported. If you're planning on travelling to these
areas, you can protect yourself by getting a yellow fever vaccine at least 10 to 14
days before travelling. Anyone can be infected with the yellow fever virus, but older
adults are at greater risk of becoming seriously ill.

Diagnosing yellow fever based on signs and symptoms can be difficult because, early
in its course, the infection can be easily confused with malaria, typhoid, dengue fever
and other viral hemorrhagic fevers. To diagnose your condition, your doctor will
likely: Ask questions about your medical and travel history, collect a blood sample
for testing. If you have yellow fever, your blood may reveal the virus itself. If not, blood
tests called enzyme-linked immunosorbent assay (ELISA) and polymerase chain reaction
(PCR) can also detect antigens and antibodies specific to the virus. Results from these tests
may take several days. No antiviral medications have proved helpful in treating yellow
fever and, as a result, treatment consists primarily of supportive care in a hospital. This
includes providing fluids and oxygen, maintaining adequate blood pressure, replacing
blood loss, providing dialysis for kidney failure, and treating any other infections
that develop. Some people receive transfusions of plasma to replace blood proteins that
improve clotting. If you have yellow fever, you may also be kept away from mosquitoes, to
avoid transmitting the disease to others.
Text 1: Questions 7-14
7. Yellow fever is common in;

A. Africa
B. South America
C. Both
D. Not given

8. Which of the following is not a sign of yellow fever?

A. Back pain
B. Vomiting
C. Nausea
D. Dry tongue

9. Signs/symptoms of toxic phase;

A. Loss of appetite
B. Yellowness of eyes
C. Brain dysfunction
D. B and C

10. Seizures may occur during;

A. Acute phase
B. Toxic phase
C. Sometimes in both the phases
D. Not given

11. Yellow fever, which is a viral disease, is spread by;

A. Aedes albopictus mosquito


B. Aedes aegypti mosquito
C. Female aedes mosquito
D. Male aedes mosquito

12. Mosquitoes transmit the virus from

A. Humans to monkeys
B. Monkeys to humans
C. Human to human
D. A&B

13. Travelling to areas where the disease is common is recommended after vaccination of;

A. 10 days
B. 12 days
C. 14 days
D. 10-14 days

14. Supportive care in hospitals includes;

A. Free medical check up


B. Free BP check up
C. Effective dialysis procedures in case of kidney failure
D. B and C
Text 2: Aortic Dissection or Dissecting Aneurysm

An aortic dissection is a serious condition in which a tear develops in the inner


layer of the aorta, the large blood vessel branching off the heart. Blood surges
through this tear into the middle layer of the aorta, causing the inner and middle
layers to separate (dissect). If the blood-filled channel ruptures through the outside
aortic wall, aortic dissection can be fatal. Aortic dissection, also called a dissecting
aneurysm, is relatively uncommon. Anyone can develop the condition, but it most
frequently occurs in men between 60 and 70 years of age. Symptoms of aortic
dissection may mimic those of other diseases, often leading to delays in diagnosis.
However, when an aortic dissection is detected early and treated promptly, your
chance of survival greatly improves.

Aortic dissection symptoms may be similar to those of other heart problems, such
as a heart attack. Typical signs and symptoms include sudden severe chest or
upper back pain (often described as a tearing, ripping or tearing sensation, that
radiates to the neck or down the back), loss of consciousness (fainting), shortness
of breath, sweating, weaker pulse in one arm compared to the other etc. If you
have signs or symptoms such as severe chest pain, fainting, sudden onset of
shortness of breath or symptoms of a stroke then seeking medical assistance is
imperative. While experiencing such symptoms doesn't always mean that you
have a serious problem, it's best to get checked out quickly because early detection
and treatment may help to save your life.

An aortic dissection occurs in a weakened area of the aortic wall. Chronic high
blood pressure may stress the aortic tissue, making it more susceptible to tearing.
You can also be born with a condition associated with a weakened and enlarged
aorta, such as Marfan syndrome or bicuspid aortic valve. Rarely, aortic
dissections may be caused by traumatic injury to the chest area, such as during
motor vehicle accidents.
Aortic dissections are divided into two groups, depending on which part of the aorta is
affected: Type A: This is the more common and dangerous type of aortic dissection. It
involves a tear in the part of the aorta just where it exits the heart or a tear extending
from the upper to lower parts of the aorta, which may extend into the abdomen.
Type B: This type involves a tear in the lower aorta only, which may also extend into
the abdomen. Risk factors for aortic dissection include Uncontrolled high blood
pressure (hypertension), found in at least two thirds of all cases, Hardening of the
arteries (atherosclerosis), Weakened and bulging artery(pre-existing aortic aneurysm),
aortic valve defect (bicuspid aortic valve), A narrowing of the aorta you're born with
(aortic coarctation)

People with certain genetic diseases are more likely to have an aortic dissection
than other people. These genetic diseases include Turner's syndrome, high blood
pressure, heart problems, and a number of other health conditions may be a result
of this disorder. Marfan syndrome: This is a condition in which connective tissue,
which supports various structures in the body, is weak. People with this disorder
often have a family history of aneurysms of the aorta and other blood vessels.
These weak blood vessels are prone to tears (dissection) and rupture easily.
Ehlers Danlos syndrome: This group of connective tissue disorders is characterized by
skin that bruises or tears easily, loose joints and fragile blood vessels. Loeys-Dietz
syndrome: This is a connective tissue disorder marked by twisted arteries, especially in
the neck. People who have Loeys-Dietz syndrome are thought to be at risk of
developing aortic dissections and aneurysms.

An aortic dissection can lead to death, due to severe internal bleeding, including into
the lining around the heart (pericardial sac), organ damage (such as kidney failure or
life-threatening damage to the intestines), strokes (possibly including paralysis), and
aortic valve damage, such as causing the aortic valve to leak (aortic regurgitation).
Detecting an aortic dissection can be tricky because the symptoms are similar to
those of a variety of health problems. Doctors often suspect an aortic dissection if the
following signs and symptoms are present: sudden tearing or ripping chest pain,
widening of the aorta on a chest X-ray, blood pressure difference between the right
and left arms.
Text 2: Questions 15-22
15. In aortic dissection a tear develops in;

A. Outer layer of aorta.


B. Inner layer of aorta.
C. Middle aorta.
D. A blood vessel branching off the heart.

16. Dissecting aneurysm is common among;

A. Men
B. Women
C. Both
D. Children

17. Symptoms of aortic dissection include;

A. Chest pain and swelling


B. Weak pulse in both arms
C. Loss of consciousness.
D. All of the above

18. Aortic dissection can also be caused due to;

A. High BP
B. Weak aortic wall
C. Inborn symptoms
D. Traumatic injury to chest during accidents

19. The most dangerous type of aortic dissection is;

A. Type A
B. Type B
C. Aortic aneurysm
D. Aortic coarctation
20. A condition in which connective tissue is weak is called;

A. Turners syndrome
B. Loeys-Dietz syndrome
C. Ehlers-Danlos syndrome
D. Marfans syndrome

21. People with Loeys-Dietz syndrome are likely to develop;

A. Aneurysms
B. Ruptured blood vessels
C. Twisted arteries in the neck
D. Aortic complications

22. Detecting aortic dissection is;

A. Easy
B. Difficult
C. Impossible
D. Sometimes possible

END OF READING TEST, THIS BOOKLET WILL BE COLLECTED


Reading test 12 : Answer Key

Part A - Answer key 1 – 7


1: C
2: D
3: A
4: B
5: C
6: A
7: B

Part A - Answer key 8 – 14


8: below twelve years
9: diagnosis of curable lesions
10: sinusitis
11: no
12: boys
13: non-parametric Mann-Whitney
14: mass and hematoma

Part A - Answer key 15 – 20


15: Hematoma
16: Statistical Analyzes
17: 61%
18: Ventriculomegaly
19: Demyelinating Disease
20: Atrophy
Reading test - part B – answer key 1 – 6

1: many disparities present in 2012-2017 widened over time


2: infantile reflex
3: places women at the centre of the care
4: disease control is a still a big problem
5: ICD purpose and uses
6: There has been widespread prevalence of the virus in Iboko

Reading test - part C – answer key

Text 1 - Answer key 7 – 14


7: Both
8: Dry tongue
9: B and C
10: Toxic phase
11: Aedes aegypti mosquito
12: A & B
13: 10-14 days
14 : B and C

Text 2 - Answer key 15 – 22


15: Inner layer of aorta.
16: Men
17: Loss of consciousness.
18: Traumatic injury to chest during accidents
19: Type A
20: Marfans syndrome
21: Aneurysms
22: Difficult
READING TEST 13
READING SUB-TEST : PART A
 Look at the four texts, A-D, in the separate Text Booklet.
 For each question, 1-20, look through the texts, A-D, to find the relevant information.
 Write your answers on the spaces provided in this Question Paper.
 Answer all the questions within the 15-minute time limit.
 Your answers should be correctly spelt.

TEXT BOOKLET - CYTOKINES

Text A

Cytokines

Cytokines are messengers secreted by various cell types in the body in response to
a wide variety of physiological stimulus. Cytokines aid in normal physiological processes
such as growth, differentiation, hematopoiesis, as well as several inflammatory and
immune responses. The cytokines as present in minimal levels in hepatic circulation
during normal physiological status and are necessary for hepatic homeostasis. However,
the cytokines have been observed to play an active role in mediating the inflammatory
progression of NAFLD as characterized by apoptotic and necrotic lesion in liver leading to
fibrosis. The cytokines involved in hepatic inflammation, are categorized under several
subfamilies – Tumor necrosis factor-α (TNF-α), Tumor growth factor β (TGF-β),
Interleukins and chemokines.
Text B

Tumor Necrosis Factor α


TNF-α is a pro-inflammatory cytokine secreted by several cell types such as neutrophils,
macrophages, T and B lymphocytes, endothelial cells, mast cells, fibroblasts etc. In liver,
hepatocytes and Kupffer cells are the major contributors of TNF- α. TNF-α plays a central
role in the initiation of inflammatory cascade and its progression from steatosis to
steatohepatitis. Experiments with mice models for obesity have shown the importance of
TNF-α in NAFLD, where anti- TNF-α drug therapy showed promising results. Increased
free fatty acid level in obesity stimulates the hepatocytes to secrete TNF induced
expression of inflammatory genes. A positive correlation has been observed between
TNF-α level in serum and degree of fibrosis in patients with NAFLD. A study with
pentoxifylline an inhibitor of TNF-α has shown suppressive effect on elevation of serum
transaminases and triglycerides in experimental NAFLD induced rats. The study also
demonstrated that NAFLD induced TNF-α expression stimulates endoplasmic reticulum
stress which further mediation the progression of steatosis to fibrosis. -Jun N-terminal
Kinase 1 (JNK1) a stress activated protein kinase is activated by TNF-α which leads to
initiation of an autocrine/paracrine loop resulting in enhanced TNF- α production in liver.
TNF- αα activates inhibitory kappa b kinase β (IKKβ) which resulting in the translocation
of NFκB into nucleus
Text C
Transforming growth factor β
TGF-β is a cytokine secreted by hepatocytes and Kupffer cells in response to degradation
changes in liver. TGF β activates the resting stellate cell by transforming them into active
myofibroblasts, which secrete extracellular matrix protein to initiate the fibrosis process.
Earlier studies have shown up regulated expression of TGF-β following experimentally
induced hepatic damage such as in CCl4 poisoning. Elevated TGF have shown in patients
with liver fibrosis. Earlier studies have shown that TGF marker for the progression of
steatohepatitis Hence detection of TGF -β level would be helpful in marking NASH stage
of NAFLD. Studies have observed polymorphisms in TGF-β1 gene in obese NAFLD patients
with advanced stages of hepatic fibrosis. Xiao and Ho have recently reported that
administration of Epigallocatechin gallate (EGCG) reduced hepatic severity in NAFLD by
suppression of TGF/SMAD pathway. Studies with TGF mice showed protective effect
against experimentally induced NAFLD with methionine and choline deficient (MCD) diet
which was mediated through smad2 activation.
Text D
Interleukins
Interleukin-6 is a proinflammatory cytokine which has been implicated in metabolic
syndrome. IL-6 also plays several other functions such as inducer of immune response,
hematopoiesis and oncogenesis. Certain studies have reported IL-6 to be antihepatotoxic
and mediate hepatic regeneration after partial hepatectomy in mice. IL-6 has also been
considered to reduce hepatic oxidative stress and to curtail mitochondrial dysfunction
role of IL-6. However, studies have reported the role of IL 6 as an acute phase
inflammatory mediator leading to secretion of inflammatory serum proteins. Hence, the
possibility of its role in pathogenesis of NAFLD cannot be ignored. A positive correlation
has been found in patients with NASH and circulating IL 6 knockout mice models have
level shown reduced severity when subjected to experimental NAFLD. Yamaguchi et al,
has shown that inhibition of IL-6 receptor with Tocilizumab enhanced hepatic steatosis
but protected against extensive hepatic damage in MCD diet induced NASH. Recently,
Hamirani et al, have observed a positive correlation between C reactive protein (CRP) and
IL6 levels and increased liver fat accumulation in NAFLD patients as verified with CT
PART A -QUESTIONS AND ANSWER SHEET
Questions 1-7
For each question, 1-7, decide which text (A, B, C or D) the information comes from.
You may use any letter more than once.

In which text can you find information about;


1. Biological cell that synthesizes the extract collagen. Answer _____________

2. Arranged in a radiating pattern like that of a star. Answer _____________

3 .Production or formation, development, and differentiation of blood cells. Answer ____________

4 .Evokes a specific functional reaction in an organ or tissue. Answer ______________

5. Hepatoprotection in most of the cases d. Answer ______________

6. Impairment of the normal processes of synthesis and elimination of triglyceride fat. Answer __

7. The tendency towards a relatively stable equilibrium between interdependent elements, especially
as maintained by physiological processes. Answer _____________

Questions 8-14
Answer each of the questions, 8-14, with a word or short phrase from one of the texts.
Each answer may include words, numbers or both. Your answers should be correctly spelt.

8 What plays an active role in mediating the inflammatory progression of NAFLD Answer ___

9 What is the term used to indicate the thickening and scarring of connective tissue? Answer ___

10 What does the TNF-α level in serum indicate in patients with NAFLD? Answer ______________

11 What reduced the hepatic severity in NAFLD patients? Answer ______________

12 What does IL-6 secrete, as an acute phase inflammatory mediator? Answer ______________

13 What is known to be enhancing or contributing more to TNF- α? Answer______________

14 What is the terms used to show the derivative, which decreases the viscosity of blood?___
Questions 15-20

Complete each of the sentences, 15-20, with a word or short phrase from one of the texts.
Each answer may include words, numbers or both. Your answers should be correctly spelt.

15. _____________ relates to a hormone which has effect only in the vicinity of the gland secreting it.

16. It can be said that disturbances in the normal functioning of the ER often lead to cell____ response.

17. ___________ also plays is reported to initiate immune response.

18. ____________ delivered results as expected

19. ______ is characterized by inflammation of the liver with concurrent fat accumulation in liver.

20 . _____ denotes a cell-produced substance that has an effect on the cell by which it is secreted

END OF PART A, THIS QUESTIONS PAPER WILL BE COLLECTED


READING SUB-TEST : PART B
In this part of the test, there are six short extracts relating to the work of health professionals .
For questions 1-6, choose the answer (A, B or C) which you think fits best according to the text.
Write your answers on the separate Answer Sheet
Questions 1-6

1. The given notice talks about;


A. US Death Rate with respect to maternity cases.
B. Scope and importance.
C. How to deal with preventable mortality?

Variety of challenges by Obstetricians and Gynecologists

The biggest challenge lies in managing the care of patients of all ages from puberty
through menopause and beyond; treating both acute and chronic health conditions;
and of course, managing risk through the pregnancy and childbirth process.
The United States has a higher ratio of maternal deaths than at least 40 other countries,
even though it spends more money per capita on maternity care than any other.
The lack of a comprehensive, confidential system of ascertainment of maternal
death designed to record and analyse every maternal death continues to subject
U.S. women to the unnecessary risk of preventable mortality. Maternal deaths
must be reviewed to make motherhood safer.

The status of maternal deaths in the United States was part of a larger report on the
global, regional, and national levels and causes of maternal mortality from 1990-
2015. The findings suggest that only 16 countries will achieve a target of a 75% reduction
in the maternal mortality ratio (or number of maternal deaths per 100,000 live births) by
2017.
2. As per the case study, pick the right statement;
A. The baby was born with the infection.
B. The infection surfaced itself when the child turns 1 year 6 months.
C. The child was normal at the age of 1 year and 6 months.

Case Study

The mother of the other patient was a white 30-year-old housewife. This was her
second pregnancy, it was planned and there had been an ovarian infection during
prenatal and she had been carrying twins, but one fetus had died. Delivery was
vaginal, birth weight was 1,260 g, gestational age was 30 weeks, SNAPE-PE was 0
and sex was male. The child presented petechiae at birth and persistent tachypnea
for several days. At 1 year and 6 months he was still excreting CMV in urine, had
normal neurological development for his corrected age and his sight and hearing
examination findings were normal.
3. Pick the right statement;
A. Improper care during pregnancy can lead to complications.
B. Continous examination is necessary to thwart complications.
C. Major complications will always lead to death.

Complications

Some complications are common to every expecting mother while others can be
specific to limited expecting females only. According to the statistics, it is found
that 90% of the females in some or the other stages of pregnancy have
knowledgeable complications. While some difficulties are easy on the body like
mild nausea and morning sickness but other complications like asthma, diabetes,
thyroid diseases and hypertension necessities to be taken care of under the
excellent guidance of gynecologists and obstetrics. Some of the medical
complications develop to be fatal to the fetus as well as the mother. Women
suffering from other medical problems like HIV, Urinary Tract Infections, Ectopic
Pregnancies and it’s should go for immunization and medical assistance from time
to time to keep the intensity of the complications under control. Some of the
medical complications like Diabetes and Hypertension are so chronic that
continue to persist in the body even after the liberation of the baby. Shunning
smoking, drinking alcohol and maintaining a healthy lifestyle decreases the
chances of having complications in pregnancy.
4. The notice gives information about;
A. Indications of splenectomy.
B. Percentage of disease conditions.
C. Various conditions, which arise due to Splenectomy.

ELECTIVE SPLENECTOMY
ITP : 57 %
Congenital spherocytosis : 12 %
Hemolytic anemia : 10 %
Hodgkin’s disease : 5 %
AIDS related thrombocytopenia : 3 %
Lymphoma : 3 %
Leukemia: 2.5 %
Others (sarcomas, splenic metastases,…)
2. SPLENECTOMY IN EMERGENCY:
Trauma

5. According to Markov model:


A. Screening for NASH in Primary Care shall be simple.
B. Screening for NASH in Diabetic Care or Clinics shall be simple.
C. Use of ultrasound is not fully recommended.

Screening For NAFLD


A recent, cost-effective analysis using a Markov model suggested that screening for NASH
in individuals with diabetes is not cost-effective at present, because of disutility associated
with available treatment. Given that liver biochemistries can be normal in patients with
NAFLD, they may not be sufficiently sensitive to serve as screening tests, whereas liver
ultrasound or TE are potentially more sensitive, but their utility as screening tools is
unproven. Some experts recently have called for “vigilance” for chronic liver disease (CLD)
in patients with type 2 diabetes, but not routine screening.
6. Pick the incorrect statement;
A. Bil (cut-off): Bil<1.5; 1.5 ≤ Bil ≤ 2.5; Bil>2.5; scores 1, 2, 3 respectively.
B. Alb (cut-off):Alb>3.5; 2.5 ≤ Alb ≤ 3.5; Alb<2.5; scores 1, 2, 3 respectively.
C. Platelets (cut-off):Plt<100; 100 ≤ Plt ≤ 150; Plt ≥150;scores 1,2,3 respectively
READING SUB-TEST : PART C
In this part of the test, there are two texts about different aspects of healthcare.
For questions 7-22, choose the answer (A, B, C or D) which you think fits best according to the text.
Write your answers on the separate Answer Sheet

Text 1: Brain Imaging Technologies


Brain imaging has greatly advanced in the last 20 years, due to a better understanding of
the electromagnetic spectrum and radiofrequency waves, in relation to protons in indivi
molecules within the cells of the brain. New technologies allow non-invasive spatial
mapping, (morphology), and observations of processes within the brain during set tasks.
sequencing scanned sections of the brain, activity between neurons in different parts of the
brain can be observed and monitored. More recent technologies using a higher frequency
resolution can identify the distribution of individual metabolites (large complex molecules), a
pharmaceutical drugs. There are a number of scanning techniques, their purpose and
limitations are described below.

A computerised tomography (CT) scans use X-rays to show the structure of the brain,
with details such as blood perfusion, (plates a and b); the resultant images are two
dimensional and of comparatively low resolution, however, the quality has been
much improved since 1998. With improved technology, the single section has now
become as multisection and the speed has increased eight times, giving well-defined
3-D pictures. A CT scan may reveal underdeveloped parts of the brain or sites of
injury from impact, tumours, lesions or infection. Before a CT scan, the patient may
drink but is asked not to eat for four hours beforehand, and not to take strenuous
exercise. A CT brain scan, the preferable scanning method by doctors, will take about
30 minutes and the patient must lie still for the duration.

An MRI scanner uses a strong magnetic field and radio waves to create pictures of
the tissues and other structures inside the brain, on a computer. The magnetic field
aligns the protons (positively charged particles) in hydrogen atoms, like tiny magnets. S
bursts of radio waves are then sent to knock the protons out of position, and as they
realign, (relaxation time), they emit radio signals which are detected by a receiving device
the scanner. The signals emitted from different tissues vary, and can, therefore, be
distinguished in the computer picture. An MRI scanner can create clear detailed pictures
of the structure of the brain and detect any abnormalities or tumours. Sometimes a dye, or tracer
such as gadolinium
may be introduced via a vein in the arm, to improve contrast in the image. Images
can be enhanced by differences in the strength of the nuclear magnetic resonance
signal recovered from different locations in the brain.

Functional magnetic resonance imaging (fMRI) can show which part of the brain
is active, or functioning, in response to the patient performing a given task, by
recording the movement of blood flow. All atoms and molecules have magnetic
resonance, emitting tiny radio wave signals with movement, because they contain
protons. Different molecules have different magnetic resonance and two
components of blood are tracked to observe brain activity. Haemoglobin in the
blood carries oxygen; oxyhaemoglobin, around the brain and when it is used up, it
becomes deoxyhaemoglobin. Where the oxygen is being ‘used up’ shows the site
of activity in the brain. The picture is made by monitoring the ratio of the tiny
wave frequencies between these two states whilst the patient carries out a task, e.g.
tapping a finger, which highlights the area of the brain functioning to carry out this task.

Positron emission tomography (PET) scanning produces a three-dimensional image


of functional processes in the brain, (not just the structure). PET is a nuclear
medicine imaging technique, which requires the patient to receive a small injection of
radioactive material (a sugar tracer; fluorodeoxyglucose), into the bloodstream. The
radioactive material causes the production of gamma-rays, these are a form of
electromagnetic radiation like X-rays, but of higher energy. The radioactive material
is transported around the body and into the brain. A ring of detectors
outside the head is used to detect pairs of gamma rays emitted indirectly by the
positron-emitting radionuclide (tracer), in each part of the brain under examination.

The single photon emission computed tomography records the signals from
gamma rays, (singly, rather than when the emissions are opposite at 180º), using
two or more synchronised gamma cameras, and the multiple 2-D images are
computed, tomographically reconstructed, to 3-D. A section may be examined
from several angles, but is slightly less clear than a PET image. A SPECT scanner
is less expensive than a PET scanner and uses longer-lived, more easily obtained
radioisotopes.
Text 1: Questions 7-14
7. According to paragraph 1, technology;

A. Has made it easier to detect what is going on in a person brain


B. Helps define how neurons act and multiply
C. Makes the mapping of the brain a very simple task
D. Can help identify how big complex parts are distributed

8. According to paragraph 2, what is true about CT?

A. It works on the principles of X-rays


B. It provides two-dimensional images
C. It can help identify which parts of the brain are developed and which are not
D. It will always just take half an hour

9. The word eitty-bitty in paragraph 3 may means;

A. Effective
B. Gigantic
C. Small
D. Strong

10. According to paragraph 3, what is not true about an MRI?

A. During the scan, the tissues produce different signals and thus easily get identified.
B. Dye may be used to improve the quality of the image produced by the scan.
C. Protons emit radio signals.
D. The receiving device collects the emitted protons.

11. According to paragraph 4, a fMRI scan;

A. Produces more efficient images of the brain than an MRI.


B. Can detect which parts of the brain are active during movement.
C. Obtains the picture by closely analysing the frequencies of the emitted waves.
D. Not given
12. According to paragraph 5, a PET is;

A. An advanced technique like an MRI.


B. A non-magnetic technique for the detection of brain functioning.
C. A technique in which a patient is given a radioactive material injection.
D. Much more powerful than other techniques.

13. According to paragraph 6, what is true about a SPECT?

A. It produces quality images like an MRI


B. It is similar to a PET in imaging
C. It relies on gamma rays for detection
D. Not given

14. According to the information provided, which technique is considered the best of all by doctors?

A. MRI
B. fMRI
C. PET
D. SPECT
Text 2: A safer way to detect heart disease
Researchers have used a specialized type of MRI to detect 88% of cases of
coronary artery disease in a group of patients with chest pain. The results suggest
that the imaging technique can detect heart disease as accurately as conventional
methods, but with much less risk. Coronary artery disease is the most common
form of heart disease and the leading cause of death in the United States. It
occurs when fat and calcium accumulate in the arteries that supply blood to the
heart. Over time, less blood reaches the heart and heart muscle dies. If the plaque
blocks the arteries completely, a heart attack occurs.

Currently, the best way to detect the disease is through a coronary angiography. A
physician threads a tube into the heart, releases a dye, and uses X-ray images to
look for decreased blood flow. But there’s a small risk in this procedure that the
tube will pierce an artery, resulting in bleeding, or else scrape plaque from artery
walls, which, once the chunks of plaque are in the bloodstream, can lead to a heart
attack or stroke. Other, noninvasive tests such as cardiac ultrasounds are less
risky, but not as accurate. Ultrasound images can be poor in patients with other
conditions such as obesity, requiring doctors to resort to invasive tests.
While an MRI allows doctors to image the body using magnets and radio waves,
until recently it could not produce clear images of dynamically voyaging objects,
such as a beating heart. In the past two years, though, stronger magnets, more
powerful computers, and new software have improved MRI scanning. “Recent
developments allow us to acquire images of the heart in motion,” says Ricardo
Cury, director of clinical cardiac MRI at Massachusetts General Hospital in Boston
and leader of the study. Doctors can now watch the heart beating in real-time and
the images are now sharp. “It’s like opening up the heart and looking at it
directly,” says Renato Santos, a cardiologist at Wake Forest University Baptist
Medical Center. “Until recently, MRI was a research tool,” says Santos. “Now it’s
really a clinical tool…ready for prime time.”
Cury combined two cardiac MRI tests to improve the technique’s ability to
diagnose coronary artery disease. In his study, published in the July issue of
Radiology, researchers at MGH, Harvard Medical School, and Beneficencia
Portuguesa Hospital in Sao Paulo, Brazil, examined 46 patients. They began with
an MRI stress test, injecting a harmless dye and medicine that stresses the heart.
As the heart pumped, they used MRI to look for decreased blood flow or evidence
that the heart was working abnormally. Next, they examined still MRI images of
the heart for damaged areas or evidence of prior heart attacks. If patients were
abnormal in one or both tests, the doctors deduced blocked arteries.

Cury’s results reinforce those of an earlier, unrelated study in which researchers


at Duke University used the same techniques to successfully diagnose coronary
artery disease in 100 patients. The results are good news for patients. The MRI
exam is short and painless. By using it to triage people with chest pain but
actually without any disease, physicians might save such patients from
unnecessary invasive procedures. In cases where the heart disease is evident,
MRI can help doctorsdecide what to do next, for instance, whether surgery to
clear or bypass a blocked artery is necessary. After surgery, doctors can use MRI
to monitor arteries for future blockages noninvasively. Cury says that the 12
percent of cases misdiagnosed in his study are less than other noninvasive tests,
and in certain cases artifacts of the study’s design. He adds that MRI’s accuracy
will increase as doctors learn to make better diagnoses from MRI images.
“Obviously 100% is ideal,” says Santos. “I think MRI is going to get us closer to
that than our traditional methods.”
Text 2: Questions 15-22
15. Paragraph 1 talks more about;

A. breakthrough in imaging.
B. How heart diseases can be detected.
C. How new techniques are more efficient than traditional techniques.
D. How a heart attack occurs.

16. According to paragraph 1, a simple cause of a heart attack is;

A. Heart doesn receive enough blood.


B. Fat gets deposited in arteries.
C. Arteries carry less blood to the heart.
D. All of these

17. What risk is mentioned in paragraph 2?

A. The tube may puncture the artery.


B. The tube can cut off plaque.
C. The artery walls may release more plaque into the bloodstream.
D. All of these

18. Paragraph 2 talks more specifically about;

A. Techniques that are risky.


B. Why conventional techniques should not be used.
C. How expensive techniques are.
D. How risky CA can be.

19. According to paragraph 3, what is true about MRI scanning?

A. MRI scans in previous years were less effective.


B. It was not possible to capture motion-based images.
C. MRI scans of the new generation are more effective with power imaging qualities.
D. use of powerful computers and software have made MRI scanning more effective.
20. According to paragraph 3, what is possible with MRI now?

A. It is easy to see the heart, live in action


B. It is easy to track heart beating
C. It is easy to get more quality-based images
D. All of these

21. According to paragraph 4, which one of the following statements is true?

A. Cury led a team of doctors and researchers at MGH.


B. Cury detected blocked arteries.
C. The team of doctors compared 46 case studies.
D. Researchers began with the stress testing.

22. In paragraph 5, the word triage may mean;

A. To help
B. To examine
C. To show
D. To provide comfort

END OF READING TEST, THIS BOOKLET WILL BE COLLECTED


Reading test 13: Answer Key

Part A - Answer key 1 – 7


1: B
2: C
3: A
4: A
5: D
6: D
7: A

Part A - Answer key 8 – 14


8: cytokines
9: fibrosis
10: degree of fibrosis
11: Epigallocatechin gallate
12: inflammatory serum proteins
13: hepatocytes and Kupffer cells
14: Pentoxifylline

Part A - Answer key 15 – 20


15: Paracrine
16: Stress
17: Il-6
18: Tnf-± drug Therapy
19: Steatohepatitis
20: Autocrine
Reading test - part B – answer key
1. Is: Scope and importance.
2. The baby was born with the infection.
3. Continous examination is necessary to thwart complications.
4. Indications of splenectomy.
5. Screening for NASH in Diabetic Care or Clinics shall be simple.
6. Platelets (cut-off):Plt<100; 100 ≤ Plt ≤ 150; Plt ≥150; scores 1, 2, 3

Reading test - part C – answer key

Text 1 - Answer key 8 – 14


7: Can help identify how big complex parts are distributed
8: It can help identify which parts of the brain are developed and which are not
9: Small
10 Is: The receiving device collects the emitted protons.
11: Can detect which parts of the brain are active during movement.
12: A technique in which a patient is given a radioactive material injection.
13: It relies on gamma rays for detection
14: MRI

Text 2 - Answer key 15 – 22


15: A breakthrough in imaging.
16: All of these
17: The tube may puncture the artery.
18: Techniques that are risky.
19: It was not possible to capture motion-based images.
20: It is easy to get more quality-based images
21: Researchers began with the stress testing.
22: To examine
READING TEST 14
READING SUB-TEST : PART A
 Look at the four texts, A-D, in the separate Text Booklet.
 For each question, 1-20, look through the texts, A-D, to find the relevant information.
 Write your answers on the spaces provided in this Question Paper.
 Answer all the questions within the 15-minute time limit.
 Your answers should be correctly spelt.

TEXT BOOKLET - CANNABIS USE DISORDER

Text A

Cannabis use disorder

Cannabis use disorder is a problematic pattern of cannabis use which leads to


impaired control over cannabis use and difficulty in ceasing use despite its harm.
Drug abuse is a global phenomenon affecting almost every country, with cigarettes,
cannabis, and alcohol being the most commonly used and abused substances.
Among them cannabis is illegal. As compared to other psychoactive substances cannabis,
most commonly known as marijuana, is the most widely used drug worldwide.
Even though there is no international consensus, young adulthood is a period where the
transition takes place from childhood to adulthood. Arnett et al. define this group as
“emerging adults” from 18 to 25 years, those who are neither children nor adults and
who are in between with their own identity and behavior. It is the age of instability, self-
focus, feeling in between, and possibilities. Young adults are the most vulnerable group
of people than any age group to cannabis dependence and related problems which
produce more years lived with disabilities. In 2013, cannabis was used by 80.6% of
current illicit drug users in the United States.
Text B

Globally 2% cause-specific disability-adjusted life years (DALYs) for young


people are attributed to illicit drug including cannabis. The study conducted
revealed that cannabis and other illegal drugs accounted for approximately $8.2
billion of the nearly $40 billion cost of substance abuse in Canada in 2002. The
probability of cannabis addiction in heavy or daily user is enormous. The
vulnerability increases in adolescents whose risk of addiction is 16%, while adults
have 5–10% risk of becoming addicted. Canadian community health survey
reported that the prevalence of cannabis dependence among adolescent and young
adult in 2012 was 5%. A three-year prospective study in the Netherlands on high risk
young adults reporting heavy use in 2013 found that nearly 40% developed
cannabis dependence.

Text C

Another study in Holland revealed that cannabis dependence was 42%. In 2013 the
prevalence of cannabis abuse or dependence was 7.4% among youth in the USA
and the rate was about half (3.55) among adolescents. National Survey on Drug
Use and Health (NSDUH) revealed that cannabis was the illicit drug with the
largest number of persons with past-year dependence or abuse in 2013. Of the 6.9
million persons aged 12 or older who were classified with illicit drug dependence
or abuse in 2013, 4.2 million persons had cannabis dependence or abuse. Another
study conducted in the USA reported that 38.5% of daily cannabis users met
criteria for cannabis dependence. A longitudinal cohort study conducted in
Australia in 2002 among young adults shows a 7% prevalence of cannabis
dependence according to DSM-IV criteria for cannabis dependence. A community
household-based survey with a cross-sectional design in Rwanda that aimed to
determine the prevalence of cannabis dependence among adolescent and young
adults shows 2.54% prevalence of cannabis dependence.
Text D

A recently published (2015) cohort study which considered cannabis abuse and
dependence as cannabis use disorder (CUD) showed the prevalence of cannabis
use disorder throughout the life to be 19.1%, with an average age of onset of 18.6 years.
Cannabis availability, regular use of cannabis, peer pressure, and
common mental disorder were factors having a significant association with
cannabis use disorder in different studies. Gateway hypothesis developed by
Kandel explained that the sequence of drug use occurring starts with legal drug and
proceeds to illegal drugs. Above all, Shashemene is a town in which Rastafarians view
Ethiopia as a promised land live. Cannabis use is a commonpractice among Rastafarians
which brought a major challenge to both youth and law enforcement in the town.
PART A -QUESTIONS AND ANSWER SHEET
Questions 1-7
For each question, 1-7, decide which text (A, B, C or D) the information comes from.
You may use any letter more than once.
In which text can you find information about;
1.increase of the drug influence. Answer ____________
2.Use of the drug in the USA. Answer _____________
3.life time prevalence of cannabis use disorder. Answer ______________
4.use of cannabis despite clinically significant distress or impairment. Answer ________
5. common determinants of drug addiction. Answer ______________
6.addiction to drug common among young community. Answer ________________
7.Prevalence of the drug abuse, drug usage. Answer ________________
Questions 8-14
Answer each of the questions, 8-14, with a word or short phrase from one of the texts.
Each answer may include words, numbers or both. Your answers should be correctly spelt.
8. From how many places data on cannabis addiction has been developed? Answer ____
9. As per the report, how many people in Canada are reported to be cannabis
addict in the first quarter of the 2nd decade of 21st century? Answer _______________
10. How many people are known to be addicted to cannabis as per the report of 2013? Answer _
11. Who are likely to be addicted more commonly? Answer _____________
12. Who are popular for making use of cannabis as its use is customary? Answer ____
13. As per the report, how many people in the USA are reported to be cannabis addict? Answer __
14. What revealed a 7 percent of young adults were cannabis dependent in Australia during 2002?
Answer ______________
Questions 15-20
Complete each of the sentences, 15-20, with a word or short phrase from one of the texts.
Each answer may include words, numbers or both. Your answers should be correctly spelt.

15. In the United States, in 2013, the drug was used by___________ of people.

16. Commonly, adults may have about_________ risk of becoming a drug addict

17. The studies in the landlocked East African country shows __________ of cannabis dependence.

18. The sequence of the drug use may end with____________.

19. It is very much likely that____________ is of higher level among drug addicts

20. _____________ explained that the sequence of drug use occurring starts with legal drug and
proceeds to illegal drugs.

END OF PART A, THIS QUESTIONS PAPER WILL BE COLLECTED


READING SUB-TEST : PART B
In this part of the test, there are six short extracts relating to the work of health professionals .
For questions 1-6, choose the answer (A, B or C) which you think fits best according to the text.
Write your answers on the separate Answer Sheet
Questions 1-6

1. Chronic urticaria;
A. Is caused by an allergic reaction to a food or drug.
B. Arises spontaneously and its cause is unknown.
C. Is a genetic disorder that is very rare.

Chronic urticarial

Chronic urticaria is a common condition characterized by recurrent hives lasting


several weeks or months and is usually idiopathic. Approximately half of the
individuals with chronic urticaria will present with episodes of angioedema that
can be severe and debilitating. There is a 47-year-old Hispanic male who presented
initially for an evaluation of chronic hives following hospitalization due to
hiveinduced anaphylaxis. The individual had a history significant for urticaria and
angioedema beginning in his early 30s. Interestingly, both the individual’s 41-
yearold sister and 12-year-old daughter were also affected by chronic urticaria and
severe angioedema. Whole exome sequencing of the proband and several family
members revealed a heterozygous variant of uncertain significance in exon 2 of
TNFAIP3, denoted as c.65G>A (p.R22Q), in all affected members.
2. Arterial aneurysms;
A. Are caused by narrowing of arteries.
B. are abnormal dilations of the peripheral arteries.
C. Occur due to change in structure and function of the Fibrillin-1.

Arterial aneurysms

Arterial aneurysms are most commonly caused by atherosclerotic disease,


especially in elderly patients aged over 60. Other etiologies, such as connective
tissue disorders, should be investigated in younger patients.. Fibrillin-1 microfibrils,
through interactions with elastin and other proteins, provide structure to elastic
and nonelastic connective tissues. In addition to the architectural functions,
Fibrillin-1 plays an important role in regulating TGF-β complexes in the extracellular
matrix. TGF-β signaling controls various processes at the cellular level, such as
cellular growth, differentiation, and apoptosis. When Fibrillin-1 is defective, it
disrupts the normal architecture of connective tissues.
3. What is correct?
A. PE occurs when a clump of material, most often a blood clot, gets wedged into
an artery in your lungs.
B. Patients with DVT are susceptible to PE.
C. PE symptoms, in most of the cases, do not get revealed easily.

Pulmonary embolism

Pulmonary embolism (PE) is regarded as an elusive diagnosis with a non-specific


clinical presentation and has a tendency to be both over- and underdiagnosed in
clinical practice. In the United States of America, venous thromboembolism (VTE)
has been reported as the 3rd commonest cause of mortality.
Most patients with PE are clinically asymptomatic. In fact, PE has been shown to be
present in 60–80% of individuals with confirmed deep vein thrombosis (DVT),
despite the absence of symptom in more than half of these patients. Cardiac arrest
following PE has an associated mortality of up to 70% within the first hour of
presentation and an overall mortality of up to 95%. Approximately 90% of episodes
of cardiac arrests occur within 1-2 hours after the onset of symptoms of PE.
4. What is the notice talking about?
A. Cleaning of the blood of toxins.
B. A major a superficial vein in the arm and disease associated with it.
C. A case report.

Clinically, the cephalic vein is preferred for haemodialysis in patients with chronic
renal failure (CRF), to remove waste products from the blood. The cut-down of the
cephalic vein in the deltopectoral groove is preferred when superior vena caval
infusion is necessary. However, cephalic veins exhibit a wide array of developmental
variations in terms of formation, course, and termination.
During routine gross anatomy dissection of the neck of the patient, a rare case of
variation of the termination of the cephalic vein in both right and left upper limbs have
been observed. Knowledge of the variations of cephalic vein is important not only for
anatomists but also for surgeons and clinicians as the vein is frequently used for
different surgical procedures and for obtaining peripheral venous access as well.
5. What does the table indicate?
A. Tenofovir goes well in coordination with Cycloferon.
B. Cycloferon goes well in coordination with Tenofovir.
C. Adefovir goes well in coordination with Cycloferon.

% of total % of dual Dual


Nb
treatment ( ) therapy ( ) therapies
Cycloferon + Adefovir 55 7.9% 43.3%
Cycloferon + Lamivudine
42 6.1% 33.0%

Adefovir + Lamivudine
9 1.3% 7.1%

Cycloferon + Tenofovir
5 0.7% 4.0%

Tenofovir + Adefovir
4 0.6% 3.2%

Tenofovir + Lamivudine
4 0.6% 3.2%

Pegasys + Tenofovir
3 0.4% 2.4%

Pegasys + Adefovir
2 0.3% 1.6%

Entecavir + Tenofovir
1 0.1% 0.8%

Interferon + Cycloferon
1 0.1% 0.8%

Entecavir + Pegasys
1 0.1% 0.8%

Total 126 18.2%


6. The products which are good moisturizers are marketed by;
A. NOW Solutions, Microfluidics, Decorte.
B. Microfluidics, Decorte, Dior.
C. Microfluidics, Decorte.

Marketed formulations of liposomes


Product name Marketed by Uses

Removes wrinkles and dark


Capture Totale Dior spots and has radiance
effect with sunscreen
Microfluidi
Dermosome Moisturizer
cs
Decorte Moisture Liposome
Decorte Moisturizer
Face Cream

Decorte Moisture Liposome Moisturizes, firms, and brightens


Decorte
Eye Cream the delicate skin around the eyes

Natural Progesterone
NOW Maintenance of healthy
Liposomal
Solutions feminine balance
Skin Cream
Hydration, boosts collagen
C-Vit
synthesis, enhances the skin’s
Liposomal Sesderma
elasticity and firmness, and
Serum
brightens the complexion

Advanced Night Repair Estée


Skin repair
Protective Recovery Complex Lauder
READING SUB-TEST : PART C
In this part of the test, there are two texts about different aspects of healthcare.
For questions 7-22, choose the answer (A, B, C or D) which you think fits best according to the text.
Write your answers on the separate Answer Sheet

Text 1: What is Creutzfeldt-Jakob Disease?

Creutzfeldt-Jakob disease is a degenerative brain disorder that leads to dementia


and, ultimately, death. Symptoms of Creutzfeldt-Jakob disease (CJD) sometimes
resemble those of other dementia-like brain disorders, such as Alzheimer`s, but
Creutzfeldt-Jakob disease usually progresses much more rapidly. Creutzfeldt Jakob
disease captured public attention in the 1990s when some people in the United
Kingdom developed a form of the disease — variant CJD (vCJD) — after eating meat
from diseased cattle. However, "classic" Creutzfeldt-Jakob disease has not been
linked to contaminated beef. Although serious, CJD is rare, and vCJD is the least
common form. Worldwide, there is an estimated one case of Creutzfeldt Jakob
disease diagnosed per million people each year, most frequently in older adults, if
not among children.

Creutzfeldt-Jakob disease is marked by rapid mental deterioration, usually within a


few months. Initial signs and symptoms of CJD typically include personality
changes, anxiety, depression, memory loss, impaired thinking, blurred vision,
insomnia, difficulty speaking, difficulty swallowing and sudden, jerky movements.
As the disease progresses, mental symptoms worsen; most people eventually lapse
into a coma. Heart failure, respiratory failure, pneumonia or other infections are
generally the cause of death. The disease usually runs its course in about seven
months, although a few people may live up to one or two years after diagnosis. In
people with the rarer CJD, psychiatric symptoms may be more prominent in the
beginning, with dementia — the loss of the ability to think, reason and remember -
developing later in the course of the illness. In addition, this variant affects people
at a younger age than classic CJD does, and appears to have the slightly longer
duration of 12 to 14 months.
Creutzfeldt-Jakob disease and its variants belong to a broad group of human and
animal diseases known as transmissible spongiform encephalopathies (TSEs). The
name derives from the spongy holes, visible under a microscope, that develops in
affected brain tissue. The cause of Creutzfeldt-Jakob disease and other TSEs
appears to be abnormal versions of a kind of protein called a prion. Normally,
these proteins are harmless, but when they`re misshapen they become infectious
and can wreak havoc on normal biological processes. The risk of CJD is low. The
disease can`t be transmitted through coughing or sneezing, touching, or sexual
contact. The three ways it develops are: Sporadically: most people with classic
CJD develop the disease for no apparent reason. CJD that occurs without
explanation is termed spontaneous CJD or sporadic CJD and accounts for the
majority of cases. By inheritance: in the United States, about 5 to 10 percent of
people with CJD have a family history of the disease or test positive for a genetic
mutation associated with CJD. This type is referred to as familial CJD. By
contamination: a small number of people have developed CJD after being exposed
to infected human tissue during a medical procedure, such as a cornea or skin
transplant. Also, because standard sterilization methods do not destroy abnormal
prions, a few people have developed CJD after undergoing brain surgery with
contaminated instruments. Cases of CJD related to medical procedures are referred
to as iatrogenic CJD. Variant CJD is linked primarily to eating beef infected with
bovine spongiform encephalopathy (BSE), the medical term for mad cow disease.

Most cases of Creutzfeldt-Jakob disease occur for unknown reasons, and no risk
factors can be identified. However, a few factors seem to be associated with
different kinds of CJD: Age: sporadic CJD tends to develop later in life, usually
around the age of 60. The onset of familial CJD occurs only slightly earlier. On the
other hand, vCJD has affected people at a much younger age, usually in their late
20s. Genetics: people with familial CJD have a genetic mutation that causes the
disease. The disease is inherited in an autosomal dominant fashion, which means
you need to inherit only one copy of the mutated gene, from either parent, to
develop the disease. If you have the mutation, the chance of passing it on to your
children is 50 percent. Genetic analysis in people with iatrogenic and variant CJD
suggests that inheriting identical copies of certain variants of the prion gene may
predispose a person to develop CJD if exposed to contaminated tissue. Exposure to
contaminated tissue: people who`ve received HGH derived from human pituitary
glands or who`ve had dura mater grafts may be at risk of iatrogenic CJD. The risk
of contracting vCJD from eating contaminated beef is difficult to determine. In
general, if countries are effectively implementing public health measures, the risk
is virtually non-existent.

Only a brain biopsy or an examination of brain tissue after death (autopsy) can
confirm the presence of Creutzfeldt-Jakob disease. But doctors can often make an
accurate diagnosis based on your medical and personal history, a neurological
exam, and certain diagnostic tests. The exam is likely to reveal such characteristic
symptoms as muscle twitching and spasms, abnormal reflexes, and coordination
problems. People with CJD may also have areas of blindness and changes in
visual-spatial perception. In addition, doctors commonly use the following tests to
help detect CJD: Electroencephalogram (EEG): using electrodes placed on your
scalp, this test measures your brain`s electrical activity. People with CJD and vCJD
show a characteristically abnormal pattern. Magnetic resonance imaging (MRI):
this technique uses radio waves and a magnetic field to create cross-sectional
images of your head and body. It`s especially useful in diagnosing brain disorders
because of its high-resolution images of the brain`s white matter and gray matter.
Spinal fluid tests: cerebrospinal fluid surrounds and cushions your brain and spinal
cord. In a test called a lumbar puncture — popularly known as a spinal tap -
doctors use a needle to withdraw a small amount of this fluid for testing. The
presence of a particular protein in spinal fluid is often an indication of CJD or vCJD.
No effective treatment exists for Creutzfeldt-Jakob disease or any of its variants. A
number of drugs have been tested - including steroids, antibiotics and antiviral
agents - and have not shown benefits. For that reason, doctors focus on alleviating
pain and other symptoms and on making people with these diseases as comfortable
as possible.
Text 1 : Questions 7-14
7. Which disease progresses faster?
A. Alzheimers
B. Jakob
C. Both Alzheimer`s and Jakob
D. Not given

8. Creutzfeldt-Jakob disease is commonly found among adults or elderly people;


A. False
B. True
C. False, because it is found among children too.
D. Not given

9. People affected with the disease may die after;


A. One year
B. Two years
C. 12-14 months
D Not given

10. Paragraph 3 talks more about;


A. Symptoms
B. Occurrence
C. Transmission
D. Prevention

11. One of the most common risk factors includes;


A. Exposure to contaminated tissue.
B. Age
C. Genetics
D. B and C
12. People who may have the human growth hormone derived from human pituitary glands
may be at risk of iatrogenic CJD.
A. 100% true
B. 100% false
C. 50% true
D. 50% false

13. Confirmation of the Creutzfeldt-Jakob disease can be done only after the death of the person.
A. True
B. False
C. True in some cases
D. Not given

14. What is most helpful in detecting CJD?


A. Electroencephalogram (EEG)
B. Spinal fluid exams
C. MRI
D. All of the above
Text 2: Heat Rash

The skin`s job is to protect the inside of the body from the outside world. It acts as
a preventive barrier against intruders that cause infection, chemicals, or ultraviolet
light from invading or damaging the body. It also plays an important role in the
body`s temperature control. One way that the body cools itself is by sweating, and
allowing that sweat or perspiration to evaporate. Sweat is manufactured in sweat
glands that line the entire body (except for a few small spots like fingernails,
toenails, and the ear canal). Sweat glands are located in the dermis or deep layer of
the skin, and are regulated by the temperature control centers in the brain. Sweat
from the gland gets to the surface of the skin via a duct. A heat rash occurs when
sweat ducts become clogged and the sweat can`t get to the surface of the skin.
Instead, it becomes trapped beneath the skin`s surface causing a mild inflammation
or rash. Heat rash is also called prickly heat or miliaria.

It is uncertain why some people get heat rashes and others don`t. The sweat gland
ducts can become blocked if excessive sweating occurs, and that sweat is not
allowed to evaporate from a specific area. Some examples of how blockage may
occur include the following: creases in the skin, for example the neck, armpit, or
groin which have skin touching adjacent skin, making it difficult for air to
circulate, therefore preventing sweat evaporation; tight clothing that prevents
sweat evaporation; bundling up in heavy clothing or sheets - this may occur when a
person tries to keep warm in the winter or when chilled because of an illness with
fever. Heavy creams or lotions can also clog sweat ducts. Babies have immature
sweat glands that aren`t able to remove the sweat they produce; they can develop
heat rash if they are exposed to warm weather, are overdressed, excessively
bundled, or have a fever. Heat rash may occur as a side effect of some medications,
for example, isotretinoin (Accutane) or clonidine (Catapres)
The most common symptoms of heat rash are red bumps on the skin, and an itchy
or prickly feeling to the skin. These are due to inflammation of the superficial
layers of the skin (the epidermis) and the prickly sensation is similar to the feeling
of mild sunburn. The symptoms of heat rash are the same in infants and adults;
however, since an infant can`t complain about the rash sensation, he or she may be
fussy. Newborns, infants, the elderly, and obese individuals with large areas with
skin-on-skin contact areas (for example, a large overlapping area of abdominal fat
or panniculus) are at risk of developing a heat rash. They are all especially at risk if
they are immobile for long periods of time and parts of the skin aren`t exposed to
circulating air, which results in the inability of the sweat ducts to "breathe"
(evaporative cooling). Heat rashes are more common in places with hot, humid,
climates because people sweat more. Intense exercise associated with lots of
sweating may cause a heat rash, especially if the clothing worn does not allow
adequate air circulation.

The appearance of a heat rash depends upon where the excess sweat gets
deposited in the skin. Tiny blisters that look like small beads of sweat are seen if the
sweat is blocked at the most superficial layers of the skin where the sweat duct
opens on the skin surface. Called miliaria crystallina, it has no symptoms other than
these "sweat bubbles." Classic heat rash or miliaria rubra occurs if the sweat causes
inflammation in the deeper layers of the epidermis. Like any other inflammation,
the area becomes red and the blisters become slightly larger. Because the sweat
glands are blocked and don`t deliver sweat to the skin`s surface, the area involved
is dry and can be irritated, itchy, and sore. This rash is also called prickly heat. Less
frequently, after repeated episodes of prickly heat, the heat rash may inflame the
deeper layer of the skin called the dermis, and cause miliaria profunda. This rash is
made up of larger, harder bumps that are more skin colored. The rash begins almost
immediately after exercise, and again no sweat can be found on the affected areas.
Rarely, this type of heat rash may potentially be dangerous if enough skin is
involved, since the lack of sweating can lead to heat-related illnesses like heat
cramps, heat exhaustion, or heat stroke.
Heat rash or prickly heat is detected by physical examination. Knowing that the
rash appears during sweating or heat, appreciating the location on the body (in skin
creases or where clothes fit tightly) and seeing what the rash looks like is enough
to make the diagnosis. As with many rashes, the health care professional may look
at the involved skin and, because of previous experience, immediately make the
diagnosis. An effective recovery process may depend more on treating heat rash
with remedies such as over-the-counter creams and sprays. Medical treatment for
heat rash may involve antibiotics if the sweat glands become infected.
Text 2 : Questions 15-22
15. Heat rash develops when;
A. Sweat ducts become clogged
B. Sweat can`t come out onto the skin
C. Skin stops developing sweat
D. None

16. One of the most common reasons given for the blockage of the sweat glands is;
A. Excessive sweat is not allowed to evaporate from the skin
B. Creases in the skin which makes circulation difficult
C. Tight clothing
D. Heavy creams and lotions

17. In babies, heat rash often develops due to;


A. Warm weather
B. Overdressing
C. Fever
D. Not given

18. Paragraph 3 talks more about;


A. Risk factors
B. Who is at risk
C. Symptoms
D. Development of the disease

19. Heat rash is common in;


A. Cold areas
B. Areas with higher humidity
C. Desert areas
D. B and C
20. What is the central idea of paragraph 4?
A. Heat rash symptoms.
B. Mode of occurrence.
C. What heat rash looks like.
D. B & C

21. Heat rash may cause miliaria profunda.


A. True in some cases
B. False
C. 100% true
D. Not given

22. Medical treatment for heat rash is effective when the;


A. Blockage is high.
B. Blockage is mild.
C. Blockage is low.
D. Home remedies are ineffective.

END OF READING TEST, THIS BOOKLET WILL BE COLLECTED


Reading test 14 : Answer Key

Part A - Answer key 1 – 7


1: B
2: C
3: D
4: A
5: D
6: A
7: C

Part A - Answer key 8 – 14


8: Six 9
Is: 5%
10: 4.2 Million
11: Young people
12: Rastafarians
13: 7.4%
14: longitudinal cohort study

Part A - Answer key 15 – 20


15: 80.6%
16: 5–10%
17: 2.54% prevalence
18: illegal drugs
19: cannabis addiction
20: Gateway hypothesis
Reading test - part B – answer key
1: B Arises spontaneously and its cause is unknown.
2: A Are caused by narrowing of arteries.
3: B Patients with DVT are susceptible to PE.
4: A Cleaning of the blood of toxins.
5: C Adefovir goes well in coordination with Cycloferon.
6: C Microfluidics, Decorte.

Reading test - part C – answer key

Text 1 - Answer key 7 – 14


7: B Jakob
8: B True
9: D Not given
10: C Transmission
11: D B and C
12: A 100% true
13: C True in some cases
14: D All of the above

Text 2 - Answer key 15 – 22


15: A Sweat ducts become clogged
16: A Excessive sweat is not allowed to evaporate from the skin
17: B Overdressing
18: B Who is at risk
19: B Areas with higher humidity
20: C What heat rash looks like.
21: A True in some cases
22: A Blockage is high.
READING TEST 15
READING SUB-TEST : PART A
 Look at the four texts, A-D, in the separate Text Booklet.
 For each question, 1-20, look through the texts, A-D, to find the relevant information.
 Write your answers on the spaces provided in this Question Paper.
 Answer all the questions within the 15-minute time limit.
 Your answers should be correctly spelt.

TEXT BOOKLET – GAUCHER DISEASE

Text A

Gaucher disease is the most common of the lysosomal storage disorders (LSDs), which are
metabolic conditions caused by genetic defects in the lysosomal system. The lysosome is an
internal cell structure that contains numerous enzymes responsible for degrading complex
cellular components. LSDs result from the absence or deficiency of a lysosomal enzyme and
the subsequent accumulation of the enzyme's particular substrate in the body. The
incidence of LSDs is estimated to range from one in 5,000 to one in 7,000 live births.
Worldwide, Gaucher disease has a prevalence estimated to range from one in 40,000 to one
in 60,000 in the general population and, though it is a panethnic disorder, in the Ashkenazi
Jewish population its frequency is markedly higher, ranging from one in 400 to one in 1,000
live births. Carrier frequency in those of Ashkenazi descent is estimated to be as high as one
in 18.
Text B
Skeletal Manifestations
The skeletal manifestations of Gaucher disease are often the most debilitating, yet the
pathogenesis of bone changes are not fully understood. Between 70% and 100% of
patients with type 1 Gaucher disease have clinical or radiographic evidence of bone
disease. Irreversible complications may influence long-term mobility and quality of life.
The spine, pelvis, and femurs are usually affected; several different mechanisms of bone
injury have been identified. The displacement of yellow marrow with red marrow because
of Gaucher cell infiltration produces both physical and biochemical changes in the bone
marrow microenvironment that can affect bone marrow vascularity and pressure,
potentially causing thrombosis, infarction, and impaired hematopoiesis.

Text C

Osteomyelitis occurs in Gaucher disease


When osteomyelitis occurs in Gaucher disease, it is usually aseptic, though it's
difficult to exclude pyogenic osteomyelitis at onset. Eventually, negative blood
cultures allow clinicians to differentiate aseptic from pyogenic osteomyelitis.
While it is impossible to predict major bone complications in patients with
Gaucher disease, risk factors include anemia and splenectomy. In untreated
patients, bone crises are reported to occur in 55% of splenectomized patients and
22% of patients with an intact spleen. Osteonecrosis is irreversible and often
precipitates fracture and joint collapse.
Text D

Enzyme replacement therapy


Enzyme replacement therapy for the treatment of Gaucher disease became
available in 1991 with the development of alglucerase (a placenta-derived GCase).
Today, however, treatment involves the iv infusion of a recombinant GCase
enzyme, which breaks down the accumulating lipid. Infusions are administered
every two weeks. Three FDA-approved enzyme replacement therapies are
currently available in the United States: imiglucerase (Cerezyme), velaglucerase
alfa (Vpriv), and taliglucerase alfa (Elelyso). Enzyme replacement therapy has
been shown to reduce the incidence of hepatosplenomegaly, normalize
hematologic values, and improve osteopenia. Risks of treatment include infusion
reactions and antibody formation, which has the potential to render the drug
inactive. Immunoglobulin G antibodies should be monitored routinely during the
first year following diagnosis, with a baseline blood sample drawn before the
patient's first infusion and blood draws repeated every three to six months. If
antibody production is high, there is a risk of anaphylaxis. Overall, infusions are
well tolerated, with the most common adverse effects being hypersensitivity
reactions, which can be managed effectively with antihistamine premedication.
PART A -QUESTIONS AND ANSWER SHEET
Questions 1-7
For each question, 1-7, decide which text (A, B, C or D) the information comes from.
You may use any letter more than once.

In which text can you find information about


1. A. condition that occurs due to an increase in the number of white cells in the blood Answer___

2 .Often, it is not possible to deal with complications Answer__________________.

3. Recorded to be more effective in dealing with the disease conditions Answer_______________.

4. homicide of virtually all other parts of the cell by the enzymes Answer_________________.

5. Almost fortnightly for effective management Answer__________________.

6 .Not-so-common when compared to other medical conditions Answer____________________.

7 .Weakening appearance Answer___________________.

Questions 8-14
Answer each of the questions, 8-14, with a word or short phrase from one of the texts.
Each answer may include words, numbers or both. Your answers should be correctly spelt.
8 .What is the common reason for Gaucher Disease? Answer_________________.

9. Is it still clear how disease appears in its various forms or not? Answer____________________.

10. ERTs available include? Answer____________________.

11 .What can lead to anaphylaxis? Answer____________________

12. What is regarded as medically clean or without infection? Answer___________________.

13 .What happens when Gaucher cell penetrate more deeply? Answer_________________.


14 .What is regarded to be difficult when it comes to assessing the patient suffering from Gaucher
Disease? Answer__________________.
Questions 15-20

Complete each of the sentences, 15-20, with a word or short phrase from one of the
texts. Each answer may include words, numbers or both. Your answers should be correctly spelt.

15. The__________ have the capacity to exterminate various other cell structures.

16. With the appearance of red marrow, the_______ and________ often get affected more badly and
may lead to infarction.

17 .The two of the common conditions that can occur due to Gaucher may include ___and __.

18 .It is requisite to monitor_______________ during the first year.

19. Enzyme replacement therapy is known to be more effective in curtailing down the_____ and
improving osteopenia.

20. Studies reveal that there are various other __________ which can make the conditions worse.

END OF PART A, THIS QUESTIONS PAPER WILL BE COLLECTED


READING SUB-TEST : PART B
In this part of the test, there are six short extracts relating to the work of health professionals .
For questions 1-6, choose the answer (A, B or C) which you think fits best according to the text.
Write your answers on the separate Answer Sheet
Questions 1-6

1. is an epidemic of infectious disease that has spread across a large region


A. Disease Transmission
B. Bioterrorism
C. Pandemics

Emerging infectious diseases

Emerging infectious diseases impact healthcare providers in the United States and
globally. Nurses play a vital role in protecting the health of patients, visitors, and
fellow staff members during routine practice and biological disasters, such as
bioterrorism, pandemics, or outbreaks of emerging infectious diseases. One vital
nursing practice is proper infection prevention procedures. Failure to practice
correctly and consistently can result in occupational exposures or disease
transmission. Infection prevention education based on existing infection prevention
competencies is critical to ensure adequate knowledge and safe practice both every
day and in times of limited resources.
2. What is the risk of passing CMV to the baby during pregnancy?
A. Baby gets infected quickly
B. There is more risk if the infection is primary
C. Low risk in the first two trimester compared to the third trimester.

Transmission

In general, 1 of every 150 to 200 babies in the USA is born with congenital CMV.
This makes CMV the most frequent congenital viral infection. Though this seems like
a large percentage of births, only 1 in 5 of these infants born with congenital CMV
will experience any adverse symptoms or long-term issues.
The virus has the potential to travel through the mother’s blood and pass through
the placenta, infecting the developing baby. If the pregnant lady contracts the virus
(primary infection) during pregnancy: It is more likely to pass on CMV to the baby if
it is primary infection during the pregnancy. If the mother has a primary CMV
infection during pregnancy, there is approximately a 40% chance of passing the virus
to the baby. The risk of transmission from mother to baby is highest if she gets a
primary CMV infection in the third trimester (40-70%) and is lowest if the primary
infection begins in the first or second trimesters (30-40%).
3. The following table talks about
A. Medication
B. Preventive measure
C. Effects of Disease

Allergic Rhinitis
Indicated as immunotherapy for short ragweed (Ambrosia artemisiifolia) pollen
induced allergic rhinitis (with or without conjunctivitis) confirmed by positive skin
test or in vitro testing for ragweed-specific IgE antibodies Initiate treatment 12
weeks before the expected onset of ragweed pollen season and maintain it
throughout the season 18-65 years: 1 tablet SL qDay; give the 1st dose in physician’s
office and observe for 30 min > 65 years: Not approved

4. What is wrong?
A. Not effective with respect to QRS complex
B. Mental disease occurs more often
C. Medication shall be used again and again

Atropine IV/IM (Rx) - Warnings


Caution in hepatic/renal impairment, BPH, CHF
Not for effective treatment of type II second or third-degree AV block with or
without a new wide QRS complex Use caution in autonomic neuropathy, myocardial
ischemia, heart failure, paralytic ileus, hepatic impairment, hiatal hernia associated
with reflux esophagitis, hyperthyroidism, myasthenia gravis, and renal impairment
Heat prostration can occur in a high environmental temperature.
Psychosis reported in sensitive individuals and with excessive doses.
When a recurrent use of atropine is essential in patients with coronary artery disease,
total dose should be restricted to 2 to 3 mg (maximum 0.03 to 0.04 mg/kg) to avoid
detrimental effects of atropine-induced tachycardia on myocardial oxygen demand
5. The notice is giving information about
A. health and safety hazards
B. Effectiveness of PPE
C. Insufficient supplies of PPE

Shortages of PPE

Multiple studies have found that the U.S. hospitals and healthcare agencies lack
sufficient PPE and even stockpiles have not provided adequate or correct supplies to
give healthcare personnel necessary PPE during past events. During the 2014 Ebola
outbreak, PPE availability was severely limited, leading to potential occupational
exposures and healthcare personnel infection. When respirators are limited,
remaining supplies can be worn for extended periods of time or re-used between
patients. However, extending the use or re-using respirators puts nurses at risk of
exposure due to auto-inoculation when removing contaminated equipment
or from reduced compliance during long-term wear.
6. According to the table given, which is correct?
A. The mean age in diabetics is significantly greater than that in non-diabetics
B. more than 60% are male
C. The difference of BMI in diabetics and non-diabetics is very less
READING SUB-TEST : PART C
In this part of the test, there are two texts about different aspects of healthcare.
For questions 7-22, choose the answer (A, B, C or D) which you think fits best according to the text.
Write your answers on the separate Answer Sheet

Text 1: Healthy Lifestyle without Prescribing Weight Loss

One of the leading approaches to improving metabolic health and thereby preventing
diabetes is recommending to individuals that they lose weight.
However, because of homeostatic responses to energy loss, this lifestyle recommendation
may not always lead to desired long-term metabolic health. It is true that in the short
term, weight loss can improve insulin sensitivity. Consuming fewer carbohydrates,
increasing activity, and/or temporarily reversing leptin resistance by decreasing the size of
fat cells can improve glucose tolerance. Leptin is a long-term fullness hormone produced
by fat cells that also has euglycemic properties. When leptin signals the hypothalamus that
adequate fat stores are present, messages are sent to eat less, expend more energy, and
normalize glucose levels. However, too much fat triggers as yet unknown chemical
messengers, making the hypothalamus resistant to leptin.

Regaining lost weight after a diet often leads to another weight loss attempt. With
repetitive loss and gain of 10-50 pounds, a pattern of weight cycling emerges. Weight
cycling is a high-risk behavior for the development of Type 2 diabetes (T2D), as regained
weight is more metabolically unhealthy because it preferentially deposits as visceral fat.
Visceral adipose tissue, as opposed to subcutaneous adipose tissue, produces more
inflammatory factors, as well as resistin and visfatin, hormones that are linked to insulin
resistance. In summary,while all weight gain can lead to negative metabolic changes,
regained weight is especially likely to promote insulin resistance and inflammation.
Emotional eating often results in consumption of excess food or poor food choices and
often leads to weight gain. Beginning a stressful new job or going through a divorce are
just two examples of life events that can spur emotional eating. Asking patients if they
have noticed a change in their eating habits, and if that coincided with any other changes
in their lives, should be part of the history if weight gain has occurred. If the patient
acknowledges overriding fullness cues and eating more than usual, the underlying stress,
emotional pain, depression, and/or anxiety could be helped in a variety of ways.

Glucocorticoids are known to reduce inflammation and are universally associated


with both weight gain and metabolic changes including hyperglycemia. While the
benefits of short-term steroids may outweigh the risks, for longer term usage,
nonsteroidal anti-inflammatories or disease-modifying drugs could be used. While the only
class of antihypertensives to cause weight gain are beta blockers, both betablockers and
thiazolidinediones can cause dysglycemia. The two classes of antihypertensives that may
prevent weight gain and improve insulin sensitivity are ACE-inhibitors and angiotensin
receptor blockers (ARBS).

Sleep hygiene can help to get sleep on time as well as sleeping more deeply.
Patients should be apprised of the need to avoid caffeine for 7 hours, and
alcohol for 2 hours before bedtime. Also turning off of “blue light” an hour before
bedtime is a good way to help increase the natural sleep hormone melatonin. For
patients who still find it difficult to fall asleep or stay asleep, recommending
melatonin supplements can help. If restorative sleep still evades the patient, it is
important to assess for obstructive sleep apnea (OSA), a major impediment of deep,
restful sleep and is associated with obesity.

A reduced activity could be a result of fatigue, chronic pain, old injuries, or newly
developing arthritis. For those suffering from pain, referral to an appropriate
specialist may be indicated. Physical therapy can also help to maximize their
mobility and to find an exercise that is also enjoyable, sustainable, and suitable for
their limitations. Research shows that threats to health, such as lack of food, sleep,
or long periods of exertion, are perceived by homeostatic sensors as threats to
human survival. Dieting, which often requires ignoring hunger may be perceived
as a threat, whereas intuitive eating, which honors internal cues is perceived as
reassuring to the body. This alternative approach to dieting was started by lay health
writers in the 1980’s and has dieticians more likely to use intuitive eating than restrictive
practices.

By having patients make sure they have healthy and delicious food available for when
ideal hunger sets in (neither starving nor hardly hungry) so they can eat until they are
satisfied, which is a key for visceral eating. Whether an advance practice nurse or a
nutritionist helpsguide the patient, this non-dieting approach to eating helps patients
replace an antagonistic relationship with the body for a nurturing one.
Text 1: Questions 7-14
7. The first paragraph talks about;
A. Fat Triggers
B .How to lose weight?
C. Healthy lifestyle
D. Nature of Leptin

8. The second paragraph talks about;


A. Risks associated with weight gain
B. Risks associated with weight loss
C. Weight loss likely to lead to weight gain
D. Unhealthy weight recycling

9. The third paragraph gives more information about;


A. Emotional eating
B. Stress eating
C. Emotional or stress eating
D. Behavioral responses

10. What do we learn about medication in the fourth paragraph?


A. Glucocorticoids are very effective
B. Recommends only the use of non-steroidal anti-inflammatories
C. Medications can lead to dysglycemia
D.it brings in more benefit to make use of steroids when the conditions are to
be dealt with immediately

11. The information in the fifth paragraph is more related to;


A. Sleep v/s Weight loss
B. New lifestyle recommendations
C. How to improve metabolic health?
D. Sleep and Weight gain
12. The word impediment in the fifth paragraph is close to;
A. Demagogue
B .Restriction
C .Disrupting
D. Retardation

13. The information in the sixth paragraph throws light on;


A. Importance of exercise
B. Importance of diet
C. Importance of exercise and diet
D. Healthy weight recycling

14.The word visceral in the last paragraph refers to;


A. Intuitive
B. Direct
C. Unnatural
D. Happy
Text 2: X-Ray Diffraction Imaging (XDI) Scanner In Diagnostic
Radiology
X-ray diffraction imaging (XDI) scanner has been developed for security screening
applications by Morpho Detection GmbH in Hamburg, Germany. The main
rationale for developing this XDI scanner is the existence of explosive materials
whose densities overlap those of common materials, such as water, leading to
unacceptably high false alarm rates for scanners employing merely transmission
xray data. As the x-ray diffraction probes molecular structure, XDI yields more
features for material identification than transmission x-rays, leading to higher
detection rates and lower false alarm rates.

The XDI scanner employs the Multiple Inverse Fan-Beam (MIFB) topology (Harding
et al, 2012). The MIFB topology is a multiple-focus, multiple-beam, multiple-
detector extension of that originally described by Harding. These extensions
increase the photon throughput by over five orders of magnitude relative to that of
the original system. The MIFB topology features an x-ray multisource, comprising a
linear array of 16 focal spots that are sequentially irradiated by a magnetically-
deflected electron beam. The accelerating voltage is 140 kV; whereas the tube DC
power is 6 kW and the beam dwell time for each focus is 200 µs.

The inherent contrast of molecular coherent scatter from body tissues is much
greater than that originating in the linear attenuation coefficient accessed by
transmission radiation, when the momentum dimension is included in tissue
discrimination. As each voxel is irradiated from several directions, a modest degree
of transmission tomosynthesis can be reconstructed from the transmission data;
The fusion of data from scatter and transmission sensors allows a significant
improvement in image quality relative to that obtained when each is separately
depicted; the conveyor belt speed is sufficient to allow an anatomical region, such
as head, thorax or abdomen, to be scanned in only a few seconds; the measured
dose imparted in an XDI scan is negligible compared with that of the natural
radiation background, taken as ~ 3 mGy / year.
The x-ray imparted by the XDI scanner was repeatedly measured with a PTW
Diados E dosimeter inserted into the center of a D100 QRM thorax phantom. The
thorax phantom, visible to the right of the picture, was inserted in a luggage bin
that was moved by conveyor belt through the scanner. The dosimeter signal was
read out through the cable. As the dosimeter was inserted into the center of the
phantom, it was shielded from radiation emitted by the x-ray multisource owing to
an overlying material; hence the skin dose will be significantly higher.

Tissue samples were measured by commercial x-ray diffractometers implementing


angular-dispersive analysis. In this form of XRD, quasi-monochromatic radiation
scattered by the sample is measured in dependence on the scatter angle. They
provide excellent momentum resolution; however, they lack tomographic sensing
capability and feature long scan times. These will be referred to in this publication
as XRD profiles. In the other cases, the diffraction profiles were measured in the
XDI scanner, implementing energy-dispersive analysis of broad-band x-radiation.
Although the momentum resolution is essentially determined by the energy
resolution of the detectors and is, therefore, inferior to that of angular-dispersive
technique, it offers direct tomographic analysis capability and is much faster.

In presenting the results of the NNMF factorization technique applied to x-ray


diffraction profiles, the intention here is to emphasize more the methodology than
its precise output. The main reason for this emphasis is that in the course of time,
as more tissues are included in the analysis, the number of base-tissue profiles
derived from NNMF is expected to increase. Moreover, the input data is a mixture
of XRD and XDI profiles. This changes both in the number and form of the base
profiles are expected when only XDI data are analyzed. Finally, the statistical
accuracy of a small set of only twelve diffraction profiles is questionable. As noted
earlier, the XDI scanner provides the unique chance to apply NNMF in vivo to
very many voxels containing healthy and diseased tissues.
Text 2: Questions 15-22
15. According to paragraph 1, XDI scanner;
A. is more powerful than any other scanners
B. can provide accurate data
C. is developed merely to deal more effectively with explosive material
D. doesn`t just transmit x-ray data

16. Paragraph 2 talks about;


A. How the scanner works?
B. Description of the scanner
C .How the image is taken, altered and transmitted
D .How it outperforms other scanners

17. Paragraph 3 talks about;


A. Features of the scanner
B .Scattering of data
C. Transmission of data
D .Scattering and transmission of data

18. Paragraph 4 talks about;


A. Image capturing
B. Transmission of information
C .Radiation dose
D. Efficiency in data capturing

19. The word close to meaning `Depth` in paragraph 5 is;


A .Momentum
B .Tomographic
C. Broad-band
D. Angular
20. The word ` Momentum ` in paragraph 5 may mean
A. Energy
B. Impulse
C. Effect
D .Upshot

21. NNMF may increase;


A. with an increase in tissues
B. with the improvement in analysis
C. with an increase in base-tissue profiles
D. with the inclusion of base-tissue profiles from NNMF

22. In the last paragraph, ` in vivo ` may mean


A. To apply externally
B. Happening or existing inside a living body
C. To be more challenging
D. To overcome complexities

END OF READING TEST, THIS BOOKLET WILL BE COLLECTED


Reading test 15 : Answer Key

Part A - Answer key 1 – 7


1: B
2: C
3: D
4: A
5: D
6: A
7: C

Part A - Answer key 8 – 14


8: Six 9
Is: 5%
10: 4.2 Million
11: Young people
12: Rastafarians
13: 7.4%
14: longitudinal cohort study

Part A - Answer key 15 – 20


15: 80.6%
16: 5–10%
17: 2.54% prevalence
18: illegal drugs
19: cannabis addiction
20: Gateway hypothesis
Reading test - part B – answer key
1: B Arises spontaneously and its cause is unknown.
2: A Are caused by narrowing of arteries.
3: B Patients with DVT are susceptible to PE.
4: A Cleaning of the blood of toxins.
5: C Adefovir goes well in coordination with Cycloferon.
6: C Microfluidics, Decorte.

Reading test - part C – answer key


Text 1 - Answer key 7 – 14
7: B Jakob
8: B True
9: D Not given
10: C Transmission
11: D B and C
12: A 100% true
13: C True in some cases
14: D All of the above

Text 2 - Answer key 15 – 22


15: A Sweat ducts become clogged
16: A Excessive sweat is not allowed to evaporate from the skin
17: B Overdressing
18: B Who is at risk
19: B Areas with higher humidity
20: C What heat rash looks like.
21: A True in some cases
22: A Blockage is high.
READING TEST 16
READING SUB-TEST : PART A
 Look at the four texts, A-D, in the separate Text Booklet.
 For each question, 1-20, look through the texts, A-D, to find the relevant information.
 Write your answers on the spaces provided in this Question Paper.
 Answer all the questions within the 15-minute time limit.
 Your answers should be correctly spelt.

TEXT BOOKLET- PSORIASIS

Text A
Psoriasis is a chronic inflammatory disease of the immune system. It mostly affects the skin
and joints, but it may also affect the fingernails, the toenails, the soft tissues of the genitals
and the inner side of the mouth. Psoriasis and psoriatic arthritis can be associated with other
diseases and conditions, including diabetes, cardiovascular disease and depression.

Psoriasis Facts
 Psoriasis is a serious medical condition.
 Approximately 7.5 million people in the United States have psoriasis and suffer from this
medical condition more helplessly.
 Psoriasis can occur at any point of life time but primarily seen in adults. Up to 40 percent of
people with psoriasis experience joint inflammation that produces symptoms of arthritis.
This condition is called psoriatic arthritis.
Psoriatic arthritis patients also experience other arthritis symptoms.
 Psoriasis usually occurs on the scalp, knees, elbows, hands and feet. Approximately 80
percent of those affected with psoriasis have a mild to moderate disease, while 20 percent
have moderate to severe psoriasis affecting more than 5 percent of the body surface area.
 Plaque psoriasis is the most common form affecting about 80 to 90 percent of psoriasis,
which is characterized by patches of raised, reddish skin covered with silvery-white scales.
Text B
Comorbidities Associated with Psoriasis;
 The incidence of Crohn`s disease and ulcerative colitis, two types of inflammatory bowel
disease, is 3.8 to 7.5 times greater in psoriasis patients than in the general population.
 Patients with psoriasis also have an increased incidence of lymphoma, heart disease,
obesity, type II diabetes and metabolic syndrome. Depression and suicide, smoking and
alcohol consumption are also more common in psoriasis patients.
 Psoriasis can have a substantial psychological and emotional impact on patients.
 The prevalence of lugubrious in patients with psoriasis may be as high as 50 percent.
Studies have shown that psoriasis patients experience physical and mental disabilities, just
like patients with other chronic illnesses such as cancer, arthritis, hypertension, heart disease
and diabetes.

Text C
Treatment Options for Psoriasis
• Topical treatments are helpful for mild to moderate psoriasis but do not tend to be
effective for treating moderate to severe psoriasis.
• Topical treatments include anthralin, coal tar, emollients, salicylic acid, tazarotene, topical
corticosteroids and forms of vitamin D. These topical medications can sometimes be used
together with other medications.
• Topical corticosteroids are available in many strengths and formulations.
• Psoriasis patients with moderate to severe psoriasis can be treated with traditional
systemics, phototherapy or biologic agents.
• In cases of more extensive psoriasis, topical agents may be used in combination with
phototherapy, or traditional systemic or biologic medications.
• Phototherapy treatment includes narrowband and broadband ultraviolet B (UVB) and
psoralen plus UVA (PUVA).
 Regular systemic treatments include acitretin, cyclosporine and methotrexate. Since
biologic therapies, sporadically propounded, target the immune system, it is important to
prevent infections during therapy. Patients need to be monitored and evaluated periodically.
Text D

The PHAROS EX-308 Excimer Laser

While mild cases of psoriasis can sometimes be treated with specific creams and ointments
(Anthralin. Topical retinoids, Calcineurin inhibitors, Coal tar and others are known to be very
much effective), many patients do not experience relief with these treatments. For those
patients, phototherapy can be a more successful option.
Phototherapy uses UV light to decrease inflammation in areas affected by psoriasis, assisting
in clearing the itchy lesions. Laser such as PHAROS EX-308 Excimer Laser allows doctors to
administer phototherapy in an especially effective way. This laser allows easy application of
highly-concentrated and customized UV light directly to the areas of affected skin, making
the treatment as effective as possible without affecting the surrounding skin.
PART A -QUESTIONS AND ANSWER SHEET
Questions 1-7
For each question, 1-7, decide which text (A, B, C or D) the information comes from.
You may use any letter more than once.

In which text can you find information about

1. More or less like conditions arising in other diseases. Answer_____________.

2. Traditionals procedures. Answer_____________.

3. An effective way of treating. Answer____________.

4. Facts with respect to new diseases that might arise. Answer___________.

5. Affect people of all ages. Answer_______________.

6. People have no choice but to endure. Answer_______________.

7. Deal with depression. Answer_____________.

Questions 8-14
Answer each of the questions, 8-14, with a word or short phrase from one of the texts.
Each answer may include words, numbers or both. Your answers should be correctly spelt.

8. What phototherapy may entail? Answer_____________.

9. What psoriasis can be connected to? Answer_______________.

10. Which therapy or what is often not suggested? Answer________________.

11. What is the initial treatment for mild to moderate psoriasis in its beginning stages?
Answer________________.

12. What is the outcome of the research conducted? Answer______________.

13. What is the advanced treatment option available for the patients? Answer________

14. What will the patients with psoriatic arthritis eventually develop? Answer________
Questions 15-20

Complete each of the sentences, 15-20, with a word or short phrase from one of the texts.
Each answer may include words, numbers or both. Your answers should be correctly spelt.

15. Patients show increasing levels of __________________.

16. The disease can have _____________ effects on sufferers.

17. Psoriasis can be associated with other diseases such as diabetes, _________ and depression.

18.___________ include acitretin, cyclosporine and methotrexate.

19. Phototherapy provides necessary help in wiping out _________.

20. Occurrence of two types of inflammatory bowel disease, is . in psoriasis patients.

END OF PART A, THIS QUESTIONS PAPER WILL BE COLLECTED


READING SUB-TEST : PART B
In this part of the test, there are six short extracts relating to the work of health professionals .
For questions 1-6, choose the answer (A, B or C) which you think fits best according to the text.
Write your answers on the separate Answer Sheet

Questions 1-6

1. Blood cell levels will be low;


A. If there are too many immunoglobulins.
B .If there are too many myeloma cells in the bone marrow.
C. If there are less antibodies.

Tests to Find Multiple Myeloma

Blood counts

The complete blood count (CBC) is a test that measures the levels of red cells, white cells, and
platelets in the blood. The most common finding is a low red blood cell count (anemia).

Quantitative immunoglobulins
This test measures the blood levels of the different antibodies. There are several different types
of antibodies in the blood: IgA, IgD, IgE, IgG, and IgM. The levels of these immunoglobulins are
measured to see if any are abnormally high or low. In multiple myeloma, the level of one type
may be high while the others are low.
2. What is right about immunoglobulin?
A. Finding a monoclonal immunoglobulin in the blood may be the first step in diagnosing
multiple myeloma.
B .Produces different types of proteins.
C .It will not produce all the exact same antibody.

Electrophoresis

The immunoglobulin produced by myeloma cells is abnormal because it is monoclonal. Serum


protein electrophoresis (SPEP) is a test that measures the immunoglobulins in the blood and
can find a monoclonal immunoglobulin. Then, another test, such as immunofixation or
immunoelectrophoresis, is used to determine the exact type of abnormal antibody (IgG or some
other type). This abnormal protein is known by several different names, including monoclonal
immunoglobulin, M protein, M spike, and paraprotein. Immunoglobulins are made up of
protein chains: 2 long (heavy) chains and 2 shorter (light) chains. Sometimes the kidneys excrete
pieces of the M protein into the urine. This urine protein, known as Bence Jones protein, is the
part of the immunoglobulin called the light chain. The tests used for finding a monoclonal
immunoglobulin in urine are called urine protein electrophoresis (UPEP) and urine
immunofixation. These are done most often on urine that has been collected over a 24-hour
period, not just on a routine urine sample.
3. What is correct about Free Light Chains?
A. Ratio of kappa and lambda is 1:1.
B. Come in handy when it is not possible to diagnose by other methods.
C. Possibly, no differences in ratio arises with differences in light chains.

Free light chains

This test measures the amount of light chains in the blood, being a possible sign of myeloma or
light chain amyloidosis. This is most helpful in the rare cases of myeloma in which no M protein
is found by SPEP. Since the SPEP measures the levels of intact (whole) immunoglobulins, it
cannot measure the amount of light chains.
This test also measures the light chain ratio which is used to see if one type of light chain is
more common than the other. Kappa and lambda, in most cases, are present in equal amounts
in the blood,. If one kind of light chain is more common than the other, the ratio will be
different, which can be a sign of myeloma.

4. The notice deals with;

A. A chromosomal structure testing.


B. FISH and its effectiveness.
C. Appropriateness of the results.

Fluorescent in situ hybridization (FISH) is similar to cytogenetic testing. It uses special


fluorescent dyes that only attach to specific parts of chromosomes. FISH can find most
chromosome changes (such as translocations and deletions) that can be seen under a
microscope in standard cytogenetic tests, as well as some changes too small to be seen with
usual cytogenetic testing. FISH can be used to look for specific changes in chromosomes. It can
be used on regular blood as well as bone marrow samples. It’s very accurate and because the
cells don’t have to grow in a dish first, results are often available within a couple of days.
5. What is right about BUN?
A. Patient may lose his / her memory.
B .Differences in electrolytes .
C. The higher the Cr levels, the greater is the effect on the kidney.

Blood chemistry tests

Levels of blood urea nitrogen (BUN) and creatinine (Cr), albumin, calcium, and other
electrolytes will be checked. BUN and Cr levels show how well your kidneys are working.
Higher levels mean that kidney function is impaired. This is common in people with myeloma.
Albumin is a protein found in the blood. Low levels can be a sign of more advanced myeloma.
Calcium levels may be higher in people with advanced myeloma. High calcium levels can cause
severe symptoms of ennui, weakness, and confusion. Levels of electrolytes such as sodium and
potassium may be affected as well.

6. Bone marrow biopsy;


A. Is painful.
B. Is painless.
C. Helps detect myeloma.

Bone marrow biopsy


In bone marrow aspiration, the back of the pelvic bone is numbed with local anesthetic. Then, a
needle is inserted into the bone, and a syringe is used to remove a small amount of liquid bone
marrow. This causes a brief sharp pain. Then for the biopsy, a needle is used to remove little
amount of bone and marrow, about 1/16- inch across and 1-inch long. Patients may feel some
pressure during the biopsy, but it usually isn’t painful. There is some soreness in the biopsy area
when the numbing medicine wears off. Most patients can go home immediately after the
procedure. A doctor will look at the bone marrow tissue under a microscope to see the
appearance, size, and shape of the cells, how the cells are arranged and to determine if there
are myeloma cells in the bone marrow and, if so, how many.
READING SUB-TEST : PART C
In this part of the test, there are two texts about different aspects of healthcare.
For questions 7-22, choose the answer (A, B, C or D) which you think fits best according to the text.
Write your answers on the separate Answer Sheet.

TEXT 1: All about Hemochromatosis

Hemochromatosis is the most common form of iron overload disease. Primary


hemochromatosis, also called hereditary hemochromatosis, is an inherited disease.
Secondary hemochromatosis is caused by anaemia, alcoholism, and other disorders. Juvenile
hemochromatosis and neonatal hemochromatosis are two additional forms of the disease.
Juvenile hemochromatosis leads to severe iron overload and liver and heart disease in
adolescents and young adults between the ages of 15 and 30. The neonatal form causes rapid
iron build-up in a baby`s liver that can lead to death.

Hemochromatosis causes the body to absorb and store too much iron. Once it grips on anyone,
it will be difficult to free that person from its clutches. The redundant iron builds up in the
body`s organs and damages them. Without treatment, the disease can cause the liver, heart,
and pancreas to fail. Iron is an essential nutrient found in many foods. The greatest amount is
found in red meat and iron-fortified breads and cereals. In the body, iron becomes part of
haemoglobin, a molecule in the blood that transports oxygen from the lungs to all body tissues.
Healthy people usually take in about 10 percent of the iron contained in the food they eat,
which meets normal dietary requirements. People with hemochromatosis absorb up to 30
percent of iron. Over time, they absorb and keep in their body between five to 20 times more
iron than the body may be in quest of. Because the body has no natural way to rid itself of the
unwanted or superfluous iron, it is stored in body tissues, specifically the liver, heart, and
pancreas.
Hereditary hemochromatosis is mainly caused by a drawback, a flaw in a gene
called HFE, which helps regulate the amount of iron absorbed from food. The two
known mutations of HFE are C282Y and H63D. C282Y is the most important because it is this
which can lead to disturbance in taking the helm. In people who inherit C282Y from both
parents, the body absorbs too much iron and hemochromatosis can result. Those who inherit
the defective gene from only one parent are carriers for the disease but usually do not develop
it; however, they still may have a little than orderly iron absorption. Neither juvenile
hemochromatosis nor neonatal hemochromatosis are caused by an HFE defect. Juvenile and
neonatal hemochromatosis are caused by a mutation in a gene called hemojuvelin.

Hereditary hemochromatosis is one of the most common genetic disorders in the United
States. It most often affects Caucasians of Northern European descent, although other ethnic
groups are also affected. About five people out of 1,000 - 0.5 percent - of the U.S. Caucasian
population carry two copies of the hemochromatosis gene and are susceptible to developing
the disease. One out of every 8 to 12 people is a carrier of one abnormal gene.
Hemochromatosis is less common in African Americans, Asian Americans, Hispanics/Latinos,
and American Indians. Although both men and women can inherit the gene defect, men are
more likely than women to be diagnosed with hereditary hemochromatosis at a younger age.
On average, men develop symptoms and are diagnosed between 30 to 50 years of age. For
women, the average age of diagnosis is about 50.

Joint pain is the most common complaint of people with hemochromatosis. Other common
symptoms include debility, abdominal pain and heart problems. However, many people have
no symptoms when they are diagnosed. If the disease is not detected and treated early, iron
may accumulate in body tissues and eventually lead to serious problems such as arthritis; liver
disease (including an enlarged liver); cirrhosis; cancer; liver failure; damage to the pancreas,
possibly causing diabetes; brain fog; heart abnormalities, such as irregular heart rhythms or
congestive heart failure; impotence; early menopause; abnormal pigmentation of the skin,
making it look gray or bronze; thyroid deficiency; damage to the adrenal glands; constant
fatigue etc.
Text 1: Questions 7-14
7. According to Paragraph 1, hemochromatosis occurs due to;
A. An excess of iron in the blood.
B .A decrease in the quantity of iron in the blood.
C. A genetic disorder.
D .Alcoholism

8. In one of the forms of hemochromatosis, which one of the following conditions occurs?
A. Anaemic people are more prone to hemochromatosis
B It can be more fatal as the quantity of the iron increases more and more
C. It can be the result of some genetic disorder but may not lead to death
D. None of this above

9. According to paragraph 2, the patient`s body shows unusual capabilities of;


A. Absorbing iron from food eaten.
B. Storing the iron derived from the food.
C. Absorption and storage of iron.
D .Retaining 20 times more than the iron required.

10. In paragraph 2, which word or phrase may mean the following: To cause someone or
something to be free from an unpleasant or harmful thing?
A. Superfluous
B. Free from clutches
C .Rid off
D. Grips on

11. According to paragraph 3, the affected people;


A. May show the presence of defective genes from both the parents.
B. May take defective genes from a single parent.
C. Show capabilities of absorbing and retaining more than the iron required for the body.
D. Show the presence of only C282Y.
12. Which word in the paragraph 3, may mean organizeor manage?
A. Take the helm
B. Orderly
C. Regulate
D. None of the above

13. Paragraph 4 deals more with;


A. Symptoms and diagnosis
B .The risk factors of hemochromatosis
C .Hereditary hemochromatosis
D. How common it is in other countries

14. Which one of the following is considered a major symptom?


A. Arthritis
B .Skin coloration
C .Joint pain
D .Brain fog
Text 2: Chronic Myelogenous Leukemia

Leukemia is a type of cancer that starts in the blood or blood-forming tissues.


There are many different types of leukemia, and treatment is different for each one.
Chronic leukemias develop in a lackadaisical way in comparison with that of acute
leukemias, which show great momentum and multiplication celerity. But CL can be just as
life threatening. Chronic myelogenous leukemia is commonly referred to as CML. Other
names for this type of cancer include chronic myeloid leukemia, chronic myelocytic
leukemia, and chronic granulocytic leukemia. This is a cancer of the white blood cells. In
CML, blast cells, or immature white blood cells, form and multiply uncontrollably; they
crowd out all the other types of necessary blood cells.

CML has different phases of progression. Which phase the disease is in determines
the appropriate treatment. The phases are based on the number of blast cells present and
include: the chronic phase, the accelerated phase, and the blast crisis phase. The Chronic
Phase: This is the earliest stage of CML, and you may have some symptoms or none at all.
During this phase, your white blood cells can still fight infections in your body. The
Accelerated Phase: In this phase, your red blood cell counts are low, and anemia (not
enough iron in your blood) may occur.
Platelet levels are also reduced, which may cause easy bruising or bleeding because
platelets help to form blood clots. The amount of blast cells increases. A fairly common
complication at this point is a swollen spleen, which may cause stomach pain. The Blast
Crisis Phase: A large number of blast cells are present in this advanced phase. Symptoms
in this phase are more severe and can be life threatening.

Genetic mutation is known to be the driving factor for this disease. Doctors do not
know what implants this initial mutation but it does happen in an unusual way. In
humans, there are 23 pairs of chromosomes. In individuals with CML, part of
chromosome 9 is switched with a piece of chromosome 22. This makes a short
chromosome 22 and a very long chromosome 9. The short chromosome 22 is called the
Philadelphia chromosome, and is present in 90 percent of CML patients. Genes from
chromosomes 9 and 22 then combine to form a gene, the BCR-ABL gene that enables
specific blood cells to multiply uncontrollably, causing CML.
Because CML generally does not cause symptoms in its early stages, the cancer is
often detected during a routine blood test. When there are symptoms, they are
general and can be symptoms of other health conditions as well. Symptoms may
include fatigue, night sweats, fever etc. If tests suggest that you may have cancer, a
bone marrow biopsy is performed. This is to get a sample of bone marrow to send
to a lab for analysis. Once diagnosed, tests will be done to explore the extent of
disease in your body. A complete blood work-up is typically ordered, along with
genetic tests done in a laboratory. Imaging tests such as an MRI, ultrasound, and
CT scan can also be used to determine the extent of the disease.

Targeted therapies are typically used first in CML treatment. These are drugs that
attack a specific part of the cancer cell to kill it. In the case of CML, these drugs
block the protein made by the BCR-ABL gene. They may include imatinib,
dasatinib, or nilotinib. These are newer therapies that have been very successful;
they are truly far from being too perilous. Chemotherapy involves using drugs to
kill cancer cells. These drugs are systemic, which means they travel through your
entire body via your bloodstream. They can be given intravenously or orally,
depending on the specific drug. They are a common cancer treatment with side
effects that may be intense, but may not lead to precarious conditions. A bone
marrow transplant (also called a blood stem cell transplant) is used when other
treatments have failed; this is because those who opt for it go for broke in most of
the cases. There is a significant chance of adverse side effects. In this type of
transplant, chemotherapy is used to kill the cancerous cells in your bone marrow
before healthy donor cells are infused into your blood to replace them. Side effects
of this procedure vary widely but can include minor things such as chills and
flushing or major complications like anemia, infections, and cataracts.
Text 2: Questions 15-22

15. According to paragraph 1, which one of the following statements is true?


A. Chronic leukemia grows at the same speed as that of acute leukemia.
B. Only acute leukemia can be much more fatal than chronic leukemia.
C. Leukemia is a condition in which blood cells start multiplying at a constant speed.
D .Acute leukemia grows at a rapid speed.

16. Which word in paragraph 2 may suggest the following meaning: slow in movement, showing
little energy or enthusiasm?
A. Momentum
B .Lackadaisical
C. Celerity
D .None of the above

17. Paragraph 2 talks more about;


A. Development stages of the leukemia.
B. Different phases of leukemia.
C. Progression of CML.
D. How CML occurs.

18. According to paragraph 3, which one of the following statements is true?


A .The gene that grows out of the fusion of genes from chromosomes 9 and 22
plays a crucial role in multiplication of the blood cells which eventually leads to CML
B. Chromosomes 9 and 22 combine in an unusual way to give birth to BCR - IBL
C .Most of the patients across the globe show the combination of the genes from chromosome 9
and chromosome 22
D .None of the above

19. Paragraph 4 talks more about;


A. CML tests
B. Symptoms of CML
C. The procedure for identification of CML
D. How CML is diagnosed
20. According to paragraph 4, which one of the following statements is correct?
A. A complete blood report will be prescribed at the end of the initial diagnosis
B .An ultrasound should be performed before the blood tests
C. If the reports are suggestive of cancer, then a bone marrow biopsy is performed
D. None of the above

21. Paragraph 5 give notice about;


A The three most common treatment options for CML
B Differences among targeted therapy, chemotherapy and bone marrow transplants
C Why a bone marrow transplant is a better option than chemotherapy
D How targeted therapy is better than the other two treatment options

22. Which word or phrase suggests the following meaning: Risky?


A Perilous
B Go for broke
C Precarious
D None of the above

END OF READING TEST, THIS BOOKLET WILL BE COLLECTED


Reading test 16 : Answer Key

Part A - Answer key 1 – 7


1: B
2: C
3: D
4: B
5: A
6: A
7: B

Part A - Answer key 8 – 14


8: UVB / UVA
9: diabetes, cardiovascular disease
10: biologic therapies
11: creams
12: experience physical and mental disabilities
13: PHAROS EX-308 Excimer Laser
14: arthritis symptoms

Part A - Answer key 15 – 20


15: depression
16: psychological
17: Cardiovascular diseases
18: Regular systemic treatments
19: itchy lesions
20: 3.8 to 7.5 times greater
Reading test - part B – answer key
1: B If there are too many myeloma cells in the bone marrow.
2: A Finding a monoclonal immunoglobulin in the blood may be the first step
in diagnosing multiple myeloma.
3: A Ratio of kappa and lambda is 1:1.
4: A A chromosomal structure testing.
5: C The higher the Cr levels, the greater is the effect on the kidney.
6: C Helps detect myeloma.

Reading test - part C – answer key


Text 1 - Answer key 7 – 14
7: B A decrease in the quantity of iron in the blood.
8: B It can be more fatal as the quantity of the iron increases more and more
9: D Retaining 20 times more than the iron required.
10: C Rid off
11: A May show the presence of defective genes from both the parents.
12: A Take the helm
13: B The risk factors of hemochromatosis
14: C Joint pain

Text 2 - Answer key 15 – 22


15: B Only acute leukemia can be much more fatal than chronic leukemia.
16: B Lackadaisical
17: C Progression of CML.
18: A The gene that grows out of the fusion of genes from chromosomes 9 and 22
plays a crucial role in multiplication of the blood cells which eventually leads to CML
19: D How CML is diagnosed
20: C If the reports are suggestive of cancer, then a bone marrow biopsy is performed Your
Answer Is:
21: A The three most common treatment options for CML
22: B Go for broke
READING TEST 17
READING SUB-TEST : PART A
 Look at the four texts, A-D, in the separate Text Booklet.
 For each question, 1-20, look through the texts, A-D, to find the relevant information.
 Write your answers on the spaces provided in this Question Paper.
 Answer all the questions within the 15-minute time limit.
 Your answers should be correctly spelt.

TEXT BOOKLET- AARSKOG-SCOTT SYNDROME

Text A

Aarskog-Scott syndrome
Aarskog-Scott syndrome is a genetic disorder that affects the development of many
parts of the body. This condition mainly affects males, although females may have
mild features of the syndrome. People with Aarskog-Scott syndrome often have
distinctive facial features, such as hypertelorism, a small nose, a vertical groove
between the base of the nose and the border of the upper lip called philtrum, and a
widow's peak hairline. They frequently have mild to moderate short stature during
childhood, but their growth usually catches up with that of their peers during
puberty. Hand abnormalities are common in this syndrome and include
brachydactyly, fifth finger abnormality called clinodactyly, cutaneous syndactyly,
and a single crease across the palm. The intellectual development of people with
Aarskog-Scott syndrome varies widely. Some may have mild learning and
behavior problems, while others have normal intelligence. In rare cases, severe
intellectual disability has been reported.
Text B

FGDI gene mutations are the only known genetic cause of Aarskog-Scott
syndrome. The FGD1 gene provides instructions for making a protein that turns on
(activates) another protein called Cdc42, which transmits signals that are important
for various aspects of development before and after birth. Mutations in the FGD1
gene lead to the production of an abnormally functioning protein. These mutations
disrupt Cdc42 signaling, leading to the wide variety of abnormalities that occur in
people with Aarskog-Scott syndrome. Only about 20 percent of people with this
disorder have identifiable mutations in the FGD1 gene. The FGD1 gene provides
instructions for making a protein that functions as a guanine nucleotide exchange
factor (GEF). GEFs turn on proteins called GTPases, which play an important role
in chemical signaling within cells. GTPases are turned off when they are attached
to a molecule called GDP and are turned on when they are bound to another
molecule called GTP.

Text C

The FGD1 protein activates the GTPase known as Cdc42 by stimulating the
exchange of GDP for GTP. Once Cdc42 is active, it transmits signals that are
critical for various aspects of development before and after birth, particularly the
development of bones. The FGD1 protein may also be involved in the maintenance
of the extracellular matrix. Through this process, the protein appears to play a role
in cell movement (migration) and the remodeling of blood vessels. More than 40
mutations in the FGD1 gene have been found to cause Aarskog-Scott syndrome.
When caused by FGD1 gene mutations, Aarskog-Scott syndrome is inherited in an
X-linked recessive pattern. The FGD1 gene is located on the X chromosome,
which is one of the two sex chromosomes. In males, one altered copy of the gene
in each cell is sufficient to cause the condition. In females, a mutation would have
to occur in both copies of the gene to cause Aarskog-Scott syndrome.
Text D

Because it is unlikely that females will have two altered copies of this gene, males
are affected by X-linked recessive disorders much more frequently than females.
Females who carry one altered copy of the FGD1 gene may show mild signs of the
condition, such as hypertelorism, short stature, or a widow's peak hairline. A
characteristic of X-linked inheritance is that fathers cannot pass X-linked traits to
their sons. Evidence suggests that Aarskog-Scott syndrome is inherited in an
autosomal dominant or autosomal recessive pattern in some families, although the
genetic cause of these cases is unknown. In autosomal dominant inheritance, one
copy of the altered gene in each cell is sufficient to cause the disorder. In
autosomal recessive inheritance, both copies of the gene in each cell have
mutations. The parents of an individual with an autosomal recessive condition each
carry one copy of the mutated gene, but they typically do not show signs and
symptoms of the condition.
PART A -QUESTIONS AND ANSWER SHEET

Questions 1-7
For each question, 1-7, decide which text (A, B, C or D) the information comes from.
You may use any letter more than once.
In which text can you find information about;
1. Remodeling of an intricate lattice of proteins. Answer _______________

2. Effects on allosomes. Answer ____________

3. Reasons not being known well. Answer ________________

4. Abnormally increased distance between two organs or body parts. Answer ____________

5. Rare disease inherited as X-linked. Answer _______________

6. Activating and deactivating proteins. Answer __________________

7. Major reason for the appearance of the disease conditions. Answer ___________________

Questions 8-14
Answer each of the questions, 8-14, with a word or short phrase from one of the texts.
Each answer may include words, numbers or both. Your answers should be correctly spelt.
8. What word or phrase may mean “short fingers”? Answer __________________

9. What is the term used to define fifth finger abnormality? Answer ___________________

10. What is the cause of Aarskog-Scott syndrome? Answer _________________

11. Where do we find FGD1 gene? Answer __________________

12. What activates Cdc42? Answer _______________

13. What may suggest curved pinky fingers? Answer ______________

14. In which pattern single copy of the altered gene in each cell can cause the disorder? Answer __
Questions 15-20
Complete each of the sentences, 15-20, with a word or short phrase from one of the texts.
Each answer may include words, numbers or both. Your answers should be correctly spelt.

15. __________ may define widely spaced eyes.

16. In____________, both copies of the gene in each cell have mutations.

17 .___________, describes a condition in which there will be a webbing of the skin between some
fingers

18. There are as many as_____________ which are known to be causal agents of the disease.

19. The proteins called __________, play an important role in chemical signalling within cells.

2.0 _________ is a vertical groove between the nose and the upper lip.

END OF PART A, THIS QUESTIONS PAPER WILL BE COLLECTED


READING SUB-TEST : PART B
In this part of the test, there are six short extracts relating to the work of health professionals .
For questions 1-6, choose the answer (A, B or C) which you think fits best according to the text. Write
your answers on the separate Answer Sheet
Questions 1-6

Atherosclerosis

The brain’s vasculature relies on the central aorta, and there is a direct correlation
between atherosclerosis of the central aorta and that of the neurovascular system.
This relation between the brain’s vascular system and the central aorta has been
evaluated recently in terms of the central arterial pressure (CAP) and the
compliance of the aorta. In the United States, there are 50 million hypertension
patients, and 3 million patients exhibit normal self-adjustment, making them
similar to patients with normal blood pressure (BP). The remaining patients exhibit
malfunctioning self-adjustment. Clinical trials suggest that patients with high
resistance hypertension are more likely to develop cerebral thrombosis compared
to patients with normal-resistance hypertension, when both groups are treated to
the same extent by using antihypertensive medication.

Ultrasound of the abdominal aorta

Patients will undergo ultrasound imaging as part of the standard clinical


surveillance programme with measurement of the maximum anteroposterior
diameter of the aneurysm. Ultrasound scans will be performed every
6±2 months. A 3.5 MHz linear array transducer will be used to provide
standard realtime longitudinal B-scan images of the AAA at the point of maximum
diameter. Maximum anteroposterior AAA diameter and distensibility (pressure
strain elastic modulus and stiffness) will be assessed. Scans will be undertaken by
accredited clinical vascular scientists with an inter observer coefficient of variation
of aortic diameter measurements of 3.5% in our laboratory.
Magnesium supplementation

The use of Magnesium supplementation in preventing arrhythmias in patients with


congestive heart failure has long been established. Magnesium deficiency can lead
to QT interval prolongation, ST-segment depression and low amplitude T waves
Magnesium also influences the movement of other ions such as potassium, sodium
and calcium across the cell membranes. The association between magnesium and
potassium is probably best demonstrated in that magnesium deficiency is often
accompanied by potassium deficiency. In patients with congestive heart failure,
both magnesium and potassium are depleted with thiazide diuretics, particularly in
patients requiring high doses of thiazide diuretics. It has been shown that the level
of potassium in muscle will not normalise unless magnesium is replaced, even
though serum potassium rises with repletion.

Ferumoxytol

Ferumoxytol (Rienso) is composed of USPIO coated with polyglucose sorbitol


carboxymethyl ether. It is supplied as an aqueous colloidal product that is
formulated with mannitol and presented in single-use vials ready for intravenous
infusion. Each vial contains 510  mg of elemental iron in a volume of
17  mL of mannitol. The ferumoxytol dose (4  mg/kg) is removed from the
vial and administered intravenously at a rate of up to 1  mL/s. The single dose
is given immediately following the baseline MRI and 24–36  h before the post
contrast scan. Blood pressure is recorded before and 30 min after
administration to monitor for hypotension. In a subset of patients (n=20), MRI
scanning and USPIO administration will be repeated at <1 month and
1 year (up to a total of 3 doses of ferumoxytol in 1  year). Each dose of
ferumoxytol equates to approximately 7% of total body iron.
Pulse wave parameter values and comparison of the estimated and measured BP
values, for three representative patients participating in the clinical study.
READING SUB-TEST : PART C
In this part of the test, there are two texts about different aspects of healthcare.
For questions 7-22, choose the answer (A, B, C or D) which you think fits best according to the text.
Write your answers on the separate Answer Sheet.

Text 1: All About Agoraphobia


A phobia is generally defined as the unrelenting fear of a situation, activity, or
thing that causes one to want to avoid it. The definition of agoraphobia is a fear of
being outside or otherwise being in a situation from which one either cannot escape
or from which escaping would be difficult or humiliating. Phobias are largely
underreported, probably because many phobia sufferers find ways to avoid the
situations to which they are phobic. The fact that agoraphobia often occurs in
combination with panic disorder makes it even more difficult to track how often it
occurs. Other facts about agoraphobia include that researchers estimate it occurs in
less than 1% to almost 7% of the population and that it is specifically thought to be
grossly underdiagnosed.

There are a number of theories about what can cause agoraphobia. One hypothesis
is that agoraphobia develops in response to repeated exposure to anxiety-provoking
events. The mental health theory that focuses on how individuals react to internal
emotional conflicts (psychoanalytic theory) describes agoraphobia as the result of a
feeling of emptiness that comes from an unresolved Oedipal conflict, which is a
struggle between the feelings the person has towards the opposite-sex parent and a
sense of competition with the same-sex parent. Although agoraphobia, like other
mental disorders, is caused by a number of factors, it also tends to run in families,
and for some people, may have a clear genetic factor contributing to its
development.
The symptoms of agoraphobia include anxiety that one will have a panic attack
when in a situation from which escape is not possible or is difficult or
embarrassing. Examples of such situations include: using public transportation,
being in open or enclosed places, being in a crowd, or outside of the home alone.
The panic attacks that can be associated with agoraphobia, like all panic attacks,
may involve intense fear, disorientation, rapid heartbeat, dizziness, or diarrhea.
Agoraphobic individuals often begin to avoid the situations that provoke these
reactions. Interestingly, the situations that people with agoraphobia avoid and the
environments that cause people with balance disorders to feel disoriented are quite
similar; this leads some cases of agoraphobia to be considered as vestibular
function (related to balance disorders) agoraphobia.

Interestingly, physicians often diagnose and treat agoraphobia, like other phobias,
when patients seek treatment for other medical or emotional problems rather than
as the primary reason that care is sought. As with other mental disorders, there is
no single, specific test for agoraphobia. The primary-care doctor or psychiatrist
will take a careful history, perform or refer to another doctor for a physical
examination, and order laboratory tests as needed. If someone has another medical
condition that he or she knows about, there may be an overlap of signs and
symptoms between the old and the new conditions. Just determining that anxiety
does not have a physical cause doesn`t immediately identify the ultimate cause.
Often, determining the cause requires the involvement of a psychiatrist, clinical
psychologist, and/or other mental health professional. In order to establish the
diagnosis of agoraphobia, the professional will likely ask questions to ensure that
the anxiety of the sufferer is truly the result of a fear of being in situations that
make it impossible, difficult, or embarrassing to escape, rather than in the context
of another emotional problem.
There are many treatments available for agoraphobia, including specific kinds of
psychotherapy as well as several effective medications. A specific form of
psychotherapy that focuses on decreasing negative, anxiety-provoking, or other
self-defeating thoughts and behaviors (called cognitive behavioral therapy) has
been found to be highly effective in treating agoraphobia. In fact, when
agoraphobia occurs along with a panic disorder, cognitive behavioral therapy -
with or without treatment with medication - is considered to be the most effective
way to both relieve symptoms and prevent their return. For example, sometimes
patients respond equally as well when treated with group cognitive behavioral
therapy or a brief course of cognitive behavioral therapy as they do when treated
with traditional cognitive therapy. Psychotherapy for agoraphobia is effective for
many people when they receive it over the Internet, which is positive news for
people who live in areas that are hundreds of miles from the nearest mental health
professional.

Another form of therapy that has been found to be effective in managing


agoraphobia includes self-exposure. In this type of intervention, the person either
imagines or puts him or herself into situations that cause increased levels of
agoraphobic anxiety, using relaxation techniques in each situation in order to
master their anxiety. As people gain access to the Internet, there is an increasing
evidence that exposure therapy can also be done effectively through that medium.
Phobias are also sometimes treated using beta-blocker medications, which block
the effects of adrenaline on the body, for example, propranolol.
Text 1: Questions 7-14

8. Tracking down Agoraphobia is;


A. Easy because it is associated with pain disorders.
B. Difficult as it is related to mood fluctuations.
C .Sometimes difficult and sometimes easy.
D .Not given

9. Agoraphobia occurs due to;


A .Anxiety-provoking events.
B. Internal emotional conflicts.
C .Unresolved Oedipal conflict.
D. All of the above.

10. Agoraphobia is;


A. Genetic
B .Non-genetic
C. Sometimes genetic
D. Can`t say

11. Virtually all of the medical conditions similar to Agoraphobia have;


A. No specific tests
B .Specific tests
C .A and B
D. It is not given

12. A patient suffering from Agoraphobia will have to undergo.


A .Physical examination
B .Laboratory tests
C. Mental Health tests
D .None
13. Cognitive behavioral therapy is considered to be highly effective;
A. With medications.
B. Without medications.
C .With or without medications.
D .In providing relief and stopping the re-occurrence of the disease.

14. Self Exposure Therapy is;


A. For mild to intense agoraphobia issues.
B. For people with years of agoraphobia.
C. A & B
D. Not given
Text 2: Xerostomia

Dry mouth is a condition that usually occurs due to a decreased production of saliva. At
times, xerostomia can make it difficult to speak and may lead to malnutrition. Extreme
dry mouth and salivary gland dysfunction can produce significant and permanent
mouth and throat disorders and can impair a person`s quality of life. Dry mouth is also
called xerostomia and it affects about 10% of all people; it is more prevalent in women
than men and it affects children in some rare cases. Disorders of saliva production
affect elderly people and those who are taking prescription and nonprescription
medications most frequently.

There are many different causes of dry mouth. Dry mouth most commonly occurs
as a side effect of medications that cause a decrease in saliva production, including
blood pressure medications, antihistamines, antidepressants, diuretics, nonsteroidal
anti-inflammatories, and many others. Other causes of dry mouth are radiation
treatments to treat cancerous tumors of the head and neck, salivary gland diseases,
diabetes, hormonal imbalance, mouth breathing, sleep apnea, and autoimmune
disorders such as Sjögren`s syndrome, rheumatoid arthritis, and systemic lupus
erythematosus. Eating disorders, such as bulimia and anorexia, are other risk factors
for developing xerostomia. Salivary production can be decreased if a major salivary
duct becomes blocked, such as from a stone or infection. Dry mouth will often occur
during pregnancy or breastfeeding due to dehydration and hormonal changes. Other
causative factors include stress, anxiety, depression, and dehydration.

People complaining of dry mouth may have trouble speaking, eating, tasting the
food, and swallowing. Frequently, a dry mouth may be most noticeable at night
during sleeping, especially in mouth-breathers. The dryness may cause chapped or
cracked lips, dry eyes, dry throat, pale gums, headaches, dizziness, bad breath or a
persistent cough. Those affected may also complain of sores in their mouth, or a
white tongue indicative of a fungal infection like yeast (candidiasis). Fungal infections
occurring in an individual with dry mouth may be associated with another underlying
disease or disorder such as Addison`s disease, HIV, or diabetes.
A burning tongue or throat, periodontitis, ulcers, sores, and inflamed soft tissue are
also all effects of oral dryness. Without a sufficient quantity of saliva to wash food
particles off the teeth, neutralize acids in the mouth, and battle the bacteria population,
a person frequently develops multiple cavities - especially around the gum line.

When the dry mouth is detected, the dentist can be helpful in obtaining a proper
diagnosis, which will help in developing a plan for management and treatment.
The dentist will inspect the main salivary glands and ducts to check for blockages
and may measure both stimulated and unstimulated salivary flow. The lips, tongue,
and oral tissues will also be inspected for dryness. Sometimes a patient will still
complain of dry mouth even if the salivary flow is adequate. Since the symptoms
of dry mouth vary greatly from individual to individual, the treatments also vary.
Sometimes treatments are given for prolonged, chronic complaint of dry mouth,
even without clinical signs of changes within the mouth. There is really no way to
prevent dry mouth, only the side effects of dry mouth. It is vital to detect, diagnose,
and treat xerostomia as early as possible to avoid the devastating effects of dry
mouth on dental and overall health.
Text 2: Questions 15-22

15. Xerostomia may lead to;


A. Malnutrition
B .Difficulty in swallowing
C. Difficulty in speaking D.
All of the above

16. Xerostomia is found in;


A .Women
B .Men
C. Children
D .Women and Men

17 Which one of these can be the cause of Dry Mouth?


A. Antihistamine and Antidepressants.
B .Blood pressure medications and non-steroidals.
C .Anti-inflammatory drugs.
D. B & C

18. Dry mouth occurs due to;


A. An injury to the salivary glands or the blocking of the salivary ducts.
B .Pregnancy in women.
C .Chronic conditions of other neck related diseases.
D. None

19. Dryness of mouth may lead to;


A. Difficulty in swallowing.
B. Difficulty in swallowing and speaking.
C .Chapped lips or cracked lips.
D. Sore throat
20. A white tongue is most commonly a symptom of;
A Diabetes
B Fungal infections.
C HIV
D Addison`s disease

21. Effects of oral dryness include;


A. Burning tongue
B. Burning throat C.
Ulcers and Sores D.
All of the above

22. According to the information above, a dentist will focus more on;
A .Assessing the stimulated or unstimulated salivary flow.
B. Inspection of the tongue and the oral tissues.
C .A & B
D. None

END OF READING TEST, THIS BOOKLET WILL BE COLLECTED


Reading test 17 : Answer Key

Part A - Answer key 1 – 7


1: C
2: C
3: D
4: D
5: A
6: B
7: B

Part A - Answer key 8 – 14


8: brachydactyly
9: clinodactyly
10: FGD1 Gene mutations
11: on X chromosome
12: FGD1 protein
13: fifth finger clinodactyly
14: autosomal dominant inheritance

Part A - Answer key 15 – 20


15: hypertelorism
16: autosomal recessive inheritance
17: cutaneous syndactyly
18: 40 genes
19: GTpases
20: philtrum
PART B (NO QUESTIONS)

Reading test - part C – answer key

Text 1 - Answer key 7 – 14


7: A Fear Of Going Outside.
8: D Not given
9: D All of the above.
10: C Sometimes genetic
11: A No specific tests
12: C Mental Health tests
13: C With or without medications.
14: A For mild to intense agoraphobia issues.

Text 2 - Answer key 15 – 22


15: A Malnutrition
16: D Women and Men
17: D B & C
19: B Difficulty in swallowing and speaking.
20: B Fungal infections.
21: D All of the above
22: C A & B
READING TEST 18
READING SUB-TEST : PART A
 Look at the four texts, A-D, in the separate Text Booklet.
 For each question, 1-20, look through the texts, A-D, to find the relevant information.
 Write your answers on the spaces provided in this Question Paper.
 Answer all the questions within the 15-minute time limit.
 Your answers should be correctly spelt.

TEXT BOOKLET -ECZEMA / DERMATITIS]

Text A

Eczema / Dermatitis
The words `eczema` and `dermatitis` are often used synonymously to describe a
polymorphic pattern of inflammation, which in the acute phase is characterized by
erythema and vesiculation, and in the chronic phase by dryness, lichenification and
fissuring. Contact dermatitis describes these patterns of reaction in response to
external agents, which may be acting either as irritants, where the T cell-mediated
immune response is not involved, or as allergens, where cell-mediated immunity is
involved. Contact dermatitis may be classified into the following reaction types:
Subjective irritancy ± idiosyncratic stinging and smarting reactions that occur
within minutes of contact, usually on the face, in the absence of visible changes.
Cosmetic or sunscreen constituents are common agents. Acute irritant contact
dermatitis is often the result of a single overwhelming exposure or a few brief
exposures to strong irritants or caustic agents. Chronic irritant contact dermatitis
occurs following repetitive exposure to weaker irritants, which may be either `wet`,
such as detergents, organic solvents, soaps, weak acids and alkalis, or `dry`, such
as low humidity air, heat, powders and dust.
Text B

Patch testing
The mainstay of diagnosis in allergic contact dermatitis is the patch test. This test
has a sensitivity and specificity of between 70% and 80%15. Patch testing involves
the reproduction under the patch tests of allergic contact dermatitis in an individual
sensitized to a particular antigen(s). The standard method involves the application of
the antigen to the skin at standardized concentrations in an appropriate vehicle and
under occlusion. The back is most commonly used principally for convenience
because of the area available, although the limbs, in particular the outer upper arms,
are also used. Various application systems are available of which the most commonly
used are Finn chambers. With this system, the investigator adds the individual
allergens to test discs that are loaded on to adhesive tape. Available are available ±
the TRUE and the Epiquic tests. There are few comparative studies between the
different systems. Pre-prepared tests are significantly more reliable than operator-
prepared tests. There is also some evidence that larger chambers may give more
reproducible tests. However, this may only apply to some allergens.
The open patch test, not so common, is used where potential irritants or sensitizers
are being assessed. It is also useful in the investigation of contact urticaria and
protein contact dermatitis. The open patch test is usually performed on the forearm
but the upper outer arm or scapular areas may also be used. The site should be
assessed at regular intervals for the first 30±60 min and a later reading should be
carried out after 3±4 days. A repeated open application test, applying the suspect
agent on to the forearm, is also useful in the assessment of cosmetics, where
irritancy or combination effects may interfere with standard patch testing. This
usually involves the application of the product twice daily for up to a week,
stopping if a reaction develops.
Text C

Photopatch testing
Where photoallergic dermatitis is suspected, photopatch testing may be carried out.
Very briefly, the standard method of photopatch testing involves the application of
the photo allergen series and any suspected materials in duplicate on either side of
the upper back. One side is irradiated with ultraviolet (UV) after an interval (1 or 2
days) and readings are taken in parallel after a further 2 days. The exact intervals
for irradiation and the dose of UVA given vary from centre to centre. The U.K.
multicentre study into photopatch testing has now been completed and published.
It is recommended that allergens be subjected to 5 J cm2 UVA and a reading to be
taken after 2 days. The incidence of photo allergy in suspected cases was low at
below 5%; however, further readings at 3 and 4 days increased the detection rate.

Text D
There are a number of aspects, which can have their effect on the accuracy of patch
testing. Principal among these are the characteristics of the individual allergens and
the method of patch testing. Some allergens are more likely to cause irritant
reactions than others. These reactions may be difficult to interpret and are easily
misclassified as positive reactions. Nickel, cobalt, potassium dichromate and carba
mix are the notable offenders in the standard series. As indicated above, preprepared
patch tests are better standardized in terms of the amount of allergen
applied and are therefore more reproducible, but are prohibitively expensive in the
U.K. Patient characteristics are also important. It is essential that the skin on the
back is free from dermatitis and that skin disease elsewhere is as well controlled as
possible. This will help to avoid the `angry back syndrome` with numerous false
positives. However, if a patient applies topical steroids to the back up to 2 days
prior to the test being applied or is taking oral corticosteroids or
immunosuppressant drugs, then there is a significant risk of false negative results.
PART A -QUESTIONS AND ANSWER SHEET

Questions 1-7
For each question, 1-7, decide which text (A, B, C or D) the information comes from.
You may use any letter more than once.

In which text can you find information about;


1. One of the most common tests. Answer _______________
2. The blockage or closing of a blood vessel or hollow organ. Answer __________
3. Important factors that may influence the testing results. Answer ____________
4 .The condition that occurs in response to excessive itching or rubbing of the skin. Answer __
5. Two different types of tests which help in diagnosing the disease condition. Answer _
6 .The process or fact of irradiating or being irradiated. Answer ___________
7 .Splitting or cracking of the skin. Answer __________

Questions 8-14

Answer each of the questions, 8-14, with a word or short phrase from one of the texts.
Each answer may include words, numbers or both. Your answers should be correctly spelt.

8. Which test is known to be more helpful with respect to the determination of


other irritants, which make it difficult to carry out other tests? Answer __________
9. What are given orally or by injection and distribute throughout the body? Answer _____
10. In which process do we apply the product two times? Answer ______________
11. What would be the outcome of the pre-prepared patch test, if there is dermatitis on the back of
the skin? Answer ______________
12. What are the common precipitants of contact dermatitis? Answer ____________
13 .What would be the outcome of the test, if the patient applies topical steroids two days prior to
the test? Answer _____________
14 .What was the proportion of photoallergic incidents in suspected cases? Answer __________
Questions 15-20
Complete each of the sentences, 15-20, with a word or short phrase from one of the
texts. Each answer may include words, numbers or both. Your answers should be correctly spelt.

15. ___________ as the cumulative irritant is known to create more worsening conditions.

16. The open patch is carried in order to examine the potentiality of the _______ or irritants.

17 .The chronic phase of a polymorphic pattern of inflammation is characterized by __________.

18. Carba mix is one of the _____________ in the standard series.

19. The standard method of testing involves the application of the___________.

20. As per recommendations, allergens be subjected to ________________.

END OF PART A, THIS QUESTIONS PAPER WILL BE COLLECTED


READING SUB-TEST : PART B
In this part of the test, there are six short extracts relating to the work of health professionals .
For questions 1-6, choose the answer (A, B or C) which you think fits best according to the text.
Write your answers on the separate Answer Sheet

Questions 1-6

1. The notice talks about;


A. How blood clots occur.
B. Blood clot doesn`t form. (if the reaction doesn`t occur the way it should)
C. Blocking of clotting reaction.

VKCFD
Inherited combined deficiency of the vitamin K dependent clotting factors is a very
rare inherited bleeding disorder that is caused by a problem with clotting Factors
II, VII, IX, and X. VKCFD can also be acquired later in life as a result of other
disorders, or certain medications such as the blood-thinning drug Coumadin.
Acquired VKCFD is more common than the inherited form. Some newborn babies
have a temporary vitamin K deficiency, which can be treated with supplements at
birth. In order to continue the chain reaction of the coagulation cascade, these four
factors need to be activated in a chemical reaction that involves vitamin K.
VKCFD is an autosomal recessive disorder, which means that both parents must
carry the defective gene in order to pass it on to their child. It also means that the
disorder affects both males and females.
2. What is correct?
A. All types of Factor I deficiency affect both males and females.
B. The fibrogen defect in impairment leads to disorder.
C. Genes can be both recessive or dominant.

Factor I deficiency

It is an umbrella term for several related disorders in males and females, known as
congenital Fibrinogen defects. Afibrinogenemia (a lack of Fibrinogen) and
hypofibrinogenemia (low levels of Fibrinogen) are quantitative defects, meaning
the amount of Fibrinogen in the blood is abnormal. Dysfibrinogenemia is a
qualitative defect in which Fibrinogen does not work the way it should.
Hypodysfibrinogenemia is a combined defect that involves both low levels of
Fibrinogen and impaired function. Afibrinogenemia is an autosomal recessive
disorder, which means that both parents must carry the defective gene in order to
pass it on to their child. Hypofibrinogenemia, dysfibrinogenemia, and
hypodysfibrinogenemia can be either recessive (both parents carry the gene) or
dominant (only one parent carries and transmits the gene).
3. What is referred to as weak muscle?
A. Chronic fatigue
B. Brachymesophalangy
C. Hypotonia

2q37 deletion syndrome

It is a condition that can affect many parts of the body. This condition is characterized
by hypotonia in infancy, mild to severe intellectual disability and developmental delay,
behavioral problems, characteristic facial features, and other physical abnormalities.
Most babies with 2q37 deletion syndrome are born with potentially chronic fatigue,
which usually improves with age. About 25 percent of people with this condition have
autism, a developmental condition that affects communication and social interaction.
The characteristic facial features associated with 2q37 deletion syndrome include a
prominent forehead, highly arched eyebrows, deep-set eyes, a flat nasal bridge, a thin
upper lip, and minor ear abnormalities. Other features of this condition can include
short stature, obesity, unusually short fingers and brachymesophalangy, sparse hair,
heart defects, seizures, and an inflammatory skin disorder called eczema. A few people
with 2q37 deletion syndrome have a rare form of kidney cancer called Wilms tumor.
4. The notice talks about;
A. Every kind of procedure is used in the process of sterilization to keep sterile the
objects or articles that are to be introduced into a wound or body cavity or that is
to penetrate the skin;
B. General overview of sterilization techniques;
C. The practices that the nurses will have to focus on; Sterile technique

Surgical asepsis is used to maintain sterilize. Use of effective sterile technique means
that no organisms are carried to the client. Microorganisms are destroyed before they
can enter the body. Sterile technique is used when changing dressings, administering
parenteral (other than the digestive tract) medications, and performing surgical and
other procedures such as urinary catheterization. With surgical asepsis, first articles
are sterilized, and then their contact with any unsterile articles is prevented. When a
sterile article touches an unsterile article, it becomes contaminated. It is no longer
sterile.
5. It is known to prevent viral infection;
A. PEG-IFN, RBV
B. Boceprevir
C. Sofosbuvir

Treatment for HCV


Before the commencement of HCV treatment, it is necessary to genotype the virus as
different genotypes require different types and duration of treatment, and the
protease inhibitors. Current therapy for genotype 1 infection is a combination of PEG-
IFN, RBV and a PI or nucleotide polymerase inhibitor, which results in high rates of
sustained virological response (SVR; a negative HCV RNA test three or six months after
the end of treatment). Boceprevir, simeprevir and telaprevir can also be used. Dual
therapy with PEG-IFN and RBV or sofosbuvir with RBV is used for genotypes 2 and 3
infections. Patients with genotype 4 infection treated with treated with sofosbuvir,
PEG-IFN and RBV have similar response rates when compared with genotype 1-
infected individuals. Small studies of genotypes 5- and 6-infected patients have shown
similar SVR rates to genotypes 2- and 3-infected ones.
6. The notice talks about;
A. Rate of heterogeneity of genitourinary sarcomas.
B. Survival rate and diagnosis.
C. Survival rates of patients with genitourinary sarcomas.

Patients with genitourinary sarcomas

Patients with genitourinary sarcomas are relatively in a bad state, when compared
with other soft tissue regions. Prognosis is relatively poor and can be explained by
the high proportion seen in high degree tumors, a large proportion of patients with
metastatic disease, large tumor and the area affected. In addition, the rarity and
heterogeneity of genitourinary sarcomas can explain the great variability in clinical
progress in different subgroups. Dissemination of urethral cancer follows the
anatomic subdivision. The anterior urethra has a lymphatic drainage system for
superficial and deep inguinal region. Posterior urethra drains the lymphatic
ganglion of the external iliac artery, hypogastric, and internal obturator muscle.
Late diagnosis is seen in one third of patients with inguinal lymphatic ganglion
metastasis and in 20% of those with pelvic ganglion metastasis.
READING SUB-TEST : PART C
In this part of the test, there are two texts about different aspects of healthcare.
For questions 7-22, choose the answer (A, B, C or D) which you think fits best according to the text.
Write your answers on the separate Answer Sheet

Text 1: Cancer and Cervical Cancer

The body is made up of trillions of living cells. Normal body cells grow, divide,
and die in an orderly fashion. During the early years of a person`s life, normal cells
divide faster to allow the person to grow. After the person becomes an adult, most
cells divide only to replace worn-out or dying cells or to repair injuries.

Cancer begins when cells in a part of the body start to grow out of control. There are
many kinds of cancer, but they all start due to out-of-control growth of abnormal cells.
Cells become cancer cells because of damage to DNA. DNA is in every cell and directs all
its actions. In a normal cell, when DNA gets damaged the cell either repairs the damage
or the cell dies. In cancer cells, the damaged DNA is not repaired, but the cell doesn`t die
like it should. Instead, this cell goes on making new cells that the body does not need.
These new cells will all have the same damaged DNA as the first cell does.

No matter where cancer may spread, it is always named after the place where it
started. For example, breast cancer that has spread to the liver is still called breast
cancer, not liver cancer. Likewise, prostate cancer that has spread to the bone is
metastatic prostate cancer, not bone cancer. Different types of cancer can behave
very differently. For example, lung cancer and breast cancer are very different
diseases. They grow at different rates and respond to different treatments. That is
why people with cancer need the treatment that is aimed at their particular kind of
cancer. Not all tumors are cancerous. Tumors that aren`t cancer are called benign.
Benign tumors can cause problems ± they can grow very large and press on healthy
organs and tissues. But they cannot grow into (invade) other tissues. Because they
can`t invade, they also can`t spread to other parts of the body (metastasize). These
tumors are almost never life threatening.
The cervix is the lower part of the uterus (womb). It is sometimes called the uterine
cervix. The body of the uterus (the upper part) is where a baby grows. The cervix
connects the body of the uterus to the vagina (birth canal). The part of the cervix
closest to the body of the uterus is called the endocervix. The part next to the vagina is
the exocervix (or ectocervix). The 2 main types of cells covering the cervix are squamous
cells (on the exocervix) and glandular cells (on the endocervix). The place where these 2
cell types meet is called the transformation zone. Most cervical cancers start in the
transformation zone and in the cells lining the cervix. These cells do not suddenly change
into cancer. Instead, the normal cells of the cervix first gradually develop pre-cancerous
changes that turn into cancer. Doctors use several terms to describe these pre-cancerous
changes, including cervical intraepithelial neoplasia (CIN), squamous intraepithelial lesion
(SIL), and dysplasia. These changes can be detected by the Pap test and treated to
prevent the development of cancer.

Cervical cancers and cervical pre-cancers are classified by how they look under a
microscope. There are 2 main types of cervical cancers: squamous cell carcinoma and
adenocarcinoma. About 80% to 90% of cervical cancers are squamous cell carcinomas.
These cancers are from the squamous cells that cover the surface of the exocervix. Under
the microscope, this type of cancer is made up of cells that are like squamous cells.
Squamous cell carcinomas most commonly begin where the exocervix joins the endocervix.

Most of the other cervical cancers are adenocarcinomas. Cervical adenocarcinomas


seem to have become more common in the past 20 to 30 years. Cervical
adenocarcinoma develops from the mucus-producing gland cells of the
endocervix. Less commonly, cervical cancers have features of both squamous cell
carcinomas and adenocarcinomas. These are called adenosquamous carcinomas or
mixed carcinomas. Although cervical cancers start from cells with pre-cancerous
changes (pre-cancers), only some of the women with pre-cancers of the cervix will
develop cancer. The change from cervical pre-cancer to cervical cancer usually
takes several years, but it can happen in less than a year. For most women, precancerous
cells will go away without any treatment. Still, in some women pre cancers turn into true
(invasive) cancers. Treating all pre-cancers can prevent almost all true cancers.
Text 1: Questions 7-14
7. When a person becomes an adult, the cells divide only to;
A Replace old cells
B Replace dying cells
C Repair injuries
D All of the above

8. The DNA in cancer cells is;


A Damaged, but can be repaired.
B Not totally damaged.
C Permanently damaged, but the cell doesn`t die.
D Damaged but creates new cells with damaged DNA.

9. Cancer in liver having its origin in breast is a;


A Liver cancer
B Breast cancer
C Prostate cancer
D Bone cancer

10. Benign tumors;


A Can grow and invade the tissues.
B Can grow and press on healthy organs and tissues.
C Are cancerous.
D Are highly fatal.

11. Most cervical cancers start in;


A Squamous cells;
B Glandular cells;
C The transformation zone;
D All the above;
12. There are............ main types of cervical cancers
A two
B three
C four
D five

13. About 80 to 90% of cervical cancers are;


A Adenocarcinoma
B Adenosquamous carcinomas
C Mixed carcinomas
D Squamous cell carcinomas

14. Pre-cancerous cells in women can go away;


A With growth in more number of new cells.
B With medications from doctors.
C Without treatment
D Can`t say
Text 2: Hepatitis - Viral Liver Infection

Hepatitis (A, B, or C) can be caused by a virus (Viral Hepatitis), drugs, alcohol,


medications, and blood transfusions. Scientists estimate that between 3.5 and 5.3
million people in the USA are living with Hepatitis. A blood test is required to
diagnose Hepatitis infection. Hepatitis A is a viral liver infection. In most cases the
body easily defeats the virus (much like the flu, which is what you may feel like
you have). Because of this it does not lead to long term liver challenges. Hepatitis
A is the most common form of Hepatitis. It is spread through the feces of a
contaminated person. This can easily be prevented by thoroughly washing hands
after using the restroom, before eating, and after changing a diaper.

Eating raw oysters and undercooked clams can increase your chances of
contracting the virus. If you are traveling in a country where Hepatitis is common
make sure you wash your hands often and well, eat cooked oysters and clams, and
use an antiviral essential oil such as Lemon to help protect yourself. Hepatitis B is
a viral liver infection. Again, most adult bodies are able to fight off the virus. In
this case, it is referred to as Acute (something that does not last long) Hepatitis B.
Hepatitis B is spread through contact with blood or bodily fluids of an infected
person. This can include unprotected sexual intercourse, sharing drug needles,
getting a tattoo with instruments that were not properly cleaned, or by sharing a
personal item such as a razor or toothbrush with an infected person.

A mother who is infected can pass the virus on to her baby during delivery. Again,
the symptoms are flu-like in nature, so it often goes undiagnosed. A person who
has Chronic (lasting three months or more) Hepatitis B may show no symptoms
until liver damage has occurred. Hepatitis B can lead to liver damage or cancer; your
doctor may want to do a biopsy to determine the amount of damage your liver has
experienced. Hepatitis C is also a viral liver infection. A few people will contract Hepatitis C
and get better. This is called Acute Hepatitis C. Most, however, will develop Chronic
Hepatitis C and go on to deal with liver damage,
cirrhosis of the liver, liver cancer, and possibly liver failure. Hepatitis C is the
number one reason for liver transplants in the USA.

Hepatitis C is spread through contact with contaminated blood. This can occur by
sharing a needle, receiving a blood transfusion or organ transplant (blood and organs have
been screened for Hepatitis in the USA since 1992), getting a tattoo with equipment that
has not been properly cleaned, and, in rare cases, a mother can pass the virus on to her
baby during birth. Scientists are not sure, but think there may be a slim possibility that the
virus may be passed through unprotected sexual intercourse.

Symptoms generally do not occur until the virus is causing damage. Again, the
symptoms are flu-like; you may also experience jaundice (yellowish eyes and skin)
after the flu-like symptoms go away. Most people discover they are infected by
having routine tests done or by donating blood or organs and the standard tests
show the infection. There is also a home test you can purchase and do if you
suspect you are infected.

If you are infected with a Hepatitis virus, or if you have been in the past, one of the
most important things that you can do is strengthen your liver. The easiest way to
do this is the Be Young Liver Cleanse: in the morning, take 1 drop of Be Young
Lemon essential oil, 1 drop of Be Young Peppermint essential oil, and 1 teaspoon
to 1 tablespoon of fresh lemon juice, followed by a glass of water. “Be Young essential
oils” are absolutely 100% pure, EOBBD tested and guaranteed to be free of synthetics and
extenders. Do not try this with an essential oil that you are not certain has been properly
cared for and tested as you do not want to increase challenges to your liver. When
properly supported, the liver has a remarkable capacity for regeneration.
Text 2: Questions 15-22

15. Hepatitis is caused by;


A Virus
B Alcohol consumption
C Medications
D All of the above

16. Which of the following spreads through feces of a contaminated person;


A Hepatitis A
B Hepatitis B
C Both
D Can`t say

17. Most adult bodies are able to fight off this virus;
A Hepatitis A virus
B Hepatitis B virus
C Both
D Can`t say

18. Hepatitis can lead to;


A Cancer
B Severe damage to the liver cells.
C Cancerous growth in the liver.
D Not given

19. In the USA people go for liver transplantation due to;


A Hepatitis A
B Hepatitis B
C Hepatitis C
D All of the above
20. Hepatitis C spreads through;
A Sharing needles
B Blood transfusion
C Organ transplantation
D All of the above

21. A patient may experience jaundice when;


A The flu-like symptoms appear.
B The flu-like symptoms disappear.
C Eyes become yellow.
D All of the above.

22. Be Young essential oils are;


A Free from extenders
B EOBBD
C Full of synthetics
D Not given

END OF READING TEST, THIS BOOKLET WILL BE COLLECTED


Reading test 18 : Answer Key

Part A - Answer key 1 – 7


1: C
2: B
3: D
4: A
5: B
6: C
7: A

Part A - Answer key 8 – 14


8: repeated open application test
9: Corticosteroids
10: repeated open application test
11: false positive result
12: cosmetic and sunscreen constituents
13: false negative result
14: below 5%

Part A - Answer key 15 – 20


Complete each of the sentences, 15-20, with a word or short phrase from one of
the texts. Each answer may include words, numbers or both.
15: Chronic irritant contact dermatitis
16: sensitizers
17: dryness, lichenification and fissuring
18: notable offenders
19: photo allergen series
20: 5 J cm2 UVA
Reading test - part B – answer key
1: B Blood clot doesn`t form. (if the reaction doesn`t occur the way it should)
2: A All types of Factor I deficiency affect both males and females.
3: C Hypotonia
4: A Every kind of procedure is used in the process of sterilization to keep sterile the
objects or articles that are to be introduced into a wound or body cavity or that is to
penetrate the skin;
5: B Boceprevir
6: C Survival rates of patients with genitourinary sarcomas.

Reading test - part C – answer key

Text 1 - Answer key 7 – 14


7: D All of the above
8: D Damaged but creates new cells with damaged DNA.
9: B Breast cancer
10: B Can grow and press on healthy organs and tissues.
11: C The transformation zone;
12: A two
13: D Squamous cell carcinomas
14: B With medications from doctors.

Text 2 - Answer key 15 – 22


15: D All of the above
16: A Hepatitis A
17: B Hepatitis B virus
18: A Cancer
19: C Hepatitis C
20: A Sharing Needles
21: B The flu-like symptoms disappear.
22: A Free from extenders
READING TEST 19
READING SUB-TEST : PART A
 Look at the four texts, A-D, in the separate Text Booklet.
 For each question, 1-20, look through the texts, A-D, to find the relevant information.
 Write your answers on the spaces provided in this Question Paper.
 Answer all the questions within the 15-minute time limit.
 Your answers should be correctly spelt.

TEXT BOOKLET- AUTISM SPECTRUM DISORDER

Text A

Autism Spectrum Disorder


Autism Spectrum Disorder (ASD) develops in early childhood. Recent population
analysis indicates that the number of cases of ASD is increasing in many countries,
particularly in technologically developed countries. The U.S. Center for Disease
Control research claims that, in some states, one of every 68 children (one of 42
boys) has a diagnosis of the ASD, a 30% increase from 2012 (IACC Strategic Plan
for Autism Spectrum Disorder Research, 2013.
Multiple dysfunctional reflex patterns are characteristic in two separate groups of
children diagnosed with autism: 1) those whose patterns were immature or
pathological and severely dysfunctional from birth, and 2) those that developed
normally but regressed into autism at age 2 or 3 unexpectedly. Reflexes of these
children may have been delayed and immature, but not noted by specialists or
parents. Their nerve system, possibly, was not resilient enough to cope with the
stress that they experienced. Alternatively, their reflexes might not have matured
and have caused the asynchronicity in their brain function development on both
cortical and extrapyramidal levels resulting in neurodevelopmental disorders
beginning around 2 years of age. An initially mild unrecognized problem can lead
to more complicated deficits with age.
Text B

Individuals diagnosed with ASD show a chronic lack of sensory motor integration
and delay of skills concerning the early motor milestones. They show a wide range
of immature reflex patterns such as Hands Pulling, Hands Supporting, Hands Grasp,
Crawling, Asymmetrical Tonic Neck Reflex, Symmetrical Tonic Neck Reflex, Babkin
Palmomental, Ocular-Vestibular, and other patterns. The MNRI program utilizes
non-invasive intervention to support the development of the neuro-sensory-motor
aspects of those reflex patterns through specific techniques and procedures that
allow restoration of links between reflex circuit components and the protection
function of a reflex to normalize their over-freezing and fight or flight reactions
seen, for example, in tactile defensiveness or deprivation. Thus, the MNRI program
works particularly with the autonomic nervous system – its sympathetic and
parasympathetic processes.
Text C

Disharmony in muscular system development and lack of regulation for muscle


tone beginning in children with ASD in their infancy results in impulsive reactions
that often turns into permanent physical characteristics and behaviors as they
grow older. For example, impulsivity may lead to poor ability for goal setting, poor
focus and following instructions, deficient inner control, hyperactivity,
disorganized and chaotic behavior, and irritability and impatience. Lack of muscle
tone regulation may later result in challenges in motor programming and control,
planning, and thus lead to poor motor-cognitive- behavior coordination. This poor
regulation is caused by a lack of balance in the excitation and inhibition processes
in the reflex circuits, including improper connectivity between alpha and gamma
motor neurons. Clinical observations show that the disharmony and lack of proper
regulation in muscle tone in children with ASD are seen mainly in: Hypertonic
muscles in the posterior dorsal plane of the body (along the spinal column -
sacrospinalis, thoracic longus, trapezius) and with the opposite hypotonic
abdominal muscles and diaphragm negatively affects development of postural
control. The child with ASD, in an attempt to release tension caused by this
disproportion of muscle tone in the back and front of their body, may often display
reactivity in behavior and impulsive movements triggering balance/equilibrium
mechanisms (balancing reflex pattern), resulting in a state of being overstimulated.
Text D

Problems in visual and auditory perception systems:


The eyes of children diagnosed with ASD show a restless state or lack of mobility
and dilated pupils. They usually have a limited, narrow visual span, poor visual
attention and focus, and hyperactive peripheral vision. Their eye movements
appear to freeze or jump rapidly in saccades. Many children with ASD demonstrate
an addictive tendency for computers and cell phones with compulsive repetition of
the same image, object, or program, often watching it at a very close range. The
child with ASD becomes over-focused, which over-stimulates their vestibular
system and static balance. The Pupillary Reflex in these children may become
hypersensitive, overstimulating the sympathetic system, with either over-reactive
or hypoactive motor activity. The visual system of children diagnosed with ASD
copes poorly with this visual chaos which leads to a visual processing disorder. A
Bonding response in infants is seen from their first months after birth. Bonding as a
behavior trait matures during their first years of life. Almost every child with ASD
assessed presented signs of inadequate bonding - lack of attachment, tactile and
interactivity defensiveness, a tendency for self-isolation, a poor imitation, and poor
learning of verbal communication. When bonding is immature, there are problems
with visual contact, focusing on the face/eyes of their mother and other adults and
poor emotional communication, inability to adequately smile, and poor labeling of
the objects in their environment.
PART A -QUESTIONS AND ANSWER SHEET

Questions 1-7
For each question, 1-7, decide which text (A, B, C or D) the information comes from.
You may use any letter more than once.
In which text can you find information about;
1. Represents the resistance to passive movement of a joint. Answer _______________
2. Associated with pupillary function. Answer ________________
3. Utilization of information and clinical experience from neurodevelopment in different ways.
Answer_____________
4. Development of autism. Answer______________
5. Possibility of development of strange characteristics as one grows. Answer ______________
6. Not existing or occurring at the same time with respect to movements or reactions. Answer____
7. Primitive reflex that normally emerges during the first year of an infant's life. Answer __

Questions 8-14
Answer each of the questions, 8-14, with a word or short phrase from one of the texts.
Each answer may include words, numbers or both. Your answers should be correctly spelt.
8. What MNRI will operate with? Answer _____________
9. How eye movements may appear? Answer ___________
10. What is the term which defines activation of the vestibular system which causes eye movement?
Answer ____________
11 ..What is related to tendency to act on a whim, displaying behaviour characterized by little or no
forethought, reflection? Answer ____________
12. What is the term used to define healthy stress? Answer__________
13. What is known to be activated as a result of turning the head to one side? Answer______
14. What is the impact visual chaos of the children with ASD? Answer ___________
Questions 15-20

Complete each of the sentences, 15-20, with a word or short phrase from one of the texts.
Each answer may include words, numbers or both. Your answers should be correctly spelt.

15. The term _______ is used to describe a rapid movement of the eye between fixation points.

16. Generally,____________ in newly born babies will be at very early stages.

17. What synchronizes hands, neck, and jaw is _____________

18. In many of the cases, reflex in affected children may turn out to be more __________ .

19. Almost all of the children with ASD show various signs of ____________ .

20 .The complexities in _______can be the result of the regulation changes in muscle tone.

END OF PART A, THIS QUESTIONS PAPER WILL BE COLLECTED


READING SUB-TEST : PART B
In this part of the test, there are six short extracts relating to the work of health professionals .
For questions 1-6, choose the answer (A, B or C) which you think fits best according to the text.
Write your answers on the separate Answer Sheet

Questions 1-6

1. What this notice talks about?


A. AIDS-related deaths dropped by more than 50%
B. AIDS-related deaths increased more significantly
C. Efforts that can lead to curbing AIDS in various countries.

UNAIDS Programme Coordinating Board

The 31st UNAIDS Programme Coordinating Board (PCB) meeting took place in Geneva
from 11-13 June 2018. There were more than 700 000 less new HIV infections estimated
globally in 2016 than in 2001. The road from 2.5 million new HIV infections in 2011 to zero
new HIV infections is a long one and significant efforts are required to accelerate HIV
prevention programmes. Sustained investments for access to antiretroviral therapy by
donors and national governments have led to record numbers of lives being saved in the
past six years.
In 2011 more than half a million fewer people died from AIDS-related illnesses than six
years earlier. It’s a dramatic turning point. Numbers can quantify, but alone cannot
express the impact of each averted death on the whole community, including its children.
The number of people dying from AIDS-related causes began to decline in the mid-2000s
because of scaled up antiretroviral therapy and the steady decline in HIV incidence since
the peak of the epidemic in 1997. In 2011, this decline continued, with evidence showing
that the drop in the number of people dying from AIDS-related causes is accelerating in
several countries.
2. The given notice explains the procedure of;
A. Use of radix Sophorae samples.
B. Ultrasonic treatment - obtaining radix.
C. Preparing radix Sophorae tonkinensis samples.

Radix Sophorae tonkinensis


Radix Sophorae tonkinensis was crushed and screened, then taking screened powder
(approximately 0.5 g) gain a respective weight, denoted M. Trichloromethane-methanol-
ammonia (40:10:1) was used to dispose of the radix Sophorae tonkinensis samples for 30
min, and all samples were subsequently obtained from organic solvent extraction with 30
min ultrasonic treatment. All disposed of samples were filtered, then 10 mL of filtrate was
measured to recover solvents to dry under decompression at 38°C to obtain the residue, the
residue was diluted by methanol, then transferred to a 10 mL volumetric flask. After mixing
and filtering with 0.45 µm filter membrane, Radix Sophorae samples were obtained. The
blank groups were treated as the samples but without radix Sophorae tonkinensis.

3. What is correct about Torcetrapib?


A. It is known to enhance health.
B. Trials performed produced negative results.
C. The drug did not get approved completely as the project was dropped in the middle.
Short note on Torcetrapib
Torcetrapib, which has been in development since the early 1990s, was supposed to raise
so-called good cholesterol, and cardiologists had hoped it would reduce the buildup of
plaques in blood vessels that can cause heart attacks. This drug actually caused an increase
in deaths and heart problems. Eighty-two people had died so far in a clinical trial, versus
51 people in the same trial who had not taken it. The GABR company gave up the project
incurring a claimed loss of $1 billion investment and not much hue and cry was raised on
the deaths of the study subjects as this was the doing of a billionaire giant manufacturer.
In case of trials or experiments, if a single death is noticed with the use of UD, a big hue
and cry would be raised.
4. The table shows
A. Comparison of Type A, B, C and G and H drugs and their resistance quality.
B. Type A is known to produce more adverse reactions than all the others present in the
table.
C. A total of 200 cases have been reported with respect to adverse reaction.

Drug Adverse Reactions


Type of ADR Number ADRS Percentage (%)
Type-A 96 103.7
Type-B 69 56.79
Type-C 23 28.39
Type G 8 7.4
Type H 4 3.7
Total 200 200

5. The notice clearly explains


A. Clinical trials using Vitamin B12.
B. Advantages of B12.
C. Study performed with focus on B12 usage.

Vitamin B12
Vitamin B12 is produced by the liver and is involved in several biochemical metabolic
reactions. It promotes the repair of damaged skin mucous membranes and vascular
endothelial cells, reduces spasm and occlusion of blood vessels, improves local blood flow
and prevents the deterioration of wound infection. In addition, it reduces the excitability of
pain fibers C and AG, leading to an analgesic effect. Vitamin B12 injections to the skin in the
radiation field benefit the wound by reducing irritation and pain, preventing rupture and
enhancing new epithelial resistance to radiation, thereby promoting healing of the skin.
Chen et al used a vitamin B12 solution to treat radiation-induced moist dermatitis. The cure
rate at 10 days was 100%, which was significantly different from the control group.
6. What is correct about the given table?
A. The age wise male patients population ranges from 4.
B. 18.18 patients were in the age group of 50-70 years.
C. 32.72 patients were in the age group of 50-60 years.

Age wise distribution of male patients showing percentage of distribution.


Age in years Male patients Age in years Female patients
20-30 4 (7.27) 20-30 0 (0)
30-40 54 (98.18) 30-40 32 (71.11)
40-50 24 (43.63) 40-50 38
50-60 18 50-60 10
60-70 10 60-70 6 (13.32)
Total 110 (110) 70-80 4 (8.88)
Total 90 (9
READING SUB-TEST : PART C
In this part of the test, there are two texts about different aspects of healthcare.
For questions 7-22, choose the answer (A, B, C or D) which you think fits best according to the text.
Write your answers on the separate Answer Sheet

Text 1: Ebola Virus and Marburg Virus

The Ebola virus and Marburg virus are related viruses that cause hemorrhagic fevers; illnesses
marked by severe bleeding (hemorrhage), organ failure and, in many cases, death. Both the
Ebola virus and Marburg virus are native to Africa, where sporadic outbreaks have occurred
for decades. The Ebola virus and Marburg virus both live in animal hosts, and humans can
contract the viruses from infected animals. After the initial transmission, the viruses can
spread from person to person through contact with bodily fluids or contaminated needles.

No drug has been approved to treat the Ebola virus or Marburg virus. People diagnosed with
the Ebola or Marburg virus receive supportive care and treatment for complications. Scientists
are coming closer to developing vaccines for these deadly diseases. In both the Ebola virus and
Marburg virus, signs and symptoms typically begin abruptly within the first five to 10 days of
infection. Early signs and symptoms include fever, severe headaches, joint and muscle aches,
chills, sore throat and weakness. Over time, symptoms become increasingly severe and may
include nausea and vomiting, diarrhea (may be bloody), red eyes, raised rash, chest pain and
coughing, stomach pain, severe weight loss, bleeding from the nose, mouth, rectum, eyes and
ears.

The Ebola virus has been found in African monkeys, chimps and other nonhuman primates. A
milder strain of Ebola has been discovered in monkeys and pigs in the Philippines. The
Marburg virus has been found in monkeys, chimps and fruit bats in Africa. The virus can be
transmitted to humans by exposure to an infected animal's bodily fluids, including blood.
Butchering or eating infected animals can spread the viruses; scientists who have operated on
infected animals as part of their research have also contracted the virus. Infected people
typically don't become contagious until they develop symptoms. Family members are often
infected as they care for sick relatives or prepare the dead for burial.
Medical personnel can be infected if they don't use protective gear such as surgical masks and
latex gloves. Medical centers in Africa are often so poor that they must reuse needles and
syringes and some of the worst Ebola epidemics have occurred because contaminated
injection equipment wasn't sterilized between uses. There's no evidence that the Ebola virus
or Marburg virus can be spread via insect bites.
Ebola and Marburg hemorrhagic fevers are difficult to diagnose because many of the early
signs and symptoms resemble those of other infectious diseases, such as typhoid and malaria.
But if doctors suspect that you have been exposed to the Ebola virus or Marburg virus, they
use laboratory tests that can identify the viruses within a few days.

Most people with Ebola or Marburg hemorrhagic fever have high concentrations of the virus
in their blood. Blood tests known as enzyme-linked immunosorbent assay (ELISA) and reverse
transcriptase polymerase chain reaction (PCR) can detect specific genes or the virus or
antibodies to them. No antiviral medications have proved effective in treating the Ebola virus
or Marburg virus infections. As a result, treatment consists of supportive hospital care; this
includes providing fluids, maintaining adequate blood pressure, replacing blood loss and
treating any other infections that develop.

As with other infectious diseases, one of the most important preventive measures for the Ebola
virus and Marburg virus is frequent hand-washing. Use soap and water, or use alcohol-based
hand rubs containing at least 60 percent alcohol when soap and water aren't available. In
developing countries, wild animals, including nonhuman primates, are sold in local markets;
avoid buying or eating any of these animals. In particular, caregivers should avoid contact with
the person's bodily fluids and tissues, including blood, semen, vaginal secretions and saliva.
People with Ebola or Marburg are most contagious during the later stages of the disease. If
you're a healthcare worker, wear protective clothing — such as gloves, masks, gowns and eye
shields - keep infected people isolated from others. Carefully disinfect and dispose of needles
and other instruments; injection needles and syringes should not be reused. Scientists are
working on a variety of vaccines that would protect people from Ebola or Marburg viruses.
Some of the results have been promising, but further testing is needed

Text 1: Questions 7-14


7. The Ebola and Marburg Viruses are native to;

A. America
B. Japan
C. Africa
D. China

8. What is right about Ebola and Marburg viruses?

A. Spread from person to person only.


B. Spread from animals to humans.
C. Spread from animals to animals.
D. Spread person to person after initial transmission from the infected animals.

9. Symptoms are typically seen within;

A. Five days
B. Ten days
C. Five to seven days
D. Five to ten days

10. In the Philippines, Ebola was discovered in;

A. Chimpanzees
B. Human primates
C. Non-human primates
D. Monkeys

11. Most known Ebola diseases occur due to;


A. Contamination
B. Bodily fluids
C. Contaminated needles and syringes
D. None

12. People with hemorrhagic fever show;

A. High number of viruses in their blood


B. Low concentrations of virus
C. High concentrations of antibodies
D. Low concentrations of antibodies

13. Pick one of the best preventive measures stated in the passage here;

A. Hand cleaning with medicinal soap.


B. Use of alcohol-based hand rubs, containing at least 60% alcohol, in absence of water &
soap.
C. Only use of soap.
D. Avoiding direct contact with patients is a necessity.

14. As a healthcare worker, you should;

A. Keep infected people totally isolated from others.


B. Not reuse needles and syringes for the second time.
C. Wear clothing such as gowns and eye shields.
D. none of the above
Text 2: A Chronic Disease - Atopic Dermatitis

Atopic dermatitis is a common chronic skin disease. It is also called atopic eczema.
Atopic is a term used to describe allergic conditions such as asthma and hay fever.
Both dermatitis and eczema mean inflammation of the skin. People with atopic
dermatitis tend to have dry, itchy and easily irritated skin. They may have times
when their skin is clear and other times when they have rash. In infants and small
children, the rash is often present on the skin around the knees and elbows and the
cheeks. In teenagers and adults, the rash is often present in the creases of the
wrists, elbows, knees or ankles, and on the face or neck.

Atopic dermatitis usually begins and ends during childhood, but some people
continue to have the disease into adulthood. If you have ever had atopic dermatitis,
you may have trouble with one or more of these: dry, sensitive skin, hand
dermatitis and skin infections. The exact cause of atopic dermatitis is unknown.
Research suggests that atopic dermatitis and other atopic diseases are genetically
determined; this means that you are more likely to have atopic dermatitis, food
allergies, asthma and/or hay fever if your parents or other family members have ever
had atopic dermatitis. These diseases may develop one after another over a period of
years. This is called the “atopic march”.

Knowing that a child with a slight wheeze has had a history of atopic dermatitis,
for example makes it easier to diagnose the subtle onset of asthma. There are many
things that make the itching and rash of atopic dermatitis worse. When you learn
more about atopic dermatitis and how to avoid things that make it worse, you may
be able to lead a healthier life.

If you have a reaction to something you touch, breathe or eat, you might have an
allergy. Allergies can trigger or worsen your atopic dermatitis symptoms. Common
causes of allergy are: dust mites, furry and feathered animals, cockroaches, pollen,
mold, foods, chemicals. Your healthcare provider may recommend allergy testing
and food challenges to see if allergies worsen itching or rashes. Allergy testing
may include skin testing, blood tests or patch tests. Many measures can be taken to
avoid things to which you are allergic. Although many of the measures can be done
for the entire home, the bedroom is the most important room to make skin friendly.
Talking with healthcare provider about what measures you can take to avoid your
allergens can be very beneficial.

Food allergies may be the cause of itching or rashes that occur immediately after
eating, especially in children. Some common food allergens include milk, eggs,
peanuts, wheat, nuts, soy and seafood. Most people are allergic to only one, two or
at the most three foods. Be aware that diet restrictions can lead to poor nutrition and
growth delay in babies and children. Talk with your healthcare provider about
maintaining a well-balanced diet.

Emotions and stress do not cause atopic dermatitis, but they may bring on itching
and scratching. Anger, frustration and embarrassment can cause flushing and
itching. Day to day stresses as well as major stressful events can lead to or worsen
the itch-scratch cycle.The medications used in atopic dermatitis include topical
steroids, topical immunomodulators, tar products, anti-infectives and
antihistamines. Steroid medicines that are applied to the skin are called topical
steroids. Topical steroids are drugs that fight inflammation; they are very helpful
when a rash is not well controlled. Topical steroids are available in many forms
such as ointments, creams, lotions and gels. It is important to know that topical
steroids are made in low to super potent strengths. Steroid pills or liquids, like
prednisone, should be avoided because of side effects and because the rash often
comes back after they are stopped.
Text 2: Questions 15-22
15. People with atopic dermatitis suffer from;

A. Hay fever
B. Asthma
C. Dry, itchy and irritated skin
D. Rashes

16. In small children, a rash is seen;

A. Around elbows
B. On the face
C. On the neck
D. Around the knees

17. People with atopic dermatitis have;

A. Dry skin
B. Skin infections
C. Hand dermatitis
D. All of the above

18. The term atopic refers to;

A. Allergic diseases
B. Asthma and hay fever.
C. Allergic conditions like hay fever.
D. Allergic conditions like asthma.

19. _____ can worsen dermatitis symptoms;

A. Allergies
B. Pollen
C. Dust
D. Mold
20. According to the information given in the passage, avoiding allergens is;

A. Easy
B. Difficult
C. Sometimes easy and sometimes difficult
D. Can say

21. Allergic conditions like asthma in patients who have had a history of atopic
dermatitis can be easily diagnosed by health professionals, this statement is;

A. Out of the paragraphs given


B. False
C. True
D. Can be true or can be false

22. Topical steroids are available in these forms:

A. Gel tubes
B. Ointments
C. Lotions
D. Ointments, creams, lotions and gels.

END OF READING TEST, THIS BOOKLET WILL BE COLLECTED


Reading test 19 : Answer Key

Part A - Answer key 1 – 7


1: C
2: D
3: A
4: A
5: C
6: A
7: B

Part A - Answer key 8 – 14


8: autonomic nervous system
9: freeze
10: Ocular-Vestibular
11: impulsivity
12: Eustress
13: asymmetrical tonic neck reflex
14: visual processing disorder

Part A - Answer key 15 – 20


15: Saccades
16: Bonding Response
17: Babkin Palmomental
18: Hypersensitive
19: Inadequate Bonding
20: Motor Programming
Reading test - part B – answer key

1: AIDS-related deaths increased more significantly


2: Preparing radix Sophorae tonkinensis samples.
3: Trials performed produced negative results.
4: Type A is known to produce more adverse reactions than all the others
present in the table.
5: Study performed with focus on B12 usage.
6: 32.72 patients were in the age group of 50-60 years.

Reading test - part C – answer key

Text 1 - Answer key 7 – 14


7: Africa
8: Spread person to person after initial transmission from the infected animals.
9: Five to ten days
10: Monkeys
11: None
12: High number of viruses in their blood
13 : Use of alcohol-based hand rubs, containing at least 60% alcohol, in absence of
water & soap.
14: Not reuse needles and syringes for the second time.

Text 2 - Answer key 15 – 22


15: Dry, itchy and irritated skin
16: Around the knees
17: All of the above
18: Asthma and hay fever.
19: Allergies
20: Easy
21: True
22: Ointments, creams, lotions and gels.
READING TES 20
READING SUB-TEST : PART A
 Look at the four texts, A-D, in the separate Text Booklet.
 For each question, 1-20, look through the texts, A-D, to find the relevant information.
 Write your answers on the spaces provided in this Question Paper.
 Answer all the questions within the 15-minute time limit.
 Your answers should be correctly spelt.

TEXT BOOKLET - GALECTIN-3

Text A

Galectin-3
It is well known that there is a close relation between obesity-induced insulin
resistance, immune cells accumulation in white adipose tissue (WAT) and
inflammation. Indeed, in obesity WAT is characterized by an increased production
and secretion of a wide range of inflammatory cytokines including TNF-alpha and
interleukin (IL)- 6, which may have local effects on endothelial, vasculature and
target adipose tissues. Therefore, activated macrophages and other antigen
presenting cells that are accumulated in elevated number in fat tissue in both types
of obese actively secrete a broad spectrum of locally produced pro-inflammatory
cytokines including galectin-3 (Gal-3). Gal-3 is a beta-galactoside-binding lectin
belonging to a multifunctional protein family, which enhances chemotaxis of
immune and antigen presenting cells, reduces insulin-stimulated glucose uptake in
myocytes and adipocytes and impairs insulin-mediated suppression of glucose
output in hepatocytes. Gal-3 may bind directly to the insulin receptor (IR) and
thereby inhibit downstream insulin resistance signaling via diminishing
interleukin-1 beta production. Therefore, Gal- 3 is a modulator of apoptosis,
necrosis and fibrosis associated with extracellular remodeling.
Text B

Gal-3 is increased in obesity and mediates inflammation and fibrosis in the heart
and vessels, as well as in the WAT. The most preclinical and clinical studies
suggest that this protein protects from inflammation in obese, while there is a large
body of evidence regarding the ability of Gal-3 to deteriorate glucose homeostasis,
modulate cell adhesion and induce pro-oxidant pathways. Interestingly, the low
serum Gal-3 concentrations are closely associated with insulin resistance in
patients with type 2 diabetes mellitus. In contrast, an inverse correlation between
serum Gal-3 and glycosylated hemoglobin in type 2 diabetes mellitus was
found. In clinical settings Gal-3 strongly independently predicts all-cause
mortality and CV mortality in the general population and in patients with known
CV disease. In fact, in cross-sectional analyses of 2946 Framingham Heart Study
participants circulating Gal-3 was associated well with abdominal adiposity,
dyslipidemia, and hypertension, but Gal-3 did not predict incident CV and
metabolic diseases after adjusting for cardiometabolic risk factors. Whether Gal-3
could be a predictive marker of the metabolically unhealthy obese is not clear,
although Gal-3 deserves further large clinical trials to understand its role in
different obese phenotypes’ development.
Text C

Natriuretic peptides (NPs) are “cardiometabolic” hormones with well-established


cardiovascular, renal, and endocrine abilities affecting sodium reabsorption and
blood pressure regulation. Although, NP levels are markers of biomechanical
cardiac stress, their role in the nature of the evolution of obese is not fully
understood. These controversies affect the clearance of NPs in obese and
pathophysiological mechanisms controlling the synthesis of them. While, NP
secretion result in a stretch of the cardiac wall and volume overload of cardiac
cavities. On the other hand, recent epidemiological and preclinical/ clinical studies
have shown that the NP system acts as a deficiency in obese patients is due to
worse clearance of NP receptors and neutral endopeptidases. Consequently, NP
system in obese is not able to mediate a wide spectrum of cardiovascular and
metabolic protective effects (i.e., vasodilation, natriuresis, diuresis, lipolysis, weight loss,
lusitropy, lipid peroxidation, and also improve mitochondrial respiration and insulin
sensitivity).

Text D
Brutsaert et al. have reported that higher levels of brain NP have associated with
decreased risk of diabetes in middle-aged adults and that the interrelation has
remained after adjustment for waist circumference, low physical activity, estimated
glomerular filtration rate and high sensitive C-reactive protein level. In contrast, it
is suggested that the low brain NP levels observed in obesity could causally
associated with the incidence of diabetes in obese individuals. The effect of brain
NPs might relate to an ability of natriuretic peptides to activate a thermogenic
program in brown and white fat tissues, increase energy expenditure and inhibit
food intake. Thus, NPs might play several metabolic roles in the development of
different phenotypes of obesity, but their predictive role in CV disease
development in obese patients is uncertain.
PART A -QUESTIONS AND ANSWER SHEET

Questions 1-7
For each question, 1-7, decide which text (A, B, C or D) the information comes from.
You may use any letter more than once.

1. Role of this metabolic hormones is not yet certain. Answer _____________

2 .Perform its role when there is a deficiency. Answer___________

3. Higher and lower level of the metabolic hormones. Answer ____________

4. Known to affect the interior surface of blood vessels and lymphatic vessels. Answer _______

5. Worsening relative sugar equilibrium. Answer _____________

6 .Introduction to carbohydrate-binding lectin whose expression is associated with inflammatory


cells. Answer ____________

7 .May have a direct impact on hemoglobin to which glucose is bound. Answer _____________

Questions 8-14
Answer each of the questions, 8-14, with a word or short phrase from one of the texts.
Each answer may include words, numbers or both. Your answers should be correctly spelt.

8. What defines the rate of myocardial relaxation? Answer___________

9. What are more associated with insulin resistance? Answer ___________

10. What can have the potential to have direct impact on insulin resistance? Answer ____

11. What defines an abnormal amount of lipids? Answer _____________

12. What are the markers of biomechanical cardiac stress? Answer______________

13 .What is associated with programmed cell death? Answer___________


14. What causes the cardiac wall to stretch and volume overload of cardiac cavities? Answer__

Questions 15-20

Complete each of the sentences, 15-20, with a word or short phrase from one of the texts.
Each answer may include words, numbers or both. Your answers should be correctly spelt.

15. ___________ can be associated with a significantly increased risk for coronary heart disease.

16 .higher levels of brain NP can be connected with the decreased risk of___________

17 .The term_____________ is referred to the thickening and scarring of connective tissue,


usually as a result of an injury.

18. It is still not clear that Gal-3 is a ______________ of the metabolically unhealthy obese

19. Excretion of sodium in the urine is a common condition and is called ______________

20. NPs may not mediate __________ of lipids.

END OF PART A, THIS QUESTIONS PAPER WILL BE COLLECTED


READING SUB-TEST : PART B
In this part of the test, there are six short extracts relating to the work of health professionals .
For questions 1-6, choose the answer (A, B or C) which you think fits best according to the text.
Write your answers on the separate Answer Sheet
Questions 1-6

1. What is correct about liver toxicity from sulfasalazine?


A. Hepatotoxicity can arise either from direct toxicity of the drug or its metabolites.
B. In rare cases, it can be immune mediated.
C. Immune-mediated toxicity is believed to be due to an idiosyncratic delayed type
hypersensitivity reaction.

Liver toxicity from sulfasalazine

Liver toxicity from sulfasalazine is a rare but serious side effect. It can range from
mild elevation in LFTs to hepatic failure and cirrhosis. The occurrence of severe
liver toxicity such as acute hepatitis as seen in our patient is <1%. After reaching
the gut, sulfasalazine is broken down by the colonic bacteria into its metabolites,
i.e., sulfapyridine and 5-aminosalicylic acid. Sulfapyridine is absorbed in the gut
and eliminated after acetylation by enzyme N-acetyltransferase which can have
variable activity based on the patient's genotype. Patients who have genotypes for
slow acetylation are found to be more predisposed to sulfasalazine-induced liver toxicity.
Injury can be hepatocellular which presents with disproportionate elevation in serum
aminotransferases or cholestasis which presents with disproportionate elevation in
alkaline phosphatase. Both patterns of liver injury can have elevation in bilirubin and
abnormal tests for liver synthetic function.
2. Adiponectin;
A. Is a protein hormone which is involved in regulating glucose levels.
B. Can have positive impact on CAD.
C. Plays a role in the development of insulin resistance.

Adiponectin

Adiponectin accounts for 0.01% of plasma protein with a half-life of 2.5 h; normal
adiponectin plasma level is 5-10 µg/mL with higher levels in females than males
due to sexual dimorphism. Adiponectin plasma forms are of two types, highmolecular-
weight and low-molecular-weight. In addition, high-molecular-weight adiponectin levels
are positively associated with CAD and negatively associated with risk of type 2 diabetes
mellitus (DM), but this is not true to the lowmolecular-weight adiponectin. Adiponectin
serum levels are inversely correlated with body mass index (BMI), visceral obesity, and
insulin resistance (IR); thus, it is regarded as an indicator and predictor of noninsulin
dependent DM, insulin resistant, and overt hyperglycemia.
3. NF induced pulmonary toxicity;
A. Is more common among women.
B. Is rare, but a serious toxic side effect may occur.
C. Can have the potential to negatively affect lungs.

NF induced pulmonary toxicity

NF induced pulmonary toxicity can be seen in three different forms, that is, due
to acute, subacute or chronic reaction. The acute form is the most common. Side
effects occurring up to the 1st month of receiving the first treatment are classified
as the acute form. Acute form develops secondary to hypersensitivity reaction
with peripheral demonstrating eosinophilia and thoracic CT showing ground glass opacity.
Chronic NF induced lung disease is seen predominantly in older women who present with
respiratory symptoms after a year or more of NF therapy. Characteristic pathologic finding
in chronic NF pulmonary toxicity is diffuse interstitial fibrosis, vascular sclerosis, fibrosis,
and thickening of the alveolar septa, interstitial inflammation, and bronchiolitis obliterans
with organising pneumonia.
4. Case Study gives information about;
A. A rare case of bilateral lower extremity edema in a young patient.
B. Talks of the effects of the low dose gabapentin therapy.
C. Worsening condition and management of the disease.

Case Study

A 46-year-old male with a past medical history of schizoaffective disorder and


chronic lower back pain, was admitted for management of worsening depression
and anxiety. He was started on gabapentin, 300mg twice daily for his back pain
and anxiety symptoms. His only other medication was hydrocodone. Over the next
few days, he started developing worsening bilateral lower extremity edema. He did
not have any cardiovascular-related symptoms. Physical exam was only significant
for 3+ pitting edema with all laboratory values and imaging being unremarkable.
Gabapentin was discontinued and his lower extremity swelling improved over
subsequent days. The incidence of pedal edema with gabapentin use is
approximately 7 to 7.5% with all studies being in elderly patients receiving doses
above 1200 mg/day. This case illustrates that lower doses of gabapentin can also
cause this adverse effect. It is important to recognize this adverse effect because
gabapentin is used in conditions like diabetic neuropathy, which is associated with
multiple comorbidities that can give rise to bilateral leg swelling. Presence of
gabapentin-induced leg swelling can thus confound the clinical picture.
5. What is correct about the survey?
A. Males were accounted for 53% of total participants
B. 43% participants had a bachelor degree of pharmacy
C. 71% were practicing as pharmacists
6. The data clearly explains that
A. The majority of the patients were between the age bracket of 29 days to 24 months
B. Over half of the patients were female
C. Male patients outnumbered female patients by 10%
READING SUB-TEST : PART C
In this part of the test, there are two texts about different aspects of healthcare.
For questions 7-22, choose the answer (A, B, C or D) which you think fits best according to the text.
Write your answers on the separate Answer Sheet

Text 1: Occupational Lung Diseases

Occupational lung diseases are a group of illnesses that are caused by either repeated,
extended exposure or a single, severe exposure to irritating or toxic substances that leads
to acute or chronic respiratory ailments. The rate of occupational lung conditions was
highest for education and health service workers in the private sector and local government
workers at 3 .8 and 5 .9 per 10,000 fulltime workers, respectively. There are two broad
categories of occupational lung diseases: (i) Diseases that are not occupation-specific, but
are aggravated at work, such as occupational asthma (ii) Diseases related to a specific
occupation, such as asbestosis, coal worker’s pneumoconiosis (black lung), berylliosis
(brown lung), and farmer’s lung. Common occupational lung diseases include
mesothelioma, occupational asthma, silicosis, asbestosis, and sick building syndrome.
Adult-onset asthma can be triggered by occupational exposures.

The estimated yearly cost of occupational injuries and illnesses is between $128
and $150 billion. Although, occupational lung diseases are often incurable, they are always
preventable. Improving ventilation, wearing protective equipment, changing work
procedures, and educating workers are key factors for prevention.
Occupational Asthma (OA) is the most common form of occupational lung disease.
Occupational asthma (also known as work-related asthma) is asthma that is caused by or
made worse by exposures in the workplace. Estimates suggest that 15 to 23 percent of new
asthma cases in adults are work related. Four states (California, New Jersey, Massachusetts,
and Michigan) tracked cases of occupational asthma over a seven-year period. During this
time, the occupations with the highest percentage of asthma cases were operators,
fabricators, and laborers (32.9%); managerial and professional specialty (20.2%), and
technical, sales, and administrative support jobs (19.2%). The four most common agents
associated with occupational asthma were miscellaneous chemicals (19.7%), cleaning
materials (11.6%), mineral and inorganic dust (11.1%), and indoor air pollutants (9.9%).
Malignant mesothelioma is a fatal type of cancer caused by exposure to asbestos.
Millions of construction and general industry workers have been exposed to
asbestos while on the job. Occupations associated with significantly higher
mesothelioma deaths include plumbers, pipefitters, and steamfitters; mechanical
engineers; electricians; and elementary school teachers. In the U.S., asbestos use
peaked in 1973 but had declined by 99.8 percent in 2007. Because mesothelioma
usually does not show up until 20 to 40 years after exposure, most of the deaths
from the disease are the result of exposures that occurred decades ago. This long
lag time means that mesothelioma deaths are expected to peak around 2010,
despite the much lower current use of asbestos. From 1999 to 2005, 18,068
malignant mesothelioma deaths were reported in the U.S. Men (81%) and
Caucasians (95%) accounted for the majority of these cases.

Silicosis is a disabling, dust-related disease and is one of the oldest occupational lung
diseases in the world. Silicosis is caused by exposure to and inhalation of airborne
crystalline silica. Dust particles from silica can penetrate the respiratory system and land
on alveoli (air sacs). This causes scar tissue to develop in the lungs and impair the
exchange of oxygen and carbon dioxide in the blood. Though symptoms of silicosis rarely
develop in less than five years, progression of the disease can lead to extreme shortness
of breath, loss of appetite, chest pains, and respiratory failure, which can cause death.
Silicosis also makes a person more susceptible to infectious diseases of the lungs, such as
tuberculosis. The death rate is generally low, but still too high considering that every one
of these deaths could have been prevented. Because of the low number of overall deaths
due to silicosis, multiple years of data are combined to provide a more accurate estimate
of the burden of this disease.
Text 1: Questions 7-14
7. The rate of OLC is reported to be higher in;

A. Healthcare service providers (private sectors)


B. Professionals in the field of education.
C. Government officials.
D. Local government workers and healthcare professionals in private industries.

8. One of these groups of diseases doesn’t come under OLD;

A. Mesothelioma and occupational asthma.


B. Occupational asthma and silicosis, asbestosis.
C. Asbestosis and mesothelioma.
D. Asbestosis and silicosis.

9. According to paragraph 2, OLDs are;

A. Incurable
B. Curable
C. Preventable but not curable
D. Curable and preventable

10. According to paragraph 2, work-related asthma means;

A. A disease which occurs due to more work


B. A disease which occurs due to less work
C. A disease which occurs due to exposure to work
D. None of the above

11. Common agents which are associated with OA in the lowest percentage are;

A. Air pollutants
B. Mineral and inorganic dust
C. Cleaning materials
D. Miscellaneous chemicals
12. The root cause of malignant mesothelioma is associated with the;

A. Use of the asbestos in the construction field


B. Exposure to asbestos on a regular basis
C. Low-quality asbestos
D. None of the above

13. The use of the asbestos was almost next to naught in the year;

A. 1997
B. 1973
C. 2007
D. 2010

14. Silicosis is;

A. More dangerous than occupational asthma.


B. More dangerous than mesothelioma.
C. Not as fatal as occupational asthma and mesothelioma.
D. Not very fatal.
Text 2: Immune System – Notes
Since inflammation in the body can lead to inflammation in the brain, we first need to
understand what inflammation is. Inflammation is part of the immune system’s response to
defend you against microbial infections. It is the body’s first line of defense against invasion
by microorganisms such as bacteria and viruses, and it is activated rapidly after infection.
The microbes are detected as foreign to the body by immune cells such as macrophages
(literally “big eater”). When macrophages encounter and recognize a foreign microorganism
they engulf the microorganism and, in addition, release a variety of cellular products into
the space around them that start and regulate further defenses that include inflammation.
Two classes of these products, known as cytokines and chemokines, leading to
inflammation. Cytokines are chemical messengers that travel away from the cells that
release them which causes alterations to the function of other cells. Chemokines also leave
the cell and attract other cells into the region. Together, they alter the blood vessels
near the site of infection, causing increased blood flow to the area and the entry of
immune system cells.

Inflammation—swelling, redness and heat—is part of the immune system’s first response to
microbial infections, but this defensive response is not limited to the bodily site of infection.
Soon after infection, a pattern develops that includes what is called the “acute phase
response (APR)” and “sickness behavior.” Fever is the most prominent feature of the APR
and for good reason: many microorganisms reproduce best at humans’ normal core body
temperature, and many of the immune system’s agents for killing them are bolstered by
elevated temperature. Sickness behaviors are well known to anyone who has had the flu.
They include reductions in activity, food intake, social interaction, mood sags; difficulty in
forming new memories; sleep changes; and sensitivity to pain increases (just think of how
even a light touch hurts when you have the flu). These changes also reduce the energetic
costs of behavior to free available energy stores to fight the infection. Fever, for example, is
quite energy intensive, requiring an extra 10 to 12 percent in energy for each degree rise. It
is obvious how all the sickness behaviors, with the exception of memory disruption, fit the
scheme of keeping us away from our usual activities. Memory disruption serves a different
purpose.
We now understand that all of the changes described above are accomplished
through the CNS. Fever, for example, occurs because the set point of temperature sensitive
cells in the hypothalamus is increased. Of course, behavior, mood, and pain are all products
of the CNS. This raises two issues: a) How does the CNS “know” what is going on in the
peripheral immune system, and b) What kinds of changes are produced in the CNS that
mediate fever and sickness behaviors? The same cytokines that participate in producing the
inflammatory response in the body also initiate the communication process to the CNS.
They accumulate in the bloodstream and thereby travel to the brain, where, although they
are large proteins and cannot readily cross the blood-brain barrier, these chemical signals
are carried across the barrier by active transport. They cross into the brain in regions where
the barrier is weak, and they bind to receptors on the insides of the cerebral vascular
blood vessels, thereby inducing the production of soluble mediators within the epithelial
cells that can cross into the brain.

The cytokine interleukin-1 beta is released in response to pathogen recognition, and a)


activates vagal fibers, b) diffuses into the brain where the barrier is weak,and c) are actively
transported across. In the brain, they act on microglia, which then produce and secrete
further cytokines that can act on neuronal cells, thereby producing sickness behaviors.

Often, a set of mechanisms that evolve to handle acute emergencies lead to outcomes that
nature did not intend if they are engaged too long. During a normal infection,
neuroinflammation and the resulting adaptive sickness behaviors persist only for several
days. However, if these responses become exaggerated or prolonged, the outcomes may
well become established, leading to cognitive impairment instead of brief memory
disruption, depression instead of reduced mood, fatigue instead of inactivity, and chronic
pain instead of acute pain. That is, physiology can become pathology when a set of
processes designed to be relatively brief becomes prolonged.
Text 2: Questions 15-22
15. The first line of defense implies;

A. Macrophages
B. Immune system
C. Inflammation
D. All of the above

16. Chemical messengers have;

A. The ability to change the functioning of other cells.


B. A greater level of mobility.
C. No capacity to move further away from cells.
D. None

17. APR develops;

A. At the time of infection.


B. After the infection.
C. After curing of the infection.
D. Before or after infection.

18. Fever occurs due to;

A. The increase in the growth of the microbes.


B. A powerless immune system.
C. The multiplication of the microbes, supported by the temperature of the human
body.
D. none

19. One of the following is not associated with sickness behaviors;

A. Loss of appetite and reduction in social interaction.


B. Reduction in social interaction and sleeping hours.
C. Change in mood and body language.
D. Body language and sleeping disorder.
20. Cytokines are described as;

A. Messenger cells.
B. Protein bodies and messenger cells.
C. Immune response bodies.
D. None

21. Cytokines cross the barrier;

A. Through active transport.


B. By bridging the blood-brain barrier.
C. By penetrating into the blood vessels.
D. Through the bloodstream and lymph vessels.

22. What happens if the responses become exaggerated?

A. Loss of memory may take place.


B. Reduction in normal mood may be noted.
C. Cognitive impairment occurs.
D. Acute pain may begin.

END OF READING TEST, THIS BOOKLET WILL BE COLLECTED


Reading test 20 : Answer Key

Part A - Answer key 1 – 7


1: D
2: C
3: D
4: A
5: B
6: A
7: B

Part A - Answer key 8 – 14


8: Lusitropy
9: Low serum Gal-3 concentrations
10: Gal-3
11: Dyslipidemia
12: NP levels
13: Gal- 3
14: NP secretion

Part A - Answer key 15 – 20


15: Interleukin 1 Beta
16: Diabetes
17: Fibrosis
18: Predictive Marker
19: Natriuresis
20 : Oxidative Degradation
Reading test - part B – answer key

1: Hepatotoxicity can arise either from direct toxicity of the drug or its metabolites.
2: Plays a role in the development of insulin resistance.
3: Can have the potential to negatively affect lungs.
4: Talks of the effects of the low dose gabapentin therapy.
5: 71% were practicing as pharmacists
6: The majority of the patients were between the age bracket of 29 days to 24 months and

Reading test - part C – answer key

Text 1 - Answer key 7 – 14


7: Local government workers and healthcare professionals in private industries.
8: Asbestosis and mesothelioma.
9: Preventable but not curable
10: None of the above
11 Is: Air pollutants
12: Exposure to asbestos on a regular basis
13: 2007
14: Not very fatal.

Text 2 - Answer key 15 – 22


15: Inflammation
16: The ability to change the functioning of other cells.
17: After the infection.
18: The multiplication of the microbes, supported by the temperature of the human body.
19: Body language and sleeping disorder.
20: None
21: Through active transport.
22: Cognitive impairment occurs.
READING TEST 21
READING SUB-TEST : PART A
 Look at the four texts, A-D, in the separate Text Booklet.
 For each question, 1-20, look through the texts, A-D, to find the relevant information.
 Write your answers on the spaces provided in this Question Paper.
 Answer all the questions within the 15-minute time limit.
 Your answers should be correctly spelt.

TEXT BOOKLET - CLOSTRIDIUM TETANI

Text A

Clostridium tetani is a toxin producing, anaerobic, gram-positive bacillus. It is a


spore forming pathogen with spores found naturally in soil, animal feces, and
manure. Typically, spores are introduced into the body via an injury that causes a
break in skin structure such as a laceration. Once C. tetani spores are within the
body, they convert to their vegetative forms and multiply within the tissues at the
site of injury. The anaerobes begin producing toxins, tetanolysin and
tetanospasmin. The tetanus toxin targets vesicle-associated membrane protein
(VAMP) which is involved in neurotransmitter release from nerve endings.
Therefore, a symptom of flaccid torpidity or immobility can be present when the
toxin binds and interferes with the release of acetylcholine at the neuromuscular
junction. The toxin can be transported in the axons and thereby reach the spinal
cord or brainstem. Once within the CNS, the toxin can be taken up by inhibitory
GABAergic or glycinergic neurons or both, where the tetanus toxins can cleave
VAMP and inhibit release of GABA and glycine. This results in rigidity and
spasms of hyperactive muscles. The toxin irreversibly binds to tissues and thereby
cannot be neutralized by TIG, once bound.
Text B

Diagnosis
There is no existing laboratory test for tetanus, thus the diagnosis is based purely
upon clinical presentation. In 2009, the Council of State and Territorial
Epidemiologists published a case definition, which stated: “in the absence of a
more likely diagnosis, an acute illness with muscle spasms or hypertonia and
diagnosis of tetanus by a health care provider.” Cases are classified as probable
provided they are clinically compatible with tetanus presentation and are reported
by a healthcare professional. There is no definition for confirmed tetanus. A case
suggestive of tetanus should be reviewed for immunization status, history of injury,
duration before a patient develops signs and symptoms, patient-specific signs,
symptoms and their progression in order to make a diagnosis. Although, not in all
the cases it is possible, however, an early intervention before the condition
becomes serious can be better understood. Majorly, it is connected more or less to
the incubation period.

Text C

Epidemiology
Since 1947 when tetanus became a nationally reportable disease in the United
States, the number of cases has got curtailed by > 95% and the number of deaths
dwindled by > 70%. Between 2001 and 2008, the Centers for Disease Control
received 233 tetanus case reports, with 197 reported patient outcomes. The case -
fatality rate for the period analyzed was 13.2%, with an annual incidence of 0.10 per
1 million and 0.23 for those ≥ 65 years of age. Tetanus-toxoid (TT) containing
vaccination status was known for 92 patients, of which 37 (40.2%) had received no
doses of TT vaccination. Appropriate prophylaxis was not administered in 49 of 51
(96.1%) patients, who were presented with an acute wound.
Text D

Treatment
According to the CDC, tetanus treatment includes the administration of TIG, a
tetanus toxoid booster, agents to control muscle spasms (benzodiazepines), and
antibiotics (metronidazole). Aggressive wound care measures should also be
initiated. TIG confers passive immunity to the toxin produced by the tetanus
pathogen, C.tetani, through neutralization of the exotoxin produced by the
bacterium. TIG is indicated for prophylaxis in patients whose immunization status
against tetanus is incomplete or uncertain. TIG is also indicated as a component of
the regimen for treatment of active cases of tetanus. Standard dosing, per the
package insert, for routine prophylaxis of adults and children of seven years or
older is 250 units of TIG given by deep intramuscular injection. Children below
seven years can receive prophylactic doses of either 4 units/kg or the entire
contents of the vial 240 units.
Treatment of active cases should be implemented immediately, with the dosage
adjusted according to the severity of the infection. Currently, the recommended
dosage range is 500 - 6,000 units, with no optimal range clearly defined. A study
conducted by Nation et al enrolled 20 patients with active tetanus and treated them
each with a single dose of between 3,000-6,000 units.
PART A -QUESTIONS AND ANSWER SHEET

Questions 1-7
For each question, 1-7, decide which text (A, B, C or D) the information comes from.
You may use any letter more than once.

In which text can you find information about;


1. Constant decrease in number of deaths due to the bacteria. Answer________________.
2. Need to evaluate the conditions and adhering to proper medical regime. Answer_______.
3. Increased tightness of muscle tone and reduced capacity of the muscle to stretch. Answer__.
4. Help decrease the bacterial invasion. Answer____________.
5. Bacteria is adept at cutting or tearing down the skin surface. Answer________________.
6 .Need of identifying and providing vaccines. Answer____________.
7. A sign of weak paralysis. Answer______________.

Questions 8-14
Answer each of the questions, 8-14, with a word or short phrase from one of the texts.
Each answer may include words, numbers or both. Your answers should be correctly spelt.

8. What is known to affect molecular assembly? Answer__________________.


9 .Where toxin eventually will lead to? Answer_______________.
10 .What is the duration for the tetanus disease to become severe? Answer_____________.
11. What is reported as not executed among patients? Answer________________.
12. What is acceptable medication for small children? Answer_______________.
13. What does the tetanus toxoid booster confers? Answer___________.
14. What symptoms can be present when the toxin binds and interferes with the release of
acetylcholine at the neuromuscular junction? Answer_______________.
Questions 15-20

Complete each of the sentences, 15-20, with a word or short phrase from one of the texts.
Each answer may include words, numbers or both. Your answers should be correctly spelt.

15. TIG cannot neutralize the toxin once it__________ .

16. The spores convert to their ____________ as they enter the body.

17 ._____________ is reported to be the actual rate by which the death occurred.

18. ____________ in appropriate ranges can be given for the children who are 7 years of age.

19. Reports showed that it can be possible to effectively deal with the disease with _____.

20. With the presence of the _______, the severity of the disease reaches to its highest degree.

END OF PART A, THIS QUESTIONS PAPER WILL BE COLLECTED


READING SUB-TEST : PART B
In this part of the test, there are six short extracts relating to the work of health professionals .
For questions 1-6, choose the answer (A, B or C) which you think fits best according to the text.
Write your answers on the separate Answer Sheet
Questions 1-6

1. What information does this notice give?


A. How the splenectomy shall be performed for better results?
B. What does the operation involve?
C. Spleen and Splenectomy

Laparoscopic splenectomy

Laparoscopic splenectomy involves making four or more small (5 ± 10 mm) incisions on the
abdomen. Carbon dioxide gas is then pumped into the abdominal cavity to provide a space
to operate in. A fiber-optic telescope and long instruments are then inserted into the
abdomen and the spleen is separated from the stomach, kidney and colon. The blood
supply of the spleen is stapled across with a special device that secures the blood vessels.
The spleen is then placed in a bag inside the abdomen. One of the small incisions is
enlarged in order to deliver the spleen out of the abdomen in this bag.
2. The manual talks about;
A. Assessment as per the protocols.
B. Functionality of the Powerheart AED.
C. STAR Biphasic Energy Protocols for Powerheart G3 AED.

The STAR Biphasic defibrillation waveform will deliver variable escalating energy that is
customized to each patient’s needs based upon a patient’s thoracic impedance. This
customization adjusts for the unique physical differences between patients. The
Powerheart G3 AED comes equipped with five different FDA cleared biphasic energy
protocols. The operator, with guidance, direction, and implementation from the designated
AED program Medical Director, may select from one of the protocols when placing the
Powerheart G3 AED into service. The Powerheart G3 AED’s factory default energy protocol
is 200-300-300 Joule (J) escalating Variable Energy (VE). The first shock is delivered within
the range of 126J-260J. Subsequent shocks are delivered within a range of 170J351J
3. The table gives more information about;
A. Natural history and outcomes of NASH.
B. Common causes of deaths.
C. Mortality rate - a comparison.

Patients with NASH

There is growing evidence that the patients with histological NASH, especially those
with some degree of fibrosis, are at higher risk for adverse outcomes such as cirrhosis
and liver-related mortality. Patients with NASH have increased overall mortality
compared to matched control populations without NASH. The most common cause of
death in patients with NAFLD is the cardiovascular disease (CVD), independent of other
metabolic comorbidities. Although liver-related mortality is the 12th leading cause of
death in the general population, it is the second or third cause of death among
patients with NASH. Cancer-related mortality is among the top three causes of death in
subjects with NASH. Patients with histological NASH have an increased liver-related
mortality rate.
4. What is right about stem cells?
A. They create, recreate cells, replenish cell supply.
B. Develop and produce different kinds of cells.
C. Generate organs.

Stem cells are homogenous biological cells those cells can distinguish into specialized
cells and can split to produce more stem cells and it’s found into multicellular
organisms. In mammals, mostly there are two types; one is embryonic stem cells,
which are isolated from the inner cell mass of blastocysts, and adult stem cells, which
are located in various tissues. And the other one is adult stem cells and progenitor cells
act as a repair system for the body, replenishing adult tissues. In a developing embryo,
stem cells distinguish into all specified cells, ectoderm, endoderm and mesoderm but
also uphold and recover the regenerative organs, such as blood, skin, or intestinal
tissues. The three sources of autologous adult stem cells in humans are Bone marrow,
Adipose tissue and Blood. Stem cells can also be taken from umbilical cord blood just
after birth. Embryonic cell lines and autologous embryonic stem cells created through
somatic cell nuclear transfer or dedifferentiation have also been proposed as assuring
candidates for future therapies. Embryonic cell lines and autologous embryonic stem
cells spawned through somatic cell nuclear transfer or dedifferentiation have also been
intended as promising candidates for future therapies.
5. What is correct as per the given notice
A. The number of DM patients increased from 2011-12.
B. Exercise can be the best antidote for DM.
C. Deaths by DM increased from 2011-2014.

DM - Overview

An estimated 23.4 million adults have diagnosed diabetes mellitus (DM), 7.6
million have undiagnosed DM, and 81.6 million have prediabetes.
Analyses of high school±aged blood donors in 2011 to 2012 reported that 10% had
prediabetes hemoglobin A1c levels and an additional 0.6% had hemoglobin A1c
≥6.5%, the threshold endorsed to diagnose DM.
A recent analysis of randomized controlled trials showed that exercise may exert
its favorable effects by significantly improving glucose tolerance and insulin
resistance. The benefits of exercise were further supported by a large intervention
project that showed that higher fitness was associated with a lower risk of incident
DM regardless of demographic characteristics and baseline risk factors.
In 2014, there were 76488 DM-related deaths
6. The notice gives information about;
A. The proposal to fund dementia studies.
B. A move by the Australian Government.
C. Achievements of National Institute of Dementia Research.

The Federal Government is providing an additional $200 million for dementia


research over the next five years. This will significantly boost funding for Australia’s
dementia research sector to over $60 million per annum. As part of the Federal
Government’s commitment to dementia research, the National Health and Medical
Research Council’s National Institute of Dementia Research was established to ensure
priority research in dementia is coordinated, funded and communicated. The Institute
collaborates with Australia’s best researchers while also drawing on the expertise of
consumers, health professionals, industry and policy makers to translate evidence into
policy and practice that works towards achieving a five-year delay in the onset of dementia
by 2025. One of the pressing issues is to build capacity in the dementia research sector by
supporting students and early career dementia researchers.
READING SUB-TEST : PART C
In this part of the test, there are two texts about different aspects of healthcare.
For questions 7-22, choose the answer (A, B, C or D) which you think fits best according to the text.
Write your answers on the separate Answer Sheet.

Text 1: Defending Mechanisms in the Human Body


The surface defences of the vertebrate body are very effective but are occasionally
breached, allowing invaders to enter the body. At this point, the body uses a host of
a nonspecific cellular and chemical devices to defend itself. We refer to this as the
second line of defence. These devices all have one property in common: they
respond to any microbial infection without pausing to determine the invader’s
identity. Although these cells and chemicals of the nonspecific immune response
roam through the body, there is a central location for the collection and distribution
of the cells of the immune system, called the lymphatic system. The lymphatic
system consists of a network of lymphatic capillaries, ducts, nodes and lymphatic
organs and, although it has other functions involved with circulation, it also stores
cells and other agents used in the immune response. These cells are distributed
throughout the body to fight infections, and also stored in the lymph nodes where
foreign invaders can be eliminated as body fluids pass through.

Macrophages (“big eaters”) are large, irregularly shaped cells that kill microbes by
ingesting them through phagocytosis, much as an amoeba ingests a food particle.
Within the macrophage, the membrane-bound vacuole containing the bacterium
fuses with a lysosome. Fusion activates lysosomal enzymes that kill the microbe by
liberating large quantities of oxygen free-radicals. Macrophages also engulf
viruses, cellular debris, and dust particles in the lungs. Macrophages circulate
continuously in the extracellular fluid, and their phagocytic actions supplement
those of the specialized phagocytic cells that are part of the structure of the liver, spleen,
and bone marrow. In response to an infection, monocytes (an undifferentiated leukocyte)
found in the blood squeeze through capillaries to enter the connective tissues. There, at the
site of the infection, the monocytes are transformed into additional macrophages.
Neutrophils are leukocytes that, like macrophages, ingest and kill bacteria by phagocytosis.
In addition, neutrophils release chemicals (some of which are identical to household bleach)
that kill other bacteria in the neighbourhood as well as neutrophils themselves. On the
other hand, natural killer cells do not attack swarming microbes directly. Instead, they kill
cells of the body that have been infected with viruses. They kill not by phagocytosis, but
rather by creating a hole in the plasma membrane of the target cell. Proteins, called
perforins, are released from the natural killer cells and insert into the membrane of the
target cell, forming a pore. This pore allows water to permeate into the target cell, which
then swells and bursts. Natural killer cells also attack cancer cells, often before the cancer
cells have had a chance to develop into a detectable tumour. The vigilant surveillance by
natural killer cells is one of the body’s most potent defences against cancer.

The cellular defences of vertebrates are enhanced by a very effective chemical defence
called the complement system. This system consists of approximately 20 different proteins
that circulate freely in the blood plasma. When they encounter a bacterial or fungal cell
wall, these proteins aggregate to form a membrane attack complex that inserts itself into
the foreign cell’s plasma membrane, forming a pore-like that produced by natural killer
cells. Water enters the foreign cell through this pore, causing the cell to swell and burst.
Aggregation of the complement proteins is also triggered by the binding of antibodies to
invading microbes. The proteins of the complement system can augment the effects of
other body defences. Some amplify the inflammatory response by stimulating histamine
release; others attract phagocytes to the area of infection; and still others coat invading
microbes, roughening the microbes’ surfaces so that phagocytes may attach to them more
readily.

There are three major categories of interferons: alpha, beta, and gamma. Almost all cells
in the body make alpha and beta interferons. These polypeptides act as messengers that
protect normal cells in the vicinity of infected cells from becoming infected. Though
viruses are still able to penetrate the neighbouring cells, the alpha and beta interferons
prevent viral replication and protein assembly in these cells. Gamma interferon is
produced only by particular lymphocytes and natural killer cells. The secretion of gamma
interferon by these cells is part of the immunological defence against infection and cancer,
as we will describe later.
Text 1: Questions 7-14
Text 2: Evolution of Ophthalmic Surgery

One of the advantages that ophthalmologists enjoy is that they are able to directly
view the pathological processes of the eye. Medical observers have been able to
document the two most common causes of blindness, cataract and corneal scarring,
since records began. Perhaps it was the chance of observation that the spontaneous
dislocation of a mature cataract from the visual axis gave some restoration of sight
that prompted the technique of ‘couching’, a form of cataract operation first
described in Sanskrit manuscripts 2000 years ago (however, there are also other
theories which show a different picture, so this particular theory is a bit ‘touchand-go’).
With the aid of a sharp instrument to penetrate the eye and dislocate the
lens, the navigational vision could be restored in an otherwise blind eye.

Effective treatment of a corneal scarring was more elusive, since complete removal
of a diseased cornea required its replacement with similar transparent tissue if the
integrity of the globe is to be preserved. Although the Greek physician Galen (AD
130±200) documented removal of corneal scars by superficial keratectomy, it was
not until 1837 that the first successful transplantation of a cornea was recorded by
an Irish physician, Bigger. He had been held captive by some Bedouin in the
Sahara, during which time he managed to restore the sight of a pet gazelle with a
homograft from another animal. It was perhaps inevitable that the earliest corneal
transplants attempted in humans were xenografts. Transient success was claimed by
Kissam in the U.S., who performed a graft with porcine tissue in 1838. The first
successful full-thickness corneal allograft was performed in 1905 by a surgeon
named Zirm working near Prague, who restored the sight of a 45-year old man
with bilateral corneal scarring from lime burns. This early success was a spur to
further experimentation, but it would not be for another 50 years that corneal
transplantation became a reproducible procedure.
The introduction of cocaine anesthesia in 1884 by ophthalmologist Karl Koller in
Vienna, inspired by his colleague in neurology Sigmund Freud, was a potent
stimulus to the development of ophthalmic surgery. Other doctors across the globe
collaborated for the development of a great surgical process. Safe and adequate
anesthesia for corneal and other anterior segment procedures could now be readily
achieved, and cataract extraction through a 180° corneal incision became a
standard procedure. Iridectomy and other glaucoma drainage procedures were also
to become commonplace within a few years. The chief technical problem with
early corneal grafting was that of fastening the transplanted tissue in place.
Although sharp trephines and scissors could be manufactured to excise the
diseased central cornea and to prepare the donor tissue, sutures at that time were
not suitable for such fine work, particularly when a watertight wound closure was
required. The tissue was held in place by ‘stay’ sutures across the surface of the
eye, which was kept closed with padding while the patient rested in bed for many
days until the wound closure was established.

The main causes of corneal failure worldwide are trachoma, vitamin A deficiency,
herpes simplex and other types of infectious keratitis. These diseases destroy the
optical function of the cornea by scarring and opacification, and by stromal melting
and thinning that cause surface topographic irregularity. Although the effect of an
external keratitis on vision may be profound, on the corneal endothelial function it
is often minimal. When grafting is carried out in these conditions, it is necessary to
replace only the diseased superficial stromal layers to restore normal corneal
clarity and optical function. By avoiding penetration of the globe, lamellar
keratoplasty gives freedom from many of the complications of a penetrating graft.
Many small incremental advances in instrumentation and surgical technique were
to make a substantial impact on graft outcome. Although surgeons had used
magnifying loupes from the outset of corneal transplantation, the introduction of
the operating microscope gave a new dimension to the accuracy of the technique.
Improved microsurgical instrumentation reduced tissue damage during surgery.
The introduction of nylon monofilament as a suture material, enabling perfect
corneal wound closure with negligible tissue reaction, was a major step forward.
Corneal wound healing is slow, and a suture may need to be left in situ for as long
as 12 to 18 months before adequate wound strength is attained, if the healing
process has been inhibited by steroid treatment. Another advancement has been the
introduction of the viscoelastic substance sodium hyaluronate, which provides
ophthalmic surgeons with a method of handling delicate eye tissues with virtually no
trauma during intraocular surgery.
Text 2: Questions 15-22

15. The most appropriate heading for paragraph 1 is;


A. Cataract and corneal scarring.
B. The evolution of surgical eye processes.
C. Cataract surgery.
D. An ancient technique of sight restoration.

16. The word touch-and-go in paragraph 1 may mean;


A. not true.
B. difficult.
C. not completely certain.
D. completely certain.

17. The most appropriate heading for paragraph 2 is;


A. Corneal surgery.
B. How corneal scarring can be treated.
C. A short history of corneal scarring.
D. A history of eye surgical attempts in humans.

17. The most appropriate heading for paragraph 2 is;


A. Corneal surgery.
B. How corneal scarring can be treated.
C. A short history of corneal scarring.
D. A history of eye surgical attempts in humans.

18. According to paragraph 2, the first successful attempt of corneal surgery was made in the
year
A. 1837
B. 1838
C. 1905
D. 1955
19. Paragraph 3 talks more about;
A. Early surgical experiences.
B. The use of anesthesia.
C. Glaucoma
D. Collaborative experiments by doctors across the globe.

20. According to paragraph 3, one of the major problems was;


A. Making the tissue soft.
B. Removing the deceased cornea.
C. Enhancing the donor issue.
D. Transferring the tissue in the right place.

21. The most appropriate heading for paragraph 4 is;


A. How to perform grafting.
B. Corneal surgery and grafting.
C. Indications for grafting.
D. Causes of cornea failure.

22. In paragraph 5, the word in situ may mean;


A. In the original place
B. Certainly
C. Without disturbing
D. Without covering

END OF READING TEST, THIS BOOKLET WILL BE COLLECTED


Reading test 21 : Answer Key

Part A - Answer key 1 – 7


1: C
2: B
3: B
4: D
5: A
6: B
7: A

Part A - Answer key 8 – 14


8: Tetanus Toxin
9: spinal cord or brainstem
10: incubation period
11: Appropriate prophylaxis
12: 250 units of TIG
13: passive immunity to the toxin
14: flaccid torpidity or immobility

Part A - Answer key 15 – 20


Complete each of the sentences, 15-20, with a word or short phrase from one of
the texts. Each answer may include words, numbers or both.
15: irreversibly binds to tissues
16: vegetative forms and multiply
17: 13.2%
18: Prophylactic doses
19: 3,000-6,000 units
20: GABA and glycine
Reading test - part B – answer key

1: B What does the operation involve?


2: C STAR Biphasic Energy Protocols for Powerheart G3 AED.
3: A Natural history and outcomes of NASH.
4: B Develop and produce different kinds of cells.
5: B Exercise can be the best antidote for DM.
6: B A move by the Australian Government.

Reading test - part C – answer key

Text 1 - Answer key 7 – 14

Text 2 - Answer key 15 – 22


15: C Cataract surgery.
16: C not completely certain.
17: A Corneal surgery.
18: A 1837
19: A Early surgical experiences.
20: D Transferring the tissue in the right place.
21: C Indications for grafting.
22: A In the original place
READING TEST 22
READING SUB-TEST : PART A
 Look at the four texts, A-D, in the separate Text Booklet.
 For each question, 1-20, look through the texts, A-D, to find the relevant information.
 Write your answers on the spaces provided in this Question Paper.
 Answer all the questions within the 15-minute time limit.
 Your answers should be correctly spelt.

TEXT BOOKLET – LYMPHOEDEMA

Text A

Phenotyping childhood lymphoedema


Primary lymphoedema is chronic oedema caused by a developmental abnormality
of the lymphatic system and is the most common type seen in the paediatric
population. In primary lymphoedema, fluid accumulates due to either abnormal
function or structure of the lymphatic system. In most cases, oedema will be
present from birth but in some cases, although the lymphatic abnormality is
presumed to be present congenitally, the swelling does not develop until some time
later. It is thought that the lymphatic system normally functions at about 10%
capacity and it is presumed that this functional redundancy allows homeostasis to
be maintained for some time in this group of patients. A population prevalence of
1.33 per 1000 for all ages has been reported but it is acknowledge that this is
probably an under estimation of the true burden of disease. A specific prevalence
figure for primary lymphoedema in the paediatric population has been estimated at
1.15 per 100,000 population, but these numbers are based on those attending a
single US clinic. A female preponderance (M:F ± 1:3) is documented, although this
may represent ascertainment bias. Primary impairment of the lymphatic drainage
system will begin surfacing due to a non-syndromic mendelian condition or as
majorly because of other undefined disorder.
Text B

Milroy disease

Milroy disease is congenital onset lymphoedema which, although rarely, transform


limbs to the nth degree or portion below the knee to the toe partially but is not
reported to affect the arms. Males and females are affected equally. In males,
hydroceles are common, affecting up to 30% of those carrying the altered gene.
Other characteristic clinical findings include upslanting `ski-jump` toenails and
prominent large calibre veins in the legs, most commonly the great (long)
saphenous veins. The causative gene was first located on chromosome 5q in 1998
and subsequently numerous causative mutations in the VEGFR3 gene have been
described. In those individuals not conforming to the Milroy phenotype, molecular
testing of VEGFR3 is not warranted. Inheritance of Milroy disease is autosomal
dominant with a penetrance of approximately 85%. This is going to be troublesome
for sure as there will be 50% that children might get it from parents who are
carrying the genes. De novo mutations have been reported so a family history is
not mandatory. For this reason, it is always worthwhile testing the parents of an
affected child before calculating recurrence risks.
Text C

Lymphoedema distichiasis

Lymphoedema distichiasis syndrome is a very unusual, not-so-arcane, dominantly


inherited condition for which the underlying genetic cause was identified by Fang
et al. Almost all individuals with lymphoedema distichiasis syndrome have
mutations in FOXC2. This condition is the association of primary, pubertal, or
post-pubertal, onset lymphoedema with aberrant eyelashes arising from the
Meibomian glands in the eyelids. Other associations include; cleft palate,
congenital heart disease, varicose veins and ptosis. Half of affected individuals will
have clinically evident varicose veins from an early age, while 100% have venous
abnormalities when assessed by ultrasound scanning. A lymphoscintigram showing
lymph reflux will necessitate the diagnosis of lymphoedema distichiasis. If
distichiasis is not present in the patient or a family member, the chance of finding a
mutation in the causative gene, FOXC2, is extremely unlikely. As distichiasis can
be difficult to see, slit lamp examination by an ophthalmologist is advised
wherever possible. Due to the increased risk of both cleft palate and congenital
heart disease, additional pre-natal scans may be recommended for a fetus at risk of
inheriting the condition.
Text D

Case Study
A 29-year-old woman with a history of renal cysts, hypertension and lymphoedema
distichiasis syndrome, was referred to ophthalmology, with bilateral blurred vision,
hyperaemia and ocular pain, developed over months.
The patient had no positive family history for lymphoedema-distichiasis or other diseases.
Clinical examination revealed stunted height (144 cm), neck webbing, bilateral
and asymmetric lymphoedema, bilateral distichiasis and keratitis. Other ocular
manifestations of lymphoedema-distichiasis, such as ptosis and strabismus, were
excluded through ophthalmological examination.
The patient`s symptoms regressed with bilateral electrolysis of the abnormal
follicles after unsuccessful attempts at epilation and follicle removal using an argon laser.
Lymphoedema-distichiasis syndrome is a rare condition, associated with
diminished quality of life, being linked with chronic keratitis, conjunctivitis and
photophobia in 75% of cases.
Distichiasis, which may be present at birth, is observed in 94% of affected individuals.
The FOXC 2 gene is the only gene in which mutations are known to cause
lymphoedema distichiasis syndrome. Its protein has a role in a variety of
developmental processes, such as formation of veins, lungs, eyes, kidneys, urinary
tract, cardiovascular system and lymphatic vessels. Any pathogenic variant of this
gene could lead to varicose veins, absence of lymphatic valves, lymphoedema, and
cardiovascular and kidney malformations.
The patient did not have a family history for this syndrome. For this reason,
lymphoedema-distichiasis syndrome in this case was a probable phenotypic
manifestation of a de novo mutation in the FOXC 2 gene.
PART A -QUESTIONS AND ANSWER SHEET

Questions 1-7
For each question, 1-7, decide which text (A, B, C or D) the information comes from.
You may use any letter more than once.

In which text can you find information about;


1. A rare disease condition, peculiarly begins at very early age. Answer_____________.
2 .Diagnosis is based on the presence of primary lymphoedema and distichiasis. Answer____.
3. Disorder is passed down (inherited) through families. Answer____________.
4 .A condition characterized by an excess of watery fluid collecting in the cavities or tissues of the
body. Answer____________.
5. It can be said that approximately 75% of affected individuals have an affected parent. Answer_.
6 No proper statistics are available. Answer____________.
7 Classically affects lower half of the legs only. Answer____________.
Questions 8-14
Answer each of the questions, 8-14, with a word or short phrase from one of the texts.
Each answer may include words, numbers or both. Your answers should be correctly spelt.

8. What can occur as a part of more complex syndromic disorder? Answer____________.


9. What is suggestive of a diagnosis of lymphoedema distichiasis? Answer___________.
10. Which syndrome is inherited in an autosomal dominant manner? Answer__________.
11 .What is that Greek Word which may mean “Two Rows?” Answer__________.
12. What is the percentage of recurrence risk for family members if one of the parents carries the
gene? Answer____________.
13. Which Greek or Latin Word may mean “From The Beginning? Answer__________.
14. Which syndrome can also present with cardiac abnormalities? Answer___________.
Questions 15-20

Complete each of the sentences, 15-20, with a word or short phrase from one of the
texts. Each answer may include words, numbers or both. Your answers should be correctly spelt.

15.___________ is possible even with more definitive treatment.

16.___________ are known to be affecting many of those with modified gene.

17. In absence of distichiasis, it could be difficult to find_____________ in the causative gene.

18. Conservative management of _____________ is with lubrication or epilation.

19 .__________ may not surface itself in an apparent way.

20. It is advisable that ______ shall undergo testing as this can minimize risk of transfer of genes

END OF PART A, THIS QUESTIONS PAPER WILL BE COLLECTED


READING SUB-TEST : PART B
In this part of the test, there are six short extracts relating to the work of health professionals .
For questions 1-6, choose the answer (A, B or C) which you think fits best according to the text. Write
your answers on the separate Answer Sheet
Questions 1-6

1. As per information given in the table, to define NAFLD;


A. Evidence of hepatic steatosis both by histology and imaging is required.
B. Secondary causes of hepatic fat accumulation needed to be taken into consideration.
C. Monogenic hereditary disorder gets revealed.
NAFLD
In the majority of patients, NAFLD is commonly associated with metabolic comorbidities
such as obesity, diabetes mellitus, and dyslipidemia. NAFLD can be categorized
histologically into nonalcoholic fatty liver (NAFL) or non-alcoholic steatohepatitis. NAFL is
defined as the presence of 5% HS without evidence of hepatocellular injury in the form of
hepatocyte ballooning (e.g., ballooning), with or without any fibrosis. A not-so common
form of monogenic disorder but the underlying genetic defect is expressed .

2. The notice is giving more information about;

A. Signs and symptoms.


B. Problems that might occur due to CMV.
C. The contagious nature of the virus.

Infected babies may have health problems that are apparent at birth or may develop
later during infancy or childhood. Although not fully understood, it is possible for
CMV to cause the death of a baby during pregnancy (pregnancy loss). Some babies
may have signs of a congenital CMV infection at birth. These signs include:
 Premature birth, Liver, lung and spleen problems,
 Small size at birth, Small head size, and Seizures.
Babies with congenital CMV infection at birth may have long-term health
problems, such as: Hearing loss, Vision loss, Intellectual disability, Small head size etc.
3. What is right about risks, specific to Laparoscopic Splenectomy?
A. Swelling on the stomach.
B. May damage lungs completely.
C. Blood vessels may rupture.
Risks Involved
Injury to the tail of the pancreas, resulting in a collection of fluid in the abdomen that may
require a further operation or drainage procedures. Bleeding from the blood vessels that
flow to the spleen requiring a return to the operating theatre. Significant distention of the
stomach that may lead to a large vomit. Occasionally some of this vomit may be inhaled
into the lungs and cause life threatening pneumonia. This is why a tube will be placed via
your nose into the stomach for the first day after the operation.
Splenunculi. Many people have tiny `extra` spleens. After the spleen is removed
they may grow and patients with blood diseases may have a recurrence of their
disease. This may require further surgery. Because the spleen is very close to the lung,
collapse of the left lung, to some measure, is quite common after splenectomy. A
physiotherapist will work with you to prevent this. It is very common to have a slight fever
on the first 1 ± 2 days after the operation because of this lung collapse.

4. The report stresses more on;


A. Shows relatively small for GA
B. Mothers who had consultations more than three times outnumbered those who had
consultations only twice or thrice.
C. Widespread prevalent of the infection.
The majority of the NB had adequate intrauterine growth (67.9%), while 26.9% were small
for gestational age (SGA) and 5.2% were large for gestational age.
Maternal age varied from 14 to 48 years (mean 25.8±7.3 years). More than 80% of the
population of recently delivered mothers were married or had a stable relationship. Two
hundred and thirty-seven (94.8%) of the 250 mothers had received some prenatal, with 175
attending four or more consultations and 62 from one to three.
5. The manual gives information about;
A. Asystole threshold rate
B. Detection rate
C. ECG operations

All ventricular fibrillation (VF) and ventricular tachycardia (VT) rhythms at or


above this rate will be classified as shockable. All rhythms below this rate will be
classified as non-shockable. This rate is programmable between 120 bpm (beats
per minute) and 240 bpm via MDLink Software by the Medical Director. The
default Detection Rate is 160 bpm
The asystole baseline-to-peak threshold is set at 0.08 mV. ECG rhythms at or
below 0.08 mV will be classified as asystole and will not be shockable.
The AED will detect noise artifacts in the ECG. Noise could be introduced by
excessive moving of the patient or electronic noise from external sources like
cellular and radio telephones. When noise is detected, the AED will issue the
prompt “ANALYSIS INTERRUPTED. STOP PATIENT MOTION” to warn the operator.
The AED will then proceed to reanalyze the rhythm and continue with the rescue.
6. What does the word sepsis mean?
A. Life-threatening complication arising due to infection.
B. Indicates release of chemicals into the bloodstream to fight the infection.
C. Higher death risk arising due to infection

Pediatric Trauma

The field has made major advances in the areas of sepsis, lung injury, traumatic
brain injury and postoperative care. The pediatric intensives role in the Trauma
Events is to give steady care during cardiorespiratory or multi-organ failure or
recuperation from surgical medications or a traumatic injury that happens to an
infant, child or adolescent. It manages the medicinal consideration of infants,
children, and teenagers, and as far as possible as a rule ranges from birth up to 18
years old. It is a zone inside a healing center, spends significant time in the
consideration of critically ill infants and children. The risk of death for injured
children is significantly lower when care is provided in pediatric trauma, which
focuses as opposed than non-pediatric trauma which is incorporated into the
Critical Care Meetings.
READING SUB-TEST : PART C
In this part of the test, there are two texts about different aspects of healthcare.
For questions 7-22, choose the answer (A, B, C or D) which you think fits best according to the text.
Write your answers on the separate Answer Sheet

Text1: Seborrheic Eczema

Seborrheic eczema, also known as seborrheic dermatitis, is a very common skin


condition that causes redness often asymmetrically, scaly patches, and dandruff. It
most often affects the scalp in an expeditious manner. It may, sometimes, begin to
flourish in oily areas of the body, such as the face, upper chest, and back. It is hard
to say how it may expand or who may get it - adults, children or newborn babies.
When infants develop this condition, it`s known as crib cap; it typically develops
within the first few weeks of life and gradually disappears over several weeks or
months. The exact cause of seborrheic eczema isn`t known. However, doctors
believe there are two main factors that can contribute to the development of the
condition. The first factor is an overproduction of oil. An excess amount of oil in
the skin might act as an irritant, causing the skin to become red and greasy. The
second contributing factor is Malassezia, which is a type of fungus that`s naturally
found in the skin`s oils. It can, sometimes, burgeon abnormally, causing the skin to
secrete more oil than usual; the increased production of oil can lead to seborrheic eczema.

The condition might also develop in infants due to hormonal changes that occur in
the mother during pregnancy. The fluctuating hormone levels are believed to
stimulate the infant`s oil glands, leading to an overproduction of oil that may
provoke this condition, begin to peeve the skin. Seborrheic eczema is a long-term
skin condition that requires ongoing treatment. However, developing a good skin
care routine and learning to recognize and eliminate triggers can help you manage
the condition effectively. The symptoms of seborrheic eczema are often aggravated
by various factors, including stress, change of seasons, and heavy alcohol use. The
types of symptoms that thrive enormously can vary from person-to-person. It`s also
possible for symptoms to occur in different parts of the body.
Seborrheic eczema tends to develop in oily areas of the body. It most often affects
the scalp, but it can also occur in the following areas: in and around the ears, on the
eyebrows, on the nose, on the back, on the upper portion of the chest etc.
Seborrheic eczema has a distinct appearance and set of symptoms: skin develops
scaly patches that flake off; the patches may be white or yellowish in color (this
problem is commonly known as dandruff and it can occur in the scalp, hair,
eyebrows, or beard); skin in the affected area tends to be greasy and oily; skin in
the affected area may be red; skin in the affected area may be itchy; hair loss may
occur in the affected area.

Doctors aren`t exactly sure why some people develop seborrheic eczema while
others don`t. However, it does appear that it develops more quickly if a close
family member has it. Other factors that contribute to its growth may include:
obesity, fatigue, poor skin care, stress, environmental factors, such as pollution, the
presence of other skin issues, such as acne etc. The symptoms are similar to those of
other skin conditions, including rosacea. To make an accurate diagnosis, a
doctor will perform a physical examination and carefully inspect the affected
areas. The doctor may also want to perform a biopsy before making a diagnosis.
During this procedure, the doctor will scrape off skin cells from the affected area;
these samples will then be sent to a laboratory for analysis. The results will help to
rule out other conditions that may be causing symptoms.

The doctor will likely recommend the patients to try some home remedies before
considering medical treatments. Dandruff shampoos are frequently used to treat
seborrheic eczema on the scalp; they usually need to be used every day for optimal
results and it is important to follow all instructions on the bottle carefully. Other
home treatments that may help manage seborrheic eczema include: using over-thecounter
(OTC) antifungal and anti-itch creams; using hypoallergenic soap and
detergent; thoroughly rinsing soap and shampoo off the skin and scalp; shaving off
a moustache or beard; and wearing loose cotton clothing to avoid skin irritation.
Text 1: Questions 7-14

7. According to paragraph 1, seborrheic eczema is;


A. Common among adults
B. Common among children
C. Common among newborn babies
D. Still a mystery to doctors

8 In paragraph 1, which medically-suitable word or phrase would mean growing or developing


more in infection?
A Asymmetrical
B Burgeon
C Flourish
D Expand

9. According to paragraph 2, which one of the following statements is true about seborrheic
eczema?
A. Infants may get this disease from their parents.
B. This disease occurs due to secretion of excess of oil by the oil glands, during
pregnancy.
C. There is no specific treatment available for this disease.
D. This disease can spread to various parts of the body as well.

10. According to paragraph 2, which word would mean: to make a bad situation worse?
A. Provoke
B. Peeve
C. Aggravate
D. Thrive

11. Paragraph 3 talks more about ……...


A. How the disease affects body parts.
B. How the symptoms develop.
C. How it can spread to other body areas.
D. Affected areas and symptoms.
12. In paragraph 3, flake off may mean .
A. to begin to fall
B. to come off a surface in small, thin pieces
C. to become more obvious
D. to begin to develop, usually on the surface

13. The most suitable heading for paragraph 4 is ………...


A. Common symptoms of seborrheic eczema
B. Common symptoms and risk factors for eczema
C. Who is at risk for seborrheic eczema?
D. How is seborrheic eczema examined?

14. The most suitable heading for paragraph 5 is ?


A. Treatment options for seborrheic eczema
B. Cost-effective management of seborrheic eczema
C. Common treatment procedures for home-based patients
D. How can you treat seborrheic eczema at home?
Text 2: Fibromyalgia

Fibromyalgia is a long-term or chronic disorder. It`s associated with widespread


pain in the muscles and bones, areas of tenderness, and general fatigue. Symptoms
like these are considered subjective. The lack of reproducible, objective tests for
this disorder plays a role in some doctors questioning the disorder altogether.
Although it`s more widely accepted in medical circles now than previously, some
doctors and researchers don`t consider fibromyalgia a de facto condition. This can
increase risk of depression, which stems from a struggle in gaining acceptance for
painful symptoms.

Researchers are closer to understanding fibromyalgia, so the stigma that surrounds


the condition is disappearing. In the past, many doctors worried that people could
use this undetectable pain as an excuse to seek out prescription pain medication.
Doctors are now finding that lifestyle changes may be better than medication in
treating and managing this condition. The more that doctors begin to accept this
diagnosis, the more likely the medical community is to explore effective ways of
treating fibromyalgia.

Fibromyalgia is often associated with areas of tenderness, which are called trigger
points or tender points. These are places on your body where even light pressure
can cause pain. Today, these points are rarely used to diagnose fibromyalgia.
Instead, they may be used as one way for doctors to narrow their list of possible
diagnoses. The pain caused by these trigger points can also be described as a
constant dull ache affecting many areas of your body. If you were to experience
this pain for at least three months, doctors may consider this a symptom of
fibromyalgia. People with this disorder may also experience: fatigue, trouble
sleeping, sleeping for long periods of time without feeling rested, headaches, depression etc.
A person used to be diagnosed with fibromyalgia if they had widespread pain and
tenderness in at least 11 of the known 18 trigger points. Doctors would check to
see how many of these points were painful by pressing firmly on them. Trigger
points are no longer the focus of diagnosis for fibromyalgia. Instead, doctors may
make a diagnosis if you report widespread pain for more than three months and
have no diagnosable medical condition that can explain the pain. Medical
researchers and doctors don`t know what causes fibromyalgia. However, thanks to
decades of research, they`re close to understanding factors that may work together
to cause it which include: Infections: Prior illnesses may trigger fibromyalgia or
make symptoms of the condition worse. Fibromyalgia often runs in families. If you
have a family member with this condition, your risk for developing it is higher.
Researchers think certain genetic mutations may play a role in this condition; those
genes haven`t yet been identified. People who experience physical or emotional
trauma may develop fibromyalgia. The condition has been linked with posttraumatic stress
disorder. Like trauma, stress can create long-reaching effects your
body deals with for months and years. Stress has been linked to hormonal
disturbances that could contribute to fibromyalgia. Doctors also don`t fully
understand the factors that cause people to experience the chronic widespread pain
associated with the condition. Some theories suggest it may be that the brain
lowers the pain threshold. Although the causes are unclear, fibromyalgia flare-ups
can be the result of stress, physical trauma, or an unrelated systemic illness like the
flu. It`s believed the brain and nervous system may garble or overreact to normal
pain signals. This incorrect interpretation could be due to an imbalance in brain chemicals.
Text 2: Questions 15-22

15. In paragraph 1, what does the word subjective mean?


A. Something that requires fact
B. Illusory
C. Not easy to understand and explain
D Something that can`t be determined or measured by tests

16. According to paragraph 1, what do some doctors feel about fibromyalgia?


A. It is a serious condition
B. It is a condition that is difficult to examine
C. It is a condition that is not real
D. It is a condition which every patient finds difficult to accept

17. According to paragraph 2, which one of the following statements is true?


A. Doctors of the 21st century know more about this condition.
B. Doctors believe change in lifestyle can be more effective in managing the condition.
C. Doctors are still finding effective ways of treating this condition.
D. Researchers are working on developing new drugs for the treatment of this condition.

18. The most suitable heading for paragraph 3 is ………


A. What are the symptoms of fibromyalgia?
B. Trigger points are significant in the diagnosis of fibromyalgia.
C. The causal agents of fibromyalgia.
D. Common and uncommon symptoms of fibromyalgia.

19. The most suitable heading for paragraph 4 .


A. Role of genetic mutation in fibromyalgia.
B. What are trigger points?
C. What causes fibromyalgia?
D. About trigger points and causal agents of fibromyalgia.
20. According to paragraph 4, which one of the following statements is true?
A. Doctors first check trigger points to identify the problem of fibromyalgia.
B. Fibromyalgia is a genetic disorder
C. Fibromyalgia is hard to detect
D. Doctors in the 21st century know more about how fibromyalgia occurs

21. According to paragraph 4, which one of the following statements is true?


A. Fibromyalgia is common among people who suffer from emotional trauma.
B. Stress is also one of the factors that often leads to fibromyalgia.
C. People who suffered long-term illness can develop fibromyalgia.
D. none of the above

22. The most suitable heading for paragraph 5 is .


A. Genetic Disorder
B. Who are at risk of developing fibromyalgia?
C. how fibromyalgia is caused?
D. common disease-growth indicators

END OF READING TEST, THIS BOOKLET WILL BE COLLECTED


Reading test 22 : Answer Key

Part A - Answer key 1 – 7


1: C
2: D
3: B
4: B
5: D
6: A
7: B

Part A - Answer key 8 – 14


8: Primary impairment of the lymphatic drainage system
9: A lymphoscintigram showing lymph reflux
10: Lymphoedema-distichiasis syndrome
11: Distichiasis
12: 50%
13: De novo
14: Lymphoedema-distichiasis syndrome

Part A - Answer key 15 – 20


15: Recurrence
16: Hydroceles
17: mutation
18: Conservative management of symptomatic distichiasis
19: symptomatic distichiasis
20: parents
Reading test - part B – answer key

1: C Monogenic hereditary disorder gets revealed.


2: A Signs and symptoms.
3: A Swelling on the stomach.
4: A Shows relatively small for GA
5: B Detection rate
6: A Life-threatening complication arising due to infection.

Reading test - part C – answer key

Text 1 - Answer key 7 – 14


7: D Still a mystery to doctors
8: B Burgeon
9: C There is no specific treatment available for this disease.
10: C Aggravate
11: D Affected areas and symptoms.
12: B to come off a surface in small, thin pieces
13: B Common symptoms and risk factors for eczema
14: D How can you treat seborrheic eczema at home?

Text 2 - Answer key 15 – 22


15: D Something that can`t be determined or measured by tests
16: C It is a condition that is not real
17: C Doctors are still finding effective ways of treating this condition.
18: A What are the symptoms of fibromyalgia?
19: D About trigger points and causal agents of fibromyalgia.
20: B Fibromyalgia is a genetic disorder
21: A Fibromyalgia is common among people who suffer from emotional trauma.
22: B Who are at risk of developing fibromyalgia?
READING TEST 23
READING SUB-TEST : PART A
 Look at the four texts, A-D, in the separate Text Booklet.
 For each question, 1-20, look through the texts, A-D, to find the relevant information.
 Write your answers on the spaces provided in this Question Paper.
 Answer all the questions within the 15-minute time limit.
 Your answers should be correctly spelt.

TEXT BOOKLET - ATRIAL FIBRILLATION (AF)

Text A

Atrial fibrillation (AF)

Atrial fibrillation (AF) is the most common cardiac arrhythmia. It affects >33 million
individuals worldwide, and its prevalence is projected to double by 2050. AF is
associated with a 5- and 2-fold increased risk of stroke and mortality, respectively.
Furthermore, AF-related strokes are associated with higher morbidity, mortality, and
health care costs compared with non-cardioembolic strokes.
The mainstay of stroke prevention remains oral anticoagulation (OAC), with vitamin K
antagonists and more recently, direct oral anticoagulants (DOAC), reducing the risk of
ischemic stroke and all-cause mortality in patients with AF. However, more than onethird
of AF patients at high risk for stroke still fail to receive effective stroke prophylaxis
in contemporary practice. Although the introduction of DOAC has overcome some of the
limitations of warfarin therapy, persistent barriers including costs, ongoing bleeding risks
with no reversal agent for most DOACs, noncompliance and high discontinuation rates
may preclude a broader use of DOAC in clinical practice.
Text B
The left atrial appendage (LAA) is a remnant of the embryonic left atrium and is
considered the main reservoir for left atrial thrombi in >90% of patients with non valvular
AF. In recent years, percutaneous LAA closure (LAAC) has rapidly grown worldwide as
an appealing alternative for the prevention of thromboembolism in patients at high risk
for stroke, with a specific focus on patients ineligible for OAC. While no specific
recommendation on LAAC was given in the 2014 American guidelines, the 2016
European guidelines for the management of AF provided a class IIb recommendation for
percutaneous LAAC in patients with AF and contraindications for long-term OAC, based
on data from the PROTECT-AF and PREVAIL trials, the only LAAC randomized trials
to date. Although none of these studies included patients ineligible for OAC, most of the
real-world registries conducted to date have focused on this target population, which
currently represents the majority of LAAC recipients. This review provides an updated
overview of current transcatheter LAAC devices and reviews the main clinical data from
LAAC randomized trials and registries, focusing on procedural and late outcomes, as
well as on future directions.

Text C
WATCHMAN and WATCHMAN FLX
The WATCHMAN device (Boston Scientific, Natick, MA) was the second dedicated
LAAC device and remains the only device studied in randomized clinical trials to date. It
consists of a self-expanding nitinol 10-strut frame, with a 160 µm permeable
polyethylene terephthalate membrane fabric cap facing the left atrium. The open distal
end is fixed by 10 active fixation anchors in 1 row. The tool will be of different
dimensions 21, 24, 27, 30, and 33 mm. The transseptal access sheath will have crucial
specifications, and may show 14F with respect to width or breadth and is available in 3
different preformed curve shapes: anterior curve, double curve (used in >90% of
procedures), and single curve. Three proximal radio-opaque markers correspond to the
approximate level of deployment for 21, 27, and 33 device sizes, respectively.
Text D

The transseptal puncture is performed under fluoroscopic and preferably


transesophageal echocardiography (TEE; bicaval view followed by short-axis view)
guidance in the inferoposterior portion of the fossa ovalis. After transseptal puncture, a
long extra-stiff J tipped 0.035-inch wire is advanced into the left upper pulmonary vein
and the transseptal sheath is exchanged over the wire for the access sheath. After
removal of dilator and guidewire, a 5F to 6F pigtail catheter is advanced through the
access sheath into the left upper pulmonary vein. By using TEE and fluoroscopic
guidance, it will be easy to create some access. In the stage pertaining to it, access sheath
and pigtail are adjusted as required. Appropriate WATCHMAN device sizing is determined
by the maximum LAA ostium diameter (measured from the circumflex artery to 1-2 cm
within the pulmonary vein ridge at 0°, 45°, 90°, and 135°) and depth (from ostium to the
tip of LAA). An oversizing of the device by 10 to 20 percent (corresponding to 2±4 mm) is
generally recommended. A fluoroscopic right (20°±30°) and caudal (20°±30°) projection,
which usually opens the mid-distal portion of the LAA is the preferred one for the
deployment of the WATCHMAN device. After accurate LAA assessment, the delivery
system is advanced into the access sheath until the distal markers of the delivery
catheter and the access sheath align. The device is then deployed with a slow
unsheathing movement.
PART A -QUESTIONS AND ANSWER SHEET
Questions 1-7
For each question, 1-7, decide which text (A, B, C or D) the information comes from.
You may use any letter more than once. Your answers should be correctly spelt.

In which text can you find information about;


1. Used for accessing the left side of the heart. Answer _____________
2. Functioning of the device. Answer ______________
3. A comparison with another disease. Answer ______________
4. A drug that helps with blocking and managing the disease condition. Answer _____________
5. Structure of the device. Answer _____________
6. Made, done, or effected through the skin. Answer _____________
7. Details about LAA imaging and LAAC closure techniques are simply beyond the study performed.
Answer _______________

Questions 8-14
Answer each of the questions, 8-14, with a word or short phrase from one of the texts.
Each answer may include words, numbers or both.
8. What is generally recommended with respect to device sizing? Answer ___________

9. When will the catheter be introduced into the left upper pulmonary vein? Answer ____

10. In how many sizes, the WATCHMAN device is available? Answer ___________

11. What will be the outer diameter of the transseptal access sheath? Answer _______________

12. What is recommended for the effective use of the Watchman Device? Answer ____

13. What is taken into account to measure the size of the device? Answer _______________

14. What is the study more focused on? Answer ______________


Questions 15-20

Complete each of the sentences, 15-20, with a word or short phrase from one of the texts.
Each answer may include words, numbers or both. Your answers should be correctly spelt.

15. The best defending procedure is more connected to the use of the ___________.

16. All three ____________ are needed to be adjusted as per the levels of the device deployed.

17. With_________ access sheath is rightly repositioned into the LAA.

18. With DOAC being introduced, the scope for the __________ enhanced more.

19. The recommended oversizing of the device is in range of___________.

20. In its final stage, the device will be deployed with_________.

END OF PART A, THIS QUESTIONS PAPER WILL BE COLLECTED


READING SUB-TEST : PART B
In this part of the test, there are six short extracts relating to the work of health professionals .
For questions 1-6, choose the answer (A, B or C) which you think fits best according to the text. Write
your answers on the separate Answer Sheet
Questions 1-6

1. As per the given notice, one of the major duties include;


A. Care professional will have to look for ways to stop bleeding.
B. It is important to maintain hemodynamic stability.
C. Support cardiac function and circulation.

Nursing care of the heart transplant patient

Nursing care of the heart transplant patient is similar to the care of any cardiac surgery
patient. Bleeding is a major concern in the early postoperative period. Chest tube
drainage is frequently monitored (initially every 15 minutes), as are the cardiac output,
pulmonary artery pressures, and CVP. Cardiac tamponade can develop, presenting as
either a sudden event or a gradual process. Chest tubes are gently milked (not stripped)
as needed to maintain patency. Atrial dysrhythmias are relatively common following
cardiac transplant. Temporary pacing wires are placed during surgery because surgical
manipulation or postoperative swelling may disrupt the conduction system.
Hypothermia is induced during surgery; postoperatively, the patient is gradually re-
warmed over a 1- to 2-hour period. Cardiac function is impaired in up to 50% of
transplanted hearts during the early postoperative period. Inotropic agents such as low-
dose dopamine, dobutamine, or milrinone may be required to bring more stability.
2. The given notice explains;
A. Why medical professionals shall join the conference?
B. What medical professionals will learn?
C. How this is different from others.

Cardiology Conference
With people from around the world focused on getting some answers concerning
Cardiology, this is your single most obvious opportunity to accomplish the greatest
accumulation of individuals from the mending focuses, Universities, bunch, etc. This
Cardiology conference rather European Cardiology Congress in 2017 will coordinate
appears, disperse information, meet with recurring pattern and potential investigators
and get name affirmation at this 3-day event. Broadly acclaimed speakers, the most
recent frameworks, methodologies, and the most current updates in Cardiology field
are indications of this conference. This World Cardiology Congress will help in
frameworks organization, B2B uniting amidst specialists and academicians.

3. The major result of the changes mention is

A. inability to see / blindness


B. ineffective accommodation and depth perception
C. extra sensitivity to glare and dry eyes

With Age Comes Weakness


The eyelids become lax with the rotation of lid margins as result flow of tear may be
disrupted. The production of tear from the lachrymal glands may decline to make the
eyes dry and burning. There may be an accumulation of fluid in the cornea turning it
hazy. The muscles of iris are weak and the lens capsule becomes stiff making it difficult
to see small objects and depth perception accurately. The lens protein gets denatured and
solidifies. The anterior aspect of the uveal tract degenerates along with degeneration of
important areas in the retina.
4. The given notice talks about;
A. Complications involved in DVT.
B. Symptoms that lead to major problems.
C. Result of the disease conditions.

DVT
The problem occurs when a part of the clot breaks off and travels through the
bloodstream to the lungs, causing a blockage called pulmonary embolism (PE). If the clot
is small, and with appropriate treatment, people can recover from PE. However, there
could be some damage to the lungs. If the clot is large, it can stop blood from reaching
the lungs and is fatal. In addition, nearly one-third of people who have a DVT will have
long-term problems caused by the damage the clot does to the valves in the vein called
post-thrombotic syndrome (PTS). People with PTS have symptoms such as swelling,
pain, discoloration, and in severe cases, scaling or ulcers in the affected part of the
body. In some cases, the symptoms can be so severe that a person becomes disabled.

5. According to the information given, what is correct?

A. Only 40% of the household could have afforded the medications.


B. 33% of households in lower middle-income countries managed to have used the mediations.
C. 25% of households in upper middle-income countries got access to medications.
Medication Survey
In a survey of 598 communities in 18 countries, the availability of aspirin, β-blockers,
ACEIs, and statins varied widely In the low-income countries, only 1 of 30 rural and
only 25 of 32 urban communities had all 4 medications available. The 4 medications
were potentially unaffordable for 60% of households in low-income countries.
Availability rose to 37% (rural) and 62% (urban) in lower middle-income countries and
73% (rural) and 80% (urban) in upper middle-income countries. Effective strategies for
delivering medications are being developed for LMIC where health system infrastructure is
underdeveloped.
6. The given notice talks about;
A. Bidil dosage use.
B. Effectiveness of BiDil in treating heart failure.
C. how it helped the community.

BiDil - Support Blacks


BiDil for in African Americans BiDil, a fixed-dose combination of two vasodilators
(hydralazine and isosorbide), is indicated as an adjunctive treatment in African
Americans with heart failure. It has been shown to reduce symptoms, decrease the
number of hospitalizations, and prolong life in Blacks. The recommended dose is one to
two tablets three times per day, although the dose may be as low as 1/2 tablet three times a
day if side effects are intolerable. The approval of this combination drug has raised the
ethical issue of race-specific FDA approval.
READING SUB-TEST : PART C
In this part of the test, there are two texts about different aspects of healthcare.
For questions 7-22, choose the answer (A, B, C or D) which you think fits best according to the text.
Write your answers on the separate Answer Sheet

Text 1: What is Zika?

The Zika virus disease is caused by the Zika virus, which is spread to people primarily
through the bite of an infected mosquito (aedes aegypti and aedes albopictus). The illness
is usually mild with symptoms lasting up to a week, and many people do not have
symptoms or will have only mild symptoms. However, a Zika virus infection during
pregnancy can cause a serious birth defect called microcephaly and other severe brain defects.

Zika is spread to people primarily through the bite of an infected aedes species mosquito
(aedes aegypti and aedes albopictus). A pregnant woman can pass Zika to her fetus
during pregnancy or around the time of birth. Also, a person with Zika can pass it to his
or her sexual partners. We encourage people, who have traveled to or live in places with
Zika to protect themselves by preventing mosquito bites and sexual transmission of Zika.

Many people infected with Zika will have no symptoms or mild symptoms that last
several days to a week. However, a Zika infection during pregnancy can cause a serious
birth defect called microcephaly and other severe fetal brain defects. Current research
suggests that Guillain-Barre syndrome (GBS), an uncommon sickness of the nervous
system, is strongly associated with Zika; however, only a small proportion of people with a
recent Zika virus infection get GBS. Once someone has been infected with Zika, it`s very
likely they`ll be protected from future infections. There is no evidence that past Zika
infection poses an increased risk of birth defects in future pregnancies.

Going to places where Zika is common is often not recommended. Travellers who go to
places with outbreaks of Zika may or may not get infected with Zika. Moreover, in
pregnant women, if the virus is caught from such places, it can cause microcephaly and
other severe fetal brain defects.
Any pregnant women who have recently travelled to an area with Zika should talk to
their doctor about their travel, even if they don`t feel sick. Pregnant women should see a
doctor if they have any Zika symptoms during their trip or within 2 weeks after
travelling. All pregnant women can protect themselves by using plenty of prevention
measures, which include but are not limited to: avoiding travel to an area with Zika;
preventing mosquito bites; and following recommended precautions against getting Zika.
The most common symptoms of the Zika virus disease are fever, rashes, joint pain, and
red eyes. Other symptoms include muscle pain and headaches. Many people infected with
Zika won`t have symptoms or will have mild symptoms, which can last for several days
to a week.

Currently, there is no evidence that a woman who has recovered from the Zika virus
infection (the virus has cleared her body) will have Zika-related pregnancy complications
in the future. Based on information about similar infections, once a person has been
infected with the Zika virus and has cleared the virus, he or she is likely to be protected
from future Zika infections. If you`re thinking about having a baby in the near future and
you or your partner live in or travelled to an area with Zika, talk with your doctor or
another healthcare provider. Men who have travelled to any areas with Zika or who have
had a Zika infection should wait at least 6 months after travel (or 6 months after
symptoms started if they get sick) before trying to conceive with their partner. Women
should wait at least 8 weeks after travel (or 8 weeks after symptoms started if they get
sick) before trying to get pregnant.
Text 1: Questions 7-14

7. According to paragraph 1, the Zika virus disease is;


A. Common
B. Uncommon
C. Severe disease
D. Without any symptoms

8. What does paragraph 2 indicate?


A. How does Zika spread from person to person
B. Modes of transmission of Zika
C. How do people get infected with Zika?
D. A & C

9. What is mentioned in paragraph 3?


A. How do people get infected with Zika?
B. What health problems can result from getting Zika?
C. How microcephaly occurs.
D. Microcephaly and birth defects.

10. According to paragraph 5, what is recommended for pregnant women?


A. They should not travel to places where the Zika virus is common.
B. They should talk to their partner before going to Zika-infected places.
C. They should inform their doctor before travelling.
D. Not given

11. In paragraph 5, how many methods of prevention from Zika are described?
A. Plenty of prevention measures
B. 2
C. 3
D. 4
12 Can a person who is completely recovered from Zika virus infection, get Zika infection again?
A. Yes
B. No
C. Depends from person to person
D. Not given

13. According to paragraph 6, which one of the following statements is true?


A. A woman`s future pregnancies will be at risk if she is infected with the Zika virus.
B. There is no evidence that infected women will not face any complications in future pregnancies.
C. Any woman who is infected with Zika is advised to wait for six months before getting pregnant.
D. B & C

14. According to paragraph 6, which of the following is not true?


A. A woman should wait for six months before they get pregnant if they have traveled to any places
with the Zika virus.
B. If the woman has been infected with the Zika virus and has cleared the virus, she is likely to be
protected from future Zika infections.
C. In most of the cases, Zika complications may occur in future pregnancies.
D. All above statements are true.
Text 2: Avian Influenza - A Virus Infections in Humans

Although avian influenza A viruses usually do not infect humans, rare cases of human
infection with these viruses have been reported. Infected birds shed the avian influenza
virus in their saliva, mucus and feces. Human infections with bird flu viruses can happen
when enough of the virus gets into a person`s eyes, nose or mouth, or is inhaled. This can
happen when the virus is in the air (in droplets or possibly dust) and a person breathes it
in, or when a person touches something that has the virus on it and then touches their
mouth, eyes or nose. Rare human infections with some avian viruses have occurred most
often after unprotected contact with infected birds or surfaces contaminated with avian
influenza viruses. However, some infections have been identified where direct contact
was not known to have occurred.

The reported signs and symptoms of low pathogenic avian influenza (LPAI) A virus
infections in humans have ranged from conjunctivitis to influenza-like illness (e.g., fever,
cough, sore throat, muscle aches) to lower respiratory disease (pneumonia) requiring
hospitalization. Highly pathogenic avian influenza (HPAI) LPAI H7N9 and HPAI Asian
H5N1 have been responsible for most human illness worldwide to date, including the
most serious illnesses and deaths. A virus infections in people have been associated with
a wide range of illness from conjunctivitis only to severe respiratory illness (e.g.
shortness of breath, difficulty breathing, pneumonia, acute respiratory distress, viral
pneumonia, respiratory failure) with multi-organ disease, sometimes accompanied by
nausea, abdominal pain, diarrhea, vomiting and sometimes neurologic changes (altered
mental status, seizures).

Detection of the avian influenza A virus infection in humans is a formidable process. It


may be difficult to assess the presence of the virus by clinical signs and symptoms alone;
laboratory testing is required. The avian influenza A virus infection is usually diagnosed
by collecting a swab from the nose or throat of the sick person during the first few days
of illness. This specimen is sent to a lab; the laboratory looks for the avian influenza A
virus either by using a molecular test, by trying to grow the virus, or both. (growing avian
influenza A viruses should only be done in laboratories with high levels of protection).
For critically ill patients, collection and testing of lower respiratory tract specimens may
lead to a diagnosis of avian influenza virus infection. For some patients who are no
longer very sick or who have fully recovered, it may be difficult to find the avian
influenza A virus in the specimen, using these methods. It is often termed a problem of
gargantuan proportion. Sometimes it may still be possible to diagnose the avian influenza
A virus infection by looking for evidence of the body`s immune response to the virus
infection by detecting specific antibodies the body has produced in response to the virus.
This is not always an option because it requires two blood specimens (one taken during the
first week of the illness and another taken 3-4 weeks later).

Analyses of available avian influenza viruses circulating worldwide suggest that most
viruses are susceptible to oseltamivir, peramivir, and zanamivir. However, some evidence
of antiviral resistance has been reported in HPAI Asian H5N1 viruses and influenza A
H7N9 viruses isolated from some human cases. Monitoring for antiviral resistance among
avian influenza A viruses is crucial and ongoing. This data directly informs WHO
antiviral treatment recommendations.
The best way to prevent an infection of the avian influenza A virus is to avoid sources of
exposure. Most human infections with avian influenza A viruses have occurred following
direct or close contact with infected poultry. People who have had contact with infected
birds may be given expert-formulated influenza antiviral drugs preventatively. While
antiviral drugs are most often used to treat flu, they can also be used to prevent infection
in someone who has been exposed to influenza viruses. When used to prevent seasonal
influenza, antiviral drugs are 70% to 90% effective. A seasonal influenza vaccination will
not prevent infection with avian influenza A viruses, but can play hardball in effectively
reducing the risk of co-infection with human and avian influenza A viruses. It`s also
possible to make a vaccine intellectually that can protect people against avian influenza
viruses. For example, the United States government always have access to the stockpiled
vaccine to protect against avian influenza A H5N1 vaccine. The stockpiled vaccine could
be used if a similar H5N1 virus were to begin transmitting easily from person to person.
Creating a candidate vaccine virus is the first step in producing a vaccine.
Text 2: Questions 15-22

15. According to paragraph 1, the virus enters the human body through.
A. The nose
B. The eyes
C. The mouth
D. Contact with an infected bird

16. According to paragraph 1, human infection with avian influenza A virus is;
A. Uncommon
B. Common
C. Rare
D. Frequent in infected places

17. According to paragraph 2, people may suffer from flu if they have been infected by
A. LPAI
B. HPAI
C. H5N1
D. LPAI H7N9

18. According to paragraph 3, what is not true?


A. A molecular test is performed to see the presence of the virus in the collected specimen.
B. Lab tests are aimed at growing the virus for the right detection purpose.
C. A specimen for testing is usually collected right after the person gets the illness by infection.
D. People who have recovered from the illness should also be diagnosed for further
confirmation.

19. According to paragraph 3, the word “gargantuan― may mean;


A. Small-scale
B. Trivial
C. Very large
D. Pint-sized
20. Which central idea is presented in paragraph 4?
A. Treatment of the avian influenza disease.
B. Effective medications for the viral disease.
C. Research on effective medicine for the avian virus disease.
D. None of the above.

21. According to paragraph 5, which word may give the meaning of experience?
A. Exposure
B. Expert
C. Intellectualism
D. None of the above

22. The phrase to play hardball in paragraph 5 may mean;


A. To be able to achieve the results.
B. To do what it takes to complete the task.
C. To work effectively.
D. To be firm and determined in order to get what you want.

END OF READING TEST, THIS BOOKLET WILL BE COLLECTED


Reading test 23 : Answer Key

Part A - Answer key 1 – 7


1: D
2: D
3: A
4: A
5: C
6: B
7: C

Part A - Answer key 8 – 14


8: Oversizing by 10 to 20 percent
9: After removal of dilator and guidewire
10: Five
11: 14 F
12: Fluoroscopic right and caudal projection
13: Maximum diameter / LAA ostium diameter
14: LAAC recipients

Part A - Answer key 15 – 20


15: oral anticoagulation (OAC)
16: radio-opaque markers
17: fluoroscopic guidance
18: warfarin therapy
19: 10% to 20%
20: unsheathing movement
Reading test - part B – answer key
1. C Support cardiac function and circulation.
2. A Why medical professionals shall join the conference?
3. C extra sensitivity to glare and dry eyes
4. A Complications involved in DVT.
5. A Only 40% of the household could have afforded the medications.
6. B Effectiveness of BiDil in treating heart failur

Reading test - part C – answer key

Text 1 - Answer key 7 – 14


7: C Severe disease
8: C How do people get infected with Zika?
9: B What health problems can result from getting Zika?
10: D Not given
11: C 3
12: B. No
13: A A woman`s future pregnancies will be at risk if she is infected with the
Zika virus.
14: C In most of the cases, Zika complications may occur in future pregnancies.

Text 2 - Answer key 15 – 22


15: B The eyes
16: A Uncommon
17: B HPAI
18: D People who have recovered from the illness should also be diagnosed
for further confirmation.
19: C Very large
20: A Treatment of the avian influenza disease.
21: A Exposure
22: D To be firm and determined in order to get what you want.
READING TEST 24
READING SUB-TEST : PART A
 Look at the four texts, A-D, in the separate Text Booklet.
 For each question, 1-20, look through the texts, A-D, to find the relevant information.
 Write your answers on the spaces provided in this Question Paper.
 Answer all the questions within the 15-minute time limit.
 Your answers should be correctly spelt.

TEXT BOOKLET - GERIATRIC PHARMACOLOGY

Text A:
Changes in gastrointestinal function
The process of aging brings about changes in gastrointestinal function such as
increase in gastric pH, deferment in process of expulsion of gastric, decreased
motility, and decreased intestinal blood flow. The intake of substances that are
actively transported from the intestinal lumen including some sugars, minerals and
vitamins may therefore be decreased in elderly patients. Apart from the
pathological or surgical alterations in gastrointestinal function such as gastrectomy,
pyloric stenosis, pancreatitis, regional enteritis and concurrent administration of
other drugs like cholestyramine and antacids may cause changes. Cholestyramine
binds and decreases the effectiveness of many drugs including thiazides,
anticoagulants, thyroxine, aspirin, PCM, and penicillin, while antacids decrease the
efficiency of the process of taking in of drugs such as chlorpromazine, tetracycline,
isoniazid.
Plasma protein concentrations may also be altered in elderly patients. Plasma
albumin concentrations are causing less increase in free concentration of acidic
drugs such as naproxen, phenytoin and warfarin. In contrast, the concentration of
α1-acid glycoprotein may be increased in the presence of chronic diseases that
frequently occur in the elderly population, potentially increasing the binding of
drugs such as antidepressants, antipsychotic drugs and β-blockers, which are
mainly bound to this protein.
Text B:

Aging Factor

Body composition, plasma protein binding, and organ blood flow help in
determining how effectively the drug is getting into every nook and corner. The
total body water and lean body mass decreases, whereas, the body fat as a
percentage of body weight increases with aging. The increased body fat is
associated with the increase in volume of distribution of fat-soluble drugs such as
the benzodiazepines, which leads to a more prolonged drug effect. Thus, it was
demonstrated that the elimination half-life of diazepam was prolonged with age
despite the fact that systemic clearance was unaltered. Change in organ blood flow
with aging may also affect the rate of its efficient movement. In most of the cases,
peripheral vascular resistance get enhanced more and more. The same goes with the
enhancement of the heart rate or cardiac output.

Text C:

Hepatic Blood Flow


Hepatic blood flow and liver mass change in proportion to body weight decrease
with aging. The rate of metabolism of many drugs by the cytochrome P450
enzyme system is decreased by 20- 40% with aging. Examples include
theophylline, propranolol, nortriptyline, alfentanil, fentanyl, alprazolam, triazolam,
diltiazem, verapamil, and levodopa. Many benzodiazepines are metabolized by
microsomal enzyme to active metabolites, which are also eliminated by hepatic
metabolism. Non-microsomal enzyme pathways may be less affected by age.
Example: Ethanol metabolism by alcohol dehydrogenase and isoniazid elimination
by acetylation are unchanged in elderly patients. Concurrent drug administration,
illness, genetics and environmental factors including smoking may have more
significant effects on hepatic drug metabolism than age.
Text D :

Toxicity In Drugs

Renal blood flow, glomerular filtration rate and tubular function all decline with
aging. In addition to physiological decline in renal function, the elderly patient is
particularly liable to renal impairment due to dehydration, congestive heart failure,
hypotension and urinary retention, or to intrinsic renal involvement, e.g., diabetic
nephropathy or pyelonephritis. As lean body mass decrease with aging, the serum
creatinine level becomes a poor indicator of (and tends to overestimate) the
creatinine clearance in older adults.
The Cockroft-Gault formula20 should be used to estimate creatinine clearance in
older adults: Creatinine clearance = {140 - age) x weight (kg) / 72 X serum creatinine
in mg/dl (For women multiplied by 0.85)
Drugs with significant toxicity that have diminished renal excretion with age
include allopurinol, aminoglycosides, amantadine, lithium, digoxin, procainamide,
chlorpropamide and cimetidine. These agents have reduced clearance, prolonged
half-lives and increased steady-state concentrations if dosages are not adjusted for
renal function.
PART A -QUESTIONS AND ANSWER SHEET

Questions 1-7
For each question, 1-7, decide which text (A, B, C or D) the information comes from.
You may use any letter more than once.

In which text can you find information about;


1. Substance which is known to decrease absorption. Answer __________
2. Various factors are known to create an effect on how drug distribution is weakened. Answer _
3. Belongs to the class of medicines called digitalis glycosides Answer __________
4 .With increase in age, various other health problems increases. Answer ____________
5. Bioavailability and absorption. Answer____________
6. Drug distribution. Answer ___________
7. Heart will pump less amount of blood through the circulatory system. Answer____________

Questions 8-14
Answer each of the questions, 8-14, with a word or short phrase from one of the texts.
Each answer may include words, numbers or both. Your answers should be correctly spelt.

9. What causes delay in gastric emptying? Answer ____________


10.9What can reduce effectiveness of blood thinners? Answer ____________
11.One of the factors that lead to decrease in body fat is? Asnwer ____________
12.How age can have its effect on cardiac output and peripheral vascular resistance? Answer__
13.What is often stamp out by Hepatic metabolism? Answer___________
14.What can have major impact on hepatic drug metabolism? Answer___________
15.What cimetidine is known to be? Answer_____________
Questions 15-20

Complete each of the sentences, 15-20, with a word or short phrase from one of the
texts. Each answer may include words, numbers or both. Your answers should be correctly spelt.

15. In most of the patients,__________ will often get transformed.

16. In most of the adults, the____________ will help signal the pathway for creatinine.

17. With steady increase in__________ , there can be increase in volume of distribution of fat-
soluble drugs.

18.__________ pathways may not show any kind of change though age increases.

19 .Many of these substances, when they are not altered as needed, are recorded to
be effective and known to enhance __________

20__________ is known to be very effective in curtailing down the absorption of drug.

END OF PART A, THIS QUESTIONS PAPER WILL BE COLLECTED


READING SUB-TEST : PART B
In this part of the test, there are six short extracts relating to the work of health professionals .
For questions 1-6, choose the answer (A, B or C) which you think fits best according to the text. Write
your answers on the separate Answer Sheet
Questions 1-6

1. The risk to the unborn baby may occur;


A. During the first half of the pregnancy.
B. When baby get primary infection through mother.
C. When infected with virus during first pregnancy.

CMV Infection

About one out of every 150 babies are born with a congenital CMV infection.
However, only about one in five babies with a congenital CMV infection will be
sick from the virus or will have long-term health problems.
If a woman is newly infected with CMV while pregnant, there is a risk that her
unborn baby will also become infected (congenital CMV). Infected babies may,
but not always, be born with a disability.
Infection during one pregnancy does not increase the risk for subsequent
pregnancies. However, if primary infection occurs, consideration should be given
to wait for at least 12 months for next pregnancy.
Studies conducted in Australia have shown that out of 1,000 live births, about 6
infants will have congenital CMV infection and 1-2 of those 6 infants (about 1 in
1000 infants overall) will have permanent disabilities of varying degree. These can
include hearing loss, vision loss, small head size, cerebral palsy, developmental
delay or intellectual disability, and in rare cases, death.
Sometimes, the virus may be reactivate while a woman is pregnant but reactivation
does not usually cause problems to the woman or to the fetus.
2. What is more related to defects?
A. SARS Co-V
B. MERS Co-V
C. Zika

Impact of re-emerging infectious diseases


New or re-emerging infectious diseases can have a huge impact on morbidity,
mortality, and costs to the affected region, and pose a significant challenge to
healthcare and public health systems. Multiple new diseases have been identified
during the past twenty years, including severe acute respiratory syndrome
coronavirus (SARS Co-V), Middle East respiratory syndrome coronavirus (MERS
Co-V), and novel strains of avian and swine influenza. In addition, multiple
existing infectious diseases have re-emerged or resurged, causing large outbreaks.
Two recent examples include Zika and Ebola. The Zika virus has caused disease in
more than 28 countries and is associated with severe natal deformity, such as
microcephaly. The 2014 Ebola virus outbreak infected almost 30,000 individuals
and resulted in more than 11,000 deaths worldwide.

3. The following manual talks about;


A. Digital blood pressure monitoring device.
B. Traditional blood pressure monitoring.
C. Accurate Blood Pressure Examination.

OMRON HEM-907XL Intellisense


Developed for the specific use in the clinical office setting and other health care
environments, this device determines blood pressure by oscillometric measurement
and displays systolic blood pressure, diastolic blood pressure, and pulse rate using
an LCD digital monitor. It has the ability to automatically measure and store up to
three sequential readings, and has a “hide” feature that hides measurements during
acquisition. The pressure measurement range for this device is 0 to 280 mmHg.
The OMRON is calibrated to the mercury manometer for routine quality assurance procedures.
4. What is known to have higher acoustical quality?
A. Littmann Cardioscope III
B. Littmann Classic II
C. A and B

Littmann Cardiology III stethoscope

The stethoscopes used for listening to Korotkoff sounds are Littmann Cardioscope III
for adults and Littmann Classic II pediatric for children.
They have a bell and diaphragm chest piece, and an acoustical rating by the manufacturer of
9 on a scale of 1-10, with a rating of 10 having the best acoustical attributes.
The construction uses a single-lumen rubber tubing connection between the ear tubes and
the chest piece. The ear tubes can be adjusted to fit the particular user at an anatomically
correct angle, and the plastic ear covers come in different sizes allowing the user to match
the best ear canal size to achieve an acoustically sealed ear fit. All parts of the stethoscope
can be cleaned for use between SPs. The bell of the stethoscope is used to auscultate the
Korotkoff sounds for blood pressure measurements.
5. Which word may indicate a plant a sapling?
A. Zygote
B. Blastocyst
C. Poppy

Implantation

Implantation takes place, when ovulation and fertilization occur. Implantation occurs in early
stage of pregnancy when the fertilized egg (zygote) treks down the fallopian tube to the
uterus and ascribes to the epithelium or uterine lining. It takes about 8 to 10 days for the
fertilized egg to reaches to the uterus. During this time, it develops into a blastocyst through
different stages of transformation instigation as a single cell dividing into 150 cells with an
outer layer the trophoblastic, a fluid filled cavity the blastocoel, and a cluster of cells on the
interior the inner cell mass. The tiny ball of cells is more or less like poppy generator. It
attaches to the epithelium during 4th week of gestation. Once it is firmly adheres, this’s
called as an embryo. The embryo then again allocates into two parts, which will become the
placenta and the fetus. An ultrasound done during the 5 to 6 weeks of gestation period that
may show the amniotic sac and yolk sac, which are forming during this time. The amniotic sac
is where baby will develop. The yolk sac will later be incorporated in a baby’s digestive tract.
This ultrasound approves that implantation has taken place.
6. The given notice gives information about;
A. Women who are now more aware of health conditions.
B. Industry insights.
C. The global gynecology devices market size.

The market size was valued at USD 10,984.1 million in 2014. Introduction of
minimally invasive procedures such as laparoscopy and high definition imaging
devices such as 3D endoscope is primarily boosting market growth. In addition,
rising prevalence of diseases, such as uterine fibrosis and sexually transmitted
diseases (STDs), associated with female reproductive organs are anticipated to
support market growth during the forecast period.
According to the United Nations, the global female population accounted for more
than 3.64 billion in 2015. Every woman visits a gynecologist at least once in her
lifetime either for pregnancy or other complications related to menstrual cycle. The
growing number of patients is likely to drive market growth during the forecast period.
Moreover, healthcare agencies are now promoting routine-checkups for early
cancer detection and other gynecological conditions. For example, The American
Cancer Society recommends annual breast cancer screening with mammography
for women aged between 40 to 44 years. Increase in routine check-ups has helped
these devices gain usage rates.
READING SUB-TEST : PART C
In this part of the test, there are two texts about different aspects of healthcare.
For questions 7-22, choose the answer (A, B, C or D) which you think fits best according to the text.
Write your answers on the separate Answer Sheet

Text1: All About Fetal Alcohol Spectrum Disorders

Fetal Alcohol Spectrum Disorders (FASDs) are an assortment of different conditions that can
occur in a person whose mother drank alcohol during pregnancy. These effects can include
physical problems and problems with behavior and learning. Often, a person with an FASD
has a mix of these problems.
FASDs are caused by a woman drinking alcohol during pregnancy when alcohol in
the mother’s blood passes to the baby through the umbilical cord. When a woman
drinks alcohol, so does her baby. There is no known safe amount of alcohol during
pregnancy or when trying to get pregnant.

To curtail down the risks of FASDs, a woman should not drink alcohol while she is
pregnant, or when she might get pregnant. This is because a woman could get
pregnant and be asymptomatic for up to 4 to 6 weeks. In the United States, nearly
half of pregnancies are unplanned. If a woman is drinking alcohol during
pregnancy, it is never too late to stop drinking. Because brain growth takes place
throughout the pregnancy, the sooner a woman stops drinking, the safer it will be
for her and her baby.

FASDs can affect every person in different ways, and can range from mild to
severe. It may not be difficult to assess why certain problems occur, however, still
they have their own appearance time and pattern. A person with an FASD might
have: abnormal facial features, such as a smooth ridge between the nose and upper
lip (this ridge is called the philtrum); small head size; shorter-than-average height;
low body weight; poor coordination etc.
Different terms are used to describe FASDs, depending on the type of symptoms.
(i) Fetal Alcohol Syndrome (FAS): FAS represents the most involved end of the
FASD spectrum. Fetal death is the most extreme outcome from drinking alcohol
during pregnancy. People with FAS might have abnormal facial features, growth
problems, and central nervous system (CNS) problems. People with FAS can have
problems with learning, memory, attention span, communication, vision, or
hearing. They might have a mix of these problems. People with FAS often have a
hard time in school and trouble getting along with others. (ii) Alcohol-Related
Neurodevelopmental Disorder (ARND): People with ARND might have
intellectual disabilities and problems with behavior and learning. They might do
poorly in school and have difficulties with math, memory, attention, judgment, and
slow, lethargic behaviour (iii) Alcohol-Related Birth Defects (ARBD): People with
ARBD might have problems with the heart, kidneys, or bones, or with hearing;
they might have a combination of these.

Diagnosing FAS can be hard because there is no medical test, like a blood test, for
it. And other disorders, such as ADHD (attention-deficit/hyperactivity disorder)
and Williams syndrome, have some symptoms like FAS. To diagnose FAS,
doctors look for: heteroclite facial features (e.g., smooth ridge between nose and
upper lip); lower-than-average height, weight, or both; central nervous system
problems (e.g., small head size, problems with attention and hyperactivity, poor
coordination); prenatal alcohol exposure; although confirmation is not required to
make a diagnosis etc.
FASDs last a lifetime. There is no cure for FASDs, but research shows that early
intervention treatment services can improve a child’s development. There are many
types of treatment options, including medication to help with some symptoms,
behavior and education therapy, parent training, and other alternative approaches. No
single treatment is effective for every child. Good treatment plans will include close
monitoring, follow-ups, and changes as needed along the way.
Text 1: Questions 7-14

7. As per the information given in paragraph 1, FASDs;


A. Occur due to alcohol consumption.
B. Are known to cause behavioral and learning disabilities.
C. Are a collection of diseases, which occur only in women.
D. Are a collection of complex, proof-less medical conditions.

8. Paragraph 2 talks more about;


A. How to prevent FASDs
B. Why FSADs women should not get pregnant?
C. What FSADs women should do when pregnant?
D. How to protect the baby from FASDs during pregnancy?

9. The most appropriate heading for paragraph 3 is _____.


A. Signs and symptoms
B. How FASDs affect babies
C. Common features of FASDs
D. None of the above

10. According to paragraph 4, what is not true about FAS?


A. People affected with the FAS show uneven growth.
B. FAS can lead to development of extra facial features.
C. People with FAS show poor memory.
D. FAS children can have health problems but they may mix well with other children.

11. According to paragraph 4, people with _____..show low agility levels.


A. ARND
B. FAS
C. ARBD
D. ARND and FAS
12. According to paragraph 5, at the time of diagnosis, most doctors look for;
A. Effects on facial features
B. Height and body weight problems
C. Problems with brain functioning
D. All of the above

13. According to paragraph 5, when is a diagnosis not required?


A. When it is known that the patients mother is an alcoholic.
B. When the features such as abnormal facial features, low body weight and lower height
become obvious.
C. When the patient shows all abnormal signs and symptoms of the FAS
D. a and c

14. According to paragraph 6, treatment for FASDs is;


A. Specific
B. Common for all conditions
C. Dependent on types of conditions
D. Depends on age
Text 2: Valley Fever

Valley fever, also called coccidioidomycosis, is an infection caused by the fungus,


Coccidioides. The fungus is known to live in the soil in the south-western United
States and parts of Mexico and Central and South America. The fungus was also
recently found in south-central Washington. People can get Valley fever by
breathing in the microscopic fungal spores from the air, although most people who
breathe in the spores don’t get sick. Usually, people who get sick with Valley fever
may get better on their own within weeks to months, but some people need
antifungal medication. Certain groups of people are at a higher risk of becoming
severely ill. It’s difficult to prevent exposure to Coccidioides in areas where it’s
common in the environment, but people who are at a higher risk of severe Valley
fever should try to avoid breathing in large amounts of dust if they’re in such localities.

Anyone who lives in or travels to the south-western United States (Arizona,


California, Nevada, New Mexico, Texas, or Utah), or parts of Mexico or Central or
South America can get Valley fever. Valley fever can affect people of any age, but
it’s most common in adults aged 60 years and over. Certain groups of people may
be at a higher risk of developing the severe forms of Valley fever, such as: people
with weakened immune systems, for example, people with HIV/AIDS; people who
have had an organ transplant; people who are taking medications such as
corticosteroids or TNF-inhibitors; pregnant women; and people who have diabetes.
The fungus that causes Valley fever, Coccidioides, doesn’t have that potential to
cross barriers; the transmission is often formidable, a mighty task that could lead to
stark failure. However, in extremely rare instances, a wound infection with
Coccidioides can spread Valley fever to someone else or the infection can be
spread through an organ transplant with an infected organ.
The most common way for someone to get Valley fever is by inhaling
Coccidioides spores that are in the air. In extremely rare cases, people can get
infected from an organ transplant if the organ donor had Valley fever, inhaling
spores from a wound infected with Coccidioides, contact with objects (such as
rocks or shoes) that have been contaminated with Coccidioides etc.
Scientists continue to study how weather and climate patterns efficaciously affect
the habitat of the fungus that causes Valley fever. Coccidioides is thought to grow
expeditiously in soil after heavy rainfall and then disperse into the air most
vigorously during hot, dry conditions. For example, hot and dry weather conditions
have been shown to parlously correlate with an increase in the number of Valley
fever cases in Arizona and in California (but to a lesser extent). The ways in which
climate change may be affecting the number of Valley fever infections, as well as
the geographic range of Coccidioides, isn’t known yet, but is a subject for further
research.

Healthcare providers rely on your medical and travel history, symptoms, physical
examinations, and laboratory tests to diagnose Valley fever. The most common
way that healthcare providers test for Valley fever is by taking a blood sample and
sending it to a laboratory to look for Coccidioides antibodies or antigens.
Healthcare providers may do imaging tests such as chest x-rays or CT scans of
your lungs to look for Valley fever pneumonia. They may also perform a tissue
biopsy, in which a small sample of tissue is taken from the body and examined
under a microscope.
Text 2: Questions 15-22

15/ According to paragraph 1, the fungus mentioned is a native of _____ .


A. US
B. Washington
C. Mexico
D. A and C

16. According to paragraph 1, treatment for valley fever is;


A. Required
B. Not required
C. Required in some specific cases
D. Not given

17. The most appropriate heading for paragraph 2 is;


A. When Valley fever may affect someone?
B. Who gets Valley fever?
C. Who can show symptoms of Valley fever?
D. Conditions that are common with Valley fever.

18. The most suitable heading for paragraph 3 is;


A. Is it contagious?
B. How can Valley fever transfer?
C. Valley fever is half contagious
D. None of the above

19. The most suitable heading for paragraph 4 is;


A. Uncommon sources of Valley fever
B. Common sources of Valley fever
C. How people may get affected with Valley fever
D. A and C
20. The most appropriate heading for paragraph 5 is;
A. Valley fever agent and its habitat.
B. Valley fever and weather.
C. How temperature affects Valley fever patients?
D. Climate and Valley fever.

21. Which word in paragraph 5 may mean quickly?


A. Efficaciously
B. Vigorously
C. Expeditiously
D. B and C

22. The most suitable heading for paragraph 6 is;


A. How valley fever is identified?
B. Common ways of identifying Valley fever.
C. Ways of identifying and treating Valley fever.
D. Three common tests for Valley fever.

END OF READING TEST, THIS BOOKLET WILL BE COLLECTED


Reading test 24: Answer Key

Part A - Answer key 1 – 7


1 :A
2:C
3:D
4:D
5:A
6:B
7:B

Part A - Answer key 8 – 14


8: Process of Aging
9 : Cholestyramine
10: Aging
11: Decreases, increases
12: benzodiazepines
13: Concurrent drug administration
14: Drug with toxicity

Part A - Answer key 15 – 20


Complete each of the sentences, 15-20, with a word or short phrase from one of
the texts. Each answer may include words, numbers or both.
15: Plasma protein concentrations
16: Serum creatinine level
17: body fat
18: Non-microsomal enzyme
19: steady-state concentrations
20: Antacid
PART B

1: When infected with virus during first pregnancy.


2: MERS Co-V
3: Digital blood pressure monitoring device.
4: A and B
5: Poppy
6: The global gynecology devices market size.

PART C
Text 1: Questions 7-14
7: Are A Collection Of Diseases, Which Occur Only In Women.
9: Common Features Of Fasds
10: People Affected With The Fas Show Uneven Growth.
11: Arbd
12: Problems With Brain Functioning
13: When The Features Such As Abnormal Facial Features, Low Body
Weight And Lower Height Become Obvious.
14: Specific

Text 2: Questions 15-22


15: A and C
16: Required in some specific cases
17: Who gets Valley fever?
18: Is it contagious?
19: Uncommon sources of Valley fever
20: Valley fever and weather.
21: Expeditiously
22: How valley fever is identified?
READING TEST 25
READING SUB-TEST : PART A
 Look at the four texts, A-D, in the separate Text Booklet.
 For each question, 1-20, look through the texts, A-D, to find the relevant information.
 Write your answers on the spaces provided in this Question Paper.
 Answer all the questions within the 15-minute time limit.
 Your answers should be correctly spelt.

TEXT BOOKLET- SPIGELIAN HERNIA - A CASE STUDY

Text A

A Spigelian hernia (SH) is an uncommon hernia comprising 0.12% to 2% of all


ventral wall hernias. Adriaan van den Spiegel, a Belgian anatomist, was the first to
describe a region known as the Spigelian fascia, which is the aponeurotic layer
located between the transverse abdominal muscle medially, and the semilunar line
laterally. Consequently, hernias in this region are referred to as SH. These
uncommon hernias are difficult to diagnose, and the cases described generally only
come from large, specialized centers. Usually the symptoms are nonspecific and
therefore a diagnosis based only on the clinical examination is difficult. In 50% of
the cases, confirmation of the diagnosis is only possible during surgery. This
disease has a higher frequency among women. SH occurs primarily in the 6th-7th
decade of life, and most often occur on the left side. Bilateral hernias appear very
rarely. The average BMI is around 29-31.50 kg/m². The most commonly
accompanying comorbidities include diabetes, peripheral vascular disease,
cerebrovascular disease, COPD, coronary heart disease, diabetes, chronic kidney
disease, and asthma.
Text B

The semilunar line extends from the cartilage of the VIII-IX rib to the pubic
tubercle and is an area of transition of the transverse abdominal muscle fibres into
the aponeurosis, whose part located between the semilunar line and the lateral
border of the rectus abdominis muscle is called the Spigelian fascia. In the widest
part of the fascia, where the inferior epigastric vessels cross, there is a region
called the Spigelian zone. The weakest part of the Spigelian fascia is an area where
the semilunar line intersects with the arcuate line (Douglas line). This area is
weakened by the lack of the posterior sheath of the rectus abdominis muscle. Most
hernias arise within the triangle formed by the arcuate line superiorly, the
semilunar line laterally and the inferior epigastric vessels inferiorly. Hernias
located inferomedially from the vascular bundle are called lower hernias, whereas
upper hernias, located superiorly to the navel are rare. 0.7% of the cases have more
than one gate. It is also known as the “lateral abdominal hernia” and the “lateral
ventral abdominal hernia”. We can also classify a hernia as an intramural hernia -
located under the aponeurosis of the external oblique muscle (98% of all cases) or
a subcutaneous hernia - where the hernia sac passes through the aponeurosis.
Text C

The etiology is not exactly known. From an embryological point of view the SH
may derive from an inherent weakness in the fusion area of the aponeuroses of the
abdominal muscles; stemming from their independent and separate development
from invading and fusing myotomes within the mesenchyme. The vascular concept
assumes that a hernia is formed in places where the vessels and nerves penetrate
through the aponeurosis. There is no clear evidence to support the validity of any
of the aforementioned concept. It is currently accepted that a multifactorial theory
is the most likely. The risk factors include obesity, collagen disorders, chronic
obstructive pulmonary disease (COPD) and previous operations. In the presented
case, it can be assumed that a hernia was caused by previous surgeries, obesity, and
chronic constipation.
Text D

SH may be idiopathic, acquired, congenital, traumatic or iatrogenic a direct


consequence of the introduction of a laparoscopic port. It can co-occur with other
abdominal hernias, which is suggestive of connective tissue disease. Hernia sacs,
most frequently are formed by the peritoneum, however, the aponeurosis of the
transverse abdominal muscle may also contribute to the structures constituting the
hernia sac. Hernia sacs usually contain a fragment of fatty tissue or a segment of
the omentum. It is less of a common occurrence for the hernia contents to enclose
tissues such as the appendix, small intestine, colon, stomach, gallbladder, blind gut,
Meckel’s diverticulum or ovarium. Due to its location, the appendix is usually a
component of abdominal hernias located on the right side of the body as was the
case in patient. There are approximately 10 cases of the appendix incarcerated in
SH described to date. All of these reported cases of SH contained incarcerated
appendix were diagnosed as periappendicular mass palpated physically, as
inflamed incarcerated appendix reveal in CT scan or in explorative laparotomy. As
a result of the non-specific clinical picture of the condition diagnosis is difficult.
PART A -QUESTIONS AND ANSWER SHEET

Questions 1-7
For each question, 1-7, decide which text (A, B, C or D) the information comes from.
You may use any letter more than once.

In which text can you find information about;

1. In some cases, hernias may appear through different routes. Answer_________


2. Relating to or denoting any disease or condition which arises spontaneously or for which the cause
is unknown. Answer___________
3. Common reason why hernia appears. Answer___________
4 .Talking about theory. Answer____________
5. Presence of one or more additional diseases or disorders co-occurring with a primary disease or
disorder. Answer ___________
6. Cases related to surgery previously performed. Answer__________
7. Unknown reasons which lead to the development of the disease. Answer____________

Questions 8-14
Answer each of the questions, 8-14, with a word or short phrase from one of the texts.
Each answer may include words, numbers or both. Your answers should be correctly spelt.

8. What is the word used to describe systemic autoimmune diseases? Answer____________


9 .How would you define lower hernias? Answer ____________
10 .What is the terms which may mean relating to illness caused by medical examination or
treatment? Answer____________
11. What is the word which means confined or imprison? Answer___________
12 .What defines a wide region superiorly from the interspinous line? Answer____________
13. What is referred to as a sheet of pearly white fibrous tissue which takes the place of a tendon in
sheet-like muscles having a wide area of attachment? Answer_____________
14. What is the development pattern of SH? Answer____________
Questions 15-20
Complete each of the sentences, 15-20, with a word or short phrase from one of the texts. Each
answer may include words, numbers or both. Your answers should be correctly spelt.

15.___________ is a fold of peritoneum connecting the stomach with other abdominal organs

16. ___________ is a condition in which sac will travel through the fibrous tissue

17. The ____________ is referred to as semilunar line.

18. ______________ in most of the cases lays the foundation for the Hernia sacs,

19 .Often, it is difficult to diagnose because the symptoms can be more___________ in nature.

20. The appendix is usually a component of ___________.

END OF PART A, THIS QUESTIONS PAPER WILL BE COLLECTED


READING SUB-TEST : PART B
In this part of the test, there are six short extracts relating to the work of health professionals .
For questions 1-6, choose the answer (A, B or C) which you think fits best according to the text.
Write your answers on the separate Answer Sheet
Questions 1-6

1. The notice gives more information about;


A. Pain in general and its outcome.
B. Effectiveness in BFA performance.
C. Implications for clinical effectiveness.

Clinical Overview

Pain is associated with decreased quality of life and is one of the most common
presenting symptoms in PC. However, it remains poorly understood and
traditional, pharmacologically-based treatment approaches are fraught with
complications. Thus, offering patients effective non-pharmacological options for
pain relief should be a priority. BFA has the potential to help patients but, given
the high prevalence of patients with chronic non-cancer pain, competing for
medical comorbidities and the need to maintain access to health care providers,
implementation of novel, non-traditional venues should be considered. We have
described a well-received and efficient manner in which to offer an integrative
modality within the context of a busy PC practice. Potential barriers that we were
able to overcome included finding adequate space for a group of this size,
obtaining training in BFA and institutional credentialing, and developing an
efficient manner in which to perform BFA. Ultimately, our clinical results are
limited by our self-selected patient population and the lack of a comparison group
and longer term follow-up to ascertain the durability of response. However, our
preliminary results suggest that BFA performed in a group setting at a VA
institution appears effective for short-term pain reduction.
2. The given notice talks more about
A. How hypoxia interferes with T lymphocyte effector function
B. formation of lymphoid organ
C. effects of hypoxia

Hypoxia Effects

When evaluating the effects of hypoxia on T-cell function, it is critical to separate the
intrinsic effects of alterations to hypoxia-sensing pathways and associated downstream
signaling from the extrinsic effects of immersion in a hypoxia microenvironment in vivo. T
cells activate HIF-1α independently of external hypoxia as a consequence of T-cell
receptor activation via a PI3K/mTOR dependent pathway. T-cell receptor-activated T cells
also increased HIF-1α mRNA synthesis by mechanisms involving protein kinase C and Ca
(2+)/calcineurin. Independently of T-cell receptor stimulation, HIF-1α mRNA is
augmented in T cells in the presence of transforming growth factor-β (TGF-β) and/or
interleukin-6 (IL-6) by a mechanism involving STAT3. In physiologic conditions, CD8+ T
cells from lymphoid organs (spleen, lymph nodes) were found to bind pimonidazole,
indicating a hypoxic state within these organs ranging from 1 to 2.5% pO2. While T cells
were functional under these conditions, overall levels of activation were higher in more
oxygenated areas.
3. What the given notice indicate:?
A. Role of key players in the immunologic antitumor response
B. Role of key players is not completely known
C. Role of metabolism

Cancer cells

Cancer cells reprogram their metabolism to rely primarily on glycolysis even in the
presence of oxygen. This shift to aerobic glycolysis with lactate production, which
is referred to as the Warburg effect, is now emerging as a hallmark of cancer. The
altered metabolism allows cancer cells to convert efficiently glucose into macromolecules
needed to sustain higher proliferative rates and consequently results in lactate
accumulation in the tumor microenvironment. Currently, lactate is regarded as a tumor-
promoting factor. Lactate accumulation in tumors has been associated with higher
incidence of metastases as well as an overall reduced survival rate of the cancer patients.
Previous work has strongly implicated that lactate is able to suppress antitumor
immunity through inactivation of cytotoxic T lymphocytes. However, it is uncertain how
the function goes on.
4. What is correct, according to the figure?
A. Only six of 42 patients contacted on day 7 (14.3%) reported increased pain
B. 7 % days after BFA treatment.
C. The mean A ±SD pain scores at the start were 7.3A±2.0.
D. At day 1 and day 7, the mean pain scores were 4.4+2.4% and 5.7A±2.6,
5. What is correct as per the given notice?
A. three has been some improvement in the way the patients used to take health medine
B. treatment by effective means have decreased mortality rate
C. Nutrient restriction has got the potential to change signaling routes.

Glioblastoma (GBM)

Glioblastoma (GBM) is a highly aggressive brain tumor and the most commonly
occurring primary malignant glioma in adults, accounting for approximately 50%
of all primary malignant brain tumor diagnoses. Standard of care consists of
surgical resection, concurrent radiation and chemotherapy, followed by adjuvant
chemotherapy. While this treatment has extended the average survival to 14.6
months and increased 2 year survival to 17%, the overall prognosis still remains
poor. GBM has proven difficult to treat due to tumor heterogeneity and the
presence of tumor microenvironments such as low pH, oxygen, and nutrients
Initially described as the Warburg Effect, tumor cells can activate alternative
metabolic pathways for production of ATP and biomolecules to circumvent
microenvironmental obstacles and fuel tumor growth. Nutrient restriction, just as
in many cases that we discover understand that is a modulator of the cellular
metabolic state and can alter the kinase signaling pathways in the cell, with glucose
playing a key role as a precursor for protein, nucleic acid, and lipid synthesis.
6. The given data best reflects
A. statistical significant differences in protein between tumor and normal tests which
was used
B. Total of 21 patients were included in the study
C. Fibronectin levels were quantified from both the tumour tissue and the adjacent
normal kidney tissue
READING SUB-TEST : PART C
In this part of the test, there are two texts about different aspects of healthcare.
For questions 7-22, choose the answer (A, B, C or D) which you think fits best according to the text.
Write your answers on the separate Answer Sheet

Text 1: Idiopathic Pulmonary Fibrosis (IPF)

Idiopathic pulmonary fibrosis (IPF) is a build-up of scar tissue in the lungs. This
scar tissue damages the lungs and makes it hard for oxygen to get in. Not getting
enough oxygen to the body can cause serious health problems and even death.
“Idiopathic” is the term used when no cause for the scarring can be found; in these
cases, doctors think the scarring starts with something that injures the lung. Scar
tissue builds up as the lungs try to repair the injury and, in time, so much scarring
forms that patients have problems breathing. IPF usually worsens over time.
However, while some patients get sick quickly, others may not feel sick for years.
Unfortunately, there is no cure for IPF, but there are treatments that may be able to
slow down the lung scarring. Understanding the condition will go a long way to
help you cope with the effects it has on your body.

The two major symptoms of IPF are shortness of breath and a persistent cough.
Other symptoms may include Fatigue and weakness, Chest pain or tightness in the
chest, Loss of appetite, Rapid weight loss etc. The causes of IPF are unknown.
There are other conditions that cause lung scarring; the lung scarring that is the
result of other conditions is often called “pulmonary ï¬ brosis”, but should be
called by the name of the cause. These other causes include the following:
Diseases, like rheumatoid arthritis and sarcoidosis, Medicines, such as those used
for certain heart conditions, Breathing in mineral dusts, such as asbestos or silica,
Allergies or overexposure to dusts, animals, or molds (There are many names for
this condition, such as “bird breeder’s lung,” “farmer’s lung,” or “humidity lung.” These
conditions are all called hypersensitivity pneumonitis).
Five million people worldwide have IPF, and it is estimated that up to 200,000
people in the United States have this condition. It usually occurs in adults between
40 and 90 years of age and it is seen more often in men than in women. Although
rare, IPF can sometimes run in families. Patients who have any symptoms of IPF
should see a pulmonologist to rule out similar conditions. The doctor will take a
number of tests, including: Breathing tests: to measure how well your lungs are
working, CT scan: to get a detailed image of your lungs, and to see if scarring has
started, Blood tests: to see if you have an infection, problems with your immune
system, or to see how much oxygen is in your blood, Bronchoscopy: to test a small
sample of lung tissue. A tube is inserted through the nose or mouth into the lung; a
light on the end of the tube lets the doctor see where to go. The doctor then takes a
small piece of lung tissue to be tested (this is called a biopsy). You usually do not
need to stay overnight in the hospital to have this done, Thoracoscopic biopsy: to
obtain larger tissue samples. This is a surgical procedure in which small incisions
are made in between the ribs. It usually requires a hospital stay and general anesthesia.

Once lung scarring forms, it cannot be removed surgically and there are currently
no medications that remove lung scarring. However, there are treatments, such as
the ones that follow, that may be able to help. Cigarette smoke not only damages
the lining of the lungs, it can also make you more likely to get a lung infection.
While some studies suggest that patients with IPF who smoke actually live longer,
these studies are not accepted by everyone, and most experts agree that you should
stop smoking. As lung scarring gets worse, many patients need extra oxygen to
help them go about their daily lives without getting too out of breath. You get this
oxygen from a tank that you carry around with you and, in later stages of IPF,
oxygen may be needed even while sleeping or resting. Oxygen is not addictive, so
you do not have to worry about using it too much. To help maintain your oxygen
levels, ask your doctor about a small, easy-to-use device called a pulse oximeter.
This device helps you to know just how much oxygen-low you require, especially during
activity.
Regular exercise can help patients with IPF. Staying in shape not only keeps your
breathing muscles strong, it also gives you more energy; this is because healthy
muscles need less oxygen to perform. Many patients with IPF lose weight because
of their disease. If you lose too much weight, your breathing muscles can become
weak and you also may not be able to fight off infections very well. A well balanced diet is
important to keep up your strength, but be wary of supplements and other nutrition
treatments that claim to improve IPF; it’s best to consult a doctor first.
Text 1: Questions 7-14 Idiopathic Pulmonary Fibrosis (IPF)

7. In IPF, patients
A. will have lung cancer
B. will have difficulty in inhalation or exhalation
C. will find it difficult to move
D. require less oxygen

8. Scar tissue develops


A. when oxygen supplied is stopped
B. when the lungs do not function properly
C. when the lungs try to repair the damage done
D. when there is more oxygen supply

9. Major symptoms of IPF are


A. fatigue and weakness
B. chest pain and breathing
C. breathing problems and coughing
D. breathing problems and weakness

10. The cause of lung scaring is


A. still not known completely
B. still not known completely
C. allergies
D. some of the common heart diseases

11. One of the simple IPF tests is


A. bronchoscopy
B. blood test
C. CT scan
D. breathing test to identify how well your lungs work
12. For lung scarring
A. no medication is available
B. medication is available
C. prevention is better
D. not given

13. Cessation means


A. to continue
B. to cease
C. to adopt
D. to gain

14 Exercises can help fight the IPF


A. true
B. false
C. sometimes true and sometimes false
D. can say
Text 2: Hyperthyroidism

The thyroid gland is a butterfly-shaped endocrine gland that is normally located in


the lower front of the neck. The thyroid’s job is to make thyroid hormones, which
are secreted into the blood and then carried to every tissue in the body. The thyroid
hormone helps the body use energy, stay warm and keep the brain, heart, muscles,
and other organs working as they should. The term hyperthyroidism refers to any
condition in which there are too many thyroid hormones produced in the body. In
other words, the thyroid gland is overactive and working too hard. Another term
that you might hear being used to describe the problem is thyrotoxicosis, which
refers to high thyroid hormone levels in the bloodstream, irrespective of their source.

The thyroid hormone plays a significant role in the pace of many processes in the
body; these processes are called your metabolism. If there is too much thyroid
hormone being produced, every function of the body tends to speed up. It is not
surprising then that some of the symptoms of hyperthyroidism are: nervousness,
irritability, increased perspiration, heart racing, hand tremors, anxiety, difficulty
sleeping, thinning of your skin, fine brittle hair and weakness in your muscles—
especially in the upper arms and thighs. Another symptom might be more frequent
bowel movements, but diarrhea is uncommon. You may lose weight despite a good
appetite and, for women, menstrual flow may lighten and menstrual periods may
occur less often. Since hyperthyroidism increases your metabolism, many
individuals initially have a lot of energy. However, as the hyperthyroidism
continues, the body tends to break down, so feeling tired is very common.

Hyperthyroidism usually begins quite slowly but in some young patients these
changes can be very abrupt. At first, the symptoms may be mistaken for simple
nervousness due to stress. If you have been trying to lose weight by dieting, you
may be pleased with your success until the hyperthyroidism, which has quickened
the weight loss, causes other problems. The most common cause (in more than
70% of people) is an overproduction of the thyroid hormone by the entire thyroid gland.
This condition is also known as Graves’ disease. Graves’ disease is caused
by antibodies in the blood that turn on the thyroid and cause it to grow and secrete
too much thyroid hormone. This type of hyperthyroidism tends to run in families
and it occurs more often in young women. Little is known about why specific
individuals get this disease.

Another type of hyperthyroidism is characterized by one or more nodules or lumps in


the thyroid that may gradually grow and increase their activity; this causes the total
output of thyroid hormones into the blood to become greater than normal. This
condition is known as toxic nodular or multinodular goiter. Also, people may
temporarily have symptoms of hyperthyroidism if they have a condition called
thyroiditis, a condition caused by a problem with the immune system or a viral
infection that causes the gland to leak stored thyroid hormone. The same symptoms
can also occur by taking too much thyroid hormone in tablet form. These last two
forms of excess thyroid hormone are only called thyrotoxicosis, since the thyroid is not
overactive.

If your physician suspects that you have hyperthyroidism, diagnosis is usually a simple
matter. A physical examination usually detects an enlarged thyroid gland and a rapid
pulse. The physician will also look for moist, smooth skin and a tremor of your
fingertips. Your reflexes are likely to be fast, and your eyes may have some
abnormalities if you have Graves’ disease. The diagnosis of hyperthyroidism will be
confirmed by laboratory tests that measure the number of thyroid hormones—
thyroxine (T4), triiodothyronine (T3) and thyroid-stimulating hormone (TSH) in your
blood. A high level of thyroid hormones in the blood plus a low level of TSH is common
with an overactive thyroid gland. If blood tests show that your thyroid is overactive,
your doctor may want to obtain a picture of your thyroid (a thyroid
scan). The scan will find out if your entire thyroid gland is overactive or whether you
have a toxic nodular goiter or thyroiditis (thyroid inflammation). A test that measures
the ability of the gland to collect iodine (a thyroid uptake) may be done at the same
time.
No single treatment is best for all patients with hyperthyroidism. The appropriate
choice of treatment will be influenced by your age, the type of hyperthyroidism that
you have, the severity of your hyperthyroidism, and any other medical conditions that
may be affecting your health, as well as your own preference. It may be a good idea to
consult with an endocrinologist who is experienced in the treatment of hyperthyroid
patients. If you are unconvinced or unclear about any thyroid treatment plan, a second
opinion is a good idea.
Text 2: Questions 15-22

15. The thyroid hormone helps with;


A. Energy consumption
B. Utilization of energy
C. Maintaining body temperature
D. Enhancing the functions of the kidney

16. In thyrotoxicosis the thyroid gland;


A. Is inactive
B. Is less active
C. Produces a greater amount of hormones than necessary
D. None of the above

17. An increase in the amount of thyroid hormones can;


A. Boost up other hormonal functions
B. Improve metabolic functions
C. Increase normal physiological functions
D. Increase pulse rate

18. Which one of these is common in thyroid diseases?


A. Loss of appetite
B. Decreased metabolism
C. Tiredness
D. None of the above

19. Hyperthyroidism can be the cause of


A. High blood pressure
B. Tiredness
C. Weight loss
D. Increase in weight, even while dieting
20. According to the information given, Graves disease occurs more commonly in;
A. Men
B. Women
C. Children
D. Adult women

21. Which one of the following suggests an overactive thyroid gland?


A. Low level of TSH
B. High level of thyroid hormones
C. High level of TSH and low level of hormones
D. High level of hormones and low level of TSH

22. According to the information given, treatment for hyperthyroidism depends more on;
A. Age
B. Only on the type of hyperthyroidism
C. The previous medical history of the patient
D. Age and type of hyperthyroidism

END OF READING TEST, THIS BOOKLET WILL BE COLLECTED


Reading test 25 : Answer Key

Part A - Answer key 1 – 7


1:B
2:D
3: B
4: C
5 :A
6: A
7: C

Part A - Answer key 8 – 14


8 : Collagen disease
9: Located inferomedially from the vascular bundle
10: Iatrogenic
11: Incarcerated
12: Spelian zone
13:aponeurosis
14: occurs in 6th-7th decade of life

Part A - Answer key 15 – 20


15:Omentum
16 : Subcutaneous Hernia
17: Spigelian Line
18: Peritoneum
19: Non-specific
20: Abdominal Hernias
Reading test - part B – answer key

1: Implications for clinical effectiveness.


2: How hypoxia interferes with T lymphocyte effector function
3 : Role of key players in the immunologic antitumor response
4 : The mean±SD pain scores at the start were 7.3±2.0.
5 : Nutrient restriction has got the potential to change signaling routes.
6: Total of 21 patients were included in the study

Reading test - part C – answer key

Text 1 - Answer key 7 – 14


Idiopathic Pulmonary Fibrosis (IPF)
7 : will have difficulty in inhalation or exhalation
8 : when the lungs try to repair the damage done
9 : breathing problems and coughing
10 : still not known completely
11 : breathing test to identify how well your lungs work
12 : no medication is available
13 : to cease
14 : true

Text 2 - Answer key 15 – 22


15: Energy consumption
16 : Produces a greater amount of hormones than necessary
17: Improve metabolic functions
18: Tiredness
19: Weight loss
20: Women
21: High level of hormones and low level of TSH
22: Age and type of hyperthyroidism
READING TEST 26
READING SUB-TEST : PART A
 Look at the four texts, A-D, in the separate Text Booklet.
 For each question, 1-20, look through the texts, A-D, to find the relevant information.
 Write your answers on the spaces provided in this Question Paper.
 Answer all the questions within the 15-minute time limit.
 Your answers should be correctly spelt.

TEXT BOOKLET - BATTENS DISEASE

Text A

Battens Disease

The Neuronal Ceroid Lipofuscinoses (NCL's), also known as Battens disease, are a
collection of congenital neurodegenerative conditions that span from prenatal life
to late adulthood with an incidence of 1:12,500. They comprise of at least 8
autosomal recessive disorders defined by having a mutation in a CLN gene, either
coding for an enzyme (CLN1 and CLN2) or a transmembrane protein (CLN3,
CLN5, CLN6 and CLN8) with all disorders having common clinical features,
including progressive visual loss to blindness, seizures, speech disturbances, motor
degeneration and intellectual decline, leading to early death LINCL has an
incidence of 0.36-0.46 per 100,000 with an age of onset of between 2-4 years, and
death commonly anticipated in the early teenage years. LINCL is caused by a
mutation of the CLN2 gene on chromosome 11p15, of which 98 mutations are
known, three of which account for the majority of cases.
Text B

The normal product of CLN2 is tripeptidyl peptidase I (TPP-1), which functions


within the lysosome to degrade N-terminal tripeptides from their substrates.
Therefore deficiencies lead to an accumulation, in particular of subunit C of
mitochondrial ATP synthase causing subsequent neuronal and retinal cell death.
This accumulation is seen on UV imaging as autofluorescent storage and on
electron microscopy as curvilinear bodies. Gene therapy is an attractive prospect
for long-term therapy in LINCL because it is a monogenic disorder. The process
involves introducing CLN2 human complementary DNA (cDNA) into the central
nervous system under the control of a promoter and in a suitable vector.
Adenoassociated viruses are considered the ideal vector as they can transduce
Postmitotic cells, mediate long-term gene expression and have an excellent safety record.
Therefore, these vectors are considered potential delivery vectors for the treatment
of LINCL. The challenge is whether using a suitable AAV vector allows sufficient
activity, expression and distribution of TPP-1 to destroy existing lysosomal storage
protein, prevent its on-going formation and consequently halt the progression of
the disease. One way of maximizing enzyme dispensation is through crosscorrection
using mannose-6-phosphate pathway, this means that CLN2 cDNA does
not need not be introduced to the whole of the CNS. The target for gene expression
is 1-5% normal expression of the gene product which is sufficient to reverse
pathology in other conditions.
Text C

One human trial evaluated the use of AAV2 vector to transfer human cCLN2
cDNA to the CNS of 10 children with LINCL aged between 3 and 10 with five
different mutation types. The study was an 18-month follow-up to vector
administration with a primary outcome measure being neurological assessment of
disease status using the modified Hamburg LINCL scale. Secondary measures
were quantitative CNS Magnetic Resonance Imaging assessment of the brain
including grey matter and ventricular volume. Control comparisons were made
with data from 4 independent untreated LINCL children who had been assessed
twice at 1-year intervals. Participants received an average dose of 2.5 × 1012
particles (1.8-3.2 × 1012) of the AAV2Hcln2 vector. The dose was shared equally
through 6 burr holes (3 in each hemisphere) of the cranial vault, and 12 cortical
locations were targeted. Assessments were made on days 7 and 14 and at 1, 6, 12
and 18 months after therapy. Adverse effects were assessed at 2 and 3 months.
Text D

While surgery itself recorded no evidence of adverse effects, post-therapy


highlighted 60 serious and 94 non-serious complications, the majority occurring
within the first 2 weeks after therapy. Serious events, including seizures and
myoclonus were considered to be consequences of drug administration, whereas
vomiting and thrombocytosis (non-serious examples) were not considered to be
caused by the operation or by tolerance to the therapy. Out of the 10 patients, 2
died, one of unknown reasons and the other during the study period from status
epilepticus, a known complication of late LINCL. Primary assessments
demonstrated that gene therapy had an effect on the progression of disease, with
an improved modified Hamburg scale score in treated subjects monitored for >6
months, and a disease progression rate which was significantly slower than in
controls. This difference was shown by the mean rate of change of the modified
Hamburg scale in the treated and untreated groups (P<0.05). Secondary measures
showed that the MRI parameters indicated a decreased rate of decline, comparable
with the primary outcomes, however, there was no statistical significance between
groups including in the grey matter volume as a percentage of total brain volume
with -2.6%/year loss of volume compared to -2.84%/year in controls (p=0.8).
PART A -QUESTIONS AND ANSWER SHEET

Questions 1-7

For each question, 1-7, decide which text (A, B, C or D) the information comes from.
You may use any letter more than once.

In which text can you find information about;

1. It is not currently known to cause disease, but causes a very mild immune response. Answer___
2. A disorder in which nerve cell activity in the brain is disturbed, causing seizures. Answer ___
3. Not so prevalent disorder that primarily affects the nervous system. Answer ___
4. Degeneration of the neurons. Answer _____________
5. Recent studies conducted on evaluation of the disease. Answer ______________
6 .After effects of the treatment. Answer _____________
7. Affected person may not be in a state to use mental power effectively. Answer ____________

Questions 8-14

Answer each of the questions, 8-14, with a word or short phrase from one of the
texts. Each answer may include words, numbers or both. Your answers should be correctly spelt.

8 What is the common reason for Battens Disease? Answer ______________


9 Which can transduce postmitotic cells? Answer_______________
10 Which is a known complication of late LINCL? Answer_______________
11 What are the two conditions which are known to arise not out of therapy ? Answer____
12 What is used to maximize the enzyme dispensation through cross-correction? Answer____
13 Which is the effective long-term therapy in LINCL? Answer____
14 What dosage of the AAV2Hcln2 vector did the participants receive? Answer___
Questions 15-20

Complete each of the sentences, 15-20, with a word or short phrase from one of the texts.
Each answer may include words, numbers or both. Your answers should be correctly spelt.

15.____________ can be described as spasmodic jerky contraction of groups of muscles.


16 . The ____________ gene provides instructions for making an enzyme called tripeptidyl peptidase 1.
17. It is always not required to introduce ____________ to CNS.
18.____________ made it all clear that the therapy can have more positive impact on the disease.
19. Out of all the mutations, _____________ are considered to be the major causes for the disease to
appear in infants and adults.
20 .The____ gene provides instructions for making a protein whose function is not well understood

END OF PART A, THIS QUESTIONS PAPER WILL BE COLLECTED


READING SUB-TEST : PART B
In this part of the test, there are six short extracts relating to the work of health professionals .
For questions 1-6, choose the answer (A, B or C) which you think fits best according to the text.
Write your answers on the separate Answer Sheet
Questions 1-6

1 .What does the notice indicates?


A. Angina pectoris is common among people.
B. There is a direct relationship between the functioning of liver and Qi blood of the
heart.
C. Various ailments can lead to angina pectoris.

Angina pectoris

Angina pectoris, whose common type is Qi-blood stagnation type is one of chest
congestion in TCM. The main cause of this type of angina is damage due to an
excess of seven emotions. The heart governs the mind and blood circulation, liver
stores of blood and controls catharsis. There is close relationship between the two.
Only if liver function is normal can Qi-blood of the heart can be calm. For elderly
individuals, who lack Qi-blood, yin and yang of the heart, spleen and kidney, are
of cold invasion, consume an inappropriate diet, have emotional disorders and
excessive fatigue, among others, can also lead to blood stasis stagnation, which can
consequently cause coronary angina pectoris of Qi-blood stagnation. The
‘Suwen Yujizhen zang’ theory has described “blockage of vessel, inability of QI to
flow freely”, which illustrates that blood stagnation can lead to Qi depression.
Therefore, accelerating blood circulation, removing blood stasis and promoting the
circulation of Qi are the main therapies.
2. The notice gives information about;
A. two different technologies used in treatment.
B. ineffectiveness of the SCS.
C. DRG and SCS comparison.

Spinal cord stimulation

For many years, Spinal cord stimulation has been used as a salvage treatment for
intractable CRPS even though many studies have not proven long-term benefit. In
multiple studies published by a European neurosurgical group, there has been great
benefit from this technology in the first year but the vast majority of patients experience a
return of symptoms by year six. Newer dorsal root ganglion (DRG) stimulation technology
may be more promising. In the most recent DRG stimulation trial, patients with CRPS have
been shown to decrease pain by fifty percent or greater in 93% of patients with chronic
intractable pain at three-month follow-up, versus 72% of patients with an SCS implant.
Unfortunately, this product is currently only FDA approved for treatment in the lower
extremity.
3 .SGB is known;
A. to be more effective in treatment.
B. to provide temporary relief from pain.
C. to be a proven medicine in certain instances.

Stellate ganglion blocks (SGB)

Stellate ganglion blocks (SGB) are the most commonly performed interventional
procedure for patients with upper extremity CRPS. The stellate ganglion is located
anterior to the 7th cervical transverse process on the anterior surface of the longus
coli muscle. It lies medial to the vertebral artery and anterolateral to the ipsilateral
common carotid artery. This ganglion can be accessed either through fluoroscopy,
CT, or ultrasound guidance. In a study published in 2006 by Ackerman and Zhang,
25 subjects underwent SGB at weekly intervals for 3 weeks. At 6 months, 40% of
patients had complete symptom relief while 24% of patients had no pain relief. In a
second study, published in 2009, three weekly blockades were completed at weekly
intervals in 22 patients with CRPS type I of the hand. Pain intensity and range of motion
were assessed two weeks after treatment. In this study patients had statistically significant
improvement in wrist ROM (P>0.001) and an overall decrease in VAS values from 8 to 1.
While most physicians do not believe that SGB alone is effective in curing the disease, we
do know that stellate ganglion blocks at least offer temporary benefits that last well
beyond the effects of local anesthetic.
4 .What is correct about metabonomics?
A. High-throughput metabolomic approach revealed the acupuncture exerting
intervention effects.
B. Metabolomics has seen a surge in popularity in recent scientific research.
C. The ultimate aim of metabonomics is to detect every small-molecule metabolite.

Metabonomics

Metabonomics as an advanced technology of systems biology has made considerable


contributions to early diagnosis of serious illness, toxicology and pharmacology. Early,
accurate diagnosis of tumours, such as liver cancer and ovarian cancer, is a distinguishing
feature and advantage of metabonomics. Valine, saturated lipids, glycine, lactate, inositol,
nucleotides, polyunsaturated fatty acids, taurine and other tumour-related metabolic
markers can be identified by metabolomics, which makes metabonomics increasingly
applicable to diagnosis, therapy and prognostic evaluation. Acupuncture has a therapeutic
effect for tumours: it can improve symptoms, prolong the lifespan of patients, improve the
quality of life, relieve cancer pain syndrome and, especially, improve the side effects of
radiotherapy and chemotherapy, eg, myelosuppression, gastrointestinal reaction, fever
etc. Acupuncture treatment of tumour possesses many features and advantages. Applying
metabonomics to research investigating acupuncture treatment of tumours both gives
play to the sensitive detection advantage of metabonomics for tumour diagnosis,
treatment assessment and exploring the mechanism of acupuncture treatment at deeper,
more extensive and more dynamic levels. In addition, there are some reports investigating
Alzheimer diseases at home and abroad, however, few in research of venereal disease.
5 .What does this notice indicate?
A. The curative effect of improving disease condition in treatment group is superior to
the control group.
B. two groups differ only by minor values.
C. total effectiveness of the control group is lesser than treatment group.

Treatment Groups - Recent Analysis


C Total
excellen effect Ineffec exacerb
Groups as effective
t ive tive ated
es rate, %
4 70.0
Control 19 9 10 2
0
Treatme 4 90.2*
28 9 4 0
nt 1
6 .What does the result indicate?
A. The results indicate that subjects in the two groups improved after treatment.
B. The laboratory indexes of the treatment group significantly declined.
C. After treatment, the laboratory indexes of the observation group were also
more declined than the control group.

Disease Treatment Results - Before and After


READING SUB-TEST : PART C
In this part of the test, there are two texts about different aspects of healthcare.
For questions 7-22, choose the answer (A, B, C or D) which you think fits best according to the text.
Write your answers on the separate Answer Sheet

Text 1: Hormonal Disorder - Adrenal Insufficiency

Adrenal insufficiency is an endocrine or hormonal disorder that occurs when the


adrenal glands do not produce enough of certain hormones. The adrenal glands are
located just above the kidneys. Adrenal insufficiency can be primary or secondary.
Primary adrenal insufficiency, also called Addison’s disease, occurs when the
adrenal glands are damaged and cannot produce enough of the hormone cortisol,
and often the hormone aldosterone. Addison’s disease affects one to four of every
100,000 people, in all age groups and both sexes. Secondary adrenal insufficiency
occurs when the pituitary gland, a bean-sized organ in the brain, fails to produce
enough adrenocorticotropin (ACTH), a hormone that stimulates the adrenal glands
to produce cortisol. If ACTH output is too low, cortisol production drops.
Eventually, the adrenal glands can shrink due to lack of ACTH stimulation.
Secondary adrenal insufficiency is much more common than Addison’s disease.

Addison's disease symptoms usually develop slowly, often over several months,
and may include muscle weakness and fatigue, weight loss and decreased appetite,
darkening of skin (hyperpigmentation), low blood pressure (even fainting), salt
cravings, low blood sugar (hypoglycemia), nausea, diarrhea or vomiting, muscle or
joint pains etc. Sometimes, however, the signs and symptoms of Addison's disease
may appear suddenly. In acute adrenal failure (Addisonian crisis), the signs and symptoms
may also include pain in your lower back, abdomen or legs, severe vomiting and diarrhea,
leading to dehydration, low blood pressure and loss of consciousness.
Your adrenal glands are composed of two sections: the interior (medulla) produces
adrenaline-like hormones; the outer layer (cortex) produces a group of hormones
called corticosteroids, which include glucocorticoids, mineralocorticoids and male
sex hormones (androgens). Glucocorticoids: These hormones, which include cortisol,
influence your body's ability to convert food fuels into energy, play a role in your immune
system's inflammatory response and help your body respond to stress. Mineralocorticoids:
These hormones, which include aldosterone, maintain your body's balance of sodium and
potassium to keep your blood pressure normal. Androgens, male sex hormones, are
produced in small amounts by the adrenal glands in both men and women. They cause
sexual development in men and influence muscle mass, libido and a sense of well-being in
men and women.

Addison's disease occurs when the cortex is damaged and doesn't produce its hormones in
adequate quantities. Doctors refer to the condition involving damageto the adrenal glands
as primary adrenal insufficiency. The failure of your adrenal glands to produce
adrenocortical hormones is most commonly the result of the body attacking itself
(autoimmune disease). For unknown reasons, your immune system views the adrenal
cortex as foreign, something to attack and destroy. Other causes of adrenal gland failure
may include tuberculosis, other infections of the adrenal glands, spreading of cancer to the
adrenal glands and bleeding into the adrenal glands. Adrenal insufficiency can also occur if
your pituitary gland is diseased. The pituitary gland produces a hormone called
adrenocorticotropic hormone (ACTH), which stimulates the adrenal cortex to produce its
hormones. Inadequate production of ACTH can lead to insufficient production of
hormones normally produced by your adrenal glands, even though your adrenal glands
aren't damaged. Doctors call this condition secondary adrenal insufficiency. Another more
common cause of secondary adrenal insufficiency occurs when people who take
corticosteroids for treatment of chronic conditions, such as asthma or arthritis, abruptly
stop taking the corticosteroids. If you have untreated Addison's disease, an Addisonian
crisis may be provoked by physical stress (such as an injury), infection or illness. All
treatment for Addison's disease involves hormone replacement therapy to correct the
levels of steroid hormones your body isn't producing. Some options for treatment include
oral corticosteroids, corticosteroid injections and androgen replacement therapy.
Text 1: Questions 7-14

7. Addisons disease effects;


A. Four of every 1,000 people.
B. One of every 1,000 people, including all age-groups.
C. One to four of every 1,000 people, including all age groups.
D. One to four of every 100,000 people.

8. Secondary adrenal insufficiency occurs due to one of these reasons;


A. More production of adrenocorticotropin (ACTH).
B. More production of cortisol.
C. Low production of aldosterone.
D. Low production of ACTH.

9. According to the passage, symptoms of Addisons disease occur;


A. Slowly
B. Suddenly
C. After a month
D. Slowly after many months

10. Cortex produces;


A. Androgens
B. Glucocorticoids
C. Mineralocorticoids
D. All of the above

11. Aldosterone maintains;


A. Body balance
B. Balance of sodium and potassium
C. High BP
D. Low BP
12. According to the information given in the passage, the statement that Androgen
influences muscle mass and physical and mental nature of men and women is;
A. True
B. False
C. Can say
D. Not given in the passage(s)

13. Doctors refer to the damage to the cortex of the kidney as;
A. Primary adrenal insufficiency.
B. Secondary adrenal insufficiency.
C. Other fatal infections.
D. Not given

14 .Secondary adrenal insufficiency also occurs because of;


A. Asthma
B. Arthritis
C. Both asthma and arthritis.
D. Stopping the use of corticosteroids.
Text 2: Carpal Tunnel Syndrome

Carpal tunnel syndrome is a condition that may be caused by repeatedly performing


stressful motions with your hand or holding your hand in the same position for long
periods of time. CTS is classified as a cumulative trauma disorder, an ailment that
attacks the body’s musculoskeletal system. The musculoskeletal system is made up of
muscles that pull on tendons and move the bones at joints. The joints are held
together by ligaments. Carpal tunnel syndrome specifically affects the sensitive nerves
of - and the blood supply that feeds – the hands and wrists.

Carpal tunnel syndrome has been around for a long time; meatpackers began
complaining of pain and loss of hand function in the 1860s. Back then, these
complaints were largely attributed to poor circulation. The nature of work has
changed over the years; today, more jobs are highly specialized and require use of
only a small number of muscles repeatedly. With the growing numbers of people
using computers and keyboards, plus the focus on better healthcare for workers,
carpal tunnel syndrome is of real concern to both employers and health-care
professionals. Recent studies have shown that carpal tunnel syndrome, like all
other cumulative trauma disorders, is on the rise while other workplace injuries
have leveled off. Many companies are turning to physical therapists for help with
designing and implementing health promotion and injury prevention programs to
protect their employees from CTS.
People with CTS usually experience feelings of numbness, weakness, tingling, and
burning in their fingers and hands. If not treated, the symptoms may escalate into
acute, persistent pain. CTS can become so crippling that people can no longer do
their work or even perform simple tasks at home. At its most extreme, carpal
tunnel syndrome forces people to undergo surgery and miss many days of work, or
prevents them from working at all because their hand functions are permanently
impaired. Carpal tunnel syndrome occurs in men and women of all ages, and is
often found in workers whose tasks require repeating the same motion in the
fingers and hand for long periods of time. CTS has surfaced among meat packers,
assembly line workers, jackhammer operators, and employees who spend hours
working at a computer or typewriter. Carpal tunnel syndrome shows up in athletes
as well as homemakers.

The U.S. Department of Labor has cited carpal tunnel syndrome, as well as other
cumulative trauma disorders, as the cause of 48 percent of all industrial workplace
illnesses. The disease affects more than five million Americans. CTS’s impact on
American businesses is devastating. It shows up in the workplace in the form of
fatigue, poor work performance, discomfort and pain, and poor
employer/employee relations. The high cost of treatment for an employee with
CTS, plus the lost productivity when that employee is absent for a long period of
time, strains the company’s ability to operate efficiently and can lead to morale
problems when other employees have to take over the absent workers’
responsibilities. Physical therapists with specialized training in cumulative trauma
disorders have been working in industrial and corporate settings for many years to
meet the health-care needs of America’s workforce. They work closely with
employers to educate employees about CTS—what causes it and how to avoid it
through proper use of the musculoskeletal system.
Physical therapists can target and correct poor work habits and improper work
designs, such as tools, furniture, equipment, and workspace. They can also assess
the potential risks of an individual and determine if that person is physically
unsuited to a particular job. Among their many responsibilities, physical therapists
teach health awareness and job safety. A typical education program includes
exercises employees can do at work and at home, adjustments to the overall work
environment and individual workstations, plus early detection of symptoms to
avoid painful and costly surgery. Physical therapists also work with employers and
their engineering departments to design and modify the work environment, helping
to remove the causal factors of CTS. If anyone has symptoms of carpal tunnel
syndrome, then consulting a physical therapist or other qualified healthcare
practitioners for an evaluation and individualized treatment is always
recommended.
Text 2: Questions 15-22

15. According to the passage, CTS;


A. Is a cumulative trauma disorder.
B. Is caused due to weakness in musculoskeletal system.
C. Occurs due to weakness in ligaments between joints.
D. All of the above

16. According to the information given, CTS is;


A. On the rise.
B. On the rise without any other cumulative trauma disorders.
C. One of the common cumulative trauma disorders that are increasing.
D. On the rise with many other cumulative trauma disorders.

17. In CTS, _____ may become permanently impaired.


A. Carpels
B. Hands
C. Feet
D. Wrists

18. CTS often occurs in;


A. Men
B. Women
C. Men and women of all ages
D. Only men of all ages

19.____ complained of pain and loss of hand function in the 1860s;


A. Meat packers
B. Assembly line workers
C. Jackhammer operators
D. Employees who spend hours at a computer
20. CTS accounts for _____.of all industrial workplace illnesses;
A. 50%
B. 48%
C. 84%
D. 60%

21. Physical therapists teach employees about;


A. Proper use of the musculoskeletal system.
B. How to avoid CTS.
C. How to increase efficiency at work.
D. Health-care needs.

22. CTS educational program includes;


A. Only detection of CTS symptoms in employees.
B. Explanation to employees on how to avoid CTS.
C. Exercises that employees can do to avoid CTS.
D. Discussion of cost involved in surgery.

END OF READING TEST, THIS BOOKLET WILL BE COLLECTED


Reading test 26: Answer Key
Part A - Answer key 1 – 7
1 :B
2 :D
3A
4:B
5:C
6:D
7 :A

Part A - Answer key 8 – 14


8: mutation of the CLN2 gene
9: Adeno-associated viruses
10: epilepticus
11 : vomiting and thrombocytosis
12: mannose-6-phosphate pathway
13: Gene therapy
14: 2.5 × 1012 particles

Part A - Answer key 15 – 20


Complete each of the sentences, 15-20, with a word or short phrase from one of
the texts. Each answer may include words, numbers or both.
15: Myoclonus
16 : Tpp1
17: Cln2 Cdna
18 : Primary Assessments
19: Three
20 :Cln6
Reading test - part B – answer key
1: Various ailments can lead to angina pectoris.
2: ineffectiveness of the SCS.
3: to provide temporary relief from pain.
4: Metabolomics has seen a surge in popularity in recent scientific research.
5: The curative effect of improving disease condition in treatment group is
superior to the control group.
6: The results indicate that subjects in the two groups improved after treatment.

Reading test - part C – answer key

Text 1 - Answer key 7 – 14


7: One to four of every 1,000 people, including all age groups.
8: Low production of ACTH.
9: Slowly after many months
10: Androgens
11: Balance of sodium and potassium
12: False
13: Primary adrenal insufficiency.
14: Stopping the use of corticosteroids.

Text 2 - Answer key 15 – 22


15: Is a cumulative trauma disorder.
16: On the rise with many other cumulative trauma disorders.
17: Wrists
18: Men and women of all ages
19: Meat packers
20: 48%
21: How to avoid CTS.
22: Exercises that employees can do to avoid CTS.
READING TEST 27
READING SUB-TEST : PART A
 Look at the four texts, A-D, in the separate Text Booklet.
 For each question, 1-20, look through the texts, A-D, to find the relevant information.
 Write your answers on the spaces provided in this Question Paper.
 Answer all the questions within the 15-minute time limit.
 Your answers should be correctly spelt.

TEXT BOOKLET - DNA SEQUENCING TECHNOLOGIES

TEXT A

DNA Sequencing Technologies


Past efforts at sequencing genes were painstaking, time consuming, and labor intensive,
such as when Gilbert and Maxam reported the sequence of 24 base pairs using a
method known as wandering-spot analysis. Thankfully, this situation began to change
during the mid-1970s, when researcher Frederick Sanger developed several faster, more
efficient techniques to sequence DNA. Indeed, Sanger's work in this area was so ground
breaking that it led to his receipt of the Nobel Prize in 1980.
Over the next several decades, technical advances automated, dramatically sped up,
and further refined the Sanger sequencing process. Also called the chain-termination or
dideoxy method, Sanger sequencing involves using a purified DNA polymerase enzyme
to synthesize DNA chains of varying lengths. The key feature of the Sanger method
reaction mixture is the inclusion of dideoxynucleotide triphosphates (ddNTPs). These
chain-terminating dideoxynucleotides lack the 3' hydroxyl (OH) group needed to form
the phosphodiester bond between one nucleotide and the next during DNA strand
elongation. Thus, when a dideoxynucleotide is incorporated into the growing strand, it
inhibits further strand extension. The result of many of these reactions is a number of
DNA fragments of varying length. These fragments are then separated by size using gel
or capillary tube electrophoresis. This procedure is sensitive enough to distinguish DNA
fragments that differ in size by only a single nucleotide.
Text B

In the modern era, whole world has experienced tremendous boost in the field of
molecular diagnosis by use of DNA sequencing technology. The human genome contains
more than 3 billion base pairs that contain all the information about our health and
wellbeing.
The first whole genome sequence of human was published successfully before
decades. It was very expensive and paid billion dollars to complete. The cost paid for,
was much more worthy as it was providing the first fundamental understanding of the
structure and biology of human genome and relation with diseases. Initially, Next
Generation Sequencing technology was generated huge amount data of human variant
later it was proved that this NGS technology is much more effective in the diagnosis
human diseases by use of bioinformatic tools to select pathogenic variants. In present
days, sequencing costs have dramatically declined and therefore it is now routinely using for
diagnosis of many rare inherited diseases including hematology and blood disorder.
Genome Wide sequence analysis is already playing an important role in the hematology
field. This new sequencing technology is going to solve the challenges that researchers in the
field of hematology are going forward.
Text C

Nowadays researchers are making disease-specific targeted NGS panel, which is helping
more quickly and precise diagnosis of specific disease in the field of hematology.
Keeping in mind the growing research in the area of molecular diagnosis, how genome
wide analysis has unlocked new avenues of research, diagnosis, and therapy for benign
hematologic disorders. Recent advances in molecular technologies, mainly next
generation sequencing, inspire us to apply these technologies as a first-line approach for
the identification of potential mutations and to determine the novel causative genes in
patients with blood disorders. Researcher have started preparing targeted NGS panels
for diagnosis of hematologic malignancies, Red cell congenital hemolytic anemia for
diagnosis of all rare cause of haemolytic anemia which covers around 70-80 genes
associated with hemoglobinopathies, which will cover gene related alpha (HBA1/2) and
beta (HBB) globin gene locus analysis, HBD sequence analysis, gene related to RBC
membrane protein disorders, RBC enzymopathies genes, congenital dyserythropoietic
anemia (CDA) and the inherited bone marrow failure syndromes (IBMFS) are a group of
rare genetic blood disorders in which there is usually some form of aplastic anemia
associated with a family history of the similar disorder.
Text D

A panel of genes already identified by WES and association studies as responsible of


CHA or modulators of the clinical course of the disease is already analyzed by mass
sequencing methodology (NGS) including between 60-70 genes. Modifiers, related to
sickle-cell formation, vascular adhesion to endothelium, tolerance to hemolysis and
acute severe events, and the analysis of the RBC glycolytic enzymes are included. The
variants obtained were studied by mapping in the GRCh38/hg38 version of the human
reference genome. For the prioritization of variants, filters related to pathogenicity and
population frequency according to the SnpEff v4.1 and Mutation Taster programs
generally used for analysis. Some are previously used clinical databases of hemolytic
anemia are Human Gene Mutation Database version Professional, ClinVar, Red Cell
Membrane Database Mutations Database, Leiden Open Variation Database ± PKLR. The
allele frequency is generally assessed in the population (1000G and ExAC) and in the
local database. Finally, most of the researchers generally used the ‘in silico’ predictions
of pathogenicity and Sorting Intolerant from Tolerant (SIFT), PolyPhen-2, Mutation
Taster, and Mutation Assessor
PART A -QUESTIONS AND ANSWER SHEET

Questions 1-7
For each question, 1-7, decide which text (A, B, C or D) the information comes from.
You may use any letter more than once.

In which text can you find information about;


1. Modern technologies of DNA sequencing Answer ___________
2. Determination of the precise sequence of nucleotides in a sample of DNA. Answer ___________
3. A source of inspiration Answer _____________
4. Early DNA sequencing technologies Answer ______________
5. Cost-effective solutions in sequencing Answer ____________
6. Clinical databases Answer ____________
7. Detection of the disease-causing genes Answer _____________

Questions 8-14

Answer each of the questions, 8-14, with a word or short phrase from one of the texts.
Each answer may include words, numbers or both. Your answers should be correctly spelt.
8. What is generally analyzed in a large population? Answer ____________
9. What is the term which defines a method in which an electric field pulls molecules across a gel
substrate or hairlike capillary fiber? Answer ____________
10. Which term may mean "performed on the computer or via computer simulation? Answer ___
11. What is known to make up the backbone of the strands of nucleic acid? Answer ____
12. What is characterized by failure of the bone marrow to produce blood? Answer ____________
13. Which method of sequencing usually comprises use of polymerase enzyme for the purpose of
building different types of chains of varied lengths? Answer ____________
14. Which sequencing technology is regarded to be more result-oriented? Answer ____________
Questions 15-20

Complete each of the sentences, 15-20, with a word or short phrase from one of the texts.
Each answer may include words, numbers or both. Your answers should be correctly spelt.

15. Red blood cell ____________ affect genes encoding red blood cell enzymes..

16. Today, researchers are busy developed targeted NGS panels which can effectively be used for the
purpose of diagnosis of_____________.

17. The variants obtained were studied by mapping in the______ of the human reference genome.

18. The key feature of the Sanger method reaction mixture is the inclusion of____________.

19.______________ analysis is effective and can solve challenges which researchers grappling with

20. NGS is considered the _____________ for detection of the disease.

END OF PART A, THIS QUESTIONS PAPER WILL BE COLLECTED


READING SUB-TEST : PART B
In this part of the test, there are six short extracts relating to the work of health professionals .
For questions 1-6, choose the answer (A, B or C) which you think fits best according to the text.
Write your answers on the separate Answer Sheet.

Questions 1-6

1. What does the report indicate?


A. There has been a recent increase in the incidence of myositis worldwide.
B. Infection of the skeletal muscles.
C. Most common in tropical areas but can also occur in temperate zones.

Primary infection of muscle

Primary infection of muscle is usually regarded as a tropical disease, and is rare although
becoming more common in temperate climates. A review of the 230 cases of primary
obturator myositis which have been reported, shows that 82% were in children under 18
years of age. The median age was 9.5 years (3 to 46), with a male:female ratio of
approximately 3:2. The median duration of symptoms prior to presentation was three
days, and the most common symptoms were fever, hip or thigh pain, and inability to fully
bear weight. In 41% there was a history of recent trauma to the hip, such as a fall or
strenuous exercise. Local trauma is a recognised initiating factor for pyomyositis and is
documented in between 21% and 66% of cases.
2. According to the notice given, what is correct?
A. Treatment cost has not gone down yet.
B. CAR-T is considered more effective.
C. After chemotherapy, CAR-T is more reliable.

Food and Drug Administration (FDA) Notice

In the year 2017, Food and Drug Administration (FDA) announced the first approval of a
CAR-T cell therapy for kids and young adults with B-cell ALL. This approval was much
celebrated and brought new hope for a more specific and efficient therapy for ALL. This
result has come through many years of research and is expected to improve the quality of
treatment of patients. It is worth noting that in addition to side effects, the high cost of
treatment is still an obstacle and the side effects. Although there are challenges to be
overcome as in any innovative research, CAR-T cell therapy seems to be the most
promising therapeutic tool against cancer, including ALL, since chemotherapy
introduction in the 1940s.
3. Huntington`s disease;
A. may result in the death of brain cells.
B. may result in loss of intelligence.
C. may result in involuntary movements.

Huntington’s disease (HD)

Huntington’s disease (HD) is a genetic neurodegenerative disorder that results in chorea,


balance and gait impairments, changes in behavior and declines in cognition. Cognitive
decline occurs early in the disease course, occasionally preceding chorea and motor
impairment, and continues to progress throughout the disease process. Cognitive deficits
include difficulty with executive function including trouble with planning and organizing,
problems with working, visual and verbal memory, and impaired concentration. While
gait dysfunction is typically thought to arise primarily from damage to the motor circuitry
of the basal ganglia, studies in elderly populations and other neurologic populations
indicate that gait dysfunction may also be related to changes in cognitive function.
4. What is correct about Hypertension?
A. For subjects with normal or subnormal hypertension,it can prevent or delay hypertension.
B. BP can be well tackled with the use of BP specific medicine.
C. Right changes in lifestyle can curtail down BP values more effectively than various other
medical therapies.

Hypertension - Prevention or Treatment

Adequate changes in lifestyle are the cornerstone for the prevention and treatment of
hypertension. Although rapid medical initiation is necessary for the patients in a high
level of risk, lifestyle changes are fundamental for the therapy. According to the previous
report, lowering effects for stable blood pressure can be equivalent to monotherapy of
medicine. Contrarily, the weak point would be the low level of compliance or adherence
associated with necessary time for adequate action. Adequate changes in lifestyle would
be effective for some group of subjects. For grade 1 hypertensive patients, it can prevent
or delay medical therapy. Moreover, for hypertensive patients continuing on medical
therapy, it can contribute to BP reduction of blood pressure and allow reduction of the
number and doses of antihypertensive agents. Appropriate changes in lifestyle would
decrease other cardiovascular risk factors and improve several clinical conditions.
5. What does the table indicate?
A. Major significant differences were observed in 24-h, daytime and nighttime SBP or DBP
when using the conventional or custom-made pillow.
B. No significant differences were observed in 24-h, daytime and nighttime SBP or DBP
when using the conventional or custom-made pillow.
C. There are significant changes in Systolic Blood Pressure and Diastolic Blood Pressure.

Comparison of 24-h blood pressure monitoring between conventional and custom-made


pillows:
6. What does the report indicate?
A. Jugular venous engorgement and diffuse pulmonary rales were not found.
B. The electrocardiogram (ECG) showed broad R waves.
C. The patient was dyspnoeic with SaO2 74%.

Events
Initial presentation at the emergency room …………..
 3-h chest pain
 Respiratory failure and the need for advanced airway support
 Electrocardiogram with broad R waves, 4 mm ST-segment downsloping in right
precordial leads, right bundle branch block (RBBB), and ST-segment elevation in
posterior leads
 Coronary angiography with total thrombotic occlusion in the proximal segment of
the circumflex artery
 Bare metal stent was placed
 The patient developed cardiogenic shock and intra-aortic balloon pump was placed;
norepinephrine, vasopressin, and dobutamine were administered with
clinical improvement

48 h post-arrival
 Electrocardiogram with disturbance of repolarization only attributable to RBBB
 The patient developed acute renal failure and haemodialysis was initiated
 After 5 days.
 During a haemodialysis session, he developed sustained ventricular tachycardia
 with degeneration in asystole
 CPR was initiated, there wasn’t a return of spontaneous circulation
 Patient decease
READING SUB-TEST : PART C
In this part of the test, there are two texts about different aspects of healthcare.
For questions 7-22, choose the answer (A, B, C or D) which you think fits best according to the text.
Write your answers on the separate Answer Sheet

Text 1: Renal Artery Stenosis

Renal artery stenosis (narrowing) is a decrease in the diameter of the renal arteries. The
resulting restriction of blood flow to the kidneys may lead to impaired kidney function
(renal failure) and high blood pressure (hypertension), referred to as renovascular
hypertension, or RVHT ("reno" for kidney and "vascular" for blood vessel).
Renovascular hypertension is as likely to occur with bilateral stenosis (when arteries to
both kidneys are narrowed) as with unilateral stenosis (when the artery to one kidney is
narrowed). The decreased blood flow to the kidneys impairs renal function. Renal artery
stenosis may cause renal failure in some patients. There is no predictable relationship
between renal failure and renal artery stenosis. Some patients have very severe bilateral
stenosis and normal renal function. Most cases of renal failure are related to diabetes,
hypertension, glomerular sclerosis, contrast nephropathy, drug toxicity and other causes

The majority of renal artery stenosis is caused by atherosclerosis (hardening and


narrowing of blood vessel wall from the inside) similar to the process that occurs in blood
vessels in the heart and other parts of the body. Risk factors for atherosclerosis include
high cholesterol levels, high blood pressure, age, cigarette smoking, diabetes etc. Less
common causes of renal artery stenosis are fibromuscular dysplasia of the vessels
(narrowing of the vessel due to internal thickening of the blood vessel wall), arteritis
(inflammation of the blood vessel), or dissection (tearing and division of the blood vessel
wall).
Narrowing of the kidney arteries is more common in individuals 50 years of age and
older. It is estimated that some degree of narrowing (greater than 50%) is found in about
18% of adults between 65-75 years of age and 42% of those older than 75 years of age.
This may be due to the fact that atherosclerosis is more common in this age group. In
younger patients, the narrowing of the renal artery is usually due to the thickening of the
artery (fibromuscular dysplasia) and it is more common in women than in men. It is
estimated that renal artery stenosis accounts for approximately 1% of mild to moderate
cases of high blood pressure. It may be responsible for more than 10% of cases of
severely elevated or difficult to treat high blood pressure (hypertension).

In general, renal artery stenosis is not associated with any obvious or specific symptoms.
Suspicious signs for renal artery stenosis include high blood pressure that responds
poorly to treatment; severe high blood pressure that develops prior to age 30 or greater
than age 50; an incidental finding (discovered through routine tests or tests performed for
another condition) of one small kidney compared to a normal sized one on the other side.
Typically, unilateral (one-sided) renal artery stenosis may be related to high blood
pressure whereas bilateral (two-sided) renal artery stenosis is more often related to
diminished kidney function.

Several tests exist to detect any evidence of renal artery stenosis, which can be divided
into imaging tests and functional tests. The imaging tests provide a picture of the blood
vessel and its anatomy and reveal the degree of narrowing. The functional tests provide
information about whether the narrowing is significant enough to cause the high blood
pressure or kidney dysfunction. Each of these tests has advantages and disadvantages.
In bilateral (both-sided) and unilateral (one-sided) renal artery stenosis associated with
high blood pressure, controlling the blood pressure with usual blood pressure medications
is the first and the safest treatment. ACE inhibitors or ARB medications with or without a
diuretic (water pill) may be tried first. In some patients, this approach may be associated
with worsening of their kidney function. Therefore, kidney function needs to be followed
closely and if worsening of kidney function is evident, these medications may need to be
stopped. It is worth noting that if renal artery stenosis is found incidentally when
performing a test for another disease and there is no evidence of kidney dysfunction or
high blood pressure then no treatment may be necessary. Sometimes even significant
stenosis may not be associated with high blood pressure or kidney dysfunction. In these
situations, periodic monitoring of blood pressure and kidney function may be advised.
Text 1: Questions 7-14

7. Renovascular hypertension is likely to occur with;


A. Bilateral stenosis.
B. Unilateral stenosis.
C. Both bilateral and unilateral stenosis.
D. When arteries to one or both kidneys are narrowed.

8. Which one of these statements is true, according to paragraph 1?


A. The increase or decrease in blood flow leads to improper functioning of the kidneys
B. Renal failure and renal artery stenosis are closely connected to each other
C. A patient may have normal renal function even if there is higher bilateral stenosis
D. In some cases, bilateral stenosis may affect renal functions too.

9. Renal Artery Stenosis is caused by;


A. Hardening of the blood vessel wall from inside.
B. Hardening of the blood vessels similar to that of the blood vessels in the heart.
C. Narrowing and hardening of the walls like that of blood vessels in the heart. D
Hardening and narrowing of the blood vessels from inside.

10. Thickening of the arteries is more common among;


A. Men
B. Women
C. Children
D. All of the above

11. According to paragraph 4, which one of the following statements is true?


A. There are no specific symptoms of the renal artery stenosis.
B. Higher BP which develops before the age of 30 or after the age of 50 can become the cause of the
renal artery stenosis.
C. Untreatable high BP can be the cause of the renal artery stenosis.
D. Differences in sizes of the kidneys can be a major cause of the renal artery stenosis.
12. Functional tests provide;
A. A clear picture of the blood vessels and functions.
B. A clear idea of whether narrowing is significant to cause high BP or kidney dysfunction.
C. Anatomy of blood vessels.
D. A clear idea of the thickening of the blood vessels.

13. The best possible treatment for renal artery stenosis is;
A. Controlling the blood pressure.
B. Using ARB medications for the quality functioning of the kidneys.
C. Improving the function of the kidney through proper medications.
D. None

14. Renal artery stenosis is closely associated with;


A. Kidney dysfunction.
B. High blood pressure.
C. Low blood pressure.
D. A & B
Text 2: Hematochezia

Rectal bleeding (hematochezia) is used to describe the presence of blood with a bowel
movement. That blood, whether it fills the toilet bowl, or is a streak on the toilet paper
when wiping, or just a few drops in the toilet bowl, is not a normal finding and should not
be ignored. The source of bleeding can be anywhere in the digestive tract, from the nose
and mouth to the rectum and anus. The color can range from bright red to maroon to
black or any shade in between, depending on how much the blood has been exposed to the
digestive juices. Anytime there is blood within the gastrointestinal system, it will
eventually be excreted in stool (feces, bowel movement, BM). The color of the stool will
depend upon the amount of blood, the source of the bleeding and how quickly the stool
moves through the digestive tract. Sometimes, the bleeding is too little to be seen by the
naked eye but can be tested for by a healthcare professional.

Depending upon where and why the bleeding has taken place in the digestive tract, the
stool consistency and color may vary greatly: the stool color may be bright red, maroon,
dark red or black. The bleeding might be hidden, unseen to the naked eye, but able to be
detected by a fecal occult blood test. There may be blood just in the bowel movement or
there may be associated feces. If the feces are formed, the blood may be mixed in with the
stool or it may just coat the surface. The stool may be well-formed or it may lose and
diarrhea like. It may be normal in shape in size or it may become pencil thin. There may
be associated with abdominal pain or the bleeding may be painless.
Hemorrhoids are the most common cause of blood in the stool. Blood vessels located in
the walls of the rectum can swell, become inflamed and bleed. Hemorrhoids can be
caused by straining at stool, diarrhea, pregnancy, obesity and prolonged sitting on the
commode. All these factors increase the pressure within the hemorrhoidal vessels,
causing them to swell. The bleeding is often associated with anal burning or itching.
Bleeding can also occur because of an anal fissure, or a split in the skin of the anus. Hard
constipated stool may cause the skin to split; other causes include pregnancy and anal
intercourse. Anal fissures are also associated with other diseases including inflammatory
bowel disease (Crohn`s disease, ulcerative colitis), cancer and infections. Anal fissures
tend to be very painful, even when sitting. The blood in the stool can also be due to
swallowed blood from a nosebleed, dental work, or other mouth injuries that cause
bleeding.
Rectal bleeding is often diagnosed by history. The health care professional may ask
questions about the circumstances surrounding the rectal bleeding including the color, the
amount of bleeding, any associated symptoms and past medical history. A variety of
medications and food can mimic blood in the stool. Iron supplements and bismuth
(Pepto-Bismol, Kaopectate) can turn stool black, as can beets and licorice. Red food
coloring and beets can turn stool into a reddish hue. Patients who take blood thinners
(anticoagulation medications) are more prone to rectal bleeding. Examples of blood
thinners include warfarin (Coumadin), enoxaparin (Lovenox), aspirin and other
antiplatelet drugs including clopidogrel (Plavix), prasugrel and rivaroxaban (Xarelto).

Physical examination is important to assess the patient`s stability. Vital signs are
important and may include orthostatic vital signs, where the blood pressure and pulse rate
are taken both lying and standing. In a patient with reduced blood volume, the blood
pressure may fall, the pulse rate may rise, and the patient may become lightheaded and
weak when standing. Palpation of the abdomen is performed to look for tender areas,
masses or enlarged organs, especially the liver and spleen. Rectal examination is
performed by inserting a finger into the rectum, with the purpose of feeling for a mass or
other abnormality. The stool color and consistency may be examined when the finger is
withdrawn. The anus also may be examined. Blood tests may be considered if there is
concern about the amount of bleeding or other associated diseases. A complete blood
count (CBC) measures the number of red blood cells, white blood cells and platelets.
Blood clotting tests include PT (protime), INR (international normalized ratio) and PTT
(partial thromboplastin time). Depending upon the situation, other tests may be ordered to
measure electrolytes, and kidney and liver functions.

Most diseases which cause rectal bleeding are likely preventable, but it is not often
possible. Hemorrhoids can be avoided with proper diet and hydration to prevent
constipation and straining to pass stool, but normal pregnancy increases the risk of
hemorrhoid formation as does the patient with an acute diarrheal illness. Avoiding
constipation also decreases the risk of diverticulosis, outpouchings in the lining of the
colon, and the risk of a diverticular bleed but this may be a consequence of a Western
diet. Alcohol abuse increases the risk of rectal bleeding in a variety of ways, from
directly irritating the lining of the GI tract, to decreasing clotting capabilities of blood.
Text 2: Questions 15-22

15. Rectal bleeding describes;


A. Blood in the bowels.
B. Blood in the digestive tract.
C. Blood in the stools.
D. Blood in the rectum.

16. Paragraph 2 talks more about;


A. Which symptoms are associated with rectal bleeding.
B. Causes of the rectal bleeding.
C. Variations in the color of the stools.
D. None

17. Blood in the stool can originate anywhere in the gastrointestinal tract.
A. False
B. True
C. Not given
D. Sometimes true and sometimes false

18. Hemorrhoids can be well defined by which one of the following?


A. Swelling of the rectal walls.
B. Inflammation and bleeding of the rectal walls.
C. Straining
D. All of the above

19. Causes of anal fissure may include;


A. Formation of the constipated stool
B. Inflammatory bowel disease
C. Cancer and infections
D. All of the above
20. According to paragraph 4, which of the following statements is true?
A. Blood thinners can cause rectal bleeding.
B. Blood thinners may sometimes cause rectal bleeding.
C. It is not fully established that rectal bleeding is the direct result of the use of blood thinners.
D. None

21. Which one of the following statements is not included in paragraph 5?


A. Physical examiners will always look for orthostatic vital signs.
B. A patient may have low blood pressure with a higher pulse rate.
C. Palpitations of the abdomen is performed to look for tender areas.
D. Rectal bleeding is identified through taking a stool sample.

22. Which one of these is the preferred blood clotting test?


A. PT
B. INR
C. PTT
D. Not given

END OF READING TEST, THIS BOOKLET WILL BE COLLECTED


Reading test 27 : Answer Key

Part A - Answer key 1 – 7


1:B
2:A
3:C
4:A
5:B
6:D
7:C

Part A - Answer key 8 – 14


8:allele frequency
9: Electrophoresis
10: In silico
11: Phosphodiester bonds
12: Aplastic anaemia
13: Sanger sequencing
14: NGS

Part A - Answer key 15 – 20


15: enzymopathies
16: hematologic malignancies
17: GRCh38/hg38 version
18: dideoxynucleotide triphosphates
19: Genome Wide sequence
20: first-line approach
Reading test - part B – answer key

1: C Most common in tropical areas but can also occur in temperate zones.
2: B CAR-T is considered more effective.
3: C may result in involuntary movements.
4: A For subjects with normal or subnormal hypertension, it can prevent or delay hypertension.
5: A Major significant differences were observed in 24-h, daytime and
nighttime SBP or DBP when using the conventional or custom-made pillow.
6: B The electrocardiogram (ECG) showed broad R waves.

Reading test - part C – answer key

Text 1 - Answer key 7 – 14

7: A Bilateral stenosis.
8: C A patient may have normal renal function even if there is higher bilateral stenosis
9: D Hardening and narrowing of the blood vessels from inside.
10: B Women
11: C Untreatable high BP can be the cause of the renal artery stenosis.
12: B A clear idea of whether narrowing is significant to cause high BP or kidney dysfunction.
13: A Controlling the blood pressure.
14: D A & B

Text 2 - Answer key 15 – 22

15: C Blood in the stools.


16: A Which symptoms are associated with rectal bleeding.
17: B True
18: D All of the above
19: D All of the above
20: A Blood thinners can cause rectal bleeding.
21: D Rectal bleeding is identified through taking a stool sample.
22: D Not given
READING TEST 28
READING SUB-TEST : PART A
 Look at the four texts, A-D, in the separate Text Booklet.
 For each question, 1-20, look through the texts, A-D, to find the relevant information.
 Write your answers on the spaces provided in this Question Paper.
 Answer all the questions within the 15-minute time limit.
 Your answers should be correctly spelt.

TEXT BOOKLET- CHRONIC FATIGUE SYNDROME (CFS)

Text A

Chronic fatigue syndrome (CFS)


Chronic fatigue syndrome (CFS) is a highly complex illness that results in
significant disability and a considerably diminished quality of life. Due to
continued questions regarding etiology, the period of onset for the illness is of
considerable interest to researchers in the field. There is fairly little strife as to
whether the illness labels CFS, ME, ME/CFS, myalgic encephalopathy, and SEID
represent one distinct condition, whether they are part of an illness spectrum, or
whether they are simply different terms used to describe the same condition. CFS,
ME, ME/CFS, and the recently named SEID, are often associated with different
case criteria. Early case criteria developed by Holmes. specify that the illness must
have a “new onset of persistent or relapsing, debilitating fatigue” without any
previous history of similar problems. Additionally, Holmes stipulate that the main
symptoms of CFS must occur over a few hours or days, indicating a sudden or
acute onset. According to Holmes, symptoms are only met if they begin at the time
of the fatigue onset or following onset. Another case criteria for CFS is referred to
as the Oxford Criteria, which stipulate that CFS involves a “definite” onset as well
as clear evidence of infection at the time of onset or first symptoms. Similar to the
Oxford Criteria, the Fukuda criteria describe the onset of the fatiguing illness as
“new” and “definite”.
Text B

The terms Myalgic Encephalomyelitis (ME) and Myalgic


Encephalomyelitis/chronic fatigue syndrome (ME/CFS) have corresponding case
criteria, which are different from the Fukuda criteria in that they require what are
considered by many to be key symptoms of the illness (e.g. post-exertional malaise
and cognitive dysfunction). The criteria for ME/CFS specify that an individual
must have a “significant degree of new onset” fatigue. Similar to the Holmes
criteria for CFS, the case criteria for ME/CFS stipulate that symptoms can only be
counted as meeting criteria if they occur or become significantly worse after the
onset of the illness. Carruthers et al. describe onset as “distinct” and assert that
most individuals experience an acute onset; however, they also assert that some
individuals are unhealthy prior to their ME/CFS onset and may not be able to
identify a specific trigger for the development of ME/CFS, or they may experience
a more “gradual” or “insidious” onset.
Text C

Hyde’s Nightingale Definition of ME stipulates that ME is both chronic and


disabling and is characterized by an acute onset. Additionally, Hyde describes ME
as an epidemic or an endemic occurring in two phases. Additionally, Hyde
indicates that ME often follows multiple, minor infections in individuals with
susceptible immune systems or immune systems that are weakened by severe
stressors. Hyde describes the initial phase of ME as the Primary Infection Phase,
which is characterized as an epidemic or endemic infectious disease with an
incubation period of between four and seven days. He describes the second phase
as the Secondary Chronic Phase, occurring with two to seven days of the Primary
Infection Phase. In this phase, Hyde asserts that there are measurable changes in
the central nervous system (CNS) of an affected individual and that this phase is
the chronic form of the disease that is most commonly depictive of ME.
Understanding the cause of ME can be like going through labyrinthine routes in
scientific discovery, as Hyde asserts that all cases of epidemic and primary ME
result from an infectious or autoimmune agent, but he also suggests that there are
often other potential causes that may go unnoticed prior to the onset of illness or as
part of the illness.
Text D

There is not one universally used or empirically derived definition of onset


duration for ME and CFS. Researchers interested in assessing the mode of illness
onset have used various definitions. Often, the distinction is made between sudden
and acute onset and insidious and gradual onset, but the duration length prescribed
to each category differs. For instance, DeLuca et al. defined sudden onset for CFS
as an unrecoverable “viral-like illness” that could be traced to a definite date.
Gradual onset was described as a “slow progression of symptoms over a period of
weeks to several months” or longer. In his study of CFS in an adolescent sample,
Bell defined sudden/acute onset as an “abrupt onset of constant and debilitating
fatigue that could be dated to a specific event or illness” . He described all other
onset patterns as gradual.
PART A -QUESTIONS AND ANSWER SHEET

Questions 1-7
For each question, 1-7, decide which text (A, B, C or D) the information comes from.
You may use any letter more than once.
In which text can you find information about;
1 .Occurs in two different phases. Answer ________
2. All patterns are gradual. Answer_________
3. He believed that ME is more complicated. Answer___________
4. Controversy over the disease conditions. Answer__________
5 .No clear definition. Answer__________
6. Theory which suggests beginning of body weakening. Answer __________
7 .Causes a spike in symptoms and a massive energy crash. Answer___________

Questions 8-14
Answer each of the questions, 8-14, with a word or short phrase from one of the
texts. Each answer may include words, numbers or both. Your answers should be correctly spelt.

8. Who thinks that ME is long lasting and ant it can be acquired extreme onset? Answer______
9 .Who defined onset as incurable? Answer_______
10. According to which principle, CFS involves a definite onset & clear evidence of infection?
Answer_____________
11. Who specified that the CFS symptoms must occur over a few hours or days, indicating an acute
onset? Answer ____________
12 .Who described that sudden onset for CFS as an unrecoverable viral-like illness? Answer__
13 .Whose statement signifies that the affected patient may go through a highly secretive and
detrimental beginning? Answer ___________
14 .Who described onset as more debilitating fatigue? Answer___________
Questions 15-20

Complete each of the sentences, 15-20, with a word or short phrase from one of the
texts. Each answer may include words, numbers or both. Your answers should be correctly spelt.

15 .____________ described the onset phase of ME as the Primary Infection

16. ME may follow ___________ infections in patients with not-so-strong immune system

17 .CFS criteria are said to have much more similarity with respect to____________

18. CFS may comprise a___________ onset and strong evidence of infection when there are first
signs of symptoms.

19. The distinction is made between _____________ and insidious/gradual onset, but the duration
length prescribed to each category differs.

20.____________ may occur due to an infectious or autoimmune agent.

END OF PART A, THIS QUESTIONS PAPER WILL BE COLLECTED


READING SUB-TEST : PART B

In this part of the test, there are six short extracts relating to the work of health professionals .
For questions 1-6, choose the answer (A, B or C) which you think fits best according to the text.
Write your answers on the separate Answer Sheet

Questions 1-6

1 What is correct about gait disturbance?


A. Pelvic tilt is common
B. Fast gait speed and improved gait pattern is the most important goal of stroke
rehabilitation
C. A hemiplegic gait may include body asymmetry, decreased weight bearing on
the affected side.

Gait disturbance

Gait disturbance is the most common problem after stroke. This problem is related
to poor ADL and mobility, and increases the risk of fall in severe cases. The body
alignment of stroke patients becomes asymmetric if they have a hemi-paralysis,
muscle weakness, motor and sensory function decrease. These problems produce a
hemiplegic gait in stroke patients. It may include poor equilibrium reaction, and
impaired selective motor control. Good body alignment is very important clinically
because asymmetry leads to inefficient energy during walking, the risk of
musculoskeletal injury in the unaffected side, and loss of bone density. Excessive
pelvic elevation and the pelvic tilt angle is directly connected to hemiplegic gait
and poor motor function in stroke patient causes an excessive pelvic tilt during gait
2. The notice is talking about;
A. Postoperative care.
B. functions of IV insulin
C. insulin dosage

IV Insulin Infusions
Patients that have required IV insulin infusions can be transitioned to subcutaneous
insulin once infusion rates are stable and glucose controlled, particularly if a diet has
been initiated. Because IV insulin has a very short half-life, the subcutaneous insulin
should be administered prior to discontinuation of IV insulin. The basal infusion rate
during fasting is a good predictor of basal subcutaneous insulin requirements, however,
providers often reduce the amount by 20% upon transition. For instance, if a patient
required 1.5 unit/hour of IV insulin overnight, this would suggest a basal need of
approximately 36 units of insulin daily. However, reducing this by 20% would result in a
starting basal dose of 30 units of insulin daily.

3. The use of molecule-altering technologies;


A. has flared up more concerns.
B. is not suitable.
C. is fast advancing.

Molecule-altering technologies
The advent of molecule-altering technologies and improved synthetic methods has
led to the finding of newer proteins and peptides that resemble human proteins and
peptides. Although, capable of producing potential therapeutic benefits, protein
molecules have serious biopharmaceutical concerns such as, poor shelf- life, rapid
degradation in the physiological environment, poor solubility, immunogenicity and
antigenicity. These concerns can be overcome by utilizing the beneficial properties of
polyethylene glycols and PEGylation. ‘PEGylation’ is the process of chemical attachment
of PEG to bioactive proteins and peptides, to modify their pharmacokinetic and
pharmacodynamic properties.
4. The notice is talking about;
A. when to perform the exercise
B. best exercise timing
C. benefits of exercise

Moderate mid-postprandial exercise

Although an uneasy consensus favoring moderate mid-postprandial exercise as


better suited for glycaemia benefits exists among researchers, pre-meal exercise
has its advantages: little risk for hypoglycemia, enhanced insulin sensitivity, and
improved muscle glycogen content and GLUT-4 protein levels. The downside of
pre-meal exercise is elevated postprandial glucose (PPG), A recent systematic
review concluded that >45 min of aerobic exercise (AE) post-meal offered
consistent glycaemia benefits. Also, the review identified resistance training as an
effective modality in this regard. The patient found a 30-min pre-meal walk
followed by another 30 minutes of post-meal walk blunted the post-meal glucose
surge better than a 60 min pre-meal or post-meal walk. The post-meal walk might
have cleared some of the extra blood glucose arrived from the liver. Split exercise
at lunchtime was comparable to mid-postprandial exercise in improving glycaemia
and oxidative stress, although there was less hyperglycemia after the meal.
5. Zidovudine can;
A. Increase decrease of lactate.
B. Lead to fatty change
C. Be used for treating anemia

Zidovudine

Nausea is another common side effect that may be present in early zidovudine use.
Other side effects of zidovudine may include granulocytopenia, myopathy, lactic
acidosis, hepatomegaly with steatosis, headache. Myopathy may occur within 6-12
months of initiating zidovudine, and has an insidious onset that involves proximal
muscle weakness and exercise-induced myalgias. The mechanism of myopathy is
believed to be mitochondrial toxicity within myocytes. Zidovudine should be used
with caution in patients who have anemia (hemoglobin less than 9.5
grams/deciliter). Reduction of hemoglobin may occur as early as 2 to 4 weeks.
Severe anemia may require dose adjustment, discontinuation, and/or blood
transfusions. Doses should be reduced until bone marrow recovers if the anemia is
significant (hemoglobin less than 7.5 grams/deciliter or reduction of greater than
25% of baseline).
6. The table
A. doesnt show a much significant difference in gentamicin doses between two groups
of patients
B. Use of gentamicin dose is significantly lower
C. Dose of 2 mg/kg is higher in noncritically ill patients.

Amikacin and gentamicin dosing in critically and noncritically ill patients


READING SUB-TEST : PART C
In this part of the test, there are two texts about different aspects of healthcare.
For questions 7-22, choose the answer (A, B, C or D) which you think fits best according to the text.
Write your answers on the separate Answer Sheet

Text 1: What is Yersiniosis?

Yersiniosis is an infectious disease caused by a bacterium of the genus Yersinia. In


the United States, most human illness is caused by one species, Y enterocolitica
(not more than one species often). Infection with Y. enterocolitica can cause a
variety of symptoms depending on the age of the person infected. Infection with Y.
enterocolitica occurs most often in young children; adults may be on a safer side.
Common symptoms in children are fever, abdominal pain, and diarrhea, which is
often bloody. Symptoms typically develop 4 to 7 days after exposure and may last
1 to 3 weeks or longer. In older children and adults, right-sided abdominal pain and
fever may be the predominant symptoms, and may be confused with appendicitis. In
a small proportion of cases, complications such as skin rashes, joint pains, or the
spread of bacteria to the bloodstream can occur.

Y. enterocolitica belongs to a family of rod-shaped bacteria. Other species of


bacteria in this family include Y. pseudotuberculosis, which causes an illness
similar to Y. enterocolitica, and Y. pestis, which cause plague. Only a few strains of
Y. enterocolitica cause illness in humans. The major animal reservoir for Y.
enterocolitica strains that causes human illness is pigs, but other strains are also
found in many other animals including rodents, rabbits, sheep, cattle, horses, dogs,
and cats. In pigs, the bacteria are most likely to be found on the tonsils.

Infection is most often acquired by eating contaminated food, especially raw or


undercooked pork products. The preparation of raw pork intestines (chitterlings)
may be particularly risky. Infants can be infected if their caretakers handle raw
chitterlings and then do not adequately clean their hands before handling the infant
or the infant’s toys, bottles, or pacifiers. Drinking contaminated unpasteurized milk
or untreated water can also transmit the infection. Occasionally Y. enterocolitica
infection occurs after contact with infected animals. On rare occasions, it can be
transmitted as a result of the bacterium passing from the stools or soiled fingers of
one person to the mouth of another person. This may happen when basic hygiene
and handwashing habits are inadequate. Rarely, is the organism transmitted
through contaminated blood during a transfusion.

Y. enterocolitica is a relatively infrequent cause of diarrhea and abdominal pain.


Based on data from the Food-borne Diseases Active Surveillance Network which
measures the burden and sources of specific diseases over time, approximately one
culture-confirmed Y. enterocolitica infection per 100,000 people occurs each year.
Children are infected more often than adults, and the infection is more common in
the winter. Y. enterocolitica infections are generally diagnosed by detecting the
organism in stools. Many laboratories do not routinely test for Y. enterocolitica, so
it is important to notify laboratory personnel when infection with this bacterium is
suspected so that special tests can be done. The organism can also be recovered
from other sites, including the throat, lymph nodes, joint fluid, urine, bile, and
blood. Uncomplicated cases of diarrhea due to Y. enterocolitica usually resolve on
their own without antibiotic treatment. However, in more severe or complicated
infections, antibiotics such as aminoglycosides, doxycycline,
trimethoprimsulfamethoxazole, or fluoroquinolones may be useful.

There are many things which can be done to prevent the infection or the spread of
the infection: Avoid eating raw or undercooked pork. Consume only pasteurized milk or
milk products. Wash hands with soap and water before eating and preparing food, after
contact with animals, and after handling raw meat. After handling raw chitterlings,
clean hands and fingernails scrupulously with soap and water before touching
infants or their toys, bottles, or pacifiers. Someone other than the food handler
should care for children while chitterlings are being prepared. Prevent
crosscontamination in the kitchen - use separate cutting boards for meat and other foods,
carefully clean all cutting boards, counter-tops, and utensils with soap and hot
water after preparing raw meat. Dispose of animal feces in a sanitary manner.
Text 1: Questions 7-14

7. Yersiniosis occurs more commonly in;


A. Children
B. Americans
C. Adults
D. Teens

8. Symptoms such as fever and diarrhea may persist for about;


A. 4 days
B. 7 days
C. 1-3 weeks
D. More than 3 weeks

9. Plague is caused by;


A. Y. pestis
B. Y. enterocolitica
C. Y. pseudotuberculosis
D. None

10. One of the following is not a common form of transmission of the parasite;
A. Contaminated unpasteurized milk
B. Blood transfusion
C. Raw or undercooked pork products
D. After contact with infected animals

11. Paragraph 4 talks about;


A. Prevention of the disease
B. Spread of the infection
C. How common the infection of Y. enterocolitica is
D. Reports by Active Surveillance Network.
12. Paragraph 5 talks about;
A. Diagnosis
B. Treatment
C. Spread of the infection and its control.
D. A & B

13. Which of these can be derived from Paragraph 5?


A. Treatment is not required for the infection caused by Y. enterocolitica.
B. Diarrhea requires no treatment
C. Complications which are mild can be resolved easily.
D. None

14. What is said about chitterlings?


A. Handling raw chitterlings with care is necessary
B. Contamination occurs more due to the careless handling chitterlings
C. Washing hands with soap before touching infants or their toys is vital
D. There should be separate caretakers to look after infants, while chitterlings are prepared
Text 2 : What is an MRI scan?

An MRI (or magnetic resonance imaging) scan is a radiology technique that uses
magnetism, radio waves, and a computer to produce images of body structures.
The MRI scanner is a tube surrounded by a giant circular magnet. The patient is
placed on a moveable bed that is inserted into the magnet. The magnet creates a
strong magnetic field that aligns the protons of hydrogen atoms, which are then
exposed to a beam of radio waves. This spins the various protons of the body, and
they produce a faint signal that is detected by the receiver portion of the MRI
scanner. The receiver information is processed by a computer, and an image is
produced. The image and resolution produced by MRI are quite detailed and can
detect tiny changes of structures within the body. For some procedures, contrast
agents, such as gadolinium, are used to increase the accuracy of the images.

An MRI scan can be used as an extremely accurate method of disease detection


throughout the body. In the head, trauma to the brain can be seen as bleeding or
swelling. Other abnormalities often found include brain aneurysms, strokes, tumors
of the brain, as well as tumors or inflammation of the spine. Neurosurgeons use an
MRI scan not only in defining brain anatomy but in evaluating the integrity of the
spinal cord after trauma. It is also used when considering problems associated with
the vertebrae or intervertebral discs of the spine. An MRI scan can evaluate the
structure of the heart and aorta, where it can detect aneurysms or tears. It provides
valuable information on glands and organs within the abdomen, and accurate
information about the structure of the joints, soft tissues, and bones of the body.
Often, surgery can be deferred or more accurately directed after knowing the
results of an MRI scan.
An MRI scan is a painless radiology technique that has the advantage of avoiding
x-ray radiation exposure. There are no known side effects of an MRI scan. The
benefits of an MRI scan relate to its precise accuracy in detecting structural
abnormalities of the body. Patients who have any metallic materials within the
body must notify their physician prior to the examination or inform the MRI staff;
metallic chips, materials, surgical clips, or foreign material can significantly distort
the images obtained by the MRI scanner. Patients who have heart pacemakers,
metal implants, or metal chips or clips in or around the eyeballs cannot be scanned
with an MRI because of the risk that the magnet may move the metal in these
areas. Similarly, patients with artificial heart valves, metallic ear implants, bullet
fragments, and chemotherapy or insulin pumps should not undergo MRI scanning.
During the MRI scan, the patient lies in a closed area inside the magnetic tube;
some patients can experience a claustrophobic sensation during the procedure.
Therefore, patients with any history of claustrophobia should relate this to the
practitioner who is requesting the test, as well as the radiology staff. A mild
sedative can be given prior to the MRI scan to help alleviate this feeling.

All metallic objects on the body are removed prior to obtaining an MRI scan.
Occasionally, patients will be given a sedative medication to decrease anxiety and
relax the patient during the MRI scan. MRI scanning requires that the patient lies
still for best accuracy; patients lie within a closed environment inside the magnetic
machine. Relaxation is important during the procedure and patients are asked to
breathe normally. Interaction with the MRI technologist is maintained throughout
the test and there are loud, repetitive clicking noises which occur during the test as
the scanning proceeds. Occasionally, patients require injections of liquid
intravenously to enhance the images which are obtained. The MRI scanning time
depends on the exact area of the body studied, but ranges from half an hour to an
hour and a half.
After the MRI scanning is completed, the computer generates visual images of the
area of the body that was scanned. These images can be transferred to film (hard
copy). A radiologist is a physician who is specially trained to interpret images of
the body. The interpretation is transmitted in the form of a report to the practitioner
who requested the MRI scan. The practitioner can then discuss the results with the
patient and/or family.
Text 2: Questions 15-22

15. According to paragraph 1, an image is produced;


A. When a signal is detected by the receiver portion of the scanner.
B. When the signal passes through the receiver, after coupling of the protons of the body.
C. When the signal is identified by the receiver which is accurately processed by a computer.
D. Only when the information, detected by the receiver is processed by a computer.

16. According to paragraph 1, what is the image like?


A. Crystal clear with no patches.
B. Self explanatory
C. Dense
D. Well enough to give a clear idea of the structures within the body.

17. According to paragraph 2, an MRI gives a clear idea about;


A. Brain aneurysms.
B. Strokes of the brain.
C. Brain tumours and spinal cord injury.
D. All

18. An MRI can;


A. Detect brain tumors.
B. Give a clear picture of spinal cord injury and other soft tissues related to it.
C. Give clear pictures to evaluate the structure of the heart and aorta.
D. All

19. Paragraph 3 talks about;


A. Detection of the diseases through an MRI scan.
B. Importance of taking an MRI scan.
C. Risks of an MRI scan.
D. None
20. Metallic materials mentioned in paragraph 3 include;
A. Surgical clips.
B. Artificial joints, metallic bone plates.
C. Only metal clips in or around the eyeballs.
D. A & B

21 .Paragraph 4 talks about;


A. How an MRI is performed.
B. How a patient prepares for an MRI scan.
C. How a patient prepares for an MRI scan and how it is performed.
D. How an MRI scan is different from others.

22. According to paragraph 5, interpreting the images implies;


A. Identifying the disease through scanning.
B. Analysis of the disease.
C. Decoding the report.
D. All of the above.

END OF READING TEST, THIS BOOKLET WILL BE COLLECTED


Reading test 28 : Answer Key

Part A - Answer key 1 – 7


1: C
2:D
3:C
4:A
5:D
6:A
7:B

Part A - Answer key 8 – 14


8: Hyde
9: DeLuca
10: Oxford Criteria
11: Holmes
12: DeLuca et al.
13: Carruthers
14: Bell

Part A - Answer key 15 – 20


15: Hyde
16: Multiple Minor
17: Fukuda Criteria
18: Sudden
19: Hydes
20: Primary Me
Reading test - part B – answer key

1: A hemiplegic gait may include body asymmetry, decreased weight bearing on the affected side.
2: Postoperative care.
3: has flared up more concerns.
4: best exercise timing
5: Lead to fatty change
6: Use of gentamicin dose is significantly lower

Reading test - part C – answer key

Text 1 - Answer key 7 – 14

7: Children
8: 1-3 weeks
9 : Y. pestis
10: Blood transfusion
11: How common the infection of Y. enterocolitica is
12: A & B
13: None
14: Washing hands with soap before touching infants or their toys is vital

Text 2 - Answer key 15 – 22

15: Only when the information, detected by the receiver is processed by a computer.
16 : Dense
17: All
18: All
19: Risks of an MRI scan.
20: A & B
21: How a patient prepares for an MRI scan and how it is performed.
22: All of the above.
READING TEST 29
READING SUB-TEST : PART A
 Look at the four texts, A-D, in the separate Text Booklet.
 For each question, 1-20, look through the texts, A-D, to find the relevant information.
 Write your answers on the spaces provided in this Question Paper.
 Answer all the questions within the 15-minute time limit.
 Your answers should be correctly spelt.

PART A -TEXT BOOKLET - DEEP VEIN THROMBOSIS

Text A

Deep vein thrombosis is a part of a condition called venous thromboembolism.


Deep vein thrombosis occurs when a blood clot (thrombus) forms in one or more
of the deep veins in the body, usually in the legs.
Deep vein thrombosis can cause leg pain or swelling, but may occur without any
symptoms. Deep vein thrombosis is a serious condition because blood clots in the veins
can break loose, travel through the bloodstream, and obstruct the lungs, blocking blood
flow.
Text B

Clinical Manifestations
A major problem associated with recognizing DVT is that the signs and symptoms are
nonspecific.
Edema: With obstruction of the deep veins comes edema and swelling of the extremity
because the outflow of venous blood is inhibited Phlegmasiaceruleadolens: Also called
massive iliofemoral venous thrombosis, the entire extremity becomes massively swollen,
tense, painful, and cool to the touch.Tenderness: Tenderness, which usually occurs later, is
produced by inflammation of the vein wall and can be detected by gently palpating the
affected extremity.
Pulmonary embolus: In some cases, signs and symptoms of a pulmonary embolus are the
first indication of DVT

Medical Management
The objectives for treatment of DVT are to prevent thrombus from growing and
fragmenting, recurrent thromboemboli, and post thrombotic syndrome.
Endovascular management; Endovascular management is necessary for DVT when
anticoagulant or thrombolytic therapy is contraindicated, the danger of pulmonary
embolism is extreme, or venous drainage is so severely compromised that permanent
damage to the extremity is likely.
Vena cava filter: A vena cava filter may be placed at the time of thrombectomy;
this filter traps late emboli and prevents pulmonary emboli.
Discharge and Home
Care Guidelines The nurse must also promote discharge and home care to the patient.
Text C

Heparin (Rx)
Strengt Route of Recommende FIRST PTT
Drug
h Administration d dosage CHECK
80 units/kg IV bolus,
THEN continuous
infusion of 18
units/kg/hr,
1unit/mL OR
2units/mL 5000 units IV bolus, 6 hours after
10units/mL Sc/IV THEN continuous starting
100units/m infusion of 1300 infusion
L units/hr, OR
250 units/kg
(alternatively, 17,500
units) SC, THEN 250
units/kg q12hr

Drug education: The nurse should teach about the prescribed anticoagulant, its
purpose, and the need to take the correct amount at the specific times prescribed.
Blood tests: The patient should be aware that periodic blood tests are necessary to
determine if a change in medication or dosage is required.
Avoid alcohol: A person who refuses to discontinue the use of alcohol should not
receive anticoagulants because chronic alcohol intake decreases their effectiveness.
Activity: Explain the importance of elevating the legs and exercising adequately.
Text D

Nursing Care Planning & Goals


The major goals for the patient include:
Demonstrate increased perfusion as individually appropriate.
Verbalize understanding of condition, therapy, regimen, side effects of
medications, and when to contact the healthcare provider.
Engage in behaviors or lifestyle changes to increase level of ease.
Verbalize sense of comfort or contentment.
Maintain position of function and skin integrity as evidenced by absence of
contractures, foot drop, decubitus, and so forth.
Maintain or increase strength and function of affected and/or compensatory body part.

Nursing Interventions
The major nursing interventions that the nurse should observe are:
Provide comfort; Elevation of the affected extremity, graduated compression
stockings, warm application, and ambulation are adjuncts to the therapy that can
remove or reduce discomfort.

Compression therapy: Graduated compression stockings reduce the caliber of the


superficial veins in the leg and increase flow in the deep veins; external
compression devices and wraps are short stretch elastic wraps that are applied from
the toes to the knees in a 50% spiral overlap; intermittent pneumatic compression
devices increase blood velocity beyond that produced by the stockings.

Positioning and exercise: When patient is on bed rest, the feet and lower legs
should be elevated periodically above the level of the heart, and active and passive
leg exercises should be performed to increase venous flow.
PART A -QUESTIONS AND ANSWER SHEET

Questions 1-7
For each of the questions, 1-7, decide which text (A, B, C or D) the information comes from. You
may use any letter more than once

In which text can you find information about?


1. Endovascular management is for DVT …………………
2. Outflow of venous blood causing extreme swelling ………………….
3. The person receiving anticoagulants should avoid alcohol.…………………
4. Deep vein thrombosis is asymptomatic sometimes …………………
5. Compression therapy reduces the caliber of the superficial veins in the leg…………………
6. The importance of keeping legs elevated …………………
7. The nursing interventions …………………

Questions 8-14

Answer each questions, 8-4, with a word or short phrase from one of the texts.
Each answer may include words, number or the both. Your answers should be correctly spelled.

8. What is the route of administration of heparin? ………………………………….


9. In which part of the body DVT normally occurs? ……………………………………..
10. When vena cava filler is used?……………………………..
11. Which management is preferred for DVT if anticoagulant or thrombolytic therapy is
contraindicated?………………………………
12. Which is the term used to describe the massive iliofemoral venous thrombosis?………………
13. Which symptom usually occurs late in DVT? ………………………………
14. DVT is a part of a condition called? …………………………………
Questions 15-20

Complete each of the sentences, 15- 20, with a word or short phrase from one of the texts. Each
answer may include words, number or both. Your answers should be correctly spelled

15. A major problem is associated with recognizing DVT is that the signs and symptoms are……

16.Tenderness which usually occurs later is produced by……………….of the vein wall

17.In some cases, signs and symptoms of a…………are the first indication.

18.First PPT check is……………… hours after starting infusion.

19.5000 units IV bolus, then continued infusion of………units/hr.

20.The patient should be aware of periodic blood tests which are necessary to determine if a change
in…………………or dosage is required.

END OF PART A, THIS QUESTIONS PAPER WILL BE COLLECTED


READING SUB-TEST : PART B
In this part of the test, there are six short extracts relating to the work of health professionals .
For questions 1-6, choose the answer (A, B or C) which you think fits best according to the text. Write
your answers on the separate Answer Sheet

Questions 1-6

1. The manual informs us that;


a. PMDD is used to prevent the incidence of assisted suicide and euthanasia
b. PMDD is a protective durable power of attorney for International Task Force on euthanasia
and assisted suicide
c. In PMDD the signer names a trusted person to make health care decisions

PROTECTIVE MEDICAL DECISIONS DOCUMENT (PMDD)

The PMDD is a protective Durable Power of Attorney for Health Care which is available from
the International Task Force on Euthanasia and Assisted Suicide. In the PMDD the signer
names a trusted person to make health care decisions in the event that the signer is
temporarily or permanently unable to make such decisions.
The PMDD, which specifically prohibits assisted suicide and euthanasia, is available in a Multi-
State version for use in most states. It is also available in statespecific versions for states
where particular requirements make a state-specific version necessary.
2. The paragraph is giving information about;
a. Types of bronchodilators
b. Uses of bronchodilators
c. Definition of bronchodilator

Bronchodilator
A bronchodilator is a substance that dilates the bronchi and bronchioles, decreasing
resistance in the respiratory airway and increasing airflow to the lungs.
Bronchodilators may be endogenous (originating naturally within the body), or they may be
medications administered for the treatment of breathing difficulties.
They are most useful in obstructive lung diseases, of which asthma and chronic obstructive
pulmonary disease are the most common conditions. Although this remains somewhat
controversial, they might be useful in bronchiolitis and bronchiectasis. They are often
prescribed but of unproven significance in restrictive lung diseases.

3.Use of ICD is;


a. Delivers an electric shock .
b. Prevents cardiac arrest in high risk patients.
c. It is a battery powered device

Implantable Cardioverter Defibrillator (ICD)?


An ICD is a battery-powered device placed under the skin that keeps track of your heart rate.
Thin wires connect the ICD to your heart. If an abnormal heart rhythm is detected the device
will deliver an electric shock to restore a normal heartbeat if your heart is beating chaotically
and much too fast. ICDs have been very useful in preventing sudden death in patients with
known, sustained ventricular tachycardia or fibrillation. Studies have shown that they may
have a role in preventing cardiac arrest in high-risk patients who haven't had, but are at risk
for, life-threatening ventricular arrhythmias.
4. Overhead radiant warmers
a. Minimize the oxygen and calories of infant
b. Is used to all neonates and infants
c. Provide neutral thermal environment for the patient

Monitoring Temperature Using an Overhead Radiant Warmer

Neonates, infants who are exposed to stressors or chilling (e.g., from undergoing numerous
procedures), and infants who have an underlying condition that interferes with
thermoregulation (e.g., prematurity) are highly susceptible to heat loss. Therefore, radiant
warmers are used for infants who have trouble maintaining body temperature. In addition,
use of a radiant warmer minimizes the oxygen and calories that the infant would expend to
maintain body temperature, thereby minimizing the effects of body temperature changes on
metabolic activity. An overhead radiant warmer warms the air to provide a neutral thermal
environment, one that is neither too warm nor too cool for the patient. The incubator
temperature is adjusted to maintain and anterior abdominal skin temperature of 36.5C
(97.7F), but at least 36C (96.8F), using servocontrol (automatic thermostat)
5. What should be done if the patient is not able to maintain flat position till the end of
Doppler ultrasound?
a. Elevate the legs of patient
b. Provide flat position according to patients comfort and document the position given
c. Educate the patient regarding the importance of maintaining flat position throughout the
procedure

To ALL staff
Subj: PROCEDURE FOR VASCULAR ASSESSMENT BY DOPPLER ULTRASOUND
The procedure should be explained to the patient and informed and understood
consent gained. Although it is not invasive it can be uncomfortable and for some
painful because the blood pressure cuff may squeeze the leg over existing
ulceration and/or oedema. Patients need to know what to expect so they can stop
the nurse from continuing should the pain become unbearable. This information
and patient’s comments on the procedure must be recorded in the patient health
record. Before carrying out the procedure the patient should rest for 10 to 20
minutes (Carter 1969 et al). The emphasis is upon obtaining the resting systolic
pressure. Time should be allowed within the nursing schedule for the patient to be
rested. The patient should also lie flat in order to minimize hydrostatic pressure
variables (Vowden and Vowden 2001). However, many patients will not be able to
lie flat and for some having their legs elevated is difficult e.g. in the case of
patients with breathing problems or arthritis. In these cases lie the patients as flat as
comfortably tolerated and/or with legs elevated as much as possible. The patient’s position
should be documented. This will contribute to consistency for future
readings and put the ABPI within a context which relates to patient positioning.
6.Antibiotics
a. Are used to treat viral infections.
b. Act by killing bacteria.
c. Are used to treat common cold or influenza.

Antibiotic

An antibiotic (from ancient Greek αντιβιοτικά antibiotiká), also called an


antibacterial, is a type of antimicrobial drug used in the treatment and prevention
of bacterial infections. They may either kill or inhibit the growth of bacteria. A
limited number of antibiotics also possess antiprotozoal activity. Antibiotics are
not effective against viruses such as the common cold or influenza; drugs which
inhibit viruses are termed antiviral drugs or antivirals rather than antibiotics.
READING SUB-TEST : PART C
In this part of the test, there are two texts about different aspects of healthcare.
For questions 7-22, choose the answer (A, B, C or D) which you think fits best according to the text.
Write your answers on the separate Answer Sheet

Text 1: Viral Infection – Yellow Fever


Yellow fever is a viral infection spread by a particular species of mosquito. It's most
common in areas of Africa and South America, affecting travellers to and residents of
those areas. In mild cases, it causes fever, headache, nausea and vomiting. But it can
become more serious, causing heart, liver and kidney problems along with bleeding
(haemorrhaging). Up to 50 percent of people with the more severe form of yellow fever
die of the disease.

There's no specific treatment for yellow fever. But getting a yellow fever vaccine before
travelling to an area in which the virus is known to exist can protect you from the disease.
During the first three to six days after you've contracted yellow fever — the incubation
period — you won't experience any signs or symptoms.
After this, the virus enters an acute phase and then, in some cases, a toxic phase that can
be life threatening.

Once the yellow fever virus enters the acute phase, you may experience signs and
symptoms including: Fever, Headache, Muscle aches, particularly in your back and knees,
Nausea, vomiting or both, Loss of appetite, Dizziness, Red eyes, face or tongue These signs
and symptoms usually improve and are gone within several days.

Although signs and symptoms may disappear for a day or two following the acute phase,
some people with acute yellow fever then enter a toxic phase. During the toxic phase,
acute signs and symptoms return and more-severe and life-threatening ones also appear.
These can include: Yellowing of your skin and the whites of your eyes (jaundice),
Abdominal pain and vomiting, sometimes of blood, Decreased urination, Bleeding from
your nose, mouth and eyes, Heart dysfunction (arrhythmia), Liver and kidney failure, Brain
dysfunction, including delirium, seizures and coma. The toxic phase of yellow fever can be
fatal.
Make an appointment to see your doctor four to six weeks before travelling to an area in
which yellow fever is known to occur. If you don't have that much time to prepare, call
your doctor anyway. Your doctor will help you determine whether you need vaccinations
and can provide general guidance on protecting your health while abroad.

Seek emergency medical care if you've recently travelled to a region where yellow fever is
known to occur and you develop severe signs or symptoms of the disease. If you develop
mild symptoms, call your doctor.

Yellow fever is caused by a virus that is spread by the Aedes aegypti mosquito.
These mosquitoes thrive in and near human habitations where they breed in even the
cleanest water. Most cases of yellow fever occur in sub-Saharan Africa and tropical South
America.

Humans and monkeys are most commonly infected with the yellow fever virus.
Mosquitoes transmit the virus back and forth between monkeys, humans or both.
When a mosquito bites a human or monkey infected with yellow fever, the virus
enters the mosquito's bloodstream and circulates before settling in the salivary glands.
When the infected mosquito bites another monkey or human, the virus then enters the
host's bloodstream, where it may cause illness.

You may be at risk of the disease if you travel to an area where mosquitoes continue to
carry the yellow fever virus. These areas include sub-Saharan Africa and tropical South
America. Even if there aren't current reports of infected humans in these areas, it doesn't
mean you're risk-free. It's possible that local populations have been vaccinated and are
protected from the disease, or that cases of yellow fever just haven't been detected and
officially reported. If you're planning on travelling to these areas, you can protect yourself
by getting a yellow fever vaccine at least 10 to 14 days before travelling. Anyone can be
infected with the yellow fever virus, but older adults are at greater risk of getting seriously
ill.
Diagnosing yellow fever based on signs and symptoms can be difficult because early in its
course, the infection can be easily confused with malaria, typhoid, dengue fever and other
viral hemorrhagic fevers.

To diagnose your condition, your doctor will likely:


Ask questions about your medical and travel history Collect a blood sample for testing

If you have yellow fever, your blood may reveal the virus itself. If not, blood tests
known as enzyme-linked immuno sorbent assay (ELISA) and polymerase chain
reaction (PCR) also can detect antigens and antibodies specific to the virus. Results
of these tests may not be available for several days.

No antiviral medications have proved helpful in treating yellow fever. As a result,


treatment consists primarily of supportive care in a hospital. This includes
providing fluids and oxygen, maintaining adequate blood pressure, replacing blood
loss, providing dialysis for kidney failure, and treating any other infections that
develop. Some people receive transfusions of plasma to replace blood proteins that
improve clotting. If you have yellow fever, you may also be kept away from
mosquitoes, to avoid transmitting the disease to others.
Text 1: Questions 7-14

7. Yellow fever is common in


A. Africa
B. South America
C. both
D. not given

8. Signs of yellow fever doesn’t include one of this


A. back pain
B. vomiting
C. nausea
D. dry tongue

9. Signs in toxic phase


A. loss of appetite
B. yellowness of eyes
C. brain dysfunction
D. B and C

10. Seizures may occur during


A. acute phase
B. toxic phase
C. sometimes in both the phases
D. not given

11. Yellow fever which is a viral disease, is spread by


A. Aedes agypti mosquito
B. Aedes aegypti mosquito
C. female mosquito
D. contamination
12. Mosquito transmits virus from
A. human to monkeys
B. monkeys to human
C. human to human
D none

13.Taking vaccine …………… days before travelling to areas where the disease is common is
recommended
A. 10 days
B. 12 days
C .14 days
D. 10-14 days

14. “It does not mean you are risk free in paragraph 8 refers to?
A. there is a greater chance of infection in some areas even though there is no recent report of
infection
B. even thought there are no current reported cases of yellow fever in some areas, there is still a
risk of getting injection
C. it is always recommended to take vaccines before travelling to Africa and South America
D. local population is not affected by yellow fever because of vaccination
Text 2: Aortic Dissection or Dissecting Aneurysm

An aortic dissection is a serious condition in which a tear develops in the inner


layer of the aorta, the large blood vessel branching off the heart. Blood surges
through this tear into the middle layer of the aorta, causing the inner and middle
layers to separate (dissect). If the blood-filled channel ruptures through the outside
aortic wall, aortic dissection is often fatal.

Aortic dissection, also called dissecting aneurysm, is relatively uncommon.


Anyone can develop the condition, but it most frequently occurs in men between
60 and 70 years of age. Symptoms of aortic dissection may mimic those of other
diseases, often leading to delays in diagnosis. However, when an aortic dissection
is detected early and treated promptly, your chance of survival greatly improves.

Aortic dissection symptoms may be similar to those of other heart problems, such
as a heart attack. Typical signs and symptoms include: Sudden severe chest or
upper back pain (often described as a tearing, ripping or shearing sensation, that
radiates to the neck or down the back), Loss of consciousness (fainting), Shortness
of breath, Sweating, Weak pulse in one arm compared to the other etc.

If you have signs or symptoms such as severe chest pain, fainting, sudden onset of
shortness of breath or symptoms of a stroke then taking medical assistance is of
preliminary importance. While experiencing such symptoms doesn't always mean
that you have a serious problem, it's best to get checked out quickly. Early
detection and treatment may help save your life.

An aortic dissection occurs in a weakened area of the aortic wall. Chronic high blood
pressure may stress the aortic tissue, making it more susceptible to tearing.
You can also be born with a condition associated with a weakened and enlarged aorta,
such as Marfan syndrome or bicuspid aortic valve. Rarely, aortic dissections may be caused
by traumatic injury to the chest area, such as during motor vehicle accidents.
Aortic dissections are divided into two groups, depending on which part of the aorta is
affected:
Type A: This is the more common and dangerous type of aortic dissection. It
involves a tear in the part of the aorta just where it exits the heart or a tear extending
from the upper to lower parts of the aorta, which may extend into the abdomen.
Type B: This type involves a tear in the lower aorta only “ which” may also extend into the
abdomen.

Risk factors for aortic dissection include: Uncontrolled high blood pressure (hypertension),
found in at least two-thirds of all cases Hardening of the arteries (atherosclerosis)
Weakened and bulging artery (pre-existing aortic aneurysm) An aortic valve defect
(bicuspid aortic valve) A narrowing of the aorta you're born with (aortic coarctation)

People with certain genetic diseases are more likely to have an aortic dissection than are
people in the general population.
These include: Turner's syndrome. High blood pressure, heart problems and a
number of other health conditions may result from this disorder.
Marfan syndrome; This is a condition in which connective tissue, which supports various
structures in the body, is weak. People with this disorder often have a family history of
aneurysms of the aorta and other blood vessels. These weak blood vessels are prone to
tears (dissection) and rupture easily.
Ehlers-Danlos syndrome; This group of connective tissue disorders is characterized
by skin that bruises or tears easily, loose joints and fragile blood vessels.
Loeys-Dietz syndrome; This is a connective tissue disorder marked by twisted arteries,
especially in the neck. People who have Loeys-Dietz syndrome are thought to be at risk for
developing aortic dissections and aneurysms.
An aortic dissection can lead to death, due to severe internal bleeding, including into the
lining around the heart (pericardial sac), Organ damage, such as kidney failure or life-
threatening damage to the intestines, Stroke, possibly including paralysis, Aortic valve
damage, such as causing the aortic valve to leak (aortic regurgitation)
Detecting an aortic dissection can be tricky because the symptoms are similar to those of a
variety of health problems. Doctors often suspect an aortic dissection if the following signs
and symptoms are present: Sudden tearing or ripping chest pain, Widening of the aorta on
chest X-ray, Blood pressure difference between right and left arms.
Text 2: Questions 15 to 22

15. In aortic dissection a tear develops in


A. outer layer of aorta
B. inner layer of aorta
C. middle aorta
D. a blood vessel branching off the heart

16. Dissecting aneurysm is common among


A. men
B. women
C. both
D. children

17. Symptoms of aortic dissection include


A. chest pain and swelling
B. weak pulse in both arms
C. loss of consciousness
D. all of the above

18. Aortic dissection can also be caused due to


A. high BP
B. weak aortic wall
C. inborn symptoms
D. traumatic injury to chest during accidents

19. The most dangerous type of aortic dissection is


A. Type A
B. Type B
C. aortic aneurism
D. aortic coarctation
20. A condition in which connective tissue is weak
A. Turner’s syndrome
B. Loeys-Dietz syndrome
C. Ehlers-Danlos syndrome
D. Marfan’s syndrome

21. People with Loeys-Dietz syndrome are likely to develop


A. aneurysms
B. ruptured blood vessels
C. twisted arteries in the neck
D. aortic complications

22.In paragraph 6 which’ refers to?


A. type B aortic dissection
B. the lower aorta
C. abdomen
D. a tear

END OF READING TEST, THIS BOOKLET WILL BE COLLECTED


Reading test 29 : Answer Key

Part A - Answer key 1 – 7


1 B
2 B
3 C
4 A
5 D
6 C
7 D

Part A - Answer key 8 – 14


8 SC/IV
9 Legs
10 Thrombectomy
11 Endovascular management
12 Phlegmasiaceruleadolens
13 Tenderness
14 Venous thromboembolism

Part A - Answer key 15 – 20


15 Nonspecific
16 Inflammation
17 Pulmonary embolus
18 6
19 1300
20 medication
Reading test - part B – answer key

1. A
2. B
3. B
4. C
5. B
6. B

Reading test - part C – answer key

Text 1 - Answer key 7 – 14


7. C
8. D
9. D
10. B
11. B
12. A/B
13. D
14. B

Text 2 - Answer key 15 – 22

15. B
16. A
17. C
18. D
19. A
20. D
21. A
22. D
READING TEST 30
READING SUB-TEST : PART A
 Look at the four texts, A-D, in the separate Text Booklet.
 For each question, 1-20, look through the texts, A-D, to find the relevant information.
 Write your answers on the spaces provided in this Question Paper.
 Answer all the questions within the 15-minute time limit.
 Your answers should be correctly spelt.

PART A -TEXT BOOKLET - HEMOPHILIA

Text A

Hemophilia results from mutations at the factor VIII or IX loci on the X chromosome and
each occurs in mild, moderate, and severe forms.
A similar level of deficiency of factor VIII or IX results in clinically indistinguishable disease
because the end result is deficient activation of factor X by the factor Xase complex
(FVIIIa/FIXa/calcium and phospholipid).
Hemophilia A is an X-linked, recessive disorder caused by the deficiency of
functional plasma clotting factor VIII (FVIII), which may be inherited or arise
from spontaneous mutation.
Hemophilia B, or Christmas disease, is an inherited, X-linked, recessive disorder
that results in the deficiency of functional plasma coagulation factor IX.
Text B
Hemophilia A
Primary sites of factor VIII (FVIII) production are thought to be the vascular endothelium
in the liver and the reticuloendothelial system.
FVIII deficiency, dysfunctional FVIII, or FVIII inhibitors lead to the disruption of the normal
intrinsic coagulation cascade, resulting in excessive hemorrhage in response to trauma
and, in severe cases, spontaneous hemorrhage.
Human synovial cells synthesize high levels of tissue factor pathway inhibitor,
resulting in a higher degree of factor Xa (FXa) inhibition, which predisposes
hemophilic joints to bleed.
This effect may also account for the dramatic response of activated factor VII (FVIIa)
infusions in patients with acute hemarthroses and FVIII inhibitors.
Bleeding into a joint may lead to synovial inflammation, which predisposes the joint to
further bleeds; a joint that has had repeated bleeds (by one definition, at least 4 bleeds
within a 6-month period) is termed a target joint.
Approximately 30% of patients with severe hemophilia A develop alloantibody inhibitors
Hemophilia B
Factor IX deficiency, dysfunctional factor IX , or factor IX inhibitors lead to
disruption of the normal intrinsic coagulation cascade, resulting in spontaneous
hemorrhage and/or excessive hemorrhage in response to trauma.
Hemorrhage sites include joints (eg, knee, elbow), muscles, central nervous system
(CNS), GI system, genitourinary (GU) system, pulmonary system, and cardiovascular
system.
Factor IX, a vitamin K–dependent single-chain glycoprotein, is synthesized first by
the hepatocyte; the precursor protein undergoes extensive posttranslational
modification before being secreted into the blood.
The intrinsic system is initiated when factor XII is activated by contact with
damaged endothelium.
In the extrinsic system, the conversion of factor X to factor Xa involves tissue
factor (TF), or thromboplastin; factor VII; and calcium ions.
FVIII and FIX circulate in an inactive form; when activated, these 2 factors
cooperate to cleave and activate factor X, a key enzyme that controls the
conversion of fibrinogen to fibrin.
Therefore, the lack of either of these factors may significantly impair clot
formation and, as a consequence, result in clinical bleeding.

Statistics and Incidences:


Hemophilia is slowly progressing among pediatric patients in all parts of the globe.
X-linked recessive inheritance | Genetics Home Reference Hemophilia A is the most
common X-linked genetic disease and the second most common factor deficiency after
von Willebrand disease (vWD).
The worldwide incidence of hemophilia A is approximately 1 case per 5000 males,
with approximately one-third of affected individuals not having a family history of
the disorder.
In the United States, the prevalence of hemophilia A is 20.6 cases per 100,000
males; in 2016, the number of people in the United States with hemophilia was
estimated to be about 20,000.
Hemophilia A occurs in all races and ethnic groups.
Because hemophilia is an X-linked, recessive condition, it occurs predominantly in
males; females usually are asymptomatic carriers.
The incidence of hemophilia B is estimated to be approximately 1 case per 25,000-
30,000 male births.
The prevalence of hemophilia B is 5.3 cases per 100,000 male individuals, with
44% of those having severe disease.
Hemophilia B is much less common than hemophilia A. Of all hemophilia cases,
80-85% are hemophilia A, 14% are hemophilia B, and the remainder are various
other clotting abnormalities.
Hemophilia B occurs in all races and ethnic groups.
Text C
Text D

Medical Management
The treatment of hemophilia may involve prophylaxis, management of bleeding
episodes, treatment of factor VIII (FVIII) inhibitors, and treatment and
rehabilitation of hemophilia synovitis.
Ryan White was an American hemophiliac who was infected with HIV from a
contaminated blood treatment (factor VIII).
Prehospital care. Rapid transport to definitive care is the mainstay of prehospital
care; prehospital care providers should apply aggressive hemostatic techniques,
assist patients capable of self-administered factor therapy, and gather focused
historical data if the patient is unable to communicate.
Emergency department care. Use aggressive hemostatic techniques; correct
coagulopathy immediately; include a diagnostic workup for hemorrhage, but never
delay indicated coagulation correction pending diagnostic testing; acute joint
bleeding and expanding, large hematomas require adequate factor replacement for
a prolonged period until the bleed begins to resolve, as evidenced by clinical
and/or objective methods; life-threatening bleeding episodes are generally initially
treated with FVIII levels of approximately 100%, until the clinicalsituation
warrants a gradual reduction in dosage.
Factor VIII and FIX concentrates. Various FVIII and FIX concentrates are
available to treat hemophilia A and B; besides improved hemostasis, continuous
infusion decreases the amount of factor used, which can result in significant
savings; obtain factor level assays daily before each infusion to establish a stable
pattern of replacement regarding the dose and frequency of administration.
Desmopressin.Desmopressin vasopressin analog, or 1-deamino-8-D-arginine
vasopressin (DDAVP), is considered the treatment of choice for mild and moderate
hemophilia A; DDAVP stimulates a transient increase in plasma FVIII levels;
DDAVP may result in sufficient hemostasis to stop a bleeding episode or to
prepare patients for dental and minor surgical procedures.
Management of bleeding Immobilization of the affected limb and the application
of ice packs are helpful in diminishing swelling and pain; early infusion upon the
recognition of initial symptoms of a joint bleed may often eliminate the need for a
second infusion by preventing the inflammatory reaction in the joint; prompt and
adequate replacement therapy is the key to preventing long-term complications.
PART A -QUESTIONS AND ANSWER SHEET
Questions 1-7
For each of the questions, 1-7, decide which text (A, B, C or D) the information comes from.
You may use any letter more than once.

1. The information regarding treatment of haemophilia is…………………..

2. Hemophilia A occurs in all the races and ethnic groups. ……………..

3. The information regarding frequency of bleeding sites in factor XIII deficiency. …………

4. Information regarding usage of aggressive hemostatic techniques………………..

5. The definition of haemophilia A…………………..

6. Information regarding the consequences of bleeding into joints ………..

7. The incidence of haemophilia A is more than hemophila B…………………..

Questions 8-14
Answer each of the questions, 8-14, with a word or short phrase from one of the texts. Each
answer may include words, numbers or both. Your answers should be correctly spelt.

8. Which types of patients have slow progression of hemophilia?……………

9. Which clotting factor is deficient in hemophilia A?……………

10.What is initiated when factor XII comes in contact with damaged endothelium?……………

11.Who are the asymptomatic carriers in the case of hemophilia?………

12.Which is the least bleeding site in factor VIII deficiency cases? ………………….

13.What is the treatment of choice for mild and moderate hemophilia A? ……………

14.Which is the most common factor deficiency disorder around the globe?……………………..
Questions 15-20

Complete each of the sentences, 15-20, with a word or short phrase from one of the texts. Each
answer may include words, numbers or both. Your answers should be correctly spelt.

15. Hemophilia B is also called as ………………….

16. A joint which has had repeated bleeds is called as ………………….

17. One third of patients with severe hemophilia A develop alloantibody………………….

18. The incidence of hemophilia B is predicted to be nearly, one case per………………. births.

19. Hemophilia is an …………………. recessive disorder.

20.……………….. synthesize high levels of tissue factor pathway inhibitors,which predisposes


hemolytic joints to bleed.

END OF PART A, THIS QUESTIONS PAPER WILL BE COLLECTED


READING SUB-TEST : PART B

In this part of the test, there are six short extracts relating to the work of health professionals .
For questions 1-6, choose the answer (A, B or C) which you think fits best according to the text.
Write your answers on the separate Answer Sheet

Questions 1-6

1. According to the passage, when it constitutes malpractice?


A. if the breach of duty o f a nurse cause no harm to the patient.
B. when a nurse commit omission during the patient care.
C. when an action of a nurse causes damage to the patient.

Elements of malpractice
To prove malpractice, all 4 of the following elements must be proven by the
plaintiff: the nurse had a duty to the patient, the nurse breached the duty, a patient
injury occurred, and there was a causal relationship between the breach of duty and
the patient injury (Reising, 2012). Therefore, in determining if malpractice has
occurred, these 4 elements must be carefully considered. First, did the nurse have a
duty to the patient? This means that the nurse was actively engaged in providing
nursing care to the patient. Second, was there a breach of that duty? In other words,
did the nurse commit an act or omission in the act of taking care of the patient and
did that act or omission result in harm to the patient. The third element is
“proximate cause.” The question here is whether the action or omission caused any
harm to the patient. If the action did not result in harm or injury, there was no
malpractice. Finally, the fourth element is damage. What harm occurred as a result
of the action of omission during the delivery of nursing care? A nurse might have a
duty to a patient and commit an action or omission during the course of nursing
care, and it might not constitute malpractice if the action or omission did not result
in harm to the patient.
2. Nasal cannula
A. Is an air delivering device for the patients
B. In a disposable plastic device, used for patients with oxygen insufficiency
C. can be used easily at home and disrupt the eating and speaking of the patient

Nasal Cannula

A variety of devices are available for delivering oxygen to the patient. Each has a
specific function and oxygen concentration. Device selection is based on the
patient’s condition and oxygen needs. A nasal cannula, also called nasal prongs, is
the most commonly used oxygen delivery device. The cannula is a disposable
plastic device with two protruding prongs for insertion into the nostrils. The
cannula connects to an oxygen source with a flow meter and, many times, a
humidifier. It is commonly used because the cannula does not impede eating or
speaking and is used easily in the home. Disadvantages of this system are that it
can be dislodged easily and can cause dryness of the nasal mucosa. A nasal
cannula is used to deliver from 1 L/minute to 6 L/minute of oxygen. Table 14-1
compares amounts of delivered oxygen for these flow rates.
3. An expressed consent to be taken during certain situations except;
A. when there is no risk involved in the treatment.
B . surgical operations and invasive procedures.
C. administration of high risk drugs.

Expressed Consent

An express consent is one the terms of which are stated in distinct and explicit
language. It may be oral or written. For the majority of relatively minor examinations or
therapeutic procedures, oral consent is employed but this should preferably be obtained
in the presence of a disinterested party. Oral consent, where properly witnessed, is as
valid as written consent, but the latter has the advantage of easy proof and permanent
form. It should be obtained when the treatment is likely to be more than mildly painful,
when it carries appreciable risk, or when it will result in diminishing of a bodily function.
Consent may be confirmed and validated adequately by means of a suitable
contemporaneous notation by the treating physician in the patient’s record.
Expressed consent in written form should be obtained for surgical operations and
invasive investigative procedures. It is prudent to obtain written consent, also where
never analgesic, narcotic or anesthetic agents will significantly affect the patient’s level of
consciousness during the treatment.
4. Irrational use of medicines is a major global problem because?
A. Medicine are over used by 50% of all patients
B. inappropriate use of medicine results in destruction of insufficient resources and
prevalent health hazards.
C. Injections are over used instead of oral formulations to treat non-bacterial infections.

Irrational use of medicines

Irrational use of medicines is a major problem worldwide. WHO estimates that


more than half of all medicines are prescribed, dispensed or sold inappropriately,
and that half of all patients fail to take them correctly. The overuse, underuse or
misuse of medicines results in wastage of scarce resources and widespread health
hazards. Examples of irrational use of medicines include: use of too many
medicines per patient ("poly-pharmacy"); inappropriate use of antimicrobials, often
in inadequate dosage, for non-bacterial infections; over-use of injections when oral
formulations would be more appropriate; failure to prescribe in accordance with
clinical guidelines; inappropriate self-medication, often of prescription-only
medicines; non-adherence to dosing regimes.
5. Application of waist restraints: The manual says us that;
A. the waist restraints and the jacket restraints are the same
B. waist restraints are used to prevent injury.
C. the potential risk of using waist restraints are more than its benefits

To :All staff

Application of Waist restraints


Waistrestraints are a form of restraint that is applied to thepatient’s torso.It is applied
overthe patient’s clothes, gown, or pajamas. When usingawaist restraint, patients can
move their extremities but cannot get out of the chair or bed.
Restraints should be used only after less-restrictive methods have failed. Ensure
compliance with ordering, assessment, and maintenance procedures. Historically,
vest or jacket restraints were used to prevent similar patient movement, but their
use has significantly decreased due to concerns for the potential risk for
asphyxiation with the device. Research suggests that waist restraints pose the same
potential risk for asphyxial death as vest restraints (Capezuti, et al., 2008).
Healthcare providers need to be aware of this potential outcome and weigh it
against possible benefit from use of the device.
6. A study of serotonin on people with anorexia nervosa found that;
A. people with anorexia are more likely to starve for long time.
B. the body produces serotonin from the food we eat
C. people with anorexia have how low count of serotonin metabolites in CSF.

SEROTONIN

Serotonin (sometimes referred to as 5-hydroxytryptophan) helps control everything


from memory and learning to sleep, mood, and appetite, researchers quickly began
to look for potential relationships between polymorphisms in serotonin receptor
genes and eating disorders. A variety of studies have found alterations in the
serotonin system in individuals currently ill with anorexia nervosa and those
recovered from the disorder. Researchers found that people who are currently
suffering from anorexia have significantly lower levels of serotonin metabolites in
their cerebrospinal fluid than individuals without an eating disorder. This is likely a
sign of starvation, since the body synthesizes serotonin from the food we eat. After
long-term recovery from anorexia, however, individuals have significantly
elevated serotonin levels (Kaye et al., 1991). In this study, the researchers found
that higher levels of serotonin correspond with levels of anxiety and obsessive
behavior. Geneticists have also found that individuals with anorexia are slightly
more likely to carry a particular variant of the 5HT2A serotonin receptor, which is
thought to increase the amount of serotonin in the brain during the non-starved
state (Gorwood et al., 2002).
READING SUB-TEST : PART C

In this part of the test, there are two texts about different aspects of healthcare.
For questions 7-22, choose the answer (A, B, C or D) which you think fits best according to the text.
Write your answers on the separate Answer Sheet

Text 1: A Hormonal Disorder – Adrenal Insufficiency

Adrenal insufficiency is an endocrine or hormonal disorder that occurs when the


adrenal glands do not produce enough of certain hormones. The adrenal glands are
located just above the kidneys. Adrenal insufficiency can be primary or secondary.
Primary adrenal insufficiency, also called Addison’s disease, occurs when the
adrenal glands are damaged and cannot produce enough of the hormone cortisol,
and often the hormone aldosterone. Addison’s disease affects one to four of every
100,000 people, in all age groups and both sexes.

Secondary adrenal insufficiency occurs when the pituitary gland, a bean-sized


organ in the brain, fails to produce enough adrenocorticotropin (ACTH), a
hormone that stimulates the adrenal glands to produce cortisol. If ACTH output is
too low, cortisol production drops. Eventually, the adrenal glands can shrink due to
lack of ACTH stimulation. Secondary adrenal insufficiency is much more common
than Addison’s disease.

Addison's disease symptoms usually develop slowly, often over several months,
and may include: muscle weakness and fatigue, weight loss and decreased appetite,
darkening of skin (hyperpigmentation), low blood pressure (even fainting), salt
cravings, low blood sugar (hypoglycemia), nausea, diarrhea or vomiting, muscle or
joint pains etc. Sometimes, however, the signs and symptoms of Addison's disease
may appear suddenly. In acute adrenal failure (addisonian crisis), the signs and
symptoms may also include: pain in your lower back, abdomen or legs, severe
vomiting and diarrhea, leading to dehydration, low blood pressure and loss of
consciousness.
Your adrenal glands are composed of two sections: the interior (medulla) produces
adrenaline-like hormones; the outer layer (cortex) produces a group of hormones
called corticosteroids, which include glucocorticoids, mineralocorticoids and male
sex hormones (androgens).
Some of the hormones the cortex produces are essential for life (glucocorticoids
and mineralocorticoids). Glucocorticoids: These hormones, which include cortisol,
influence your body's ability to convert food fuels into energy, play a role in your
immune system's inflammatory response and help your body respond to stress.
Mineralocorticoids: These hormones, which include aldosterone, maintain your
body's balance of sodium and potassium to keep your blood pressure normal.

Androgens, male sex hormones, are produced in small amounts by the adrenal
glands in both men and women. They cause sexual development in men and
influence muscle mass, libido and a sense of well-being in men and women.

Primary adrenal insufficiency


Addison's disease occurs when the cortex is damaged and doesn't produce its
hormones in adequate quantities. Doctors refer to the condition involving damage
to the adrenal glands as primary adrenal insufficiency. The failure of your adrenal
glands to produce adrenocortical hormones is most commonly the result of the
body attacking itself (autoimmune disease). For unknown reasons, your immune
system views the adrenal cortex as foreign, something to attack and destroy. Other
causes of adrenal gland failure may include: tuberculosis, other infections of the
adrenal glands, spreading of cancer to the adrenal glands and bleeding into the
adrenal glands.
Secondary adrenal insufficiency
Adrenal insufficiency can also occur if your pituitary gland is diseased. The
pituitary gland produces a hormone called adrenocorticotropic hormone (ACTH),
which stimulates the adrenal cortex to produce its hormones. Inadequate
production of ACTH can lead to insufficient production of hormones normally
produced by your adrenal glands, even though your adrenal glands aren't damaged.
Doctors call this condition secondary adrenal insufficiency.
Another more common cause of secondary adrenal insufficiency occurs when
people who take corticosteroids for treatment of chronic conditions, such as asthma
or arthritis, abruptly stop taking the corticosteroids.
If you have untreated Addison's disease, an addisonian crisis may be provoked by
physical stress, such as an injury, infection or illness. All treatment for Addison's
disease involves hormone replacement therapy to correct the levels of steroid
hormones your body isn't producing. Some options for treatment include: Oral
corticosteroids, Corticosteroid injections, Androgen replacement therapy.
Part C -Text 1: Questions 7-14
7. Addison’s disease effects
A. Four of every 1,000 people
B. One of every 1,000 people, including all age-groups
C. One to four of every 1,000 people, including all age groups
D. One to four of every 100,000 people

8. Secondary adrenal insufficiency occurs due to one of these reasons


A. More production of adrenocorticotropin (ACTH)
B. More production of cortisol
C. Low production of aldosterone
D. Low production of ACTH

9. According to the passage(s), symptoms of Addison’s disease occur


A. Slowly
B. Suddenly
C. After a month
D. Slowly after many months

10. Cortex produces


A. Androgens
B. Glucoscorticods
C. Mineralocorticods
D. All of the above

11. Aldosterone maintains


A. Body balance
B. Balance of sodium and potassium
C. High BP
D. Low BP
12. According to the information given in the passage, the statement that “Androgen influences
muscle massandphysicalandmentalnatureofmenand women” is
A. True
B. False
C. Can’t say
D. Not given in the passage(s)

13. Doctors refer to the damage to the cortex of the kidney as


A. Primary adrenal insufficiency
B. Secondary adrenal insufficiency
C. Other fatal infections
D. Not given

14. Secondary adrenal insufficiency in the paragraph 8 refers to


A The condition where kidney is diseased
B. In which adrenal glands are damaged
C. The kidney is slightly damaged causing decreased release of certain hormones
D. The condition in which adrenal glands are not damaged
Text 2: Carpal Tunnel Syndrome

Carpal tunnel syndrome is a condition that may be caused by repeatedly performing


stressful motions with your hand or holding your hand in the same position for long periods
of time. CTS is classified as a cumulative trauma disorder,an ailment that attacks the body’s
musculoskeletal system. The musculoskeletal system is made up of muscles that pull on
tendons and move the bones at joints. The joints are held together by ligaments. Carpal
tunnel syndrome specifically affects the sensitive nerves of - and the blood supply that
feeds – the hands and wrists.

Carpal tunnel syndrome has been around for a long time; meat packers began complaining
of pain and loss of hand function in the 1860s. Back then, these complaints were largely
attributed to poor circulation. The nature of work has changed over the years; today, more
jobs are highly specialized and require use of only a small number of muscles repeatedly.
With the growing numbers of people using computers and keyboards, plus the focus on
better health-care for workers, carpal tunnel syndrome is of real concern to both employers
and health-care professionals.

Recent studies have shown that carpal tunnel syndrome, like all other cumulative trauma
disorders, is on the rise while other workplace injuries have leveled off.
Many companies are turning to physical therapists for help with designing and
implementing health promotion and injury prevention programs to protect their employees
from CTS.

People with CTS usually experience feelings of numbness, weakness, tingling, and
burning in their fingers and hands. If not treated, the symptoms may escalate into
acute, persistent pain. CTS can become so crippling that people can no longer do
their work or even perform simple tasks at home. At its most extreme, carpal
tunnel syndrome forces people to undergo surgery and miss many days of work, or
prevents them from working at all because their hand functions are permanently impaired.
Carpal tunnel syndrome occurs in men and women of all ages, and is often found
in workers whose tasks require repeating the same motion in the fingers and hand
for long periods of time. CTS has surfaced among meat packers, assembly line
workers, jackhammer operators, and employees who spend hours working at a
computer or typewriter. Carpal tunnel syndrome shows up in athletes as well as
homemakers.

The U.S. Department of Labor has cited carpal tunnel syndrome, as well as other
cumulative trauma disorders, as the cause of 48 percent of all industrial workplace
illnesses. The disease affects more than five million Americans.

CTS’simpactonAmericanbusinessesisdevastating.Itshowsupintheworkplacein the form of


fatigue, poor work performance, discomfort and pain, and poor
employer/employee relations. The high cost of treatment for an employee with
CTS, plus the lost productivity when that employee is absent for a long period of
time,strainsthecompany’sabilitytooperateefficientlyandcanleadtomorale
problems when other employees have to take over the absent workers’ responsibilities.

Physical therapists with specialized training in cumulative trauma disorders have


been working in industrial and corporate settings for many years to meet the
health-care needs of America’s work force. They work closely with employers to
educate employees about CTS—what causes it and how to avoid it through proper
use of the musculoskeletal system.

Physical therapists can target and correct poor work habits and improper work
designs, such as tools, furniture, equipment, and work space. They also can assess
the risk potential of an individual and determine if that person is physically
unsuited for a particular job. Among their many responsibilities, physical therapists
teach health awareness and job safety.
A typical education program includes exercises employees can do at work and at
home, adjustments to the overall work environment and individual work stations,
plus early detection of symptoms to avoid painful and costly surgery.

Physical therapists also work with employers and their engineering departments to
design and modify the work environment, helping to remove the causal factors of
CTS. If anyone has symptoms of carpal tunnel syndrome then consulting a
physical therapist or other qualified health care practitioner for an evaluation and
individualized treatment is always recommended
Part C -Text 2: Questions 15-22
15. According to the passage, CTS
A. is a cumulative trauma disorder
B. is caused due to weakness in musculoskeletal system
C. occurs due to weakness in ligaments between joints
D. all of the above

16. According to the information given, CTS


A. is on the rise
B. is on the rise without any other cumulative trauma disorders
C. is one of the common cumulative trauma disorders that is on rise
D. is on the rise with many other cumulative trauma disorders

17. “In paragraph 4 permanently impaired refers. ”


A. total loss of carpal function
B. total loss of finger and hand function
C. cannot move because tarsal bones loose function
D. wrists does not work

18. CTS often occurs in


A. men
B. women
C. men and women of all ages
D. only men of all ages

19. …………complained of pain and loss of hand function in the 1860s


A. meat packers
B. assembly line workers
C. jackhammer operators
D. employees who spend hours at a computer
20. CTS accounts for ……….of all industrial workplace illnesses
A. 50 %
B. 48%
C. 84%
D. 60%

21. Physical therapists teach employees about


A. proper use of the musculoskeletal system
B. how to avoid CTS
C. how to increase efficiency at work
D. health-care needs

22. One of the impacts of CTS on American businesses


A. low productivity because of absence of workers due to CTS
B. absence of workers due to CTS condition
C. unfair employee treatment
D. none

END OF READING TEST, THIS BOOKLET WILL BE COLLECTED


Reading test 30 : Answer Key

Part A - Answer key 1 – 7


1. D
2. B
3. C
4. D
5. A
6. B
7. B

Part A - Answer key 8 – 14


8. pediatric patients
9. factor viii/ f viii
10.the intrinsic system
11.femeles / women
12.renal
13.desmopressin / desmopressin vasopresin amlog / ddavp
14.von willebrand disease (vwd)

Part A - Answer key 15 – 20


15.christmas disease
16.target joint
17.inhibitors hemophilia b
18.25,000-30,000 male
19.x-linked
20.human synovial cells
Reading test - part B – answer key
1. C
2. B
3. A
4. B
5. C
6. B

Reading test - part C – answer key

Text 1 - Answer key 7 – 14


7. C
8. D
9. D
10. A
11. B
12. B
13. A
14. D

Text 2 - Answer key 15 – 22

15. A
16. D
17. B
18. C
19. A
20. B
21. B
22. A
READING TEST 31
READING SUB-TEST : PART A
 Look at the four texts, A-D, in the separate Text Booklet.
 For each question, 1-20, look through the texts, A-D, to find the relevant information.
 Write your answers on the spaces provided in this Question Paper.
 Answer all the questions within the 15-minute time limit.
 Your answers should be correctly spelt.

PART A -TEXT BOOKLET – OSTEOPOROSIS

Text A

Osteoporosis is classified as a metabolic bone disorder.


Osteoporosis occurs when the creation of new bone doesn’t keep up with the removal of
old bone. Osteoporosis causes bones to become weak and brittle — so brittle that a fall or
even mild stresses such as bending over or coughing can cause a fracture.
Classification-Osteoporosis may be classified into two types:
Primary osteoporosis: Primary osteoporosis occurs in women after menopause and
in men later in life, but it is not merely a consequence of aging but of failure to
develop optimal peak bone mass during childhood, adolescence, and young
adulthood. Secondary osteoporosis: Secondary osteoporosis is the result of medications or
other conditions and diseases that affect bone metabolism.
Text B
Statistics and Incidences
Osteoporosis is the most prevalent bone disease in the world.
More than 10 million Americans have osteoporosis and an additional 33.6 million
have osteopenia, the precursor to osteoporosis. It is projected that one of every two
Caucasian women and one of every five men will have an osteoporosis-related fracture at
some point in their lives. The costs incurred from treating osteoporosis-related fractures in
the United States are estimated at $20 billion annually. The prevalence of osteoporosis in
women older than 80 years is 50%. The average 75-year-old woman has lost 25% of her
cortical bone and 40% of her trabecular bone. With the aging of the population, the
incidence of fractures (more than 1.5 million osteoporotic fractures per year), pain, and
disability associated with osteoporosis is increasing.
Causes
The causes of osteoporosis and their effects on bone include:
Genetics: Small-framed, non obese
Caucasian women are at greatest risk; Asian women of slight build are at risk for low peak
bone mineral density;
African American women are less susceptible to osteoporosis.
Age: Osteoporosis occurs in men at a lower rate and at an older age, as it is believed that
testosterone and estrogen are important in achieving and maintaining bone mass, so risk
for osteoporosis increases with increasing age.
Nutrition: A low calcium intake, low vitamin D intake, high phosphate intake, and
inadequate calories reduce nutrients needed for bone remodeling.
Physical exercise: A sedentary lifestyle, lack of weight-bearing exercise, and low
weight and body mass index increases the risk for osteoporosis because bones need
stress for bone maintenance.
Lifestyle choices: Too much consumption of caffeine and alcohol, smoking, and
lack of exposure to sunlight reduces osteogenesis in bone remodeling.
Medications: Intake of corticosteroids, antiseizure medications, heparin, and
thyroid hormone affects calcium absorption and metabolism.
Text C
Text D

Treatment overview
Although a diagnosis of osteoporosis is based on the results of your bone mineral
density scan (DEXA or DXA scan), the decision about what treatment you need, if
any, is also based on a number of other factors. These include your:
age
sex
risk of fracture
previous injury history
If you've been diagnosed with osteoporosis because you've had a fracture, you
should still receive treatment to try to reduce your risk of further fractures.
You may not need or want to take medication to treat osteoporosis. However, you
should ensure you're maintaining sufficient levels of calcium and vitamin D. To
achieve this, your healthcare team will ask you about your diet and may
recommend making changes or taking supplements.

Medication for osteoporosis:


A number of different medications are used to treat osteoporosis. Your doctor will
discuss the treatments available and make sure the medicines are right for you. A
number of factors are taken into consideration before deciding which medication to
use. These include your:
age
bone mineral density (measured by your T score)
risk factors for fracture
Bisphosphonates:
Bisphosphonates slow the rate that bone is broken down in your body. This
maintains bone density and reduces the risk of fracture.
There are a number of different bisphosphonates, including: alendronate ,ibandronate
,risedronate, zoledronic acid

They're given as a tablet or injection.

You should always take bisphosphonates on an empty stomach with a full glass of
water. Stand or sit upright for 30 minutes after taking them. You'll also need to
wait between 30 minutes and 2 hours before eating food or drinking any other
fluids. Bisphosphonates usually take 6 to 12 months to work, and you may need to
take them for 5 years or longer. You may also be prescribed calcium and vitamin D
supplements to take at a different time to the bisphosphonate.

The main side effects associated with bisphosphonates include: irritation to the oesophagus
(the tube that food passes through from the mouth to the stomach) swallowing problems
(dysphagia) ,stomach pain. Not everyone will experience these side effects.

Selective oestrogen receptor modulators (SERMs)


SERMs are medications that have a similar effect on bone as the hormone oestrogen. They
help to maintain bone density and reduce the risk of fracture, particularly of the spine.

Raloxifene is the only type of SERM available for treating osteoporosis. It's taken as a daily
tablet.
Side effects associated with raloxifene include:hot flushes, leg cramps a potential increased
risk of blood clots

Parathyroid hormone (teriparatide)


Parathyroid hormone is produced naturally in the body. It regulates the amount of
calcium in bone. Parathyroid hormone treatments (human recombinant parathyroid
hormone or teriparatide) are used to stimulate cells that create new bone
(osteoblasts). They're given by injection.
PART A -QUESTIONS AND ANSWER SHEET

Questions 1-7
For each of the questions, 1-7, decide which text (A, B, C or D) the information comes from.
You may use any letter more than once

In which text can you find information about?


1. Osteoporosis causes bone to become weak and brittle. ……………
2. It is not required to take medications to treat osteoporosis…………………..
3. Over consumption of caffeine and alcohol may reduce osteogenesis…………………
4. One fifth of white women aged 65 have osteoporosis ……………………
5. Certain number of aspects is considered before treating osteoporosis.……………….
6. The major side effect of bisphosphonates. …………………
7. The incidence of fractures associated with osteoporosis is increasing with the aging of population.
…………….

Questions 8-14
Answer each of the questions, 8-4, with a word or short phrase from one of the texts.
Each answer may include words, number of the both. Your answers should be correctly spelled.

8. Which is the most prevalent bone disease in the world?………………………………


9. What are the two hormones which help in maintaining bone mass in men?…………………
10.Which medication is used to reduce the rate of broken bone?…………………………………….
11.Which test is performed to diagnose osteoporosis?……………………………….
12.What are the two types of osteoporosis?…………………………………
13.What is the percentage of Caucasian women, who are expected to have osteoporosis related
fracture in their lives?……………………………….
14.What is the estimated cost involved to treat osteoporosis annually in the US?…………………
Questions 15-20

Complete each of the sentences, 15- 20, with a word or short phrase from one of the texts.
Each answer may include words, number or both. Your answers should be correctly spelled

15. Formation of new bone is called as ………………….

16. The risk of osteoporosis increases with ………………….

17. Consumption of anticoagulants may affects …………………. And metabolism

18. Hot flushes, leg cramps and blood clots are the side effects of………………….

19. …………………….million osteoporotic fractures are noted with the aging of population every year.

20. Patients with osteoporosis should maintain sufficient levels of……………………

END OF PART A ,THIS BOOKLET WILL BE COLLECTED


READING SUB-TEST : PART B

In this part of the test, there are six short extracts relating to the work of health professionals .
For questions 1-6, choose the answer (A, B or C) which you think fits best according to the text. Write
your answers on the separate Answer Sheet

Questions 1-6

1. If a person does not have an advance directive…………………………….


a. Critical decisions are made by court
b. Health care provider & and institutions will make critical decisions for him / her.
c. His relatives are responsible for these decisions

COMBINATION ADVANCE DIRECTIVE

A combination advance directive is a signed, witnessed (or notarized) document


which contains specific written directions that are to be followed by a named
agent. Since it is not possible to predict all circumstances that may be faced in the
future or to cover all possible interventions, specific directions may severely limit
thediscretionandflexibilitythattheagentneedsandmayrestricttheagent’s
authority in a way the signer did not intend. In addition, the specific written
directions may not be altered through discussions between the signer and the agent.
Any changes necessitate a new document to reflect nuances or changed directions.
It is important that all adults consider who will make medical decisions for them if
they are temporarily or permanently unable to make them for themselves. Unless a
person has an advance directive, many health care providers and institutions will
make critical decisions for him/her or a court may appoint a guardian who is
unfamiliar with the person’s values and wishes.
2. The paragraph gives information regarding
a. The production, types and functions of corticosteroids
b. The wide range of physiological processes where corticosteroids are involved
c. Two main classes of corticosteroids and their functions

CORTICOSTEROIDS
Corticosteroids are a class of steroid hormones that are produced in the adrenal
cortex of vertebrates, as well as the synthetic analogues of these hormones. Two
main classes of corticosteroids, glucocorticoids and mineralocorticoids, are
involved in a wide range of physiological processes, including stress response,
immune response, and regulation of inflammation, carbohydrate metabolism,
protein catabolism, blood electrolyte levels, and behavior

3. The main purpose of the cooling blanket is to


a. Act as a hyperthermia pad to reduce body temperature.
b. Can be preset to maintain a specific body temperature, usually used to lower body
temperature
c. Device continually monitors the patient’s body temperature

USING A COOLING BLANKET


A cooling blanket, or hypothermia pad, is a blanket-sized Aquathermia pad that
conducts a cooled solution, usually distilled water, through coils in a plastic
blanket or pad. Placing a patient on a hypothermia blanket or pad helps to lower
body temperature. The nurse monitors the patient’s body temperature and can reset
the blanket setting accordingly. The blanket also can be preset to maintain a
specificbodytemperature;thedevicecontinuallymonitorsthepatient’sbody
temperature using a temperature probe (which is inserted rectally or in the
esophagus, or placed on the skin) and adjusts the temperature of the circulating
liquid accordingly.
4.The notice is giving information about…
A. How the use of restraints is authorized
B. Ways of applying restraints to different body parts
C. What restraint is and its examples

RESTRAINT

A restraint is any manual method, physical or mechanical device, material or equipment that
immobilizes or reduces the ability of a patient to move his/her arms, legs, body or head
freely (e.g. –Safe keeper bed, Posey bed, safety mitt, locked belt, or soft limb restraint); or a
restraint is a drug or medication when it is used as a restriction to manage the patient’s
behavior or restrict the patient’s freedom of movement and is not standard treatment or
dosage for the patient’s condition A restraint does not include devices, such as
orthopedically prescribed devices, surgical dressings or bandages, protective helmets, or
other methods that involve the physical holding of a patient for the purpose of conducting
routine physical examinations or tests, or to protect the patient from falling out of bed, or
to permit the patient to participate in activities without the risk of physical harm.
5. The purpose of this email is to
a. Report on how the confidential information is accessed in health care system
b. The importance of considering the security of patient documentation in any circumstances
c. Remind staff about the unimportance of maintaining confidentiality

To All Staffs
Subject: DOCUMENTATION GUIDELINES

Health care professionals should view the security of client documentation as a


serious issue. Failure to comply with legislation, falsifying information or providing
information without the client or agency’s consent may constitute professional misconduct.
Sharing confidential information is only acceptable in an effort to support the provision of
quality care with health care team members who are a part of the client’s circle of care.
Documentation, in any format, should be maintained in areas where the information cannot
be easily accessed by casual observers or those not directly involved in the care of the client.
Health records maintained in a client’s home should be stored in a manner to reduce the risk
of family members or others (e.g., visitors, guests) accessing confidential information.
Agencies should have policies outlining who has access the health records and how clients
and their family members are made aware of the importance of maintaining confidentiality.
6. What should be considered while selecting analgesics?
a. Its various effects on peripheral and central nervous system.
b. The different types and their respective effects.
c. Their severity and response to other medication

ANALGESICS

An analgesic or painkiller is any member of the group of drugs used to achieve


analgesia, relief from pain. Analgesic drugs act in various ways on the peripheral
and central nervous systems. They are distinct from anesthetics, which temporarily
affect, and in some instances completely eliminate, sensation. Analgesics include
paracetamol (known in North America as acetaminophen or simply APAP), the
nonsteroidal anti-inflammatory drugs (NSAIDs) such as the salicylates, and opioid
drugs such as morphine and oxycodone. When choosing analgesics, the severity
and response to other medication determines the choice of agent; the World Health
Organization (WHO) pain ladder specifies mild analgesics as its first step.
READING SUB-TEST : PART C

In this part of the test, there are two texts about different aspects of healthcare.
For questions 7-22, choose the answer (A, B, C or D) which you think fits best according to the text.
Write your answers on the separate Answer Sheet

Text 1: Idiopathic Pulmonary Fibrosis (IPF)

Idiopathic pulmonary fibrosis (IPF) is a build-up of scar tissue in the lungs. This
scar tissue damages the lungs and makes it hard for oxygen to get in. Not getting
enough oxygen to the body can cause serious health problems and even death.
“Idiopathic” is the term used when no cause for the scarring can be found; in
these cases, doctors think the scarring starts by something that injures the lung.
Scar tissue builds up as the lungs try to repair the injury and, in time, so much
scarring forms that patients have problems breathing.
IPF usually worsens over time. However, while some patients get sick quickly,
Reading Test 3 others may not feel sick for years. Unfortunately, there is no cure
for IPF, but there are treatments that may be able to slow down the lung scarring.
Understanding the condition will go a long way to help you cope with the effects it
has on your body.
The two major symptoms of IPF are shortness of breath and a persistent cough.
Other symptoms may include:
 Fatigue and weakness
 Chest pain or tightness in the chest
 Loss of appetite
 Rapid weight loss

The causes of IPF are unknown. There are other conditions that cause lung scarring; the
lung scarring that is the result of other conditions is often called “p ulmonary fibrosis”, but
should be called by the name of the cause.
These other causes include the following:
 Medicines, such as those used for certain heart conditions
 Breathing in mineral dusts, such as asbestos or silica
 Allergies or overexposure to dusts, animals, or molds (There are many names for this
condition, such as “bird breeder’s lung,” “farmer’s lung,” or “humidifier lung.” These
conditions are all called hypersensitivity pneumonitis).
Five million people worldwide have IPF, and it is estimated that up to 200,000
people in the United States have this condition. It usually occurs in adults between
40 and 90 years of age and it is seen more often in men than in women. Although
rare, IPF can sometimes run in families.
Patients who have any symptoms of IPF should see a pulmonologist to rule out
similar conditions. The doctor will take a number of tests, including:
 Breathing tests: to measure how well your lungs are working.
 CT scan: to get a detailed image of your lungs, and to see if scarringhas started.
 Blood tests: to see if you have an infection, problems with your immune system, or to
see how much oxygen is in your blood.
 Bronchoscopy: to test a small sample of lung tissue. A tube is inserted through the nose
or mouth into the lung; a light on the end of the tube lets the doctor see where to go.
The doctor then takes a small piece of lung tissue to be tested (this is called a biopsy).
You usually do not need to stay overnight in the hospital to have this done.
 Thoracoscopic biopsy: to obtain larger tissue samples. This is a surgical procedure in
which small incisions are made in between the ribs. It usually requires a hospital stay
and general anesthesia.
Treatment: Once lung scarring forms, it cannot be removed surgically and there are
currently no medications that remove lung scarring. However, there are treatments,
such as the ones that follow, that may be able to help.
Smoking Cessation: Cigarette smoke not only damages the lining of the lungs, it
can also make you more likely to get a lung infection. While some studies suggest
that patients with IPF who smoke actually live longer, these studies are not
accepted by everyone, and most experts agree that you should stop smoking.

Supplemental oxygen: As lung scarring gets worse, many patients need extra
oxygen to help them go about their daily lives without getting too out of breath.
You get this oxygen from a tank that you carry around with you and, in later stages
of IPF, oxygen may be needed even while sleeping or resting. Oxygen is not
addictive, so you do not have to worry about using it too much. To help maintain
your oxygen levels, ask your doctor about a small, easy-to-use device called a
pulse oximeter. This device helps you to know just how much oxygen-flow you
require, especially during activity.
Exercise: Regular exercise can help patients with IPF. Staying in shape not only
keeps your breathing muscles strong, it also gives you more energy. This is
because healthy muscles need less oxygen to perform work.
Nutrition Many patients with IPF lose weight because of their disease. If you lose
too much weight, your breathing muscles can become weak. You also may not be
able to fight off infections very well. A well-balanced diet is important to keep up
your strength. Be wary of supplements and other nutrition treatments that claim to
improve IPF.
Part C -Text 1: Questions 7-14

7. In IPF, patients
A. will have lung cancer
B. will have difficulty in inhalation or exhalation
C. will find it difficult to move
D. require less oxygen

8. Scar tissue develops


A. when oxygen supplied is stopped
B. when the lungs do not function properly
C. when the lungs try to repair the damage done
D. when there is more oxygen supply

9. Major symptoms of IPF are


A. fatigue and weakness
B. chest pain and breathing
C. breathing problems and coughing
D. breathing problems and weakness

10. The cause of lung scaring is


A. still not known completely
B. known
C. allergies
D. some of the common heart diseases

11.One of the simple IPF tests is


A. bronchoscopy
B. blood test
C. CT scan
D. breathing test to identify how well your lungs work

12. For lung scarring


A. no medication is available
B. medication is available
C. prevention is better
D. not given
13. Cessation means
A. to continue
B. to cease
C. to adopt
D. to gain

14. The term ‘Idiopathic’ in the passage 1 indicates?


A. the disease which occurs randomly without family history or gender
B. when the route cause of a disease is unknown.
C. the disease which occurs in an ideal way
D. The disease which occurs all of a sudden causing death.
Text 2: Hyperthyroidism

The thyroid gland is a butterfly-shaped endocrine gland that is normally located in the
lower front of the neck. The thyroid’s job is to make thyroid hormones, which are
secreted into the blood and then carried to every tissue in the body. The thyroid
hormone helps the body use energy, stay warm and keep the brain, heart, muscles, and
other organs working as they should.

The term hyperthyroidism refers to any condition in which there are too many
thyroid hormones produced in the body. In other words, the thyroid gland is
overactive and working too hard. Another term that you might hear being used to
describe the problem is thyrotoxicosis, which refers to high thyroid hormone levels
in the blood stream, irrespective of their source.

The thyroid hormone plays a significant role in the pace of many processes in the
body; these processes are called your metabolism. If there is too much thyroid
hormone being produced, every function of the body tends to speed up. It is not
surprising then that some of the symptoms of hyperthyroidism are: nervousness,
irritability, increased perspiration, heart racing, hand tremors, anxiety, difficulty
sleeping, thinning of your skin, fine brittle hair and weakness in your muscles—
especially in the upper arms and thighs.

You may have more frequent bowel movements, but diarrhea is uncommon. You
may lose weight despite a good appetite and, for women, menstrual flow may
lighten and menstrual periods may occur less often. Since hyperthyroidism
increases your metabolism, many individuals initially have a lot of energy.
However, as the hyperthyroidism continues, the body tends to break down, so
feeling tired is very common.
Hyperthyroidism usually begins quite slowly but in some young patients these
changes can be very abrupt. At first, the symptoms may be mistaken for simple
nervousness due to stress. If you have been trying to lose weight by dieting, you
may be pleased with your success until the hyperthyroidism, which has quickened
the weight loss, causes other problems.

The most common cause (in more than 70% of people) is an overproduction of the
thyroid hormone by the entire thyroid gland. This condition is also known as
Graves’disease.Graves’diseaseiscausedbyantibodiesinthebloodthatturnon
the thyroid and cause it to grow and secrete too much thyroid hormone. This type
of hyperthyroidism tends to run in families and it occurs more often in young
women. Little is known about why specific individuals get this disease.

Another type of hyperthyroidism is characterized by one or more nodules or lumps


in the thyroid that may gradually grow and increase their activity; this causes the
total output of thyroid hormones into the blood to become greater than normal.
This condition is known as toxic nodular or multi nodular goiter. Also, people may
temporarily have symptoms of hyperthyroidism if they have a condition called
thyroiditis, a condition caused by a problem with the immune system or a viral
infection that causes the gland to leak stored thyroid hormone. The same symptoms
can also occur by taking too much thyroid hormone in tablet form. These last two
forms of excess thyroid hormone are only called thyrotoxicosis, since the thyroid is
not overactive.

If your physician suspects that you have hyperthyroidism, diagnosis is usually a


simple matter. A physical examination usually detects an enlarged thyroid gland
and a rapid pulse. The physician will also look for moist, smooth skin and a tremor
of your fingertips. Your reflexes are likely to be fast, and your eyes may have some
abnormalities if you have Graves’disease.
The diagnosis of hyperthyroidism will be confirmed by laboratory tests that
measure the amount of thyroid hormones— thyroxine (T4) and triiodothyronine
(T3)—and thyroid-stimulating hormone (TSH) in your blood. A high level of
thyroid hormone in the blood plus a low level of TSH is common with an
overactive thyroid gland. If blood tests show that your thyroid is overactive, your
doctor may want to obtain a picture of your thyroid (a thyroid scan). The scan will
find out if your entire thyroid gland is overactive or whether you have a toxic
nodular goiter or thyroiditis (thyroid inflammation). A test that measures the ability
of the gland to collect iodine (a thyroid uptake) may be done at the same time
No single treatment is best for all patients with hyperthyroidism. The appropriate
choice of treatment will be influenced by your age, the type of hyperthyroidism
that you have, the severity of your hyperthyroidism, other medical conditions that
may be affecting your health, and your own preference. It may be a good idea to
consult with an endocrinologist who is experienced in the treatment of
hyperthyroid patients. If you are unconvinced or unclear about any thyroid
treatment plan, a second opinion is a good idea.
Part C -Text 2: Questions 15-22

15. The thyroid hormone helps with


A. energy consumption
B. utilization of energy
C maintaining body temperature
D. enhancing the functions of the kidney

16. In thyrotoxicosis
A. the thyroid gland is inactive
B. the thyroid gland is less active
C. the thyroid gland produces a greater amount of hormones then necessary
D. none of the above

17. An increase in the amount of thyroid hormones can


A. boost up other hormonal functions
B. improve metabolic functions
C. increase normal physiological functions
D. increase pulse rate

18. Which one of these is common in thyroid diseases?


A. loss of appetite
B. decreased metabolism
C. tiredness
D. none of the above

19. Hyperthyroidism can be the cause of …………


A. high BP
B. tiredness
C. weight loss
D. increase in weight, even while dieting

20. According to the information given, “Grave’s disease” occurs more commonly in
A. men
B. women
C. children
D. adult women
21. In hyperthyroidism, the level of thyroid hormones is
A. considerably higher
B. very low
C. much higher
D. normal

22. ‘graves disease’ in paragraph 6 is?


A. condition in which too much thyroid harmone is produced due to some antibodies in the blood
B. over production of thyroid hormone due to the over activity of immature thyroid gland
C. unconditional production of TSH
D. condition in which less thyroid harmone secreated by entire thyroid gland

END OF READING TEST, THIS BOOKLET WILL BE COLLECTED


Reading test 31: Answer Key

Part A - Answer key 1 – 7


1. A
2. D
3. B
4. C
5. D
6. D
7. B

Part A - Answer key 8 – 14

8. osteoporosis
9. testosterone , estrogen
10.bisphossphonates
11.dexa or dxa scan
12.primary &secondary
13.50%
14.$20 billion

Part A - Answer key 15 – 20

15.osteogenesis
16.age
17.calcium absorption
18.raloxifene
19.more than 1.5
20.calcium , vitamin d
Reading test - part B – answer key
1. B
2. A
3. B
4. C
5. B
6. C

Reading test - part C – answer key

Text 1 - Answer key 7 – 14


7. B
8. C
9. C
10. A
11. D
12.A
13.B
14. B

Text 2 - Answer key 15 – 22


15. A
16.C
17. B
18. C
19.C
20. B
21.C
22. A
READING TEST 32
READING SUB-TEST : PART A
 Look at the four texts, A-D, in the separate Text Booklet.
 For each question, 1-20, look through the texts, A-D, to find the relevant information.
 Write your answers on the spaces provided in this Question Paper.
 Answer all the questions within the 15-minute time limit.
 Your answers should be correctly spelt.

TEXT BOOKLET - ANGINA PECTORIS

Text A

Cardiovascular disease is the leading cause of death in the United States for men
and women of all racial and ethnic groups.
Angina pectoris is a clinical syndrome usually characterized by episodes or
paroxysms of pain or pressure in the anterior chest. The cause is insufficient
coronary blood flow, resulting in a decreased oxygen supply when there is
increased myocardial demand for oxygen in response to physical exertion or
emotional stress.
Text B

Risk factors

a. Family history
b. Increasing age, particularly women
c. Gender: men; women, especially after menopause (estrogen seems to provide some
protection)
d. Race; risk appears higher in African-Americans
e. Cigarette smoking contributes to vasoconstriction, platelet activation, arterial smooth
muscle cell proliferation, and reduced oxygen availability
f. Hypertension; widened QRS complex (bundle branch block)
g. Hyperlipidemia: increased total cholesterol; increased LDL (high: 130 to150 mg/dL;
very high: 160 mg/dL or more); increased ratio of total cholesterol orLDL to HDL; low
HDL (less than 40 mg/dL); HDL greater than 60 mg/dL seems tohelp protect against
coronary artery disease (CAD); increased triglycerides (high:200 to 499 mg/dL; very
high: 500 mg/dL or more)
h. Obesity (particularly abdominal obesity)
i. Sedentary lifestyle (contributes to obesity and reduced HDL)
j. Type 2 diabetes
k. Stress; an innate, competitive, aggressive type A personality seems less important
than amount of stress and client’s psychologic response)
l. Metabolic syndrome: cluster of signs including hyperlipidemia, low HDL level,
abdominal obesity, increased BP, insulin resistance, increased levels of C-reactive
protein, and increased fibrinogen level
Text C
Text D

Management of acute MI

A. Improvement of perfusion
(1) Administration of aspirin immediately
(2) Beta blockers or angiotensin II receptor blockers for left ventricular systolic
dysfunction (LVSD)
(3) Thrombolytic therapy within 30 minutes of arrival; anticoagulants
(4) IV nitroglycerin
(5) ACEIs
(6) Antidysrhythmics to maintain cardiac function
(7) PCI within 90 minutes of arrival at emergency department
(8) Intraaortic balloon pump that inflates during diastole and deflates during systole to
decrease cardiac workload by decreasing after load and increasing myocardial perfusion
for cardiogenic shock
(9) Aspirin, beta blocker, and possible antilipidemic prescribed at discharge

B. Promotion of comfort and rest


(1) Analgesics (e.g., IV morphine) to reduce pain, anxiety, and cardiac Work load by
decreasing preload and after load
(2) Oxygen to improve tissue oxygenation
(3) Maintenance of bed or chair rest to decrease oxygen tissue demands
(4) Diet therapy: 2g sodium diet or clear liquids, depending on presence of nausea
PART A -QUESTIONS AND ANSWER SHEET
Questions 1-7
For each of the questions, 1-7, decide which text (A, B, C or D) the information comes from.
You may use any letter more than once.

1. How to assess chest pain? ............................................................................

2. The major cause of angina pectoris ..............................................................

3. Metabolic syndrome may also contribute to angina …………….…………..

4. It is required to take ECG for patients with chest pain………………….……….

5. How acute MI can be managed? ………………………………...

6. Importance of pain medications in management of chest pain…………………..

7. Greater levels of high density lipoprotein in the body protects from cardiac illness.…………

Questions 8-14
Answer each of the questions, 8-4, with a word or short phrase from one of the texts.
Each answer may include words, number of the both. Your answers should be correctly spelled.
8. What is the main symptom of angina pectoris?………………………………….

9. What contributes to obesity and reduced HDL?………………………..

10.Which is the leading cause of death in the US for both the genders?…………………………..

11.What should be administered within half an hour of arrival of client with angina?……………

12.What should be avoided if the client exhibits chest pain which is angina unlikely?…………..

13.What is recommended in order to reduce oxygen tissue demands?………..

14.What is the next step involved in the angina assessment protocol, if the client’s resting ECG is
normal?……………………………………………….
Questions 15-20

Complete each of the sentences, 15- 20, with a word or short phrase from one of the texts. Each
answer may include words, number or both. Your answers should be correctly spelled

15.………………should be administered within one and half hour of arrival.

16.Beta blockers or angiotensin II receptor blockers to be administered for……………..

17.The risk of angina appears higher in……………..

18.…………is used to reduce cardiac workload which works by decreasing after load and increasing
myocardial perfusion.

19.……………contributes to vasoconstriction and platelet activation

20.…………… sodium diet or clear liquids are recommended depending on presence of nausea

END OF PART A, THIS QUESTIONS PAPER WILL BE COLLECTED


READING SUB-TEST : PART B

In this part of the test, there are six short extracts relating to the work of health professionals .
For questions 1-6, choose the answer (A, B or C) which you think fits best according to the text.
Write your answers on the separate Answer Sheet

Questions 1-6

1. According to the paragraph, a living will covers certain situation, except,


a) during the need of resuscitation, ventilation and dialysis
b) to receive only pain medication
c) when there is no life threatening injury is present

A living will addresses many of the medical procedures common in life threatening
situations, such as resuscitation via electric shock, ventilation and
dialysis. One can choose to allow some of these procedures or none of them. One
can also indicate whether he or she wishes to donate his or her organs and tissues
after death. Even if he or she refuses life-sustaining care, he or she can express the
desire to receive pain medication throughout his or her final hours.
In most states, one can extend his or her living will to cover situations where he or
she has no brain activity or where doctors expect him or her to remain unconscious
for the rest of his or her life, even if a terminal illness or life-threatening injury
isn’tpresent.Becausethesesituationscanoccurtoanypersonatanyage,it’sagood idea for all
adults to have a living will.
2. The passage says regarding suprapubic catheter that
a) It is better than indwelling catheters
b) It is preferred for short-term urinary drainage
c) It may cause damage to the urethra, if it is not secured with sutures or tape

Suprapubic catheter

A suprapubic catheter may be used for long-term continuous urinary drainage. This
type of catheter is surgically inserted through a small incision above the pubic area.
Suprapubic bladder drainage diverts urine from the urethra when injury, stricture,
prostatic obstruction, or gynecologic or abdominal surgery has compromised the
flow of urine through the urethra. A suprapubic catheter is often preferred over
indwelling urethral catheters for long-term urinary drainage. Suprapubic catheters
are associated with decreased risk of contamination with organisms from fecal
material, elimination of damage to the urethra, a higher rate of patient satisfaction,
and lower risk of catheter-associated urinary tract infections. The drainage tube is
secured with sutures or tape. Care of the patient with a suprapubic catheter
includes skin care around the insertion site; care of the drainage tubing and
drainage bag is the same as for an indwelling catheter.
3. Evaluation of breast cancer by using PET/CT scan
a) provides accurate results than the PET scan alone
b) to be compared with PET or CT scan alone
c) has been receiving no attention because it’s results to be compared with

the PET or CT scan alone.

The Role of PET/CT for Evaluating Breast Cancer

Positron emission tomography combined with computed tomography (PET/CT)


has been receiving increasing attention during the recent years for making the
diagnosis, for determining the staging and for the follow-up of various malignancies.
The PET/CT findings of 58 breast cancer patients (age range: 34 79 years old, mean age:
50 years) were retrospectively compared with the PET or CT scans alone. PET/CT was
found to be better than PET or CT alone for detecting small tumors or multiple
metastases, for accurately localizing lymph node metastasis and for monitoring the
response to chemotherapy in breast cancer patients.
4. The purpose of the guidelines is;
a) To conduct competitions in environmental health, between the health care providers
regarding management of pediatric asthma
b) Merge environmental management of asthma with child health care
c) To provide environmental instructions to the patients

Extract from“Environmental Management of Pediatric Asthma:


Guidelines for Health Care Providers”

These guidelines are aimed at integrating environmental management of asthma


into pediatric health care. These documents outline competencies in environmental
health relevant to pediatric asthma that should be mastered by health care
providers, and outlines the environmental interventions that should be
communicated to patients. The guidelines contain three components: Competencies
(Competencias en Espanol): An outline of the knowledge and skills that health care
providers and health professional students should master and demonstrate in order
to incorporate management of environmental asthma triggers into pediatric
practice. Environmental History Form: A quick, user-friendly document that can
be used as an intake tool to help determine pediated patients' environmental asthma
triggers. Environmental Intervention Guidelines: Follow-up questions and
intervention solutions to environmental asthma triggers.
5. While dispensing dangerous drugs
a) RN must document the preparation and performance time
b) RN must follow the written drug dispersing procedure as well as nurseprotocol
c) Pharmacist and physician may need to sign the procedure

Subject: DISPENSING DANGEROUS DRUGS

RNs are authorized to dispense dangerous drugs only under the following conditions: The
dispensing is in accordance with a written drug dispensing procedure and under the
authority of an order issued in conformity with a nurse protocol. There must be
documented preparation and performance (i.e., ability to perform) specific to dispensing
dangerous drugs based on a written dispensing procedure. Documentation should
include that each RN has read and understands the drug dispensing procedure. A copy of
the drug dispensing procedure must be
accessible in each of the specific settings where RNs dispense under nurse protocols and
be available upon request. The procedure must be signed by the pharmacist and
physician who have established it.
6. A metabotropic receptors
a) form an ion channel pore
b) are coupled with G-proteins
c) are directly linked with ion channels on the plasma membrane

A metabotropic receptor

A metabotropic receptor is a type of membrane receptor of eukaryotic cells that


acts through a second messenger. It may be located at the surface of the cell or in
vesicles. Based on their structural and functional characteristics, the
neurotransmitter receptor can be classified into two broad categories: metabotropic
and ionotropic receptors. Ionotropic receptors form an ion channel pore. In
contrast, metabotropic receptors are indirectly linked with ion channels on the
plasma membrane of the cell through signal transduction mechanisms, often G
proteins. Hence, G protein-coupled receptors are inherently metabotropic. Other
examples of metabotropic receptors include tyrosine kinases and
guanylyl cyclase receptors. Both receptor types are activated by specific
neurotransmitters. When an ionotropic receptor is activated, it opens a channel that
allows ions such as Na+, K+, or Cl-to flow. In contrast, when a metabotropic
receptor is activated, a series of intracellular events are triggered that can also
result in ion channels opening or other intracellular events, but involve a range of
second messenger chemicals
READING SUB-TEST : PART C

In this part of the test, there are two texts about different aspects of healthcare.
For questions 7-22, choose the answer (A, B, C or D) which you think fits best according to the text.
Write your answers on the separate Answer Sheet

Text 1: Ebola Virus and Marburg Virus

The Ebola virus and Marburg virus are related viruses that cause hemorrhagic
fevers — illnesses marked by severe bleeding (hemorrhage), organ failure and, in
many cases, death. Both the Ebola virus and Marburg virus are native to Africa,
where sporadic outbreaks have occurred for decades.

The Ebola virus and Marburg virus both live in animal hosts, and humans can
contract the viruses from infected animals. After the initial transmission, the
viruses can spread from person to person through contact with bodily fluids or
contaminated needles.

No drug has been approved to treat the Ebola virus or Marburg virus. People
diagnosed with the Ebola or Marburg virus receive supportive care and treatment
for complications. Scientists are coming closer to developing vaccines for these
deadly diseases.

In both the Ebola virus and Marburg virus, signs and symptoms typically begin
abruptly within the first five to 10 days of infection. Early signs and symptoms
include: fever, severe headaches, joint and muscle aches, chills, sore throat and
weakness. Over time, symptoms become increasingly severe and may include:
nausea and vomiting, diarrhea (may be bloody), red eyes, raised rash, chest pain
and coughing, stomach pain, severe weight loss, bleeding from the nose, mouth,
rectum, eyes and ears.
Ebola virus has been found in African monkeys, chimps and other nonhuman
primates. A milder strain of Ebola has been discovered in monkeys and pigs in the
Philippines. The Marburg virus has been found in monkeys, chimps and fruit bats
in Africa.

The virus can be transmitted to humans by exposure to an infected animal's bodily


fluids, including blood. Butchering or eating infected animals can spread the
viruses; scientists who have operated on infected animals as part of their research
have also contracted the virus.

Infected people typically don't become contagious until they develop symptoms.
Family members are often infected as they care for sick relatives or prepare the
dead for burial. Medical personnel can be infected if they don't use protective gear
such as surgical masks and latex gloves. Medical centers in Africa are often so
poor that they must reuse needles and syringes and some of the worst Ebola
epidemics have occurred because contaminated injection equipment wasn't
sterilized between uses. There's no evidence that the Ebola virus or Marburg virus
can be spread via insect bites.

Ebola and Marburg hemorrhagic fevers are difficult to diagnose because many of
the early signs and symptoms resemble those of other infectious diseases, such as
typhoid and malaria. But if doctors suspect that you have been exposed to the
Ebola virus or Marburg virus, they use laboratory tests that can identify the viruses
within a few days.

Most people with Ebola or Marburg hemorrhagic fever have high concentrations of
the virus in their blood. Blood tests known as enzyme-linked immunosorbent assay
’(ELISA)’ and reverse transcriptase polymerase chain reaction ‘(PCR)’ can detect specific
genes or the virus or antibodies to them.
No antiviral medications have proved effective in treating Ebola virus or Marburg
virus infection. As a result, treatment consists of supportive hospital care. This
includes providing fluids, maintaining adequate blood pressure, replacing blood
loss and treating any other infections that develop.

As with other infectious diseases, one of the most important preventive measures
for Ebola virus and Marburg virus is frequent hand-washing. Use soap and water,
or use alcohol-based hand rubs containing at least 60 percent alcohol when soap
and water aren't available. In developing countries, wild animals, including
nonhuman primates, are sold in local markets. Avoid buying or eating any of these
animals. In particular, caregivers should avoid contact with the person's body fluids and
tissues, including blood, semen, vaginal secretions and saliva. People with Ebola or
Marburg are most contagious in the later stages of the disease.

If you're a health care worker, wear protective clothing — such as gloves, masks,
gowns and eye shields. Keep infected people isolated from others. Carefully
disinfect and dispose of needles and other instruments. Injection needles and
syringes should not be reused.

Scientists are working on a variety of vaccines that would protect people from
Ebola or Marburg viruses. Some of the results have been promising, but further
testing is needed.
Part C -Text 1: Questions 7-14

7. The Ebola and Marburg Viruses are native to


A. America
B. Japan
C .Africa
D. China

8. According to the passage(s), the Ebola and Marburg viruses


A. spread from person to person only
B. spread from animals to humans
C. spread from animals to animals
D. spread person to person after initial transmission from the infected animals

9. One of these statements is true, according to the information given in the passage(s)
A. scientists have developed vaccines for the treatment of Ebola and Marburg diseases
B. scientists are closer to developing an effective vaccine for the treatment
C. it is not possible to fight the diseases caused by Ebola and Marburg
D. scientists have discovered the causes of the transmission of the viruses from animal to human

10. Symptoms are typically seen within


A. five days
B. ten days
C. five to seven days
D. five to ten days

11. In the Philippines, Ebola was discovered in


A. chimpanzees
B. human primates
C. non-human primates
D. monkeys
12. Most known Ebola diseases occur due to
A. contamination
B. bodily fluids
C. contaminated needles and syringes
D. none

13. People with hemorrhagic fever show


A. high number of viruses in their blood
B. low concentrations of virus
C. high concentrations of antibodies
D. low concentrations of antibodies

14. As a health care worker, you


A. should keep infected people totally isolated from others
B. should not reuse needles and syringes for the second time
C. should wear clothing such as gowns and eye shields
D. none of the above
Text 2: A Chronic Disease - Atopic Dermatitis

Atopic dermatitis is a common chronic skin disease. It is also called atopic eczema.
‘Atopic’ is a term used to describe allergic conditions such as asthma and hay fever. Both
dermatitis and eczema mean inflammation of the skin. People with atopic dermatitis tend
to have dry, itchy and easily irritated skin.
They may have times when their skin is clear and other times when they have rash.

In infants and small children, the rash is often present on the skin around the knees
and elbows and the cheeks. In teenagers and adults, the rash is often present in the
creases of the wrists, elbows, knees or ankles, and on the face or neck.

Atopic dermatitis usually begins and ends during childhood, but some people
continue to have the disease into adulthood. If you have ever had atopic dermatitis,
you may have trouble with one or more of these: dry, sensitive skin, hand
dermatitis and skin infections.

The exact cause of atopic dermatitis is unknown. Research suggests that atopic
dermatitis and other atopic diseases are genetically determined; this means that you
are more likely to have atopic dermatitis, food allergies, asthma and/or hay fever if
your parents or other family members have ever had atopic dermatitis. These
diseases may develop one after another over a period of years.
This is called the atopic march.

Recognizing that a person with atopic dermatitis is at a higher risk of developing


one of these diseases is important for parents, patients and health care providers.
Knowing that a child with a slight wheeze has had a history of atopic dermatitis,
for example makes it easier to diagnose the subtle onset of asthma. There are many
things that make the itching and rash of atopic dermatitis worse. When you learn
more about atopic dermatitis and how to avoid things that make it worse, you may
be able to lead a healthier life.
If you have a reaction to something you touch, breathe or eat, you might have an
allergy. Allergies can trigger or worsen your atopic dermatitis symptoms. Common
causes of allergy are: dust mites, furry and feathered animals, cockroaches, pollen,
mold, foods, chemicals.

Your health care provider may recommend allergy testing and food challenges to
see if allergies worsen itching or rashes. Allergy testing may include skin testing,
blood tests or patch tests. Many measures can be taken to avoid things to which
you are allergic. Although many of the measures can be done for the entire home,
the bedroom is the most important room to make skin friendly. Talking with health care
provider about what measures you can take to avoid your allergens can be very beneficial.

Food allergies may be the cause of itching or rashes that occur immediately after
eating, especially in children. Some common food allergens include milk, eggs,
peanuts, wheat, nuts, soy and seafood. Most people are allergic to only one, two or
at the most three foods. Be aware that diet restrictions can lead to poor nutrition
and growth delay in babies and children. Talk with your health care provider about
maintaining a well-balanced diet.

Emotions and stress do not cause atopic dermatitis, but they may bring on itching
and scratching. Anger, frustration and embarrassment can cause flushing and
itching. Day to day stresses as well as major stressful events can lead to or worsen
the itch-scratch cycle. The medications used in atopic dermatitis include: Topical
steroids, Topical immuno modulators, Tar products, Antiinfectives, Antihistamines.

Steroid medicines that are applied to the skin are called topical steroids. Topical
steroids are drugs that fight inflammation. They are very helpful when rash is not
well controlled. Topical steroids are available in many forms such as ointments,
creams, lotions and gels. It is important to know that topical steroids are made in
low to super potent strengths. Steroid pills or liquids, like prednisone, should be avoided
because of side effects and because the rash often comes back after they are stopped.
Part C -Text 2: Questions 15-22
15. People with atopic dermatitis suffer from
A. hay fever
B. asthma
C. dry, itchy and irritated skin
D. rashes

16. In small children, a rash is seen


A. around elbows
B. on the face
C. on the neck
D. around the knees

17. People with atopic dermatitis have


A. dry skin
B. skin infections
C. hand dermatitis
D. all of the above

18. The term atopic in the passage 1 refers to


A. allergic diseases
B. asthma and hay fever
C. allergic conditions like hay fever
D. allergic conditions like asthma

19. ………….can worsen dermatitis symptoms


A. allergies
B. pollen
C. dust
D. mold
20. According to the information given in the passage(s), avoiding allergens is
A. easy
B. difficult
C. sometimes easy and sometimes difficult
D.can’tsay

21. Allergic conditions like asthma in patients who have had a history of atopic
dermatitis can be easily diagnosed by health professionals,‖ this statement is
A. out of the paragraphs given
B. false
C. true
D. can be true or can be false

22. According to information given, common food allergens can be in


A. milk, egg, fish
B. fish, nuts, soya been
C. nuts, eggs, wheat
D. milk, cereals, sea food

END OF READING TEST, THIS BOOKLET WILL BE COLLECTED


Reading test 32 : Answer Key
Part A - Answer key 1 – 7

1. C
2. A
3. B
4. C
5. D
6. D
7. B

Part A - Answer key 8 – 14

8. chest pain
9. sedentary life style
10.cardiovascular disease
11.thrombolytic therapy / anticoagulants
12.unnecessary tests.
13.bedrest/chair rest
14.exercise ECG

Part A - Answer key 15 – 20

15.Pic
16.LVSD
17.African – American
18.Intraortic balloon pump
19.Cigarette smoking
20.2G
Reading test - part B – answer key
1. C
2. A
3. A
4. B
5. B
6. B

Reading test - part C – answer key

Text 1 - Answer key 7 – 14


7. C
8. D
9. B and D
10. D
11. D
12.C
13. A
14. B

Text 2 - Answer key 15 – 22


15. C
16. D
17. D
18. B
19. A
20. A
21. C
22. C
READING TEST 33
READING SUB-TEST : PART A
 Look at the four texts, A-D, in the separate Text Booklet.
 For each question, 1-20, look through the texts, A-D, to find the relevant information.
 Write your answers on the spaces provided in this Question Paper.
 Answer all the questions within the 15-minute time limit.
 Your answers should be correctly spelt.

PART A -TEXT BOOKLET - ANAPHYLAC TIC SHOCK

Text A
Anaphylactic shock occurs rapidly and is life-threatening.
Anaphylactic shock is a systemic, type I hypersensitivity reaction that often has fatal
consequences. Anaphylaxis causes the immune system to release a flood of chemicals that
can cause a person to go into shock.
Text B

Pathophysiology:
Anaphylaxis occurs in an individual after re exposure to an antigen to which that person
has produced a specific lgE antibody.
Re exposure: Upon re exposure to the sensitized allergen, the allergen may cross-link the
mast cell or basophil surface-bound allergen-specific IgE resulting in cellular
degranulation as well as de novo synthesis of mediators.
Binding: Immunoglobulin E (IgE) binds to the antigen (the foreign material that provokes
the allergic reaction).
Activation: Antigen-bound IgE then activates receptors on mast cells and basophils.
Inflammatory mediators release. This leads to the release of inflammatory mediators such
as histamine.
Histamine release: Many of the signs and symptoms of anaphylaxis are attributable to
binding of histamine to its receptors; binding to H1 receptors mediates pruritus,
rhinorrhea, tachycardia, and bronchospasm.
Prostaglandin D2: Prostaglandin D2 mediates bronchospasm and vascular dilatation,
principle manifestations of anaphylaxis.
Leukotriene C4: Leukotriene C4 is converted into LTD4 and LTE4, mediators of
hypotension, bronchospasm, and mucous secretion during anaphylaxis in addition to
acting as chemotactic signals for eosinophils and neutrophils.
Causes
Allergy symptoms aren’t usually life-threatening, but a severe allergic reaction can lead to
anaphylaxis.
Food allergies: The most common anaphylaxis triggers in children are food allergies,
such as to peanuts, and tree nuts, fish, shellfish and milk.
Medication allergies: Certain medications, including antibiotics, aspirin and other overthe-
counter pain relievers, and the intravenous (IV) contrast used in some imaging tests.
Insect allergies: Stings from bees, yellow jackets, wasps, hornets and fire ants.
Latex allergy: Latex allergy develops after many previous exposures to latex.
Clinical Manifestations:
An anaphylactic reaction produces the following symptoms:
Anxiety: The first symptoms usually include a feeling of impending doom or fright.
Skin reactions: Skin reactions such as hives, itching, and flushed or pale skin follow.
Shortness of breath: Constriction of the airways and a swollen tongue or throat could
cause wheezing and troubled breathing.
Hypotension: A low blood pressure occurs as one of the major symptoms of shock.
Tachycardia: The heart compensates through pumping faster and trying to deliver blood to
all body systems.
Dizziness: The patient may feel dizzy which could lead to fainting.

Medical Management:
Treatment of anaphylactic shock include:
Remove antigen: Removing the causative antigen such as discontinuing an antibiotic
agent could stop the progression of shock.
Administer medications: Administer medications that restore vascular tone and provide
emergency support of basic life functions.
Cardiopulmonary resuscitation: If cardiac arrest and respiratory arrest are imminent or
have occurred, cardiopulmonary resuscitation is performed.
Endotracheal intubation: Endotracheal intubation or tracheostomy may be necessary to
establish an airway.
Intravenous therapy: IV lines are inserted to provide access for administering fluids
Text C
Text D

Quick reference anaphylaxis algorithm


Rapid assessment:
Airway: look for and relieve airway obstruction; call for help early if there are signs of
obstruction. Remove any traces of allergen remaining (eg, nut fragments caught in teeth,
with a mouthwash; bee stings without compressing any attached venom sacs).
Breathing: look for and treat bronchospasm and signs of respiratory distress.
Circulation: colour, pulse and BP.
Disability: assess whether responding or unconscious.
Exposure: assess skin with adequate exposure, but avoid excess heat loss.
Consider anaphylaxis when there is compatible history of rapid-onset severe allergic-type
reaction with respiratory difficulty and/or hypotension, especially if there are skin changes
present.
Give high-flow oxygen - using a mask with an oxygen reservoir (greater than 10 litres min-1
to prevent reservoir collapse).
Lay the patient flat: Raise the legs (with care, as this may worsen any breathing problems).
In pregnant patients, use a left lateral tilt of at least 15° (to avoid caval compression).
Adrenaline (epinephrine) intramuscularly (IM) in the anterolateral aspect of the middle
third of the thigh (safe, easy, effective):
PART A -QUESTIONS AND ANSWER SHEET

Questions 1-7
For each of the questions, 1-7, decide which text (A, B, C or D) the information comes from.
You may use any letter more than once

1. Food allergy may cause anaphylactic shock……………………….

2. History of previous exposure leads to anaphylaxis in persons with repeated exposure to the same antigen
……………………….

3. Individuals may feel itching after theexposure ………………………….

4. One fifth of the allergic reactions is due to egg ……………………

5. Immediate action to be taken when a person goes into anaphylactic shock………………….

6. Anaphylactic shock may also cause death ……………………….

7. H1 receptors may cause breathing difficulty ……………………….

Questions 8-14
Answer each of the questions, 8-4, with a word or short phrase from one of the texts.
Each answer may include words, number of the both. Your answers should be correctly spelled.

8. What should be given in order to maintain respiration?……………………………

9. What can be avoided by removing the antigen from the body?……………………………

10. What is released by the immune system during anaphylaxis?………………………….

11. What is the most common cause of anaphylaxis in children?…………………………

12. Which allergy may occur after several previous exposures?……………………….

13. Which position is given to pregnant women in order to maintain breathing?……………………..

14. What should be checked to assess circulation?…………………….


Questions 15-20

Complete each of the sentences, 15- 20, with a word or short phrase from one of the texts.
Each answer may include words, number or both. Your answers should be correctly spelled

15. Anaphylactic shock is a ………………….type 1 systemic reaction.

16. One of the major symptoms of shock is…….

17.…………………. is given intramuscularly to stop progression of shock.

18.…………….. symptoms usually do not cause fatal consequences.

19.………………….activates FcRI receptors on mast cells and basophils.

20. Medications to be administered to restore………. and provide emergency support.

END OF PART A, THIS QUESTIONS PAPER WILL BE COLLECTED


READING SUB-TEST : PART B

In this part of the test, there are six short extracts relating to the work of health professionals .
For questions 1-6, choose the answer (A, B or C) which you think fits best according to the text. Write
your answers on the separate Answer Sheet

Questions 1-6

1. In advance directive
a. Patient can make decisions about his or her medical treatment
b. An attending physician can withhold medical interventions from a terminally ill patient
c. Family members have the authority to interpret it to the patient along with physician.

LIVING WILL

A Living Will is the oldest type of health care advance directive. It is a signed, witnessed
(ornotarized) documentcalleda“declaration”or“directive.”Most declarations instruct an
attending physician to withhold or withdraw medical interventions from its signer if he/she is
in a terminal condition and is unable to make decisions about medical treatment.
Sinceanattendingphysicianwhomaybeunfamiliarwiththesigner’swishesand values has the
power and authority to carry out the signer’s directive, certain terms contained in the
document may be interpreted by the physician in a manner that was not intended by the
signer. Family members and others who are familiar with the signer’s values and wishes have
no legal standing to interpret the meaning of the directive.
2. Leukotriene modifiers:
a. Are used to treat condition associated with an allergic response
b. Are released from mast cell , basophils and eosinophils
c. Cause airway constriction

LEUKOTRIENE MODIFIERS

Leukotriene modifiers may be used for the treatment of certain conditions that are
associated with an allergic response. Leukotriene modifiers prevent the action of leukotrienes
in the body. Leukotrienes are substances that are released from mast cells, basophils and
eosinophils. These substances cause airway constriction (a narrowing of the airways),
increased mucus production, swelling and inflammation in the lungs. This causes symptoms
such as wheezing, cough, post nasal drip and shortness of breath. Leukotriene modifiers
include two types of agents: Leukotriene-receptor antagonists, such as montelukast and
zafirlukast, prevent leukotrienes from binding to their receptors Leukotriene synthesis
inhibitors, such as zileuton, block the enzyme, 5- lipoxygenase, which is necessary for the
formation of leukotrienes.
3. Chemical dot thermometers
a. Change color according to the body temperature.
b. Are used usually in pediatric settings.
c. To be kept in rectum for 1 minute.

CHEMICAL DOT THERMOMETERS

Chemical dot thermometers (liquid crystal skin contact thermometers) are sometimes used
as alternatives in pediatric settings. These single-use, disposable, flexible thermometers have
specific chemical mixtures in circles on the thermometer that change color to measure
temperature increments of two tenths of a degree. Place the thermometer in the mouth with
the dot side (sensor) down, into the posterior sublingual pocket. Keep this type of
thermometer in the mouth for 1 minute, in the axilla 3 minutes, and in the rectum 3 minutes.
Read the color change 10 to 15 seconds after removing the thermometer. Read away from
any heat source. Wearable, continuous-use chemical dot thermometers are available. These
are placed under the axilla and must remain in place at least 2 to 3 minutes before taking the
first reading; continuously thereafter. Replace thermometer and assess the underlying skin
every 48 hours.
4. Choose the correct answer;
a. Automatic electronic equipment always provides accurate reading.
b. Incomplete deflation of cuff may lead to arterial perfusion.
c. The cuff consists of pressure sensor, which enhances reading.

ASSESSING BLOOD PRESSURE USING AN ELECTRONIC DEVICE

Automatic, electronic equipment is often used to monitor blood pressure in acute


care settings, during anesthesia, postoperatively, or any time frequent assessments
are necessary. This unit determines blood pressure by analyzing the sounds of
blood flow or measuring oscillations. The machine can be set to take and record
blood pressure readings at preset intervals. Irregular heart rates, excessive patient
movement, and environmental noise can interfere with the readings. Because
electronic equipment is more sensitive to outside interference, these readings are
susceptible to error. The cuff is applied in the same manner as the auscultatory
method, with the microphone or pressure sensor positioned directly over the artery.
When using an automatic blood pressure device for serial readings, check the
cuffed limb frequently. Incomplete deflation of the cuff between measurements can
lead to inadequate arterial perfusion and venous drainage, compromising the circulation in
the limb.
5. Which statement is incorrect?
a. Epinephrine and insulin are administered subcutaneously
b. The medications which are administered subcutaneously are absorbed over a long period
of time
c. Medications are administered into the skin by using a special needle.

TO ALL STAFF
SUBJECT: SUBCUTANEOUS (SUB-Q) INJECTION

Subcutaneous injections are necessary for patients receiving certain medications.


Patients who need medications such as epinephrine and insulin must receive their
required dosage by way of subcutaneous injections. Nurses must use special
needles to administer the injections to patients. These specialized needles are
designed to enter the fatty area just beneath the skin’s surface. The medications
that nurses administer into the fatty layer under the skin by way of subcutaneous
injections are absorbed over several hours. These medications require that patients
receive a slow-release dosage over time.
6. An expectorant
a. Is an agent which acts by dissolving various chemical bonds within secretions
b. Works mainly by lubricating the irritated respiratory tract
c. Works by signaling the body to increase the hydration of secretions

EXPECTORANT AND MUCOLYTIC

An expectorant (from the Latin expectorare, to expel or banish) works by signaling


the body to increase the amount or hydration of secretions, resulting in more yet
clearer secretions and as a byproduct lubricating the irritated respiratory tract. One
expectorant, guaifenesin, is commonly available in many cough syrups. Often the
term "expectorant" is incorrectly extended to any cough medicine, since it is a
universal component. A mucolytic agent is an agent which dissolves thick mucus
and is usually used to help relieve respiratory difficulties. It does so by dissolving
various chemical bonds within secretions, which in turn can lower the viscosity by
altering the mucin-containing components. Alternatively, attacking the affinity
between secretions and the biological surfaces is another avenue, which is used by
adhesives and surfactants.
READING SUB-TEST : PART C

In this part of the test, there are two texts about different aspects of healthcare.
For questions 7-22, choose the answer (A, B, C or D) which you think fits best according to the text.
Write your answers on the separate Answer Sheet

Text 1: Cancer and what is cervical cancer?

The body is made up of trillions of living cells. Normal body cells grow, divide,
and die in an orderly fashion. During the early years of a person's life, normal cells
divide faster to allow the person to grow. After the person becomes an adult, most
cells divide only to replace worn-out or dying cells or to repair injuries.

Cancer begins when cells in a part of the body start to grow out of control. There
are many kinds of cancer, but they all start because of out-of-control growth of
abnormal cells. Cells become cancer cells because of damage to DNA. DNA is in
every cell and directs all its actions. In a normal cell, when DNA gets damaged the
cell either repairs the damage or the cell dies. In cancer cells, the damaged DNA is
not repaired, but the cell doesn’t die like it should. Instead, this cell goes on
making new cells that the body does not need. These new cells will all have the
same damaged DNA as the first cell does.

No matter where a cancer may spread, it is always named after the place where it
started. For example, breast cancer that has spread to the liver is still called breast
cancer, not liver cancer. Likewise, prostate cancer that has spread to the bone is
metastatic prostate cancer, not bone cancer.

Different types of cancer can behave very differently. For example, lung cancer
and breast cancer are very different diseases. They grow at different rates and
respond to different treatments. That is why people with cancer need treatment that
is aimed at their particular kind of cancer.
Not all tumors are cancerous. Tumors that aren’t can cancer called benign.
Benign tumors can cause problems – they can grow very large and press on healthy
organs and tissues. But they cannot grow into (invade) other tissues. Because they
can’tinvade,theyalsocan’tspreadtootherpartsofthebody(metastasize). These
tumors are almost never life threatening.

The cervix is the lower part of the uterus (womb). It is sometimes called the uterine
cervix. The body of the uterus (the upper part) is where a baby grows. The cervix
connects the body of the uterus to the vagina (birth canal). The part of the cervix
closest to the body of the uterus is called the endocervix. The part next to the
vagina is the exocervix (or ectocervix). The 2 main types of cells covering the
cervix are squamous cells (on the exocervix) and glandular cells (on the
endocervix). The place where these 2 cell types meet is called the transformation
zone. Most cervical cancers start in the transformation zone.

Most cervical cancers begin in the cells lining the cervix. These cells do not
suddenly change into cancer. Instead, the normal cells of the cervix first gradually
develop precancerous changes that turn into cancer. Doctors use several terms to
describe these pre-cancerous changes, including cervical intraepithelial neoplasia
(CIN), squamous intraepithelial lesion (SIL), and dysplasia. These changes can be
detected by the Pap test and treated to prevent the development of cancer.

Cervical cancers and cervical pre-cancers are classified by how they look under a
microscope. There are 2 main types of cervical cancers: squamous cell carcinoma
and adenocarcinoma. About 80% to 90% of cervical cancers are squamous cell
carcinomas. These cancers are from the squamous cells that cover the surface of
the exocervix. Under the microscope, this type of cancer is made up of cells that
are like squamous cells. Squamous cell carcinomas most commonly begins where
the exocervix joins the endocervix.
Most of the other cervical cancers are adenocarcinomas. Cervical adenocarcinomas
seem to have becoming more common in the past 20 to 30 years. Cervical
adenocarcinoma develops from the mucus-producing gland cells of the endocervix.
Less commonly, cervical cancers have features of both squamous cell carcinomas
and adenocarcinomas. These are called adenosquamous carcinomas or mixed carcinomas.

Although cervical cancers start from cells with pre-cancerous changes (precancers), only
some of the women with pre-cancers of the cervix will develop
cancer. The change from cervical pre-cancer to cervical cancer usually takes
several years, but it can happen in less than a year. For most women, pre-cancerous
cells will go away without any treatment. Still, in some women pre-cancers turn
into true (invasive) cancers. Treating all pre-cancers can prevent almost all true cancers.
Part C -Text 1: Questions 7-14

7. When a person becomes an adult, the cells divide only to ...........


A. replace old cells
B. replace dying cells
C. repair injuries
D. all of the above

8. In cancer cells
A. DNA is damaged (but it can be repaired)
B. DNA is not totally damaged
C. DNA is permanently damaged, but the cell doesn't die
D. DNA is damaged but the cell doesn't die, instead, gives new cells, containing damaged DNA

9. Breast cancer that has spread to the liver is called


A. liver cancer
B. breast cancer
C. prostate cancer
D. bone cancer

10. Benign tumors


A. can spread to the other parts of the body
B. cannot spread to the other parts of the body
C. are cancerous
D. are highly fatal

11. Most cervical cancers start


A. in squamous cells
B. in glandular cells
C. in the transformation zone
D. all of the above
12. There are ........... main types of cervical cancers
A. two
B. three
C. four
D. five

13. About 80% to 90% of cervical cancers are ............


A. adenocarcinoma
B. a denosquamous carcinomas
C. mixed carcinomas
D. squamous cell carcinomas

14. “all tumors are cancerous” the statement given in paragraph 5 is


A. exactly true
B. exactly false
C. sometimes true
D. sometime false
Text 2: Hepatitis – Viral Liver Infection

Hepatitis (A, B, or C) can be caused by a virus (Viral Hepatitis), drugs, alcohol,


medications, and blood transfusions. Scientists estimate that between 3.5 and 5.3
million people in the USA are living with Hepatitis. A blood test is required to
diagnose Hepatitis infection.

Hepatitis A is a viral liver infection. In most cases the body easily defeats the virus
(much like the flu, which is what you may feel like you have). Because of this it
does not lead to long term liver challenges. Hepatitis A is the most common form
of Hepatitis. It is spread through the feces of a contaminated person. This can
easily be prevented by thoroughly washing hands after using the restroom, before
eating, and after changing a diaper.

Eating raw oysters and undercooked clams can increase your chances of
contracting the virus. If you are traveling in a country where Hepatitis is common
make sure you wash your hands often and well, eat cooked oysters and clams, and
use an antiviral essential oil such as Lemon to help protect yourself.

Hepatitis B is a viral liver infection. Again, most adult bodies are able to fight off
the virus. In this case, it is referred to as Acute (something that does not last long)
Hepatitis B. Hepatitis B is spread through contact with blood or bodily fluids of an
infected person. This can include unprotected sexual intercourse, sharing drug
needles, getting a tattoo with instruments that were not properly cleaned, or by
sharing a personal item such as a razor or toothbrush with an infected person.

A mother who is infected can pass the virus on to her baby during delivery. Again,
the symptoms are flu-like in nature so it often goes undiagnosed. A person who has
Chronic (lasting three months or more) Hepatitis B may show no symptoms until liver
damage has occurred. Hepatitis B can lead to liver damage or cancer; your doctor may
want to do a biopsy to determine the amount of damage your liver has experienced.
Hepatitis C is also a viral liver infection. A few people will contract Hepatitis C
and get better. This is called Acute Hepatitis C. Most, however, will develop
Chronic Hepatitis C and go on to deal with liver damage, cirrhosis of the liver,
liver cancer, and possibly liver failure. Hepatitis C is the number one reason for
liver transplants in the USA.

Hepatitis C is spread through contact with contaminated blood. This can occur by
sharing a needle, receiving a blood transfusion or organ transplant (blood and
organs have been screened for Hepatitis in the USA since 1992), getting a tattoo
with equipment that has not been properly cleaned, and, in rare cases, a mother can
pass the virus on to her baby during birth. Scientists are not sure, but think there
may be a slim possibility that the virus may be passed through unprotected sexual
intercourse.

Symptoms generally do not occur until the virus is causing damage. Again, the
symptoms are flu-like; you may also experience jaundice (yellowish eyes and skin)
after the flu-like symptoms go away. Most people discover they are infected by
having routine tests done or by donating blood or organs and the standard tests
show the infection. There is also a home test you can purchase and do if you
suspect you are infected.

If you are infected with a Hepatitis virus, or if you have been in the past, one of the
most important things that you can do is strengthen your liver. The easiest way to
do this is the Be Young Liver Cleanse. It is simply to take upon rising in the
morning 1 drop of Be Young Lemon essential oil, one drop of Be Young
Peppermint essential oil, and 1 teaspoon to 1 tablespoon of fresh lemon juice,
followed by a glass of water.
‘ Be Young essential oils ’ are absolutely 100% pure, EOBBD tested and
guaranteed to be free of synthetics and extenders. Do not try this with an essential
oil that you are not certain has been properly cared for and tested as you do not
want to increase your liver challenge. When properly supported the liver has a
remarkable capacity for regeneration.
Part C -Text 2: Questions 15-22
15. Hepatitis is caused by
A. virus
B. alcohol consumption
C. medications
D. all of the above

16. This spreads mainly through feces of contaminated person


A. hepatitis A
B. hepatitis B
C. both
D. can't say

17. Most adult bodies are able to fight off this virus A hepatitis
A. virus B hepatitis
B. virus
C. both
D. can't say

18. Hepatitis can lead to


A. cancer
B. more damage to the cells of liver
C. cancerous growth in liver
D. not given

19. In the USA people go for liver transplantation because of


A. hepatitis A
B. hepatitis B
C. hepatitis C
D. all of the above

20. Hepatitis C spreads through


A. sharing needles
B. blood transfusion
C. organ transplantation
D. all of the above
21. A patient may experience jaundice when
A. the flu-like symptoms appear
B. the flu-like symptoms disappear
C. eyes become yellow
D. all of the above

22. “Be Young essential oils”In the final paragraphare


A. free from extenders
B. EOBBD
C. full of synthetics
D. not given

END OF READING TEST, THIS BOOKLET WILL BE COLLECTED


Reading test 33 : Answer Key

Part A - Answer key 1 – 7

Part A - Answer key 8 – 14

Part A - Answer key 15 – 20


Reading test - part B – answer key
1. B
2. A
3. A
4. C
5. C
6. C

Reading test - part C – answer key

Text 1 - Answer key 7 – 14


7. D
8. D
9. B
10. B
11.C
12. A
13. D
14.D

Text 2 - Answer key 15 – 22


15. D
16. A
17. B
18. A
19. C
20. D
21. B
22. A
READING TEST 34
READING SUB-TEST : PART A
 Look at the four texts, A-D, in the separate Text Booklet.
 For each question, 1-20, look through the texts, A-D, to find the relevant information.
 Write your answers on the spaces provided in this Question Paper.
 Answer all the questions within the 15-minute time limit.
 Your answers should be correctly spelt.
TEXT BOOKLET
PART A -QUESTIONS AND ANSWER SHEET

Questions 1-7

For each of the questions, 1-6, decide which text (A, B, C or D) the information comes from.
You may use any letter more than once.

In which text can you find information about


1. the type of injuries that may lead to tetanus?
2. signs that a patient may have tetanus?
3. how to decide whether a tetanus vaccine is necessary?
4. an alternative name for tetanus?
5. possible side-effects of a particular tetanus medication?
6. other conditions which are associated with tetanus?

Questions 7-13
Complete each of the sentences, 7-14, with a word or short phrase from one of the texts.
Each answer may include words, numbers or both. Your answers should be correctly spelt.

Patients at increased risk of tetanus:


7. If a patient has been touching ___________ or earth, they are more susceptible to tetanus.

8. Any ___________ lodged in the site of an injury will increase the likelihood of tetanus.

9. Patients with ___________ fractures are prone to tetanus.

10. Delaying surgery on an injury or burn by more than ____increases the probability of tetanus.

11. If a burns patient has been diagnosed with ____ they are more liable to contract tetanus.

12. A patient who is ____ or a regular recreational drug user will be at greater risk of tetanus.

Management of tetanus-prone injuries:

13. Clean the wound thoroughly and prescribe ___________ if necessary, followed by tetanus
vaccine and HTIG as appropriate.
14. Where will a patient suffering from tetanus first experience muscle contractions?
Questions 15-20

Answer each of the questions, 15-20, with a word or short phrase from one of the texts. Each
answer may include words, numbers or both. Your answers should be correctly spelt.

15. What can muscle spasms in tetanus patients sometimes lead to?

16. If you test for tetanus using a spatula, what type of reaction will confirm the condition?

17. How many times will you have to vaccinate a patient who needs a full course of tetanus vaccine?

18. What should you give a drug user if you're uncertain of their vaccination history?

19. What size of needle should you use to inject HTIG?

20. What might a patient who experienced an adverse reaction to HTIG be unable to stop doing?

END OF PART A, THIS QUESTIONS PAPER WILL BE COLLECTED


READING SUB-TEST : PART B

In this part of the test, there are six short extracts relating to the work of health professionals .
For questions 1-6, choose the answer (A, B or C) which you think fits best according to the text. Write
your answers on the separate Answer Sheet

Questions 1-6

1. Nursing staff can remove a dressing if


A. a member of the surgical team is present.
B. there is severe leakage from the wound.
C. they believe that the wound has healed.

Post-operative dressings

Dressings are an important component of post-operative wound management. Any


dressings applied during surgery have been done in sterile conditions and should ideally
be left in place, as stipulated by the surgical team. It is acceptable for initial dressings to be
removed prematurely in order to have the wound reviewed and, in certain situations, apply
a new dressing. These situations include when the dressing is no longer serving its purpose
(i.e. dressing falling off, excessive exudate soaking through the dressing and resulting in a
suboptimal wound healing environment) or when a wound complication is suspected.
2. As explained in the protocol, the position of the RUM container will ideally
A. encourage participation in the scheme.
B. emphasise the value of recycling.
C. facilitate public access to it.

Unwanted medicine: pharmacy collection protocol


A Returned Unwanted Medicine (RUM) Project approved container will be delivered by the
wholesaler to the participating pharmacy. The container is to be kept in a section of the
dispensary or in a room or enclosure in the pharmacy to which the public does not have
access. The container may be placed in a visible position, but out of reach of the public, as
this will reinforce the message that unwanted prescription drugs can be returned to the
pharmacy and that the returned medicines will not be recycled.
Needles, other sharps and liquid cytotoxic products should not be placed in the container,
but in one specifically designed for such waste.
3. The report mentioned in the memo suggests that
A. data about patient errors may be incomplete.
B. errors by hospital staff can often go unreported.
C. errors in prescriptions pose the greatest threat to patients.

Memo: Report on oral anti-cancer medications

Nurse Unit Managers are directed to review their systems for the administration of oral
anti-cancer drugs, and the reporting of drug errors. Serious concerns have been raised in a
recent report drawing on a national survey of pharmacists.
Please note the following paragraph quoted from the report:
Incorrect doses of oral anti-cancer medicines can have fatal consequences. Over the
previous four years, there were three deaths and 400 patient safety issues involving oral
anti-cancer medicines. Half of the reports concerned the wrong dosage, frequency, quantity
or duration of oral anti-cancer treatment. Of further concern is that errors on the part of
patients may be under-reported. In light of these reports, there is clearly a need for
improved systems covering the management of patients receiving oral therapy
4. What point does the training manual make about anaesthesia workstations?
A. Parts of the equipment have been shown to be vulnerable to failure.
B. There are several ways of ensuring that the ventilator is working effectively.
C. Monitoring by health professionals is a reliable way to maintain patient safety.

Anaesthesia Workstations

Studies on safety in anaesthesia have documented that human vigilance alone is


inadequate to ensure patient safety and have underscored the importance of monitoring
devices. These findings are reflected in improved standards for equipment design,
guidelines for patient monitoring and reduced malpractice premiums for the use of
capnography and pulse oximetry during anaesthesia. Anaesthesia workstations integrate
ventilator technology with patient monitors and alarms to help prevent patient injury in
the unlikely event of a ventilator failure. Furthermore, since the reservoir bag is part of
the circuit during mechanical ventilation, the visible movement of the reservoir bag is
confirmation that the ventilator is functioning
5.In cases of snakebite, the flying doctor should be aware of
A. where to access specific antivenoms.
B. the appropriate method for wound cleaning.
C. the patients most likely to suffer complications.

Memo to Flying Doctor staff: Antivenoms for snakebite

Before starting treatment:


• Do not wash the snakebite site.
• If possible, determine the type of snake by using a 'snake-venom detection kit' to test a
bite site swab or, in systemic envenoming, the person's urine. If venom detection is not
available or has proved negative, seek advice from a poisons information centre.
• Testing blood for venom is not reliable.
• Assess the degree of envenoming; not all confirmed snakebites will result in systemic
envenoming; risk varies with the species of snake.
• People with pre-existing renal, hepatic, cardiac or respiratory impairment and those
taking anticoagulant or antiplatelet drugs may have an increased risk of serious outcome
from snakebite. Children are also especially at increased risk of severe envenoming because
of smaller body mass and the likelihood of physical activity immediately after a bite.
6. What was the purpose of the BMTEC forum?
A. to propose a new way of carrying out cleaning audits
B. to draw conclusions from the results of cleaning audits
C. to encourage more groups to undertake cleaning audits

Cleaning Audits

Three rounds of environmental cleaning audits were completed in 2013-2014. Key


personnel in each facility were surveyed to assess the understanding of environmental
cleaning from the perspective of the nurse unit manager, environmental services manager
and the director of clinical governance. Each facility received a report about their
environmental cleaning audits and lessons learned from the surveys. Data from the 15 units
were also provided to each facility for comparison purposes.
The knowledge and experiences from the audits were shared at the BMTEC Forum in
August 2014. This forum allowed environmental services managers, cleaners, nurses and
clinical governance to discuss the application of the standards and promote new and
improved cleaning practice. The second day of the forum focused on auditor training and
technique with the view of enhancing internal environmental cleaning auditing by the
participating groups.
READING SUB-TEST : PART C

In this part of the test, there are two texts about different aspects of healthcare.
For questions 7-22, choose the answer (A, B, C or D) which you think fits best according to the text.
Write your answers on the separate Answer Sheet

Text 1: Does homeopathy 'work'?

For many, homeopathy is simply unscientific, but regular users hold a very different view.
Homeopathy works by giving patients very dilute substances that, in larger doses, would
cause the very symptoms that need curing. Taking small doses of these substances
- derived from plants, animals or minerals - strengthens the body's ability to heal and
increases resistance to illness or infection. Or that is the theory. The debate about its
effectiveness is nothing new. Recently, Australia's National Health and Medical Research
Council (NHMRC) released a paper which found there were 'no health conditions for
which there was reliable evidence that homeopathy was effective'. This echoed a report
from the UK House of Commons which said that the evidence failed to show a 'credible
physiological mode of action' for homeopathic products, and that what data were available
showed homeopathic products to be no better than placebo. Yet Australians spend at least
$11 million per year on homeopathy.

So what's going on? If Australians - and citizens of many other nations around the world -
are voting with their wallets, does this mean homeopathy must be doing something right?
'For me, the crux of the debate is a disconnect between how the scientific and medical
community view homeopathy, and what many in the wider community are getting out of it,'
says Professor Alex Broom of the University of Queensland. 'The really interesting question
is how can we possibly have something that people think works, when to all intents and
purposes, from a scientific perspective, it doesn't?'
Part of homeopathy's appeal may lie in the nature of the patient-practitioner consultation. In
contrast to a typical 15-minute GP consultation, a first homeopathy consultation might take
an hour and a half. 'We don't just look at an individual symptom in isolation. For us, that
symptom is part of someone's overall health condition,' says Greg Cope, spokesman for the
Australian Homeopathic Association. 'Often we'll have a consultation with someone and find

details their GP simply didn't have time to.' Writer Johanna Ashmore is a case in point. She
sees her homeopath for a one-hour monthly consultation. 'I feel, if I go and say I've got this
health concern, she's going to treat my body to fight it rather than just treat the symptom.'
Most people visit a homeopath after having received a diagnosis from a 'mainstream'
practitioner, often because they want an alternative choice to medication, says Greg Cope.
'Generally speaking, for a homeopath, their preference is if someone has a diagnosis from a
medical practitioner before starting homeopathic treatment, so it's rare for someone to
come and see us with an undiagnosed condition and certainly if they do come undiagnosed,
we'd want to refer them on and get that medical evaluation before starting a course of
treatment,' he says.

Given that homeopathic medicines are by their very nature incredibly dilute - and, some
might argue, diluted beyond all hope of efficacy - they are unlikely to cause any adverse
effects, so where's the harm? Professor Paul Glasziou, chair of the NHMRC's Homeopathy
Working Committee, says that while financial cost is one harm, potentially more harmful are
the non-financial costs associated with missing out on effective treatments. 'If it's just a cold,
I'm not too worried. But if it's for a serious illness, you may not be taking disease-modifying
treatments, and most worrying is things like HIV which affect not only you, but people around
you,' says Glasziou. This is a particular concern with homeopathic vaccines, he says, which
jeopardise the 'herd immunity' - the immunity of a significant proportion of the population -
which is crucial in containing outbreaks of vaccine-preventable diseases.

The question of a placebo effect inevitably arises, as studies repeatedly seem to suggest that
whatever benefits are being derived from homeopathy are more a product of patient faith
rather than of any active ingredient of the medications. However, Greg Cope dismisses this
argument, pointing out that homeopathy appears to benefit even the sceptics: 'We might
see kids first, then perhaps Mum and after a couple of years, Dad will follow and, even
though he's only there reluctantly, we get wonderful outcomes. This cannot be explained
simply by the placebo effect.' As a patient, Johanna Ashmore is aware scientific research
does little to support homeopathy but can still see its benefits. 'If seeing my homeopath each
month improves my health, I'm happy. I don't care how it works, even if it's all in the mind - I
just know that it does.

But if so many people around the world are placing their faith in homeopathy, despite
the evidence against it, Broom questions why homeopathy seeks scientific validation.
The problem, as he sees it, lies in the fact that 'if you're going to dance with conventional
medicine and say "we want to be proven to be effective in dealing with discrete physiological
conditions", then you indeed do have to show efficacy. In my view this is not about broader
credibility per se, it's about scientific and medical credibility - there's actually quite a lot of
cultural credibility surrounding homeopathy within the community but that's not replicated
in the scientific literature.'
Part C -Text 1: Questions 7-14

7. The two reports mentioned in the first paragraph both concluded that homeopathy

A. could be harmful if not used appropriately.


B. merely works on the same basis as the placebo effect.
C. lacks any form of convincing proof of its value as a treatment.
D. would require further investigation before it was fully understood.

8. When commenting on the popularity of homeopathy, Professor Broom shows his

A. surprise at people's willingness to put their trust in it.


B. frustration at scientists' inability to explain their views on it.
C. acceptance of the view that the subject may merit further study.
D. concern over the risks people face when receiving such treatment.

9. Johanna Ashmore's views on homeopathy highlight

A. how practitioners put their patients at ease.


B. the key attraction of the approach for patients.
C. how it suits patients with a range of health problems.
D. the opportunities to improve patient care which GPs miss.

10. In the fourth paragraph, it is suggested that visits to homeopaths

A. occasionally depend on a referral from a mainstream doctor.


B. frequently result from a patient's treatment preferences.
C. should be preceded by a visit to a relevant specialist.
D. often reveal previously overlooked medical problems.

11. What particularly concerns Professor Glasziou?

A. the risks to patients of relying on homeopathic vaccinations


B. the mistaken view that homeopathic treatments can only do good
C. the way that homeopathic remedies endanger more than just the user
D. the ineffectiveness of homeopathic remedies against even minor illnesses
12. Greg Cope uses the expression 'wonderful outcomes' to underline

A. the ability of homeopathy to defy its scientific critics.


B. the value of his patients' belief in the whole process.
C. the claim that he has solid proof that homeopathy works.
D. the way positive results can be achieved despite people's doubts.

13. From the comments quoted in the sixth paragraph, it is clear that Johanna Ashmore is

A. prepared to accept that homeopathy may depend on psychological factors.


B. happy to admit that she was uncertain at first about proceeding.
C. sceptical about the evidence against homeopathic remedies.
D. confident that research will eventually validate homeopathy.

14. What does the word 'this' in the final paragraph refer to?

A. the continuing inability of homeopathy to gain scientific credibility


B. the suggestion that the scientific credibility of homeopathy is in doubt
C. the idea that there is no need to pursue scientific acceptance for homeopathy
D. the motivation behind the desire for homeopathy to gain scientific acceptance
Text 2: Brain-controlled prosthetics
Paralysed from the neck down by a stroke, Cathy Hutchinson stared fixedly at a drinking straw
in a bottle on the table in front of her. A cable rose from the top of her head, connecting her to
a robot arm, but her gaze never wavered as she mentally guided the robot arm, which was
opposite her, to close its grippers around the bottle, then slowly lift the vessel towards her
mouth. Only when she finally managed to take a sip did her face relax. This example illustrates
the strides being taken in brain-controlled prosthetics. But Hutchinson's focused stare also
illustrates the one crucial feature still missing from prosthetics. Her eyes could tell her where the
arm was, but she couldn't feel what it was doing.

Prosthetics researchers are now trying to create prosthetics that can 'feel'. It's a daunting
task: the researchers have managed to read signals from the brain; now they must write
information into the nervous system. Touch encompasses a complicated mix of information
- everything from the soft prickliness of wool to the slipping of a sweaty soft-drink can. The
sensations arise from a host of receptors in the skin, which detect texture, vibration, pain,
temperature and shape, as well as from receptors in the muscles, joints and tendons that
contribute to 'proprioception' - the sense of where a limb is in space. Prosthetics are being
outfitted with sensors that can gather many of these sensations, but the challenge is to get the
resulting signals flowing to the correct part of the brain.

For people who have had limbs amputated, the obvious way to achieve that is to route the
signals into the remaining nerves in the stump, the part of the limb left after amputation. Ken
Horch, a neuroprosthetics researcher, has done just that by threading electrodes into the
nerves in stumps then stimulating them with a tiny current, so that patients felt like their fingers
were moving or being touched. The technique can even allow patients to distinguish basic
features of objects: a man who had lost his lower arms was able to determine the difference
between blocks made of wood or foam rubber by using a sensor-equipped prosthetic hand.
He correctly identified the objects' size and softness more than twice as often as would have
been expected by chance. Information about force and finger position was delivered from the
prosthetic to a computer, which prompted stimulation of electrodes implanted in his upper-arm
nerves.
As promising as this result was, researchers will probably need to stimulate hundreds or
thousands of nerve fibres to create complex sensations, and they'll need to keep the devices
working for many years if they are to minimise the number of surgeries required to replace
them as they wear out. To get around this, some researchers are instead trying to give
patients sensory feedback by touching their skin. The technique was discovered by accident
by researcher Todd Kuiken. The idea was to rewire arm nerves that used to serve the hand,
for example, to muscles in other parts of the body. When the patient thought about closing his
or her hand, the newly targeted muscle would contract and generate an electric signal, driving
movement of the prosthetic.

However, this technique won't work for stroke patients like Cathy Hutchinson. So some
researchers are skipping directly to the brain. In principle, this should be straightforward.
Because signals from specific parts of the body go to specific parts of the brain, scientists
should be able to create sensations of touch or proprioception in the limb by directly activating
the neurons that normally receive those signals. However, with electrical stimulation, all neurons
close to the electrode's tip are activated indiscriminately, so 'even if I had the sharpest needle in
the Universe, that could create unintended effects', says Arto Nurmikko, a neuroengineer. For
example, an attempt to create sensation in one finger might produce sensation in other parts of
the hand as well, he says.

Nurmikko and other researchers are therefore using light, in place of electricity, to activate
highly specific groups of neurons and recreate a sense of touch. They trained a monkey to
remove its hand from a pad when it vibrated. When the team then stimulated the part of its
brain that receives tactile information from the hand with a light source implanted in its skull,
the monkey lifted its hand off the pad about 90% of the time. The use of such techniques in
humans is still probably 10-20 years away, but it is a promising strategy.
Even if such techniques can be made to work, it's unclear how closely they will approximate
natural sensations. Tingles, pokes and vibrations are still a far cry from the complicated
sensations that we feel when closing a hand over an apple, or running a finger along a table's
edge. But patients don't need a perfect sense of touch, says Douglas Weber, a bioengineer.
Simply having enough feedback to improve their control of grasp could help people to perform
tasks such as picking up a glass of water, he explains. He goes on to say that patients who
wear cochlear implants, for example, are often happy to regain enough hearing to hold a phone
conversation, even if they're still unable to distinguish musical subtleties.
Part C -Text 2: Questions 15-22

15. What do we learn about the experiment Cathy Hutchinson took part in?

A. It required intense concentration.


B. It failed to achieve what it had set out to do.
C. It could be done more quickly given practice.
D. It was the first time that it had been attempted.

16. The task facing researchers is described as 'daunting' because

A. signals from the brain can be misunderstood.


B. it is hard to link muscle receptors with each other.
C. some aspects of touch are too difficult to reproduce.
D. the connections between sensors and the brain need to be exact.

17. What is said about the experiment done on the patient in the third paragraph?

A. There was statistical evidence that it was successful.


B. It enabled the patient to have a wide range of feeling.
C. Its success depended on when amputation had taken place.
D. It required the use of a specially developed computer program.

18. What drawback does the writer mention in the fourth paragraph?

A. The devices have a high failure rate.


B. Patients might have to undergo too many operations.
C. It would only be possible to create rather simple sensations.
D. The research into the new technique hasn't been rigorous enough.
19. What point is made in the fifth paragraph?

A. Severed nerves may be able to be reconnected.


B. More research needs to be done on stroke victims.
C. Scientists' previous ideas about the brain have been overturned.
D. It is difficult for scientists to pinpoint precise areas with an electrode.

20. What do we learn about the experiment that made use of light?

A. It can easily be replicated in humans.


B. It worked as well as could be expected.
C. It may have more potential than electrical stimulation.
D. It required more complex surgery than previous experiments.

21. In the final paragraph, the writer uses the phrase 'a far cry from' to underline

A. how much more there is to achieve.


B. how complex experiments have become.
C. the need to reduce people's expectations.
D. the differences between types of artificial sensation.

22. Why does Weber give the example of a cochlear implant?

A. to underline the need for a similar breakthrough in prosthetics


B. to illustrate the fact that some sensation is better than none
C. to highlight the advances made in other areas of medicine
D. to demonstrate the ability of the body to relearn skills

END OF READING TEST ,THIS BOOKLET WILL BE COLLECTED


Reading test 34 : Answer Key

Part A - Answer key 1 – 7


1B
2A
3c
4A
5D
6A
7 organic matter

Part A - Answer key 8 – 14


8 foreign bodies
9 compound
10 6/six hours
11 systemic sepsis
12 immuno(-)suppressed
13 antibiotics
14 (in) (the) jaw

Part A - Answer key 15 – 20


15 broken bones
16 (a) bite reflex
17 5/five (times)
18 (a) booster dose OR booster doses
19 twenty-three/23 gauge
20 crying
Reading test - part B – answer key

1 B there is severe leakage from the wound.


2 A encourage participation in the scheme.
3 A data about patient errors may be incomplete.
4 B There are several ways of ensuring that the ventilator is working effectively.
5 c the patients most likely to suffer complications.
6 B to draw conclusions from the results of cleaning audits

Reading test - part C – answer key

Text 1 - Answer key 7 – 14


7 c lacks any form of convincing proof of its value as a treatment.
8 A surprise at people's willingness to put their trust in it.
9 B the key attraction of the approach for patients.
10 B frequently result from a patient's treatment preferences.
11 c the way that homeopathic remedies endanger more than just the user
12 D the way positive results can be achieved despite people's doubts.
13 A prepared to accept that homeopathy may depend on psychological factors.
14 D the motivation behind the desire for homeopathy to gain scientific acceptance

Text 2 - Answer key 15 – 22


15 A It required intense concentration.
16 D the connections between sensors and the brain need to be exact.
17 A There was statistical evidence that it was successful.
18 B Patients might have to undergo too many operations.
19 D It is difficult for scientists to pinpoint precise areas with an electrode.
20 c It may have more potential than electrical stimulation.
21 A how much more there is to achieve.
22 B to illustrate the fact that some sensation is better than none
READING TEST 35
READING SUB-TEST : PART A
 Look at the four texts, A-D, in the separate Text Booklet.
 For each question, 1-20, look through the texts, A-D, to find the relevant information.
 Write your answers on the spaces provided in this Question Paper.
 Answer all the questions within the 15-minute time limit.
 Your answers should be correctly spelt.

TEXT BOOKLET

READING
PART A -QUESTIONS AND ANSWER SHEET

Questions 1-7

For each question, 1-7, decide which text (A, B, C or D) the information comes from.
You may use any letter more than once.

In which text can you find information about


1. the drug treatment required?
2. which parts of the body can be affected?
3. the various ways calories can be introduced?
4. who to contact to help the patient after they leave hospital?
5. what kind of dressing to use?
6. how long to give drug therapy to the patient?
7. what advice to give the patient regarding needle use?

Questions 8-14
Complete each of the sentences, 8-14, with a word or short phrase from one of the texts. Each
answer may include words, numbers or both. Your answers should be correctly spelt.

Patients at increased risk of tetanus:


8. Which two drugs can you use to treat the clostridium species of pathogen?
9. Which common metabolic condition may occur with NF?
1O. What complication can a patient suffer from if NF isn't treated quickly enough?
11. What procedure can you use with a wound if the patient can't be operated on?
12. What should the patient be told to use to clean an injection site?
13. Which two drugs can be used if you can't use vancomycin?
14. What kind of infection should you use tetracyclines for?
Questions 15-20

Answer each of the questions, 15-20, with a word or short phrase from one of the texts.
Each answer may include words, numbers or both. Your answers should be correctly spelt.

15. The average proportion of patients who die as a result of contracting NF is

16. Patients who have eaten ___________ may be infected with Aeromonas hydrophilia.

17. Patients with Type 2 infection usually present with infected

18. Type 1 NF is also known as ___________

19. The patient needs to be aware of the need to keep glycated haemoglobin levels lower than ___

20. The patient will need a course of ___________ to regain fitness levels after returning home.

END OF PART A, THIS QUESTIONS PAPER WILL BE COLLECTED


READING SUB-TEST : PART B

In this part of the test, there are six short extracts relating to the work of health professionals .
For questions 1-6, choose the answer (A, B or C) which you think fits best according to the text. Write
your answers on the separate Answer Sheet

Questions 1-6

1. The policy document tells us that


A. stop dates aren't relevant in all circumstances.
B. anyone using EPMA can disregard the request for a stop date.
C. prescribers must know in advance of prescribing what the stop date should be.

Prescribing stop dates

Prescribers should write a review date or a stop date on the electronic prescribing system
EPMA or the medicine chart for each antimicrobial agent prescribed. On the EPMA, there
is a forced entry for stop dates on oral antimicrobials. There is not a forced stop date on
EPMA for IV antimicrobial treatment - if the prescriber knows how long the course of
IV should be, then the stop date can be filled in. If not known, then a review should be
added to the additional information, e.g. 'review after 48 hrs'. If the prescriber decides
treatment needs to continue beyond the stop date or course length indicated, then it is their
responsibility to amend the chart. In critical care, it has been agreed that the routine use of
review/stop dates on the charts is not always appropriate.
2. The guidelines inform us that personalised equipment for radiotherapy
A. is advisable for all patients.
B. improves precision during radiation.
C. needs to be tested at the first consultation.

Guidelines: Radiotherapy Simulation Planning Appointment

The initial appointment may also be referred to as the Simulation Appointment. During
this appointment you will discuss your patient's medical history and treatment options,
and agree on a radiotherapy treatment plan. The first step is usually to take a CT scan of
the area requiring treatment. The patient will meet the radiation oncologist, their registrar
and radiation therapists. A decision will be made regarding the best and most comfortable
position for treatment, and this will be replicated daily for the duration of the treatment.
Depending on the area of the body to be treated, personalised equipment such as a face
mask may be used to stabilise the patient's position. This equipment helps keep the patient
comfortable and still during the treatment and makes the treatment more accurate.
3. The purpose of these instructions is to explain how to
A. monitor an ECG reading.
B. position electrodes correctly.
C. handle an animal during an ECG procedure.

CT200CV Veterinarian Electrocardiograph User Manual


Animal connections

Good electrode connection is the most important factor in recording a high quality ECG.
By following a few basic steps, consistent, clean recordings can be achieved.
1. Shave a patch on each forelimb of the animal at the contact site.
2. Clean the electrode sites with an alcohol swab or sterilising agent.
3. Attach clips to the ECG leads.
4. Place a small amount of ECG electrode gel on the metal electrode of the limb strap or
adapter clip. 5. Pinch skin on animal and place clips on the shaved skin area of the animal
being tested. The animal must be kept still.
6. Check the LCD display for a constant heart reading.
7. If there is no heart reading, you have a contact problem with one or more of the leads.
8. Recheck the leads and reapply the clips to the shaven skin of the animal.
4. The group known as 'impatient patients' are more likely to continue with a course of
prescribed medication if
A. their treatment can be completed over a reduced period of time.
B. it is possible to link their treatment with a financial advantage.
C. its short-term benefits are explained to them.

Medication adherence and impatient patients

A recent article addressed the behaviour of people who have a 'taste for the present
rather than the future'. It proposed that these so-called 'impatient patients' are unlikely
to adhere to medications that require use over an extended period. The article proposes
that, an 'impatience genotype' exists and that assessing these patients' view of the future
while stressing the immediate advantages of adherence may improve adherence rates
more than emphasizing potentially distant complications. The authors suggest that rather
than attempting to change the character of those who are 'impatient', it may be wise to
ascertain the patient's individual priorities, particularly as they relate to immediate gains.
For example, while advising an 'impatient' patient with diabetes, stressing improvement
in visual acuity rather than avoidance of retinopathy may result in greater medication
adherence rates. Additionally, linking the cost of frequently changing prescription lenses
when visual acuity fluctuates with glycemic levels may sometimes provide the patient with
an immediate financial motivation for improving adherence.
4. The memo reminds nursing staff to avoid
A. x-raying a patient unless pH readings exceed 5.5.
B. the use of a particular method of testing pH levels.
C. reliance on pH testing in patients taking acid-inhibiting medication.

Checking the position of a nasogastric tube

It is essential to confirm the position of the tube in the stomach by one of the following:
• Testing pH of aspirate: gastric placement is indicated by a pH of less than 4, but may
increase to between pH 4-6 if the patient is receiving acid-inhibiting drugs.
• Blue litmus paper is insufficiently sensitive to adequately distinguish between levels of
acidity of aspirate.
• X-rays: will only confirm position at the time the X-ray is carried out. The tube may have
moved by the time the patient has returned to the ward. In the absence of a positive aspirate
test, where pH readings are more than 5.5, or in a patient who is unconscious or on a
ventilator, an X-ray must be obtained to confirm the initial position of the nasogastric tube.
6. This extract informs us that
A. the amount of oxytocin given will depend on how the patient reacts.
B. the patient will go into labour as soon as oxytocin is administered.
C. the staff should inspect the oxytocin pump before use.

Extract from guidelines: Oxytocin

1 Oxytocin Dosage and Administration


Parenteral drug products should be inspected visually for particulate matter and
discoloration prior to administration, whenever solution and container permit. Dosage of
Oxytocin is determined by the uterine response. The dosage information below is based
upon various regimens and indications in general use.

1.1 Induction or Stimulation of Labour


Intravenous infusion (drip method) is the only acceptable method of administration for
the induction or stimulation of labour. Accurate control of the rate of infusion flow is
essential. An infusion pump or other such device and frequent monitoring of strength of
contractions and foetal heart rate are necessary for the safe administration of Oxytocin
for the induction or stimulation of labour. If uterine contractions become too powerful, the
infusion can be abruptly stopped, and oxytocic stimulation of the uterine musculature will
soon wane.
READING SUB-TEST : PART C

In this part of the test, there are two texts about different aspects of healthcare.
For questions 7-22, choose the answer (A, B, C or D) which you think fits best according to the text.
Write your answers on the separate Answer Sheet

Text 1 : Phobia pills

An irrational fear, or phobia, can cause the heart to pound and the pulse to race. It can
lead to a full-blown panic attack - and yet the sufferer is not in any real peril. All it takes
is a glimpse of, for example, a spider's web for the mind and body to race into panicked
overdrive. These fears are difficult to conquer, largely because, although there are no
treatment guidelines specifically about phobias, the traditional way of helping the sufferer
is to expose them to the fear numerous times. Through the cumulative effect of these
experiences, sufferers should eventually feel an increasing sense of control over their
phobia. For some people, the process is too protracted, but there may be a short cut. Drugs
that work to boost learning may help someone with a phobia to 'detrain' their brain, losing
the fearful associations that fuel the panic.

The brain's extraordinary ability to store new memories and forge associations is so well
celebrated that its dark side is often disregarded. A feeling of contentment is easily evoked
when we see a photo of loved ones, though the memory may sometimes be more idealised
than exact. In the case of a phobia, however, a nasty experience with, say, spiders, that
once triggered a panicked reaction, leads the feelings to resurge whenever the relevant
cue is seen again. The current approach is exposure therapy, which uses a process called
extinction learning. This involves people being gradually exposed to whatever triggers
their phobia until they feel at ease with it. As the individual becomes more comfortable with
each situation, the brain automatically creates a new memory - one that links the cue with
reduced feelings of anxiety, rather than the sensations that mark the onset of a panic attack.
Unfortunately, while it is relatively easy to create a fear-based memory, expunging that
fear is more complicated. Each exposure trial will involve a certain degree of distress in
the patient, and although the process is carefully managed throughout to limit this, some
psychotherapists have concluded that the treatment is unethical. Neuroscientists have been
looking for new ways to speed up extinction learning for that same reason.

One such avenue is the use of 'cognitive enhancers' such as a drug called 0-cycloserine or
DCS. DCS slots into part of the brain's 'NMDA receptor' and seems to modulate the neurons'
ability to adjust their signalling in response to events. This tuning of a neuron's firing is
thought to be one of the key ways the brain stores memories, and, at very low doses, DCS
appears to boost that process, improving our ability to learn. In 2004, a team from Emory
University in Atlanta, USA, tested whether DCS could also help people with phobias. A pilot
trial was conducted on 28 people undergoing specific exposure therapy for acrophobia - a
fear of heights. Results showed that those given a small amount of DCS alongside their
regular therapy were able to reduce their phobia to a greater extent than those given a
placebo. Since then, other groups have replicated the finding in further trials.
For people undergoing exposure therapy, achieving just one of the steps on the long
journey to overcoming their fears requires considerable perseverance, says Cristian Sirbu,
a behavioural scientist and psychologist. Thanks to improvement being so slow, patients -
often already anxious - tend to feel they have failed. But Sirbu thinks that DCS may make it
possible to tackle the problem in a single 3-hour session, which is enough for the patient to
make real headway and to leave with a feeling of satisfaction. However, some people have
misgivings about this approach, claiming that as it doesn't directly undo the fearful response
which is deep-seated in the memory, there is a very real risk of relapse.

Rather than simply attempting to overlay the fearful associations with new ones, Merel Kindt
at the University of Amsterdam is instead trying to alter the associations at source. Kindt's
studies into anxiety disorders are based on the idea that memories are not only vulnerable
to alteration when they're first laid down, but, of key importance, also at later retrieval. This
allows for memories to be 'updated', and these amended memories are re-consolidated by
the effect of proteins which alter synaptic responses, thereby maintaining the strength of
feeling associated with the original memory. Kindt's team has produced encouraging results
with arachnophobic patients by giving them propranolol, a well-known and well-tolerated
beta-blocker drug, while they looked at spiders. This blocked the effects of norepinephrine
in the brain, disrupting the way the memory was put back into storage after being retrieved,
as part of the process of reconsolidation. Participants reported that while they still don't like
spiders, they were able to approach them. Kindt reports that the benefit was still there three
months after the test ended.
Text 1: Questions 7-14
7.In the first paragraph, the writer says that conventional management of phobias can be
problematic because of

A. the lasting psychological effects of the treatment.


B. the time required to identify the cause of the phobia.
C. the limited choice of therapies available to professionals.
D. the need for the phobia to be confronted repeatedly over time.

8. In the second paragraph, the writer uses the phrase 'dark side' to reinforce the idea that

A. memories of agreeable events tend to be inaccurate.


B. positive memories can be negatively distorted over time.
C. unhappy memories are often more detailed than happy ones.
D. unpleasant memories are aroused in response to certain prompts.

9. In the second paragraph, extinction learning is explained as a process which

A. makes use of an innate function of the brain.


B. encourages patients to analyse their particular fears.
C. shows patients how to react when having a panic attack.
D. focuses on a previously little-understood part of the brain.

10. What does the phrase 'for that same reason' refer to?

A. the anxiety that patients feel during therapy


B. complaints from patients who feel unsupported
C. the conflicting ethical concerns of neuroscientists
D. psychotherapists who take on unsuitable patients
11. In the fourth paragraph, we learn that the drug called DCS

A. is unsafe to use except in small quantities.


B. helps to control only certain types of phobias.
C. affects how neurons in the brain react to stimuli.
D. increases the emotional impact of certain events.

12. In the fifth paragraph, some critics believe that one drawback of using DCS is that

A. its benefits are likely to be of limited duration.


B. it is only helpful for certain types of personality.
C. few patients are likely to complete the course of treatment.
D. patients feel discouraged by their apparent lack of progress.

13. In the final paragraph, we learn that Kindt's studies into anxiety disorders focused on how

A. proteins can affect memory retrieval.


B. memories are superimposed on each other
C. negative memories can be reduced in frequency.
D. the emotional force of a memory is naturally retained.

14. The writer suggests that propranolol may

A. not offer a permanent solution for patients' phobias.


B. increase patients' tolerance of key triggers.
C. produce some beneficial side-effects.
D. be inappropriate for certain phobias.
Text 2: Challenging medical thinking on placebos

Dr Damien Finniss, Associate Professor at Sydney University's Pain Management and


Research Institute, was previously a physiotherapist. He regularly treated football players
during training sessions using therapeutic ultrasound. 'One particular session', Finniss
explains, 'I treated five or six athletes. I'd treat them for five or ten minutes and they'd say,
"I feel much better" and run back onto the field. But at the end of the session, I realised the
ultrasound wasn't on.' It was a light bulb moment that set Finniss on the path to becoming a
leading researcher on the placebo effect.

Used to treat depression, psoriasis and Parkinson's, to name but a few, placebos have
an image problem among medics. For years, the thinking has been that a placebo is
useless unless the doctor convinces the patient that it's a genuine treatment - problematic
for a profession that promotes informed consent. However, a new study casts doubt on
this assumption and, along with a swathe of research showing some remarkable results
with placebos, raises questions about whether they should now enter the mainstream as
legitimate prescription items. The study examined five trials in which participants were told
they were getting a placebo, and the conclusion was that doing so honestly can work.
'If the evidence is there, I don't see the harm in openly administering a placebo,' says Ben
Colagiuri, a researcher at the University of Sydney. Colagiuri recently published a meta-analysis
of thirteen studies which concluded that placebo sleeping pills, whose genuine
counterparts notch up nearly three million prescriptions in Australia annually, significantly
improve sleep quality. The use of placebos could therefore reduce medical costs and the
burden of disease in terms of adverse reactions.

But the placebo effect isn't just about fake treatments. It's about raising patients' expectations
of a positive result; something which also occurs with real drugs. Finniss cites the 'openhidden'
effect, whereby an analgesic can be twice as effective if the patient knows they're
getting it, compared to receiving it unknowingly. 'Treatment is always part medical and part
ritual,' says Finniss. This includes the austere consulting room and even the doctor's clothing.
But behind the performance of healing is some strong science. Simply believing an analgesic
will work activates the same brain regions as the genuine drug. 'Part of the outcome of what
we do is the way we interact with patients,' says Finniss.

That interaction is also the focus of Colagiuri's research. He's looking into the 'nocebo'
effect, when a patient's pessimism about a treatment becomes self-fulfilling. 'If you give a
placebo, and warn only 50% of the patients about side effects, those you warn report more
side effects,' says Colagiuri. He's aiming to reverse that by exploiting the psychology of
food packaging. Products are labelled '98% fat-free' rather than '2% fat' because positive
reference to the word 'fat' puts consumers off. Colagiuri is deploying similar tactics. A drug
with a 30% chance of causing a side effect can be reframed as having a 70% chance of not
causing it. 'You're giving the same information, but framing it a way that minimises negative
expectations,' says Colagiuri.

There is also a body of research showing that a placebo can produce a genuine biological
response that could affect the disease process itself. It can be traced back to a study from the
1970s, when psychologist Robert Ader was trying to condition taste-aversion in rats. He gave
them a saccharine drink whilst simultaneously injecting Cytoxan, an immune-suppressant
which causes nausea. The rats learned to hate the drink due to the nausea. But as Ader
continued giving it to them, without Cytoxan, they began to die from infection. Their immune
system had 'learned' to fail by repeated pairing of the drink with Cytoxan. Professor Andrea
Evers of Leiden University is running a study that capitalises on this conditioning effect and
may benefit patients with rheumatoid arthritis, which causes the immune system to attack the
joints. Evers' patients are given the immunosuppressant methotrexate, but instead of always
receiving the same dose, they get a higher dose followed by a lower one. The theory is that
the higher dose will cause the body to link the medication with a damped-down immune
system. The lower dose will then work because the body has 'learned' to curb immunity as a
placebo response to taking the drug. Evers hopes it will mean effective drug regimes that use
lower doses with fewer side effects.
The medical profession, however, remains less than enthusiastic about placebos. 'I'm one
of two researchers in the country who speak on placebos, and I've been invited to lecture at
just one university,' says Finniss. According to Charlotte Blease, a philosopher of science, this
antipathy may go to the core of what it means to be a doctor. 'Medical education is largely
about biomedical facts. 'Softer' sciences, such as psychology, get marginalised because it's
the hard stuff that's associated with what it means to be a doctor.' The result, says Blease,
is a large, placebo-shaped hole in the medical curriculum. 'There's a great deal of medical
illiteracy about the placebo effect ... it's the science behind the art of medicine. Doctors need
training in that.'
Text 2: Questions 15-22

15. A football training session sparked Dr Finniss' interest in the placebo effect because

A. he saw for himself how it could work in practice.


B. he took the opportunity to try out a theory about it.
C. he made a discovery about how it works with groups.
D. he realised he was more interested in research than treatment.

16. The writer suggests that doctors should be more willing to prescribe placebos now because

A. research indicates that they are effective even without deceit.


B. recent studies are more reliable than those conducted in the past.
C. they have been accepted as a treatment by many in the profession.
D. they have been shown to relieve symptoms in a wide range of conditions.

17. What is suggested about sleeping pills by the use of the verb 'notch up'?

A. they may have negative results


B. they could easily be replaced
C. they are extremely effective
D. they are very widely used

18. What point does the writer make in the fourth paragraph?

A. The way a treatment is presented is significant even if it is a placebo.


B. The method by which a drug is administered is more important than its content.
C. The theatrical side of medicine should not be allowed to detract from the science.
D. The outcome of a placebo treatment is affected by whether the doctor believes in it.

19. In researching side effects, Colagiuri aims to

A. discover whether placebos can cause them.


B. reduce the number of people who experience them.
C. make information about them more accessible to patients.
D. investigate whether pessimistic patients are more likely to suffer from them.
20. What does the word '!!' in the sixth paragraph refer to?

A. a placebo treatment
B. the disease process itself
C. a growing body of research
D. a genuine biological response

21. What does the writer tell us about Ader's and Evers' studies?

A. Both involve gradually reducing the dosage of a drug.


B. Evers is exploiting a response which Ader discovered by chance.
C. Both examine the side effects caused by immunosuppressant drugs.
D. Evers is investigating whether the human immune system reacts to placebos as Ader's rats did.

22. According to Charlotte Blease, placebos are omitted from medical training because

A. there are so many practical subjects which need to be covered.


B. those who train doctors do not believe that they work.
C. they can be administered without specialist training.
D. their effect is more psychological than physical.

END OF READING TEST ,THIS BOOKLET WILL BE COLLECTED


Reading test 35 : Answer Key

Part A - Answer key 1 – 7


1B
2A
3c
4D
5c
6B
7D

Part A - Answer key 8 – 14

8 clindamycin (and) penicillin


9 diabetes mellitus
10 septic shock
11 VACI vacuum-assisted closure
12 alcohol pads
13 daptomycin (and) linezolid
14 vibrio (infection)

Part A - Answer key 15 – 20

15 32.2%
16 seafood
17 limbs
18 polymicrobial
19 7%
20 physical therapy
Reading test - part B – answer key

1. A, stop dates aren't relevant in all circumstances.


2. B, improves precision during radiation.
3. B, position electrodes correctly.
4. c, its short-term benefits are explained to them.
5. B, the use of a particular method of testing pH levels.
6. A, the amount of oxytocin given will depend on how the patient reacts.

Reading test - part C – answer key

Text 1 - Answer key 7 – 14


7. D, the need for the phobia to be confronted repeatedly over time.
8. D, unpleasant memories are aroused in response to certain prompts.
9. A, makes use of an innate function of the brain.
10. A, the anxiety that patients feel during therapy
11. c , affects how neurons in the brain react to stimuli.
12. A, its benefits are likely to be of limited duration.
13. D, the emotional force of a memory is naturally retained.
14. B, increase patients' tolerance of key triggers.

Text 2 - Answer key 15 – 22


15. A, he saw for himself how it could work in practice.
16. A, research indicates that they are effective even without deceit.
17. D, they are very widely used
18. A, The way a treatment is presented is significant even if it is a placebo.
19. B, reduce the number of people who experience them.
20. C, a growing body of research
21. B, Evers is exploiting a response which Ader discovered by chance.
22. D, their effect is more psychological than physical.

END OF READING TEST, THIS BOOKLET WILL BE COLLECTED

Potrebbero piacerti anche